26.06.2024

Π€ΠΎΡ€ΠΌΡƒΠ»Π° Смкости цилиндричСского кондСнсатора: Π€ΠΎΡ€ΠΌΡƒΠ»Π° Смкости кондСнсатора, Π‘

Π‘ΠΎΠ΄Π΅Ρ€ΠΆΠ°Π½ΠΈΠ΅

Π€ΠΎΡ€ΠΌΡƒΠ»Π° Смкости кондСнсатора, Π‘

Если q – Π²Π΅Π»ΠΈΡ‡ΠΈΠ½Π° заряда ΠΎΠ΄Π½ΠΎΠΉ ΠΈΠ· ΠΎΠ±ΠΊΠ»Π°Π΄ΠΎΠΊ кондСнсатора, Π° – Ρ€Π°Π·Π½ΠΎΡΡ‚ΡŒ ΠΏΠΎΡ‚Π΅Π½Ρ†ΠΈΠ°Π»ΠΎΠ² ΠΌΠ΅ΠΆΠ΄Ρƒ Π΅Π³ΠΎ ΠΎΠ±ΠΊΠ»Π°Π΄ΠΊΠ°ΠΌΠΈ, Ρ‚ΠΎ Π²Π΅Π»ΠΈΡ‡ΠΈΠ½Π° C, равная:

Β  Β 

называСтся Π΅ΠΌΠΊΠΎΡΡ‚ΡŒΡŽ кондСнсатора. Π­Ρ‚ΠΎ постоянная Π²Π΅Π»ΠΈΡ‡ΠΈΠ½Π°, которая зависит Ρ‚ΠΎ Ρ€Π°Π·ΠΌΠ΅Ρ€ΠΎΠ² ΠΈ устройства кондСнсатора.

Рассмотрим Π΄Π²Π° ΠΎΠ΄ΠΈΠ½Π°ΠΊΠΎΠ²Ρ‹Ρ… кондСнсатора, Ρ€Π°Π·Π½ΠΈΡ†Π° ΠΌΠ΅ΠΆΠ΄Ρƒ ΠΊΠΎΡ‚ΠΎΡ€Ρ‹ΠΌ Π·Π°ΠΊΠ»ΡŽΡ‡Π°Π΅Ρ‚ΡΡ Ρ‚ΠΎΠ»ΡŒΠΊΠΎ Π² Ρ‚ΠΎΠΌ, Ρ‡Ρ‚ΠΎ ΠΌΠ΅ΠΆΠ΄Ρƒ ΠΎΠ±ΠΊΠ»Π°Π΄ΠΊΠ°ΠΌΠΈ ΠΎΠ΄Π½ΠΎΠ³ΠΎ Π²Π°ΠΊΡƒΡƒΠΌ (ΠΈΠ»ΠΈ часто говорят Π²ΠΎΠ·Π΄ΡƒΡ…), ΠΌΠ΅ΠΆΠ΄Ρƒ ΠΎΠ±ΠΊΠ»Π°Π΄ΠΊΠ°ΠΌΠΈ Π΄Ρ€ΡƒΠ³ΠΎΠ³ΠΎ находится диэлСктрик. Π’ Ρ‚Π°ΠΊΠΎΠΌ случаС ΠΏΡ€ΠΈ Ρ€Π°Π²Π½Ρ‹Ρ… зарядах Π½Π° кондСнсаторах Ρ€Π°Π·Π½ΠΎΡΡ‚ΡŒ ΠΏΠΎΡ‚Π΅Π½Ρ†ΠΈΠ°Π»ΠΎΠ² Π²ΠΎΠ·Π΄ΡƒΡˆΠ½ΠΎΠ³ΠΎ кондСнсатора Π±ΡƒΠ΄Π΅Ρ‚ Π² Ρ€Π°Π· мСньшС, Ρ‡Π΅ΠΌ ΠΌΠ΅ΠΆΠ΄Ρƒ ΠΎΠ±ΠΊΠ»Π°Π΄ΠΊΠ°ΠΌΠΈ Π²Ρ‚ΠΎΡ€ΠΎΠ³ΠΎ. Π—Π½Π°Ρ‡ΠΈΡ‚ Π΅ΠΌΠΊΠΎΡΡ‚ΡŒ кондСнсатора с диэлСктриком (C) Π² Ρ€Π°Π· большС, Ρ‡Π΅ΠΌ Π²ΠΎΠ·Π΄ΡƒΡˆΠ½ΠΎΠ³ΠΎ ():

Β  Β 

Π³Π΄Π΅ – диэлСктричСская ΠΏΡ€ΠΎΠ½ΠΈΡ†Π°Π΅ΠΌΠΎΡΡ‚ΡŒ диэлСктрика.

Π—Π° Π΅Π΄ΠΈΠ½ΠΈΡ†Ρƒ Смкости кондСнсатора ΠΏΡ€ΠΈΠ½ΠΈΠΌΠ°ΡŽΡ‚ Π΅ΠΌΠΊΠΎΡΡ‚ΡŒ Ρ‚Π°ΠΊΠΎΠ³ΠΎ кондСнсатора, ΠΊΠΎΡ‚ΠΎΡ€Ρ‹ΠΉ Π΅Π΄ΠΈΠ½ΠΈΡ‡Π½Ρ‹ΠΌ зарядом (1 Кл) заряТаСтся Π΄ΠΎ разности ΠΏΠΎΡ‚Π΅Π½Ρ†ΠΈΠ°Π»ΠΎΠ², Ρ€Π°Π²Π½ΠΎΠΉ ΠΎΠ΄Π½ΠΎΠΌΡƒ Π²ΠΎΠ»ΡŒΡ‚Ρƒ (Π² БИ). Π•Π΄ΠΈΠ½ΠΈΡ†Π΅ΠΉ Смкости кондСнсатора (ΠΊΠ°ΠΊ ΠΈ любой эклСктичСской Смкости) Π² ΠΌΠ΅ΠΆΠ΄ΡƒΠ½Π°Ρ€ΠΎΠ΄Π½ΠΎΠΉ систСмС Π΅Π΄ΠΈΠ½ΠΈΡ† (БИ) слуТит Ρ„Π°Ρ€Π°Π΄ (Π€).

Π€ΠΎΡ€ΠΌΡƒΠ»Π° элСктричСской Смкости плоского кондСнсатора

ПолС ΠΌΠ΅ΠΆΠ΄Ρƒ ΠΎΠ±ΠΊΠ»Π°Π΄ΠΊΠ°ΠΌΠΈ плоского кондСнсатора ΠΎΠ±Ρ‹Ρ‡Π½ΠΎ ΡΡ‡ΠΈΡ‚Π°ΡŽΡ‚ ΠΎΠ΄Π½ΠΎΡ€ΠΎΠ΄Π½Ρ‹ΠΌ. Π•Π³ΠΎ ΠΎΠ΄Π½ΠΎΡ€ΠΎΠ΄Π½ΠΎΡΡ‚ΡŒ Π½Π°Ρ€ΡƒΡˆΠ°Π΅Ρ‚ΡΡ Ρ‚ΠΎΠ»ΡŒΠΊΠΎ ΠΎΠΊΠΎΠ»ΠΎ ΠΊΡ€Π°Π΅Π². ΠŸΡ€ΠΈ вычислСнии Смкости плоского кондСнсатора этими ΠΊΡ€Π°Π΅Π²Ρ‹ΠΌΠΈ эффСктами часто ΠΏΡ€Π΅Π½Π΅Π±Ρ€Π΅Π³Π°ΡŽΡ‚. Π­Ρ‚ΠΎ слСдуСт Π΄Π΅Π»Π°Ρ‚ΡŒ, Ссли расстояниС ΠΌΠ΅ΠΆΠ΄Ρƒ пластинами ΠΌΠ°Π»ΠΎ Π² сравнСнии с ΠΈΡ… Π»ΠΈΠ½Π΅ΠΉΠ½Ρ‹ΠΌΠΈ Ρ€Π°Π·ΠΌΠ΅Ρ€Π°ΠΌΠΈ. Для расчСта Смкости плоского кондСнсатора ΠΏΡ€ΠΈΠΌΠ΅Π½ΡΡŽΡ‚ Ρ„ΠΎΡ€ΠΌΡƒΠ»Ρƒ:

Β  Β 

Π³Π΄Π΅ – элСктричСская постоянная; S – ΠΏΠ»ΠΎΡ‰Π°Π΄ΡŒ ΠΊΠ°ΠΆΠ΄ΠΎΠΉ (ΠΈΠ»ΠΈ наимСньшСй) пластины; d – расстояниС ΠΌΠ΅ΠΆΠ΄Ρƒ пластинами.

ЭлСктричСская Π΅ΠΌΠΊΠΎΡΡ‚ΡŒ плоского кондСнсатора, ΠΊΠΎΡ‚ΠΎΡ€Ρ‹ΠΉ содСрТит N слоСв диэлСктрика Ρ‚ΠΎΠ»Ρ‰ΠΈΠ½Π° ΠΊΠ°ΠΆΠ΄ΠΎΠ³ΠΎ , ΡΠΎΠΎΡ‚Π²Π΅Ρ‚ΡΡ‚Π²ΡƒΡŽΡ‰Π°Ρ диэлСктричСская ΠΏΡ€ΠΎΠ½ΠΈΡ†Π°Π΅ΠΌΠΎΡΡ‚ΡŒ i-Π³ΠΎ слоя , Ρ€Π°Π²Π½Π°:

Β  Β 

Π€ΠΎΡ€ΠΌΡƒΠ»Π° элСктричСской Смкости цилиндричСского кондСнсатора

ЦилиндричСский кондСнсатор прСдставляСтся собой Π΄Π²Π΅ соосных (ΠΊΠΎΠ°ΠΊΡΠΈΠ°Π»ΡŒΠ½Ρ‹Ρ…) цилиндричСскиС проводящиС повСрхности, Ρ€Π°Π·Π½ΠΎΠ³ΠΎ радиуса, пространство ΠΌΠ΅ΠΆΠ΄Ρƒ ΠΊΠΎΡ‚ΠΎΡ€Ρ‹ΠΌΠΈ заполняСт диэлСктрик. ЭлСктричСская Π΅ΠΌΠΊΠΎΡΡ‚ΡŒ цилиндричСского кондСнсатора вычисляСтся ΠΊΠ°ΠΊ:

Β  Β 

Π³Π΄Π΅ l – высота Ρ†ΠΈΠ»ΠΈΠ½Π΄Ρ€ΠΎΠ²; – радиус внСшнСй ΠΎΠ±ΠΊΠ»Π°Π΄ΠΊΠΈ; – радиус Π²Π½ΡƒΡ‚Ρ€Π΅Π½Π½Π΅ΠΉ ΠΎΠ±ΠΊΠ»Π°Π΄ΠΊΠΈ.

Π€ΠΎΡ€ΠΌΡƒΠ»Π° элСктричСской Смкости сфСричСского кондСнсатора

БфСричСским кондСнсатором Π½Π°Π·Ρ‹Π²Π°ΡŽΡ‚ кондСнсатор, ΠΎΠ±ΠΊΠ»Π°Π΄ΠΊΠ°ΠΌΠΈ ΠΊΠΎΡ‚ΠΎΡ€ΠΎΠ³ΠΎ ΡΠ²Π»ΡΡŽΡ‚ΡΡ Π΄Π²Π΅ концСнтричСскиС сфСричСскиС проводящиС повСрхности, пространство ΠΌΠ΅ΠΆΠ΄Ρƒ Π½ΠΈΠΌΠΈ Π·Π°ΠΏΠΎΠ»Π½Π΅Π½ΠΎ диэлСктриком. Π•ΠΌΠΊΠΎΡΡ‚ΡŒ Ρ‚Π°ΠΊΠΎΠ³ΠΎ кондСнсатора находят ΠΊΠ°ΠΊ:

Β  Β 

Π³Π΄Π΅ – радиусы ΠΎΠ±ΠΊΠ»Π°Π΄ΠΎΠΊ кондСнсатора.

ΠŸΡ€ΠΈΠΌΠ΅Ρ€Ρ‹ Ρ€Π΅ΡˆΠ΅Π½ΠΈΡ Π·Π°Π΄Π°Ρ‡ ΠΏΠΎ Ρ‚Π΅ΠΌΠ΅ Β«Π•ΠΌΠΊΠΎΡΡ‚ΡŒ кондСнсатора»

Ρ„ΠΎΡ€ΠΌΡƒΠ»Π° для расчСта элСктричСской Смкости

ΠšΠΎΠ½Π΄Π΅Π½ΡΠ°Ρ‚ΠΎΡ€ – радиоэлСктронный ΠΏΡ€ΠΈΠ±ΠΎΡ€, способный Π½Π°ΠΊΠ°ΠΏΠ»ΠΈΠ²Π°Ρ‚ΡŒ ΠΈ ΠΎΡ‚Π΄Π°Π²Π°Ρ‚ΡŒ заряд. Как ΠΏΡ€Π°Π²ΠΈΠ»ΠΎ, Π½Π° Π΅Π³ΠΎ корпусС даСтся информация ΠΎ Π΅Π³ΠΎ Смкости, Π½ΠΎ ΠΈΠ½ΠΎΠ³Π΄Π° трСбуСтся самому Ρ€Π°ΡΡΡ‡ΠΈΡ‚Π°Ρ‚ΡŒ этот Π½ΠΎΠΌΠΈΠ½Π°Π». ΠšΠΎΠ½Π΄Π΅Π½ΡΠ°Ρ‚ΠΎΡ€Π°ΠΌΠΈ ΠΌΠΎΠ³ΡƒΡ‚ Π²Ρ‹ΡΡ‚ΡƒΠΏΠ°Ρ‚ΡŒ ΠΈ ΠΏΡ€ΠΎΠ²ΠΎΠ΄Π½ΠΈΠΊΠΈ, ΠΎΠ½ΠΈ Ρ‚Π°ΠΊΠΆΠ΅ ΠΎΠ±Π»Π°Π΄Π°ΡŽΡ‚ ΠΎΠΏΡ€Π΅Π΄Π΅Π»Π΅Π½Π½ΠΎΠΉ Π΅ΠΌΠΊΠΎΡΡ‚ΡŒΡŽ. Для расчСта сущСствуСт нСсколько Ρ„ΠΎΡ€ΠΌΡƒΠ» Смкости кондСнсатора, ΠΈΡ… ΠΈ рассмотрим.

Π’ Ρ‡Π΅ΠΌ измСряСтся Π΅ΠΌΠΊΠΎΡΡ‚ΡŒ кондСнсатора

Π§Ρ‚ΠΎ Ρ‚Π°ΠΊΠΎΠ΅ заряд Π΅Ρ‰Π΅ проходят Π² школС, ΠΊΠΎΠ³Π΄Π° ΡΠ±ΠΎΠ½ΠΈΡ‚ΠΎΠ²ΡƒΡŽ ΠΏΠ°Π»ΠΎΡ‡ΠΊΡƒ Π½Π°Ρ‚ΠΈΡ€Π°ΡŽΡ‚ ΠΎ ΡˆΠ΅Ρ€ΡΡ‚ΡΠ½ΡƒΡŽ Ρ‚ΠΊΠ°Π½ΡŒ ΠΈ подносят ΠΊ малСньким кусочкам Π±ΡƒΠΌΠ°Π³ΠΈ. Под дСйствиСм элСктромагнитных сил Π±ΡƒΠΌΠ°Π³Π° ΠΏΡ€ΠΈΠ»ΠΈΠΏΠ°Π΅Ρ‚ ΠΊ ΠΏΠ°Π»ΠΎΡ‡ΠΊΠ΅. ΠŸΠΎΠ΄ΠΎΠ±Π½Ρ‹ΠΉ заряд накапливаСтся Π² кондСнсаторС. Но для Π½Π°Ρ‡Π°Π»Π° познакомимся с самим кондСнсатором.

ΠŸΡ€ΠΎΡΡ‚Π΅ΠΉΡˆΠΈΠΌ кондСнсатором ΡΠ²Π»ΡΡŽΡ‚ΡΡ Π΄Π²Π΅ мСталличСскиС пластины, Ρ€Π°Π·Π΄Π΅Π»Π΅Π½Π½Ρ‹Π΅ диэлСктриком. ΠžΡ‚ качСства диэлСктрика зависит, ΠΊΠ°ΠΊ Π΄ΠΎΠ»Π³ΠΎ энСргия заряТСнного кондСнсатора ΠΌΠΎΠΆΠ΅Ρ‚ ΡΠΎΡ…Ρ€Π°Π½ΡΡ‚ΡŒΡΡ. На этих пластинах, ΠΎΠ½ΠΈ Π΅Ρ‰Π΅ Π½Π°Π·Ρ‹Π²Π°ΡŽΡ‚ΡΡ ΠΎΠ±ΠΊΠ»Π°Π΄ΠΊΠ°ΠΌΠΈ, накапливаСтся Ρ€Π°Π·Π½ΠΎΠΈΠΌΠ΅Π½Π½Ρ‹ΠΉ заряд. Как это происходит?

ЭлСктричСский заряд, Π° Π² случаС с ΠΌΠ΅Ρ‚Π°Π»Π»Π°ΠΌΠΈ это элСктроны, способСн ΠΏΠ΅Ρ€Π΅ΠΌΠ΅Ρ‰Π°Ρ‚ΡŒΡΡ ΠΏΠΎΠ΄ дСйствиСм элСктродвиТущСй силы (э. Π΄. с.). ΠŸΠΎΠ΄ΠΊΠ»ΡŽΡ‡Π°Ρ мСталличСскиС пластинки ΠΊ источнику Ρ‚ΠΎΠΊΠ°, ΠΌΡ‹ ΠΏΠΎΠ»ΡƒΡ‡Π°Π΅ΠΌ Π·Π°ΠΌΠΊΠ½ΡƒΡ‚ΡƒΡŽ Ρ†Π΅ΠΏΡŒ, Π½ΠΎ Ρ€Π°Π·Π΄Π΅Π»Π΅Π½Π½ΡƒΡŽ диэлСктриком. ЭлСктростатичСскоС ΠΏΠΎΠ»Π΅ ΠΏΡ€ΠΎΡ…ΠΎΠ΄ΠΈΡ‚ этот диэлСктрик, замыкая Ρ†Π΅ΠΏΡŒ, Π° элСктроны, дойдя Π΄ΠΎ прСпятствия, ΠΎΡΡ‚Π°Π½Π°Π²Π»ΠΈΠ²Π°ΡŽΡ‚ΡΡ ΠΈ ΡΠΊΠ°ΠΏΠ»ΠΈΠ²Π°ΡŽΡ‚ΡΡ.

ΠŸΠΎΠ»ΡƒΡ‡Π°Π΅Ρ‚ΡΡ, Π½Π° ΠΎΠ΄Π½ΠΎΠΉ ΠΎΠ±ΠΊΠ»Π°Π΄ΠΊΠ΅ Π½Π°Π±Π»ΡŽΠ΄Π°Π΅Ρ‚ΡΡ ΠΈΠ·Π±Ρ‹Ρ‚ΠΎΠΊ элСктронов, ΠΈ эта пластина ΠΈΠΌΠ΅Π΅Ρ‚ ΠΎΡ‚Ρ€ΠΈΡ†Π°Ρ‚Π΅Π»ΡŒΠ½Ρ‹ΠΉ Π·Π½Π°ΠΊ, Π° Π½Π° Π΄Ρ€ΡƒΠ³ΠΎΠΉ пластинС элСктронов нСдостаСт Π½Π°ΡΡ‚ΠΎΠ»ΡŒΠΊΠΎ ΠΆΠ΅, Π·Π½Π°ΠΊ Π½Π° этой ΠΎΠ±ΠΊΠ»Π°Π΄ΠΊΠ΅, ΠΊΠΎΠ½Π΅Ρ‡Π½ΠΎ ΠΆΠ΅, Π±ΡƒΠ΄Π΅Ρ‚ ΠΏΠΎΠ»ΠΎΠΆΠΈΡ‚Π΅Π»ΡŒΠ½Ρ‹ΠΌ.

Π’ΠΎΡ‚ Ρ‚Π΅ΠΏΠ΅Ρ€ΡŒ Π½ΡƒΠΆΠ½Π° для опрСдСлСния Смкости кондСнсатора Ρ„ΠΎΡ€ΠΌΡƒΠ»Π°, ΠΎΠΏΡ€Π΅Π΄Π΅Π»ΡΡŽΡ‰Π°Ρ, ΠΊΠ°ΠΊΠΎΠΉ заряд способСн размСстится Π½Π° ΠΊΠΎΠ½ΠΊΡ€Π΅Ρ‚Π½ΠΎΠΌ кондСнсаторС.


Π’ качСствС Π΅Π΄ΠΈΠ½ΠΈΡ†Ρ‹ измСрСния Π² ΠΌΠ΅ΠΆΠ΄ΡƒΠ½Π°Ρ€ΠΎΠ΄Π½ΠΎΠΉ систСмС (БИ) Π΅ΠΌΠΊΠΎΡΡ‚ΡŒ опрСдСляСтся Π² Π€Π°Ρ€Π°Π΄Π°Ρ….

Много это ΠΈΠ»ΠΈ ΠΌΠ°Π»ΠΎ — Π΅ΠΌΠΊΠΎΡΡ‚ΡŒ Π² 1Π€? Π§Ρ‚ΠΎΠ±Ρ‹ кондСнсатор ΠΎΠ±Π»Π°Π΄Π°Π» Π΅ΠΌΠΊΠΎΡΡ‚ΡŒΡŽ Π² 1Π€, ΠΎΠ½ Π΄ΠΎΠ»ΠΆΠ΅Π½ ΡΠΎΠ΄Π΅Ρ€ΠΆΠ°Ρ‚ΡŒ Π² сСбС заряд Π² 1К (ΠΊΡƒΠ»ΠΎΠ½) ΠΈ ΠΏΡ€ΠΈ этом напряТСниС ΠΌΠ΅ΠΆΠ΄Ρƒ ΠΎΠ±ΠΊΠ»Π°Π΄ΠΊΠ°ΠΌΠΈ Π΄ΠΎΠ»ΠΆΠ½ΠΎ Ρ€Π°Π²Π½ΡΡ‚ΡŒΡΡ 1 Π²ΠΎΠ»ΡŒΡ‚Ρƒ.


Π˜Π½Ρ‚Π΅Ρ€Π΅ΡΠ½ΠΎ. Π§Ρ‚ΠΎ Ρ‚Π°ΠΊΠΎΠ΅ заряд Π² 1 ΠΊΡƒΠ»ΠΎΠ½? Если Π΄Π²Π° ΠΏΡ€Π΅Π΄ΠΌΠ΅Ρ‚Π°, ΠΊΠ°ΠΆΠ΄Ρ‹ΠΉ ΠΈΠ· ΠΊΠΎΡ‚ΠΎΡ€Ρ‹Ρ… ΠΈΠΌΠ΅Π΅Ρ‚ заряд Π² ΠΎΠ΄ΠΈΠ½ ΠΊΡƒΠ»ΠΎΠ½ Ρ€Π°Π·ΠΌΠ΅ΡΡ‚ΠΈΡ‚ΡŒ Π² Π²Π°ΠΊΡƒΡƒΠΌΠ΅ Π½Π° расстоянии ΠΎΠ΄ΠΈΠ½ ΠΌΠ΅Ρ‚Ρ€, Ρ‚ΠΎ сила притяТСния ΠΌΠ΅ΠΆΠ΄Ρƒ Π½ΠΈΠΌΠΈ Π±ΡƒΠ΄Π΅Ρ‚ Ρ€Π°Π²Π½Π° силС притяТСния Π·Π΅ΠΌΠ»Π΅ΠΉ Ρ‚Π΅Π»Π° массой Π² ΠΎΠ΄ΠΈΠ½ ΠΌΠΈΠ»Π»ΠΈΠΎΠ½ Ρ‚ΠΎΠ½Π½.

Как ΠΈ любая Π±ΡƒΠΊΠ²Π°Π»ΡŒΠ½Π°Ρ Π΅ΠΌΠΊΠΎΡΡ‚ΡŒ ΠΎΠ΄ΠΈΠ½ ΠΈ Ρ‚ΠΎΡ‚ ΠΆΠ΅ кондСнсатор ΠΌΠΎΠΆΠ΅Ρ‚ Π²ΠΌΠ΅Ρ‰Π°Ρ‚ΡŒ Ρ€Π°Π·Π½ΠΎΠ΅ количСство заряда.

Рассмотрим ΠΏΡ€ΠΈΠΌΠ΅Ρ€.

  • Π’ Ρ‚Ρ€Π΅Ρ…Π»ΠΈΡ‚Ρ€ΠΎΠ²ΡƒΡŽ Π±Π°Π½ΠΊΡƒ Π²Ρ…ΠΎΠ΄ΠΈΡ‚ Ρ‚Ρ€ΠΈ Π»ΠΈΡ‚Ρ€Π° Π²ΠΎΠ·Π΄ΡƒΡ…Π°. Π•Π³ΠΎ Ρ…Π²Π°Ρ‚ΠΈΡ‚ для дыхания, допустим, Π½Π° 3 ΠΌΠΈΠ½ΡƒΡ‚Ρ‹. Но Ссли Π²ΠΎΠ·Π΄ΡƒΡ… Π·Π°ΠΊΠ°Ρ‡Π°Ρ‚ΡŒ ΠΏΠΎΠ΄ ΠΊΠ°ΠΊΠΈΠΌ-Ρ‚ΠΎ Π΄Π°Π²Π»Π΅Π½ΠΈΠ΅ΠΌ, Ρ‚ΠΎ Π΅ΠΌΠΊΠΎΡΡ‚ΡŒ Ρ‚Π°ΠΊ ΠΈ останСтся Ρ‚Ρ€ΠΈ Π»ΠΈΡ‚Ρ€Π°, ΠΎΠ΄Π½Π°ΠΊΠΎ Π΄Ρ‹ΡˆΠ°Ρ‚ΡŒ ΠΌΠΎΠΆΠ½ΠΎ Π±ΡƒΠ΄Π΅Ρ‚ дольшС. Π’Π°ΠΊ устроСн Π°ΠΊΠ²Π°Π»Π°Π½Π³ для Π½Ρ‹Ρ€ΡΠ»ΡŒΡ‰ΠΈΠΊΠΎΠ². ΠŸΠΎΠ»ΡƒΡ‡Π°Π΅Ρ‚ΡΡ, количСство Π²ΠΎΠ·Π΄ΡƒΡ…Π° Π² Π±Π°Π½ΠΊΠ΅ зависит ΠΎΡ‚ давлСния, ΠΊΠΎΡ‚ΠΎΡ€ΠΎΠ΅ Π² Π½Π΅ΠΉ создаСтся. Π’ΠΎΡ‡Π½ΠΎ Ρ‚Π°ΠΊ ΠΆΠ΅ Π΅ΡΡ‚ΡŒ нСкая Π·Π°Π²ΠΈΡΠΈΠΌΠΎΡΡ‚ΡŒ ΠΌΠ΅ΠΆΠ΄Ρƒ Ρ€Π°Π·Π»ΠΈΡ‡Π½Ρ‹ΠΌΠΈ силами, Π²Π»ΠΈΡΡŽΡ‰ΠΈΠΌΠΈ Π½Π° Π΅ΠΌΠΊΠΎΡΡ‚ΡŒ.

Π€ΠΎΡ€ΠΌΡƒΠ»Π° Смкости плоского кондСнсатора

ΠŸΡ€Π΅ΠΆΠ΄Π΅ Ρ‡Π΅ΠΌ ΡƒΠ·Π½Π°Ρ‚ΡŒ, ΠΏΠΎ ΠΊΠ°ΠΊΠΎΠΉ Ρ„ΠΎΡ€ΠΌΡƒΠ»Π΅ вычисляСтся Π΅ΠΌΠΊΠΎΡΡ‚ΡŒ плоского кондСнсатора, рассмотрим Ρ„ΠΎΡ€ΠΌΡƒΠ»Ρƒ для ΠΎΠ΄ΠΈΠ½ΠΎΡ‡Π½ΠΎΠ³ΠΎ ΠΏΡ€ΠΎΠ²ΠΎΠ΄Π½ΠΈΠΊΠ°. Она ΠΈΠΌΠ΅Π΅Ρ‚ Π²ΠΈΠ΄:

  • Π³Π΄Π΅ Q – заряд,
  • Ο† – ΠΏΠΎΡ‚Π΅Π½Ρ†ΠΈΠ°Π».

Как Π²ΠΈΠ΄Π½ΠΎ Π΅ΠΌΠΊΠΎΡΡ‚ΡŒ кондСнсатора, Ρ„ΠΎΡ€ΠΌΡƒΠ»Π° ΠΊΠΎΡ‚ΠΎΡ€ΠΎΠ³ΠΎ здСсь ΠΏΡ€ΠΈΠ²Π΅Π΄Π΅Π½Π°, Π±ΡƒΠ΄Π΅Ρ‚ Ρ‚Π΅ΠΌ большС, Ρ‡Π΅ΠΌ больший заряд способСн Π½Π°ΠΊΠ°ΠΏΠ»ΠΈΠ²Π°Ρ‚ΡŒΡΡ Π½Π° Π½Π΅ΠΌ ΠΏΡ€ΠΈ Π½Π΅Π·Π½Π°Ρ‡ΠΈΡ‚Π΅Π»ΡŒΠ½ΠΎΠΌ ΠΏΠΎΡ‚Π΅Π½Ρ†ΠΈΠ°Π»Π΅. Π§Ρ‚ΠΎΠ±Ρ‹ Π»Π΅Π³Ρ‡Π΅ это Π±Ρ‹Π»ΠΎ ΠΏΠΎΠ½ΡΡ‚ΡŒ, рассмотрим ΠΏΠΎΠ»ΡƒΡ‡ΠΈΠ²ΡˆΠΈΠ΅ ΡˆΠΈΡ€ΠΎΠΊΠΎΠ΅ распространСниС плоскиС кондСнсаторы Ρ€Π°Π·Π½Ρ‹Ρ… Ρ€Π°Π·ΠΌΠ΅Ρ€ΠΎΠ².

Для получСния качСствСнного кондСнсатора Π²Π°ΠΆΠ½Ρ‹ Π»ΡŽΠ±Ρ‹Π΅ ΠΌΠ΅Π»ΠΎΡ‡ΠΈ:

  1. ровная ΠΏΠΎΠ²Π΅Ρ€Ρ…Π½ΠΎΡΡ‚ΡŒ ΠΊΠ°ΠΆΠ΄ΠΎΠΉ ΠΎΠ±ΠΊΠ»Π°Π΄ΠΊΠΈ;
  2. ΠΎΠ±Π΅ пластинки ΠΏΠΎ всСй ΠΏΠ»ΠΎΡ‰Π°Π΄ΠΈ Π΄ΠΎΠ»ΠΆΠ½Ρ‹ Ρ€Π°ΡΠΏΠΎΠ»Π°Π³Π°Ρ‚ΡŒΡΡ Π½Π° ΠΎΠ΄ΠΈΠ½Π°ΠΊΠΎΠ²ΠΎΠΌ расстоянии;
  3. Ρ€Π°Π·ΠΌΠ΅Ρ€Ρ‹ ΠΎΠ±ΠΊΠ»Π°Π΄ΠΎΠΊ Π΄ΠΎΠ»ΠΆΠ½Ρ‹ Π±Ρ‹Ρ‚ΡŒ строго ΠΈΠ΄Π΅Π½Ρ‚ΠΈΡ‡Π½Ρ‹ΠΌΠΈ;
  4. ΠΎΡ‚ качСства диэлСктрика, располоТСнного ΠΌΠ΅ΠΆΠ΄Ρƒ пластинками, Π±ΡƒΠ΄Π΅Ρ‚ Π·Π°Π²ΠΈΡΠ΅Ρ‚ΡŒ Ρ‚ΠΎΠΊ ΡƒΡ‚Π΅Ρ‡ΠΊΠΈ;
  5. Π΅ΠΌΠΊΠΎΡΡ‚ΡŒ Π½Π°ΠΏΡ€ΡΠΌΡƒΡŽ зависит ΠΎΡ‚ расстояния ΠΌΠ΅ΠΆΠ΄Ρƒ ΠΎΠ±ΠΊΠ»Π°Π΄ΠΊΠ°ΠΌΠΈ, Ρ‡Π΅ΠΌ ΠΎΠ½ΠΎ мСньшС, Ρ‚Π΅ΠΌ большС Π΅ΠΌΠΊΠΎΡΡ‚ΡŒ.

Π’Π΅ΠΏΠ΅Ρ€ΡŒ обратимся ΠΊ плоскому кондСнсатору. Π€ΠΎΡ€ΠΌΡƒΠ»Π° опрСдСлСния Смкости кондСнсатора нСсколько отличаСтся ΠΎΡ‚ ΠΏΡ€ΠΈΠ²Π΅Π΄Π΅Π½Π½ΠΎΠΉ Π²Ρ‹ΡˆΠ΅:

  • Π³Π΄Π΅ S – ΠΏΠ»ΠΎΡ‰Π°Π΄ΡŒ ΠΎΠ΄Π½ΠΎΠΉ ΠΎΠ±ΠΊΠ»Π°Π΄ΠΊΠΈ,
  • Ξ΅rΒ — диэлСктричСская ΠΏΡ€ΠΎΠ½ΠΈΡ†Π°Π΅ΠΌΠΎΡΡ‚ΡŒ диэлСктрика,
  • Ξ΅0 — элСктричСская постоянная,
  • d – расстояниС ΠΌΠ΅ΠΆΠ΄Ρƒ ΠΎΠ±ΠΊΠ»Π°Π΄ΠΊΠ°ΠΌΠΈ.

ЭлСктричСская постоянная выраТаСтся числом 8,854187817Γ—10-12.


Π’Π½ΠΈΠΌΠ°Π½ΠΈΠ΅! Π­Ρ‚Π° Ρ„ΠΎΡ€ΠΌΡƒΠ»Π° справСдлива Ρ‚ΠΎΠ»ΡŒΠΊΠΎ Ρ‚ΠΎΠ³Π΄Π°, ΠΊΠΎΠ³Π΄Π° расстояниС ΠΌΠ΅ΠΆΠ΄Ρƒ пластинами Π½Π°ΠΌΠ½ΠΎΠ³ΠΎ мСньшС ΠΈΡ… ΠΏΠ»ΠΎΡ‰Π°Π΄ΠΈ.

ΠŸΠΎΠΏΡ€ΠΎΠ±ΡƒΠ΅ΠΌ Ρ€Π°Π·ΠΎΠ±Ρ€Π°Ρ‚ΡŒΡΡ с ΠΊΠ°ΠΆΠ΄ΠΎΠΉ ΠΏΠ΅Ρ€Π΅ΠΌΠ΅Π½Π½ΠΎΠΉ ΠΏΠΎΠ΄Ρ€ΠΎΠ±Π½Π΅Π΅. ΠŸΠ»ΠΎΡ‰Π°Π΄ΡŒ измСряСтся Π² ΠΌ2, Ρ‚ΠΎΡ‡Π½Π΅Π΅, приводится ΠΊ этой Π²Π΅Π»ΠΈΡ‡ΠΈΠ½Π΅. А Π²ΠΎΡ‚ ΠΏΡ€ΠΎΠ½ΠΈΡ†Π°Π΅ΠΌΠΎΡΡ‚ΡŒ диэлСктрика ΠΌΠΎΠΆΠ΅Ρ‚ ΠΎΠ±ΠΎΠ·Π½Π°Ρ‡Π°Ρ‚ΡŒΡΡ ΠΏΠΎ-Ρ€Π°Π·Π½ΠΎΠΌΡƒ.

Π’ России это Ξ΅rΒ (Ρ‚Π°ΠΊΠΆΠ΅ ΠΎΠ·Π½Π°Ρ‡Π°Π΅Ρ‚ ΠΎΡ‚Π½ΠΎΡΠΈΡ‚Π΅Π»ΡŒΠ½Π°Ρ ΠΏΡ€ΠΎΠ½ΠΈΡ†Π°Π΅ΠΌΠΎΡΡ‚ΡŒ), Π² англоязычной Π»ΠΈΡ‚Π΅Ρ€Π°Ρ‚ΡƒΡ€Π΅ встрСчаСтся Ξ΅aΒ (Ρ‚Π°ΠΊΠΆΠ΅ ΠΎΠ·Π½Π°Ρ‡Π°Π΅Ρ‚ Π°Π±ΡΠΎΠ»ΡŽΡ‚Π½Π°Ρ ΠΏΡ€ΠΎΠ½ΠΈΡ†Π°Π΅ΠΌΠΎΡΡ‚ΡŒ), Π° Ρ‚ΠΎ ΠΌΠΎΠΆΠ΅Ρ‚ ΠΈ вовсС ΠΈΡΠΏΠΎΠ»ΡŒΠ·ΠΎΠ²Π°Ρ‚ΡŒΡΡ Π±Π΅Π· индСкса, просто Ξ΅. О Ρ‚ΠΎΠΌ, Ρ‡Ρ‚ΠΎ здСсь ΠΈΡΠΏΠΎΠ»ΡŒΠ·ΡƒΠ΅Ρ‚ΡΡ диэлСктричСская ΠΏΡ€ΠΎΠ½ΠΈΡ†Π°Π΅ΠΌΠΎΡΡ‚ΡŒ диэлСктрика ΠΌΠΎΠΆΠ½ΠΎ ΠΏΠΎΠ½ΡΡ‚ΡŒ ΠΈΠ· контСкста.

Π”Π°Π»ΡŒΡˆΠ΅ ΠΈΠ΄Π΅Ρ‚ Ξ΅0. Π­Ρ‚ΠΎ ΡƒΠΆΠ΅ вычислСнноС Π·Π½Π°Ρ‡Π΅Π½ΠΈΠ΅, измСряСмоС Π² Π€/ΠΌ. ПослСдняя пСрСмСнная – d. Π˜Π·ΠΌΠ΅Ρ€Π΅Π½Π½ΠΎΠ΅ расстояниС Ρ‚Π°ΠΊΠΆΠ΅ приводится ΠΊ ΠΌΠ΅Ρ‚Ρ€Ρƒ. Π•ΠΌΠΊΠΎΡΡ‚ΡŒ кондСнсатора, Ρ„ΠΎΡ€ΠΌΡƒΠ»Π° ΠΊΠΎΡ‚ΠΎΡ€ΠΎΠ³ΠΎ сСйчас рассматриваСтся, ΠΏΠΎΠΊΠ°Π·Ρ‹Π²Π°Π΅Ρ‚ ΡΠΈΠ»ΡŒΠ½ΡƒΡŽ Π·Π°Π²ΠΈΡΠΈΠΌΠΎΡΡ‚ΡŒ ΠΎΡ‚ расстояния ΠΎΠ±ΠΊΠ»Π°Π΄ΠΎΠΊ. ΠŸΠΎΡΡ‚ΠΎΠΌΡƒ ΡΡ‚Π°Ρ€Π°ΡŽΡ‚ΡΡ это расстояниС ΠΏΠΎ возмоТности ΡΠΎΠΊΡ€Π°Ρ‰Π°Ρ‚ΡŒ. ΠŸΠΎΡ‡Π΅ΠΌΡƒ этот ΠΏΠΎΠΊΠ°Π·Π°Ρ‚Π΅Π»ΡŒ Ρ‚Π°ΠΊ Π²Π°ΠΆΠ΅Π½?

Π˜Π΄Π΅Π°Π»ΡŒΠ½Ρ‹ΠΌΠΈ условиями для получСния наибольшСй Смкости – это отсутствиС ΠΏΡ€ΠΎΠΌΠ΅ΠΆΡƒΡ‚ΠΊΠ° ΠΌΠ΅ΠΆΠ΄Ρƒ ΠΎΠ±ΠΊΠ»Π°Π΄ΠΊΠ°ΠΌΠΈ, Ρ‡Π΅Π³ΠΎ, ΠΊΠΎΠ½Π΅Ρ‡Π½ΠΎ, Π΄ΠΎΠ±ΠΈΡ‚ΡŒΡΡ Π½Π΅Π²ΠΎΠ·ΠΌΠΎΠΆΠ½ΠΎ. Π§Π΅ΠΌ Π±Π»ΠΈΠΆΠ΅ находятся Ρ€Π°Π·Π½ΠΎΠΈΠΌΠ΅Π½Π½Ρ‹Π΅ заряды, Ρ‚Π΅ΠΌ сильнСС сила притяТСния, Π½ΠΎ здСсь Π²ΠΎΠ·Π½ΠΈΠΊΠ°Π΅Ρ‚ компромисс.

ΠŸΡ€ΠΈ ΡƒΠΌΠ΅Π½ΡŒΡˆΠ΅Π½ΠΈΠΈ Ρ‚ΠΎΠ»Ρ‰ΠΈΠ½Ρ‹ диэлСктрика, Π° ΠΈΠΌΠ΅Π½Π½ΠΎ ΠΎΠ½ раздСляСт Ρ€Π°Π·Π½ΠΎΠΈΠΌΠ΅Π½Π½Ρ‹Π΅ заряды, Π²ΠΎΠ·Π½ΠΈΠΊΠ°Π΅Ρ‚ Π²Π΅Ρ€ΠΎΡΡ‚Π½ΠΎΡΡ‚ΡŒ Π΅Π³ΠΎ пробоя ΠΈΠ·-Π·Π° разности ΠΏΠΎΡ‚Π΅Π½Ρ†ΠΈΠ°Π»ΠΎΠ² Π½Π° ΠΎΠ±ΠΊΠ»Π°Π΄ΠΊΠ°Ρ…. Π‘ Π΄Ρ€ΡƒΠ³ΠΎΠΉ стороны, ΠΊΠ°ΠΊ ΡƒΠΆΠ΅ Π³ΠΎΠ²ΠΎΡ€ΠΈΠ»ΠΎΡΡŒ, ΠΏΡ€ΠΈ ΡƒΠ²Π΅Π»ΠΈΡ‡Π΅Π½ΠΈΠΈ напряТСния увСличиваСтся количСство зарядов. Π’ΠΎΡ‚ ΠΈ приходится Π²Ρ‹Π±ΠΈΡ€Π°Ρ‚ΡŒ ΠΌΠ΅ΠΆΠ΄Ρƒ Π΅ΠΌΠΊΠΎΡΡ‚ΡŒΡŽ ΠΈ Ρ€Π°Π±ΠΎΡ‡ΠΈΠΌ напряТСниСм кондСнсатора.

Π•ΡΡ‚ΡŒ другая Ρ„ΠΎΡ€ΠΌΡƒΠ»Π° для плоского ΠΏΠ΅Ρ€Π΅ΠΌΠ΅Π½Π½ΠΎΠ³ΠΎ кондСнсатора:

Π—Π΄Π΅ΡΡŒ диэлСктричСская ΠΏΡ€ΠΎΠ½ΠΈΡ†Π°Π΅ΠΌΠΎΡΡ‚ΡŒ ΠΎΠ±ΠΎΠ·Π½Π°Ρ‡Π΅Π½Π° Π±ΡƒΠΊΠ²ΠΎΠΉ Ξ΅, Ο€ = 22/7 β‰ˆ 3,142857142857143, d – Ρ‚ΠΎΠ»Ρ‰ΠΈΠ½Π° диэлСктрика. Π€ΠΎΡ€ΠΌΡƒΠ»Π° ΠΏΡ€Π΅Π΄Π½Π°Π·Π½Π°Ρ‡Π΅Π½Π° для кондСнсатора, состоящСго ΠΈΠ· Π½Π΅ΡΠΊΠΎΠ»ΡŒΠΊΠΈΡ… пластин.

Допустимая Ρ‚ΠΎΠ»Ρ‰ΠΈΠ½Π° диэлСктрика d Ρ‚Π°ΠΊΠΆΠ΅ зависит ΠΎΡ‚ Ξ΅r, Ρ‡Π΅ΠΌ Π²Ρ‹ΡˆΠ΅ коэффициСнт, Ρ‚Π΅ΠΌ Ρ‚ΠΎΠ½ΡŒΡˆΠ΅ ΠΌΠΎΠΆΠ½ΠΎ ΠΈΡΠΏΠΎΠ»ΡŒΠ·ΠΎΠ²Π°Ρ‚ΡŒ диэлСктрик, Ρ‚Π΅ΠΌ Π±ΠΎΠ»ΡŒΡˆΡƒΡŽ Π΅ΠΌΠΊΠΎΡΡ‚ΡŒ Π±ΡƒΠ΄Π΅Ρ‚ ΠΈΠΌΠ΅Ρ‚ΡŒ кондСнсатор. Π­Ρ‚ΠΎ Π±Ρ‹Π» самый слоТный ΠΌΠ°Ρ‚Π΅Ρ€ΠΈΠ°Π», дальшС Π±ΡƒΠ΄Π΅Ρ‚ Π»Π΅Π³Ρ‡Π΅.

Π€ΠΎΡ€ΠΌΡƒΠ»Π° Смкости цилиндричСского кондСнсатора

Π’Π΅ΠΏΠ΅Ρ€ΡŒ ΠΏΠΎΠ³ΠΎΠ²ΠΎΡ€ΠΈΠΌ ΠΎ Ρ‚ΠΎΠΌ, ΠΊΠ°ΠΊ Π½Π°ΠΉΡ‚ΠΈ Π΅ΠΌΠΊΠΎΡΡ‚ΡŒ кондСнсатора цилиндричСской Ρ„ΠΎΡ€ΠΌΡ‹. К Π½ΠΈΠΌ относятся кондСнсаторы, состоящиС ΠΈΠ· Π΄Π²ΡƒΡ… мСталличСских Ρ†ΠΈΠ»ΠΈΠ½Π΄Ρ€ΠΎΠ², вставлСнных ΠΎΠ΄ΠΈΠ½ Π² Π΄Ρ€ΡƒΠ³ΠΎΠΉ. Для раздСлСния ΠΌΠ΅ΠΆΠ΄Ρƒ Π½ΠΈΠΌΠΈ располоТСн диэлСктрик. Π€ΠΎΡ€ΠΌΡƒΠ»Π° Смкости кондСнсатора выглядит ΡΠ»Π΅Π΄ΡƒΡŽΡ‰ΠΈΠΌ ΠΎΠ±Ρ€Π°Π·ΠΎΠΌ:

Π—Π΄Π΅ΡΡŒ Π²ΠΈΠ΄ΠΈΠΌ нСсколько Π½ΠΎΠ²Ρ‹Ρ… ΠΏΠ΅Ρ€Π΅ΠΌΠ΅Π½Π½Ρ‹Ρ…:

  • l – высота Ρ†ΠΈΠ»ΠΈΠ½Π΄Ρ€Π°;
  • R1 ΠΈ R2 – радиус ΠΏΠ΅Ρ€Π²ΠΎΠ³ΠΎ ΠΈ Π²Ρ‚ΠΎΡ€ΠΎΠ³ΠΎ (внСшнСго) Ρ†ΠΈΠ»ΠΈΠ½Π΄Ρ€ΠΎΠ²;
  • ln – это Π½Π΅ пСрСмСнная, Π° матСматичСский символ Π½Π°Ρ‚ΡƒΡ€Π°Π»ΡŒΠ½ΠΎΠ³ΠΎ Π»ΠΎΠ³Π°Ρ€ΠΈΡ„ΠΌΠ°. На Π½Π΅ΠΊΠΎΡ‚ΠΎΡ€Ρ‹Ρ… ΠΊΠ°Π»ΡŒΠΊΡƒΠ»ΡΡ‚ΠΎΡ€Π°Ρ… ΠΎΠ½ имССтся.

ВсСгда Π½ΡƒΠΆΠ½ΠΎ ΠΏΠΎΠΌΠ½ΠΈΡ‚ΡŒ, Ρ‡Ρ‚ΠΎ всС Π²Π΅Π»ΠΈΡ‡ΠΈΠ½Ρ‹ Π΄ΠΎΠ»ΠΆΠ½Ρ‹ ΠΏΡ€ΠΈΠ²ΠΎΠ΄ΠΈΡ‚ΡŒΡΡ ΠΊ Π΅Π΄ΠΈΠ½ΠΎΠΉ систСмС, Π² ΠΏΡ€ΠΈΠ²Π΅Π΄Π΅Π½Π½ΠΎΠΉ Π½ΠΈΠΆΠ΅ Ρ‚Π°Π±Π»ΠΈΡ†Π΅ ΡƒΠΊΠ°Π·Π°Π½Ρ‹ ΠΌΠ΅ΠΆΠ΄ΡƒΠ½Π°Ρ€ΠΎΠ΄Π½Ρ‹Π΅ систСмы Π΅Π΄ΠΈΠ½ΠΈΡ† (БИ).

Из Π½Π΅Π΅ Π²ΠΈΠ΄Π½ΠΎ, Ρ‡Ρ‚ΠΎ всС расстояния Π½ΡƒΠΆΠ½ΠΎ ΠΏΡ€ΠΈΠ²ΠΎΠ΄ΠΈΡ‚ΡŒ ΠΊ ΠΌΠ΅Ρ‚Ρ€Ρƒ.

Π•Ρ‰Π΅ стоит ΠΎΠ±Ρ€Π°Ρ‰Π°Ρ‚ΡŒ Π²Π½ΠΈΠΌΠ°Π½ΠΈΠ΅ Π½Π° качСство диэлСктрика. Если Ρ‚ΠΎΠ»Ρ‰ΠΈΠ½Π° диэлСктрика влияСт Ρ‚ΠΎΠ»ΡŒΠΊΠΎ Π½Π° Π΅ΠΌΠΊΠΎΡΡ‚ΡŒ кондСнсатора, Ρ‚ΠΎ Π΅Π³ΠΎ качСство Π·Π°Ρ‚Ρ€Π°Π³ΠΈΠ²Π°Π΅Ρ‚ ΡΠΎΡ…Ρ€Π°Π½Π½ΠΎΡΡ‚ΡŒ энСргии. Π”Ρ€ΡƒΠ³ΠΈΠΌΠΈ словами, кондСнсатор с качСствСнным диэлСктриком Π±ΡƒΠ΄Π΅Ρ‚ ΠΈΠΌΠ΅Ρ‚ΡŒ мСньший саморазряд.

ΠžΠΏΡ€Π΅Π΄Π΅Π»ΠΈΡ‚ΡŒ качСство ΠΌΠΎΠΆΠ½ΠΎ ΠΏΠΎ числу, стоящСму Π²ΠΎΠ·Π»Π΅ вСщСства, Ρ‡Π΅ΠΌ ΠΎΠ½ΠΎ большС, Ρ‚Π΅ΠΌ Π»ΡƒΡ‡ΡˆΠ΅ качСство. Π‘Ρ€Π°Π²Π½Π΅Π½ΠΈΠ΅ производится ΠΏΠΎ Π²Π°ΠΊΡƒΡƒΠΌΡƒ, Π·Π½Π°Ρ‡Π΅Π½ΠΈΠ΅ ΠΊΠΎΡ‚ΠΎΡ€ΠΎΠ³ΠΎ Ρ€Π°Π²Π½ΠΎ Π΅Π΄ΠΈΠ½ΠΈΡ†Π΅.

Π€ΠΎΡ€ΠΌΡƒΠ»Π° Смкости сфСричСского кондСнсатора

ПослСднСС Ρ‡Ρ‚ΠΎ ΠΎΡΡ‚Π°Π»ΠΎΡΡŒ Ρ€Π°Π·ΠΎΠ±Ρ€Π°Ρ‚ΡŒ – Ρ„ΠΎΡ€ΠΌΡƒΠ»Ρƒ опрСдСлСния Смкости кондСнсатора, состоящСго ΠΈΠ· Π΄Π²ΡƒΡ… сфСр. ΠŸΡ€ΠΈΡ‡Π΅ΠΌ ΠΎΠ΄Π½Π° сфСра находится Π²Π½ΡƒΡ‚Ρ€ΠΈ Π΄Ρ€ΡƒΠ³ΠΎΠΉ. Π€ΠΎΡ€ΠΌΡƒΠ»Π° ΠΈΠΌΠ΅Π΅Ρ‚ ΡΠ»Π΅Π΄ΡƒΡŽΡ‰ΠΈΠΉ Π²ΠΈΠ΄:

Из ΠΏΡ€ΠΈΠ²Π΅Π΄Π΅Π½Π½Ρ‹Ρ… ΠΏΠ΅Ρ€Π΅ΠΌΠ΅Π½Π½Ρ‹Ρ… здСсь всС Π·Π½Π°ΠΊΠΎΠΌΠΎ. Π‘Ρ‚ΠΎΠΈΡ‚ ΠΎΠ±Ρ€Π°Ρ‚ΠΈΡ‚ΡŒ Π²Π½ΠΈΠΌΠ°Π½ΠΈΠ΅ лишь Π½Π° сам кондСнсатор.

ΠšΡ€ΠΎΠΌΠ΅ своСй Π½Π΅ΠΎΠ±Ρ‹Ρ‡Π½ΠΎΠΉ Ρ„ΠΎΡ€ΠΌΡ‹ Ρƒ Π½Π΅Π³ΠΎ Π΅ΡΡ‚ΡŒ свои особСнности: Π²Π½ΡƒΡ‚Ρ€ΠΈ ΠΌΠ°Π»ΠΎΠΉ сфСры Π½ΠΈΠΊΠ°ΠΊΠΎΠ³ΠΎ заряда Π½Π΅Ρ‚, ΠΎΠ½ образуСтся Π½Π° внСшнСй части ΠΌΠ°Π»ΠΎΠΉ сфСры ΠΈ Π²Π½ΡƒΡ‚Ρ€Π΅Π½Π½Π΅ΠΉ части большого ΡˆΠ°Ρ€Π°. Π’Π°ΠΊΠΆΠ΅ заряд отсутствуСт ΠΈ Π½Π° внСшнСй сторонС внСшнСй сфСры.

Π’Π°ΠΊ ΠΆΠ΅ ΠΊΠ°ΠΊ ΠΈ всС Π΄Ρ€ΡƒΠ³ΠΈΠ΅ кондСнсаторы, сфСры Ρ€Π°Π·Π΄Π΅Π»Π΅Π½Ρ‹ диэлСктриком. Π’ΠΎΠ»Ρ‰ΠΈΠ½Π° ΠΈ качСство диэлСктрика ΠΎΠΊΠ°Π·Ρ‹Π²Π°ΡŽΡ‚ Ρ‚Π°ΠΊΠΎΠ΅ ΠΆΠ΅ влияниС Π½Π° Π΅ΠΌΠΊΠΎΡΡ‚ΡŒ, ΠΊΠ°ΠΊ Π² случаС с Π΄Ρ€ΡƒΠ³ΠΈΠΌΠΈ кондСнсаторами.

ПослС Ρ‚ΠΎΠ³ΠΎ ΠΊΠ°ΠΊ Π±Ρ‹Π»ΠΈ рассмотрСны Ρ„ΠΎΡ€ΠΌΡƒΠ»Ρ‹, стоит ΠΈΡΠΏΡ€ΠΎΠ±ΠΎΠ²Π°Ρ‚ΡŒ ΠΈΡ… Π½Π° ΠΏΡ€Π°ΠΊΡ‚ΠΈΠΊΠ΅. Рассмотрим, ΠΊΠ°ΠΊ Π½Π°ΠΉΡ‚ΠΈ Π΅ΠΌΠΊΠΎΡΡ‚ΡŒ кондСнсатора ΠΊΠ°ΠΆΠ΄ΠΎΠ³ΠΎ Π²ΠΈΠ΄Π°.

ΠŸΡ€ΠΈΠΌΠ΅Ρ€Ρ‹ Ρ€Π΅ΡˆΠ΅Π½ΠΈΡ Π·Π°Π΄Π°Ρ‡

НачнСм с плоского кондСнсатора. Π€ΠΎΡ€ΠΌΡƒΠ»Π° для этого Π²ΠΈΠ΄Π°:

Допустим, Ρƒ нас Π΅ΡΡ‚ΡŒ ΡΠ»Π΅Π΄ΡƒΡŽΡ‰ΠΈΠ΅ значСния:

  • Π² качСствС диэлСктрика возьмСм ΡΠ»ΡŽΠ΄Ρƒ Ρ‚ΠΎΠ»Ρ‰ΠΈΠ½ΠΎΠΉ 0,02 ΠΌΠΌ, Ξ΅ = 6;
  • кондСнсатор ΠΊΠ²Π°Π΄Ρ€Π°Ρ‚Π½Ρ‹ΠΉ со сторонами Π² 7 ΠΌΠΌ.

ΠžΠΏΡ€Π΅Π΄Π΅Π»ΡΠ΅ΠΌ ΠΏΠ»ΠΎΡ‰Π°Π΄ΡŒ пластин: 7Γ—7 = 49 ΠΌΠΌ2.

ΠŸΡ€ΠΈΠ²ΠΎΠ΄ΠΈΠΌ ΠΊ Π΅Π΄ΠΈΠ½ΠΎΠΉ систСмС: 4,9Γ—10-5 = 0,000049 ΠΌ2. Π’ΠΎΠ»Ρ‰ΠΈΠ½Π° диэлСктрика 0,02Γ—10-5 = 0,00002 ΠΌ. ЭлСктричСская постоянная 8,854187817Γ—10-12.

ΠŸΠΎΠ΄ΡΡ‚Π°Π²Π»ΡΠ΅ΠΌ Π² Ρ„ΠΎΡ€ΠΌΡƒΠ»Ρƒ ΠΈ высчитываСм Ρ‡ΠΈΡΠ»ΠΈΡ‚Π΅Π»ΡŒ: 6Γ—8,854187817Γ—10-12 Γ—4,9Γ—10-5, сокращаСм ΠΈ Ρ€Π΅ΡˆΠ°Π΅ΠΌ 6Γ—49Γ—8,854187817Γ—10-17 = 2,603131218198Γ—10-14.

Π”Π΅Π»ΠΈΠΌ Π½Π° Ρ‚ΠΎΠ»Ρ‰ΠΈΠ½Ρƒ диэлСктрика: 2,603131218198Γ—10 / 2Γ—10 = 1301,565609099Γ—10 = 1,301565609099Γ—10. Π¨Π΅ΡΡ‚ΡŒ Π½ΡƒΠ»Π΅ΠΉ – это тысячи ΠΈΠ»ΠΈ приставка Β«ΠΌΠΈΠΊΡ€ΠΎΒ», получаСтся ΠΎΠΊΡ€ΡƒΠ³Π»Π΅Π½ΠΎ 1,3 ΠΌΠΊΠ€.

Π’ΠΎΠ·ΠΌΠΎΠΆΠ½ΠΎ, ΠΏΡ€ΠΈ вычислСнии Π±Ρ‹Π»Π° Π΄ΠΎΠΏΡƒΡ‰Π΅Π½Π° ошибка, Π½ΠΎ это Π½Π΅ экзамСн ΠΏΠΎ ΠΌΠ°Ρ‚Π΅ΠΌΠ°Ρ‚ΠΈΠΊΠ΅. Π’Π°ΠΆΠ½ΠΎ ΠΏΠΎΠ½ΡΡ‚ΡŒ сам ΠΌΠ΅Ρ‚ΠΎΠ΄ вычислСния.

Π€ΠΎΡ€ΠΌΡƒΠ»Π° для цилиндричСского кондСнсатора:

Π’Ρ‹Π±ΠΈΡ€Π°Π΅ΠΌ значСния:

  • l = 1 см;
  • R1 = 0,25 ΠΌΠΌ;
  • R2 = 0,26 ΠΌΠΌ;
  • Ξ΅ = 2.

ПодгоняСм ΠΏΠΎΠ΄ Π΅Π΄ΠΈΠ½ΡƒΡŽ систСму: l — 1 см = 1Γ—10-2 = 0,01 ΠΌ; R1 – 0,25 ΠΌΠΌ = 0,0025 ΠΌ; R2 – 0,26 ΠΌΠΌ = 0,0026 ΠΌ.

ΠŸΠΎΠ΄ΡΡ‚Π°Π²Π»ΡΠ΅ΠΌ значСния Π² Ρ‡ΠΈΡΠ»ΠΈΡ‚Π΅Π»ΡŒ: 2Γ—3,142857142857143Γ—8,854187817Γ—10-12Γ—2Γ—0,01 1,11Γ—10-12. Находим Π·Π½Π°ΠΌΠ΅Π½Π°Ρ‚Π΅Π»ΡŒ: 0,26:0,25 = 1,04.

Находим Π½Π°Ρ‚ΡƒΡ€Π°Π»ΡŒΠ½Ρ‹ΠΉ Π»ΠΎΠ³Π°Ρ€ΠΈΡ„ΠΌ, ΠΎΠ½ Ρ€Π°Π²Π΅Π½ ΠΏΡ€ΠΈΠΌΠ΅Ρ€Π½ΠΎ 0,39. Π§ΠΈΡΠ»ΠΈΡ‚Π΅Π»ΡŒ Π΄Π΅Π»ΠΈΠΌ Π½Π° Π·Π½Π°ΠΌΠ΅Π½Π°Ρ‚Π΅Π»ΡŒ: 1,11Γ—10-12/0,39 = 2,85Γ—10-12.

Число с 12 нулями это приставка Β«ΠΏΠΈΠΊΠΎΒ», ΠΏΠΎΠ»ΡƒΡ‡Π°Π΅ΠΌ 2,85 ΠΏΠ€.

Π€ΠΎΡ€ΠΌΡƒΠ»Π° для сфСричСского кондСнсатора:

Π’Ρ‹Π±ΠΈΡ€Π°Π΅ΠΌ значСния:

  • Ξ΅= 4;
  • r1= 5 см;
  • r2= 5,01 см.

Π‘Π½ΠΎΠ²Π° всС подгоняСм: 5 см = 0,05 ΠΌ; 5,01 см = 0,0501 ΠΌ. ЗаполняСм Ρ‡ΠΈΡΠ»ΠΈΡ‚Π΅Π»ΡŒ. 4Γ—3,142857142857143Γ—4Γ—8,854187817Γ—10-12Γ—0,05Γ—0,0501 1,11Γ—10-12 ВычисляСм Π·Π½Π°ΠΌΠ΅Π½Π°Ρ‚Π΅Π»ΡŒ: 0,0501 – 0,05 = 0,01. ΠŸΡ€ΠΎΠΈΠ·Π²ΠΎΠ΄ΠΈΠΌ Π΄Π΅Π»Π΅Π½ΠΈΠ΅: 1,11Γ—10-12Γ—0,01 = 1,11Γ—10-10. Π‘Π½ΠΎΠ²Π° ΠΏΠΎΠ»ΡƒΡ‡ΠΈΠ»ΠΈ ΠΏΠΈΠΊΠΎΡ„Π°Ρ€Π°Π΄Ρ‹, Π° ΠΈΠΌΠ΅Π½Π½ΠΎ 1,11 ΠΏΠ€.

ΠŸΠΎΡ…ΠΎΠΆΠΈΠ΅ ΠΌΠ°Ρ‚Π΅Ρ€ΠΈΠ°Π»Ρ‹ Π½Π° сайтС:

ΠŸΠΎΠ½Ρ€Π°Π²ΠΈΠ»Π°ΡΡŒ ΡΡ‚Π°Ρ‚ΡŒΡ — подСлись с Π΄Ρ€ΡƒΠ·ΡŒΡΠΌΠΈ!

Β 

ЭлСктричСство ΠΈ ΠΌΠ°Π³Π½Π΅Ρ‚ΠΈΠ·ΠΌ

ΠŸΠΎΠ²Ρ‹ΡˆΠ΅Π½ΠΈΡ Смкости ΠΏΡ€ΠΎΠ²ΠΎΠ΄Π½ΠΈΠΊΠ° ΠΌΠΎΠΆΠ½ΠΎ Π΄ΠΎΡΡ‚ΠΈΠ³Π½ΡƒΡ‚ΡŒ Π½Π΅ Ρ‚ΠΎΠ»ΡŒΠΊΠΎ ΡƒΠ²Π΅Π»ΠΈΡ‡Π΅Π½ΠΈΠ΅ΠΌ Π΅Π³ΠΎ Ρ€Π°Π·ΠΌΠ΅Ρ€ΠΎΠ², Π½ΠΎ ΠΈ Π·Π° счСт приблиТСния ΠΊ Π½Π΅ΠΌΡƒ Π΄Ρ€ΡƒΠ³ΠΎΠ³ΠΎ ΠΏΡ€ΠΎΠ²ΠΎΠ΄Π½ΠΈΠΊΠ°. ΠŸΡ€ΠΈΠΌΠ΅Ρ€Π°ΠΌΠΈ ΠΌΠΎΠ³ΡƒΡ‚ ΡΠ»ΡƒΠΆΠΈΡ‚ΡŒ плоский кондСнсатор, сфСричСский кондСнсатор ΠΈ Π΄Ρ€. ΠœΡ‹ вычислим ΠΈΡ… Смкости, исходя ΠΈΠ· Π΄Π°Π½Π½Ρ‹Ρ… ΠΎΠΏΡ€Π΅Π΄Π΅Π»Π΅Π½ΠΈΠΉ ΠΈ Π³Π΅ΠΎΠΌΠ΅Ρ‚Ρ€ΠΈΠΈ кондСнсатора.

Плоский кондСнсатор (рис. 2.11).Β 

Рис. 2.12.Β  ЭлСктричСскоС ΠΏΠΎΠ»Π΅ идСального плоского кондСнсатора

Π˜Π΄Π΅Π°Π»ΡŒΠ½Ρ‹ΠΉ плоский кондСнсатор прСдста­вляСт собой Π΄Π²Π΅ мСталличСскиС ΠΏΠ°Ρ€Π°Π»Π»Π΅Π»ΡŒΠ½Ρ‹Π΅ пластины, Π»ΠΈΠ½Π΅ΠΉΠ½Ρ‹Π΅ Ρ€Π°Π·ΠΌΠ΅Ρ€Ρ‹ ΠΊΠΎΡ‚ΠΎΡ€Ρ‹Ρ… ΠΌΠ½ΠΎΠ³ΠΎ большС расстояния Β ΠΌΠ΅ΠΆΠ΄Ρƒ Π½ΠΈΠΌΠΈ. ΠŸΡƒΡΡ‚ΡŒ ΠΏΠ»ΠΎΡ‰Π°Π΄ΡŒ ΠΊΠ°ΠΆΠ΄ΠΎΠΉ ΠΈΠ· пластин Ρ€Π°Π²Π½Π° Β (рис. 2.12). На ΠΎΠ΄Π½Ρƒ пластину ΠΏΠΎΠΌΠ΅Ρ‰Π΅Π½ заряд , Π½Π° Π΄Ρ€ΡƒΠ³ΡƒΡŽ β€”  Если пластины достаточно Π²Π΅Π»ΠΈΠΊΠΈ, Ρ‚ΠΎ ΠΈΡ… ΠΌΠΎΠΆΠ½ΠΎ ΡΡ‡ΠΈΡ‚Π°Ρ‚ΡŒ «бСсконСчными» Π² Ρ‚ΠΎΠΌ смыслС, Ρ‡Ρ‚ΠΎ допустимо ΠΏΡ€Π΅Π½Π΅Π±Ρ€Π΅Ρ‡ΡŒ Β«ΠΊΡ€Π°Π΅Π²Ρ‹ΠΌΠΈΒ» эффСктами β€” распрСдСлСниями зарядов ΠΈ конфигурациями ΠΏΠΎΠ»Π΅ΠΉ Π²Π±Π»ΠΈΠ·ΠΈ ΠΈΡ… ΠΊΡ€Π°Π΅Π².

Π’ΠΎΠ³Π΄Π° заряды Ρ€Π°ΡΠΏΡ€Π΅Π΄Π΅Π»ΡΡŽΡ‚ΡΡ ΠΏΠΎ Π²Π½ΡƒΡ‚Ρ€Π΅Π½Π½ΠΈΠΌ повСрхностям пластин практичСски Ρ€Π°Π²Π½ΠΎΠΌΠ΅Ρ€Π½ΠΎ, с постоянной ΠΏΠ»ΠΎΡ‚Π½ΠΎΡΡ‚ΡŒΡŽ. Π Π°Π·Π½ΠΎΡΡ‚ΡŒ ΠΏΠΎΡ‚Π΅Π½Ρ†ΠΈΠ°Π»ΠΎΠ² ΠΌΠ΅ΠΆΠ΄Ρƒ ΠΎΠ±ΠΊΠ»Π°Π΄ΠΊΠ°ΠΌΠΈ Ρ€Π°Π²Π½Π° ΠΈΠ½Ρ‚Π΅Π³Ρ€Π°Π»Ρƒ ΠΎΡ‚ напряТСнности поля, взятому ΠΏΠΎ Π»ΡŽΠ±ΠΎΠΌΡƒ ΠΏΡƒΡ‚ΠΈ ΠΌΠ΅ΠΆΠ΄Ρƒ Π½ΠΈΠΌΠΈ:

Рис. 2.12.Β  ЭлСктричСскоС ΠΏΠΎΠ»Π΅ идСального плоского кондСнсатора

Π’ΠΈΠ΄Π΅ΠΎ 2.9. ГСомСтрия Ρ€Π΅Π°Π»ΡŒΠ½ΠΎΠ³ΠΎ плоского кондСнсатора ΠΈ распрСдСлСниС заряда Π½Π° Π΅Π³ΠΎ пластинах.

Π’ΠΎΠ³Π΄Π° заряды Ρ€Π°ΡΠΏΡ€Π΅Π΄Π΅Π»ΡΡŽΡ‚ΡΡ ΠΏΠΎ Π²Π½ΡƒΡ‚Ρ€Π΅Π½Π½ΠΈΠΌ повСрхностям пластин практичСски Ρ€Π°Π²Π½ΠΎΠΌΠ΅Ρ€Π½ΠΎ, с постоянной ΠΏΠ»ΠΎΡ‚Π½ΠΎΡΡ‚ΡŒΡŽ . Π Π°Π·Π½ΠΎΡΡ‚ΡŒ ΠΏΠΎΡ‚Π΅Π½Ρ†ΠΈΠ°Π»ΠΎΠ² ΠΌΠ΅ΠΆΠ΄Ρƒ ΠΎΠ±ΠΊΠ»Π°Π΄ΠΊΠ°ΠΌΠΈ Ρ€Π°Π²Π½Π° ΠΈΠ½Ρ‚Π΅Π³Ρ€Π°Π»Ρƒ ΠΎΡ‚ напряТСнности поля, взятому ΠΏΠΎ Π»ΡŽΠ±ΠΎΠΌΡƒ ΠΏΡƒΡ‚ΠΈ ΠΌΠ΅ΠΆΠ΄Ρƒ Π½ΠΈΠΌΠΈ:Β 

Β 

(2. 10)

ПолС, создаваСмоС двумя бСсконСчными ΠΏΠ°Ρ€Π°Π»Π»Π΅Π»ΡŒΠ½Ρ‹ΠΌΠΈ плоскостями, заряТСнными Ρ€Π°Π·Π½ΠΎΠΈΠΌΠ΅Π½Π½ΠΎ с ΠΎΠ΄ΠΈΠ½Π°ΠΊΠΎΠ²Ρ‹ΠΌΠΈ плотностями, являСтся ΠΎΠ΄Π½ΠΎΒ­Ρ€ΠΎΠ΄Π½Ρ‹ΠΌ, ΠΈ Π΅Π³ΠΎ Π½Π°ΠΏΡ€ΡΠΆΠ΅Π½Π½ΠΎΡΡ‚ΡŒ Ρ€Π°Π²Π½Π°Β  Β (см. (2.3)).

ΠΠ°ΠΏΡ€ΡΠΆΠ΅Π½Π½ΠΎΡΡ‚ΡŒ поля Π² пространствС, ΠΎΠΊΡ€ΡƒΠΆΠ°ΡŽΡ‰Π΅ΠΌ пластины, ΠΌΠΎΠΆΠ½ΠΎ ΡΡ‡ΠΈΡ‚Π°Ρ‚ΡŒ Ρ€Π°Π²Π½ΠΎΠΉ Π½ΡƒΠ»ΡŽ, Ссли ΠΏΡ€Π΅Π½Π΅Π±Ρ€Π΅Ρ‡ΡŒ ΠΊΡ€Π°Π΅Π²Ρ‹ΠΌΠΈ эффСктами. Π˜Π½Ρ‚Π΅Π³Ρ€ΠΈΡ€ΡƒΡ вдоль силовой Π»ΠΈΠ½ΠΈΠΈ (ΠΊΠΎΡ‚ΠΎΡ€Ρ‹Π΅ ΠΎΡ€Ρ‚ΠΎΠ³ΠΎΠ½Π°Π»ΡŒΠ½Ρ‹ пластинам), ΠΏΠΎΠ»ΡƒΡ‡Π°Π΅ΠΌ

(2.11)

ΠžΡ‚ΡΡŽΠ΄Π° Π½Π°Ρ…ΠΎΠ΄ΠΈΠΌ Π΅ΠΌΠΊΠΎΡΡ‚ΡŒ плоского кондСнсатора:

(2.12)

ЦилиндричСский кондСнсатор. ЦилиндричСский кондСнсатор прСдставляСт собой Π΄Π²Π° ΠΊΠΎΠ°ΠΊΡΠΈΠ°Π»ΡŒΠ½Ρ‹Ρ… Π΄Π»ΠΈΠ½Π½Ρ‹Ρ… проводящих Ρ†ΠΈΠ»ΠΈΠ½Π΄Ρ€Π° радиусами  и Β ΠΈ Π΄Π»ΠΈΠ½ΠΎΠΉ . ΠŸΡ€Π΅Π΄ΠΏΠΎΠ»Π°Π³Π°Ρ, Ρ‡Ρ‚ΠΎΒ , ΠΌΡ‹ ΠΈ Π² этом случаС ΠΏΡ€Π΅Π½Π΅Π±Ρ€Π΅Π³Π°Π΅ΠΌ ΠΊΡ€Π°Π΅Π²Ρ‹ΠΌΠΈ эффСктами. ЛинСйная ΠΏΠ»ΠΎΡ‚Π½ΠΎΡΡ‚ΡŒ заряда Π½Π° Ρ†ΠΈΠ»ΠΈΠ½Π΄Ρ€Π°Ρ… Ρ€Π°Π²Π½Π° . ΠœΡ‹ ΡƒΠΆΠ΅ Π²Ρ‹Π²Π΅Π»ΠΈ Π²Ρ‹Ρ€Π°ΠΆΠ΅Π½ΠΈΠ΅ для элСктричСского поля Π΄Π»ΠΈΠ½Π½ΠΎΠ³ΠΎ заряТСнного Ρ†ΠΈΠ»ΠΈΠ½Π΄Ρ€Π° (см.Β (1.17)):

(2.13)

ЭлСктричСскоС ΠΏΠΎΠ»Π΅ Π½Π°ΠΏΡ€Π°Π²Π»Π΅Π½ΠΎ ΠΏΠΎ радиусу Ρ†ΠΈΠ»ΠΈΠ½Π΄Ρ€ΠΎΠ². Π˜Π½Ρ‚Π΅Π³Ρ€ΠΈΡ€ΡƒΡ ΠΏΠΎ этому ΠΏΡƒΡ‚ΠΈ ΠΎΡ‚ ΠΎΠ΄Π½ΠΎΠΉ ΠΎΠ±ΠΊΠ»Π°Π΄ΠΊΠΈ ΠΊ Π΄Ρ€ΡƒΠ³ΠΎΠΉ, Π½Π°Ρ…ΠΎΠ΄ΠΈΠΌ Ρ€Π°Π·Π½ΠΎΡΡ‚ΡŒ ΠΏΠΎΡ‚Π΅Π½Ρ†ΠΈΠ°Π»ΠΎΠ² ΠΌΠ΅ΠΆΠ΄Ρƒ ΠΎΠ±ΠΊΠ»Π°Π΄ΠΊΠ°ΠΌΠΈ:

Β 

(2.14)

ΠžΡ‚ΡΡŽΠ΄Π° слСдуСт Π²Ρ‹Ρ€Π°ΠΆΠ΅Π½ΠΈΠ΅ для Смкости цилиндричСского кондСнсатора:

(2.15)

Π’ случаС, ΠΊΠΎΠ³Π΄Π° Π·Π°Π·ΠΎΡ€ ΠΌΠ΅ΠΆΠ΄Ρƒ ΠΎΠ±ΠΊΠ»Π°Π΄ΠΊΠ°ΠΌΠΈΒ  , ΠΌΠΎΠΆΠ½ΠΎ ΠΈΡΠΏΠΎΠ»ΡŒΠ·ΠΎΠ²Π°Ρ‚ΡŒ ΠΏΠ΅Ρ€Π²Ρ‹ΠΉ Ρ‡Π»Π΅Π½ разлоТСния Π»ΠΎΠ³Π°Ρ€ΠΈΡ„ΠΌΠ° Π² ряд Π’Π΅ΠΉΠ»ΠΎΡ€Π°

Β 

Ρ‡Ρ‚ΠΎ ΠΏΡ€ΠΈΠ²ΠΎΠ΄ΠΈΡ‚ ΠΊ Π²Ρ‹Ρ€Π°ΠΆΠ΅Π½ΠΈΡŽ

(2. 16)

Π’ скобках стоит ΠΏΡ€ΠΎΠΈΠ·Π²Π΅Π΄Π΅Π½ΠΈΠ΅ Π΄Π»ΠΈΠ½Ρ‹ окруТности Ρ†ΠΈΠ»ΠΈΠ½Π΄Ρ€Π° Π½Π° Π΅Π³ΠΎ высоту, Ρ‡Ρ‚ΠΎ Ρ€Π°Π²Π½ΠΎ ΠΏΠ»ΠΎΡ‰Π°Π΄ΠΈ повСрхности Ρ†ΠΈΠ»ΠΈΠ½Π΄Ρ€Π° (ΠΏΠ»ΠΎΡ‰Π°Π΄ΠΈ ΠΎΠ±ΠΊΠ»Π°Π΄ΠΎΠΊ). Π’.Β ΠΎ. ΠΌΡ‹ воспроизвСли Π² этом ΠΏΡ€Π΅Π΄Π΅Π»Π΅ Π²Ρ‹Ρ€Π°ΠΆΠ΅Π½ΠΈΠ΅ (2.12) для Смкости плоского кондСнсатора.

БфСричСский кондСнсатор. БфСричСский кондСнсатор образуСтся двумя концСнтричСскими сфСрами радиусам Β ΠΈΒ . Π˜Π½Ρ‚Π΅Π³Ρ€ΠΈΡ€ΡƒΡ вдоль радиуса ΡƒΠΆΠ΅ Ρ…ΠΎΡ€ΠΎΡˆΠΎ Π·Π½Π°ΠΊΠΎΠΌΠΎΠ΅ Π²Ρ‹Ρ€Π°ΠΆΠ΅Π½ΠΈΠ΅

Β 

ΠΏΠΎΠ»ΡƒΡ‡Π°Π΅ΠΌ Ρ€Π°Π·Π½ΠΎΡΡ‚ΡŒ ΠΏΠΎΡ‚Π΅Π½Ρ†ΠΈΠ°Π»ΠΎΠ² ΠΌΠ΅ΠΆΠ΄Ρƒ ΠΎΠ±ΠΊΠ»Π°Π΄ΠΊΠ°ΠΌΠΈ:

Β 

(2.17)

ΠΎΡ‚ΠΊΡƒΠ΄Π°

(2.18)

Если внСшний радиус бСсконСчно Π²Π΅Π»ΠΈΠΊ Β (физичСски  это Π·Π½Π°Ρ‡ΠΈΡ‚, Ρ‡Ρ‚ΠΎΒ ), Ρ‚ΠΎ Π²Ρ‹Ρ‡ΠΈΡ‚Π°Π΅ΠΌΡ‹ΠΌ Π² Π·Π½Π°ΠΌΠ΅Π½Π°Ρ‚Π΅Π»Π΅ ΠΌΠΎΠΆΠ½ΠΎ ΠΏΡ€Π΅Π½Π΅Π±Ρ€Π΅Ρ‡ΡŒ, ΠΈ ΠΌΡ‹ ΠΏΡ€ΠΈΡ…ΠΎΠ΄ΠΈΠΌ ΠΊ Ρ„ΠΎΡ€ΠΌΡƒΠ»Π΅ (2. 9) для Смкости ΡƒΠ΅Π΄ΠΈΠ½Π΅Π½Π½ΠΎΠΉ сфСры. Π’ ΠΎΠ±Ρ€Π°Ρ‚Π½ΠΎΠΌ случаС, ΠΊΠΎΠ³Π΄Π° Π·Π°Π·ΠΎΡ€ ΠΌΠ΅ΠΆΠ΄Ρƒ ΠΎΠ±ΠΊΠ»Π°Π΄ΠΊΠ°ΠΌΠΈ Β ΠΌΠΎΠΆΠ½ΠΎ ΠΏΠΎΠ»ΠΎΠΆΠΈΡ‚ΡŒ Π² числитСлС  ЗамСчая, Ρ‡Ρ‚ΠΎ Β Π΅ΡΡ‚ΡŒ ΠΏΠ»ΠΎΡ‰Π°Π΄ΡŒ ΠΎΠ±ΠΊΠ»Π°Π΄ΠΎΠΊ, ΠΌΡ‹ снова ΠΏΡ€ΠΈΡ…ΠΎΠ΄ΠΈΠΌ ΠΊ Ρ„ΠΎΡ€ΠΌΡƒΠ»Π΅ (2.12).

Π’ΠΈΠ΄Π΅ΠΎ 2.10. ВлияниС диэлСктрика Π½Π° распрСдСлСниС зарядов Π½Π° ΠΏΡ€ΠΎΠ²ΠΎΠ΄Π½ΠΈΠΊΠ΅ ΠΈ Π΅Π³ΠΎ Π΅ΠΌΠΊΠΎΡΡ‚ΡŒ.

Π—Π°Π΄Π°Ρ‡Π°. ΠšΠΎΠ½Π΄Π΅Π½ΡΠ°Ρ‚ΠΎΡ€, ΠΈΡΠΏΠΎΠ»ΡŒΠ·ΡƒΠ΅ΠΌΡ‹ΠΉ Π² Ρ‡ΠΈΠΏΠ΅ Π·Π°ΠΏΠΎΠΌΠΈΠ½Π°ΡŽΡ‰Π΅Π³ΠΎ устрой­ства ΠΊΠΎΠΌΠΏΡŒΡŽΡ‚Π΅Ρ€Π°, ΠΈΠΌΠ΅Π΅Ρ‚ Π΅ΠΌΠΊΠΎΡΡ‚ΡŒΒ Β ΠΈ заряТаСтся Π΄ΠΎ разности ΠΏΠΎΡ‚Π΅Π½Ρ†ΠΈΠ°Π»ΠΎΠ²Β . Каково число  избыточных элСктронов Π½Π° Π΅Π³ΠΎ ΠΎΡ‚Ρ€ΠΈΡ†Π°Ρ‚Π΅Π»ΡŒΠ½ΠΎΠΉ ΠΎΠ±ΠΊΠ»Π°Π΄ΠΊΠ΅? Π’ ΠΊΠ°ΠΊΠΎΠΉ массС Π²ΠΎΠ΄Ρ‹ ΠΏΠΎΠ»Π½ΠΎΠ΅ число всСх Π°Ρ‚ΠΎΠΌΠ½Ρ‹Ρ… элСктронов Ρ€Π°Π²Π½ΠΎΒ ?

РСшСниС. Заряд кондСнсатора Ρ€Π°Π²Π΅Π½ . Π§Ρ‚ΠΎΠ±Ρ‹ Π½Π°ΠΉΡ‚ΠΈ число ΠΈΠ·Π±Ρ‹Ρ‚ΠΎΡ‡Π½Ρ‹Ρ… элСктронов, Π½Π°Π΄ΠΎ Ρ€Π°Π·Π΄Π΅Π»ΠΈΡ‚ΡŒ Β Π½Π° заряд элСктрона: Β ΠŸΠΎΡ‡Ρ‚ΠΈ Π΄Π²Π° ΠΌΠΈΠ»Π»ΠΈΠΎΠ½Π° элСктронов, ΠΌΠ½ΠΎΠ³ΠΎ это ΠΈΠ»ΠΈ ΠΌΠ°Π»ΠΎ? Для этого Π½Π°ΠΉΠ΄Π΅ΠΌ массу Π²ΠΎΠ΄Ρ‹ с Ρ‚Π΅ΠΌ ΠΆΠ΅ числом элСктро­нов. ΠœΠΎΠ»Π΅ΠΊΡƒΠ»Π° Π²ΠΎΠ΄Ρ‹  содСрТит Π΄Π²Π° Π°Ρ‚ΠΎΠΌΠ° Β ΠΈ ΠΎΠ΄ΠΈΠ½ Π°Ρ‚ΠΎΠΌ , Ρ‚ΠΎ Π΅ΡΡ‚ΡŒ всСго 10 элСктронов. Π‘Ρ‚Π°Π»ΠΎ Π±Ρ‹Ρ‚ΡŒ, Π² ΠΈΠ½Ρ‚Π΅Ρ€Π΅ΡΡƒΡŽΡ‰Π΅ΠΉ нас массС Π²ΠΎΠ΄Ρ‹ Π΄ΠΎΠ»ΠΆΠ½ΠΎ ΡΠΎΠ΄Π΅Ρ€ΠΆΠ°Ρ‚ΡŒΡΡΒ ΠΌΠΎΠ»Π΅ΠΊΡƒΠ». Число ΠΌΠΎΠ»Π΅ΠΊΡƒΠ» Π² ΠΎΠ΄Π½ΠΎΠΌ ΠΌΠΎΠ»Π΅ Ρ€Π°Π²Π½ΠΎ Β Ρ‚ΠΎ Π΅ΡΡ‚ΡŒ Π½Π°Π΄ΠΎ Π²Π·ΡΡ‚ΡŒ моля. ΠœΠΎΠ»ΡΡ€Π½Ρ‹ΠΉ вСс Π²ΠΎΠ΄Ρ‹ Ρ€Π°Π²Π΅Π½ Β ΠΊΠ³/кмоль, Ρ‚Π°ΠΊ Ρ‡Ρ‚ΠΎ искомая масса составляСт ΠΊΠ³, Ρ‚ΠΎ Π΅ΡΡ‚ΡŒ ΠΊΡ€Π°ΠΉΠ½Π΅ ΠΌΠ°Π»Π°.Β Β  Миллион частиц β€” ΠΌΠ½ΠΎΠ³ΠΎ Π² ΠΌΠΈΡ€Π΅ элСктронов, Π½ΠΎ совсСм ΠΌΠ°Π»ΠΎ Π² ΠΌΠ°ΡΡˆΡ‚Π°Π±Π°Ρ… нашСго ΠΌΠΈΡ€Π°.

ЦилиндричСский кондСнсатор — ЭнциклопСдия ΠΏΠΎ ΠΌΠ°ΡˆΠΈΠ½ΠΎΡΡ‚Ρ€ΠΎΠ΅Π½ΠΈΡŽ XXL







Для Ρ†ΠΈΠ»ΠΈΠ½Π΄Ρ€Π° с внСшним ΠΈ Π²Π½ΡƒΡ‚Ρ€Π΅Π½Π½ΠΈΠΌ Π΄ΠΈΠ°ΠΌΠ΅Ρ‚Ρ€Π°ΠΌΠΈ Π’ ΠΈ Π± соотвСтствСнно ΠΈ осСвой Π΄Π»ΠΈΠ½ΠΎΠΉ 1 (диэлСктрик цилиндричСского кондСнсатора изоляция коаксиального кабСля)  [c.87]

Π•ΠΌΠΊΠΎΡΡ‚ΡŒ цилиндричСского кондСнсатора (см. рис. 5,3, Π°) Ρ€Π°ΡΡΡ‡ΠΈΡ‚Ρ‹Π²Π°ΡŽΡ‚ ΠΏΠΎ Ρ„ΠΎΡ€ΠΌΡƒΠ»Π΅  [c.150]

Π•ΠΌΠΊΠΎΡΡ‚ΡŒ цилиндричСского кондСнсатора  [c.399]

Π‘ β€” Π΅ΠΌΠΊΠΎΡΡ‚ΡŒ цилиндричСского кондСнсатора Π² см.  [c.113]

ОсСвоС ΡƒΠ΄Π»ΠΈΠ½Π΅Π½ΠΈΠ΅ ΠΈ ΡƒΠ³ΠΎΠ» закручивания ΠΈΠ·ΠΌΠ΅Ρ€ΡΡŽΡ‚ΡΡ мСханичСскими ΠΈΠ½Π΄ΠΈΠΊΠ°Ρ‚ΠΎΡ€Π°ΠΌΠΈ. Π˜Π·ΠΌΠ΅Ρ€Π΅Π½ΠΈΠ΅ Π²Π΅Π»ΠΈΡ‡ΠΈΠ½Ρ‹ измСнСния Π΄ΠΈΠ°ΠΌΠ΅Ρ‚Ρ€Π° ΠΎΠ±Ρ€Π°Π·Ρ†Π° этими ΠΏΡ€ΠΈΠ±ΠΎΡ€Π°ΠΌΠΈ Π΄Π°Π΅Ρ‚ локальноС Π·Π½Π°Ρ‡Π΅Π½ΠΈΠ΅ Π΄Π΅Ρ„ΠΎΡ€ΠΌΠ°Ρ†ΠΈΠΈ. Π‘Ρ€Π΅Π΄Π½Π΅Π΅ Π·Π½Π°Ρ‡Π΅Π½ΠΈΠ΅ Ρ€Π°Π΄ΠΈΠ°Π»ΡŒΠ½ΠΎΠΉ Π΄Π΅Ρ„ΠΎΡ€ΠΌΠ°Ρ†ΠΈΠΈ ΠΌΠΎΠΆΠ½ΠΎ ΠΏΠΎΠ»ΡƒΡ‡ΠΈΡ‚ΡŒ с ΠΏΠΎΠΌΠΎΡ‰ΡŒΡŽ Смкостного Π΄Π°Ρ‚Ρ‡ΠΈΠΊΠ°, ΠΏΡ€Π΅Π΄ΡΡ‚Π°Π²Π»ΡΡŽΡ‰Π΅Π³ΠΎ собой цилиндричСский кондСнсатор, Π²Π½ΡƒΡ‚Ρ€Π΅Π½Π½Π΅ΠΉ ΠΎΠ±ΠΊΠ»Π°Π΄ΠΊΠΎΠΉ ΠΊΠΎΡ‚ΠΎΡ€ΠΎΠ³ΠΎ являСтся испытуСмый ΠΎΠ±Ρ€Π°Π·Π΅Ρ†, внСшнСй β€” Ρ†ΠΈΠ»ΠΈΠ½Π΄Ρ€ ΠΈΠ· Π΄Π²ΡƒΡ… ΠΏΠΎΠ»ΠΎΠ²ΠΈΠ½ (рис. 1).  [c.238]












ΠŸΠΎΠ»ΡŒΠ·ΡƒΡΡΡŒ Ρ„ΠΎΡ€ΠΌΡƒΠ»ΠΎΠΉ цилиндричСского кондСнсатора  [c.239]

Π•ΠΌΠΊΠΎΡΡ‚ΡŒ цилиндричСского кондСнсатора (Ρ„ΠΈΠ³. 4)  [c.331]

Π“ΠΎΡ€ΠΈΠ·ΠΎΠ½Ρ‚Π°Π»ΡŒΠ½Ρ‹ΠΉ цилиндричСский кондСнсатор встроСн Π² сухопарник ΠΈ ΠΎΡ‚Π΄Π΅Π»Π΅Π½ ΠΎΡ‚ основного ΠΏΠ°Ρ€ΠΎΠ²ΠΎΠ³ΠΎ пространства Π²Π½ΡƒΡ‚Ρ€Π΅Π½Π½ΠΈΠΌ ΠΊΠΎΠΆΡƒΡ…ΠΎΠΌ. ΠžΠΏΡ€Π΅ΡΠ½ΠΈΡ‚Π΅Π»ΠΈ рассчитаны Π½Π° ΠΏΠΎΠ²Ρ‹ΡˆΠ΅Π½Π½ΠΎΠ΅ солСсодСрТа-Π½ΠΈΠ΅ дистиллята (50 Π»Π³Π³/Π»).  [c.215]










ΠŸΡ€ΠΈΠ½Ρ†ΠΈΠΏ дСйствия ΠΈΠ·ΠΌΠ΅Ρ€ΠΈΡ‚Π΅Π»ΡŒΠ½ΠΎΠΉ части Ρ‚ΠΎΠΏΠ»ΠΈΠ²ΠΎΠΌΠ΅Ρ€Π° основан Π½Π° ΠΈΠ·ΠΌΠ΅Π½Π΅Π½ΠΈΠΈ элСктричСской Смкости Π΄Π°Ρ‚Ρ‡ΠΈΠΊΠΎΠ² Ρ‚ΠΎΠΏΠ»ΠΈΠ²ΠΎΠΌΠ΅Ρ€Π° (ΠΏΡ€Π΅Π΄ΡΡ‚Π°Π²Π»ΡΡŽΡ‰ΠΈΡ… собой цилиндричСскиС кондСнсаторы) ΠΏΡ€ΠΈ ΠΈΠ·ΠΌΠ΅Π½Π΅Π½ΠΈΠΈ уровня Ρ‚ΠΎΠΏΠ»ΠΈΠ²Π° Π² Π±Π°ΠΊΠ°Ρ….  [c.247]

ΠŸΡ€ΠΎΡΡ‚ΠΎΡ‚Π° согласования цилиндричСского кондСнсатор ΠΉ элСктростатичСской фокусировки с ΠΎΡ‚ΠΊΠ»ΠΎΠ½ΡΡŽΡ‰Π΅ΠΉ  [c.52]



Π€ΠΈΠ³. 78. Π‘Ρ…Π΅ΠΌΠ° пластинчато ΠΎ ΠΈ цилиндричСского кондСнсатора.

ΠΠ°ΠΏΡ€ΡΠΆΠ΅Π½Π½ΠΎΡΡ‚ΡŒ поля Π² цилиндричСском кондСнсаторС  [c.25]

ΠŸΠΎΡ‚Π΅Π½Ρ†ΠΈΠ°Π» Π²Π½ΡƒΡ‚Ρ€ΠΈ цилиндричСского кондСнсатора ΠΈ r=Ui β€” Π•Π³ 1ΠΏ Ri  [c.26]

Π•ΠΌΠΊΠΎΡΡ‚ΡŒ цилиндричСского кондСнсатора 2-.e  [c.26]



Рис. 2.3. Участок изоляции ΠΌΠ΅ΠΆΠ΄Ρƒ Π·Π»Π΅ΠΊΡ‚Ρ€ΠΎΠ΄Π°-ΠΌΠ½ Π² Π²ΠΈΠ΄Π΅ Π΄Π²ΡƒΡ… ΠΊΠΎΠ°ΠΊΡΠΈΠ°Π»ΡŒΠ½Ρ‹Ρ… ΠΈΠΈΠ»ΠΈΠ½Π΄Ρ€ΠΎΠ² (цилиндричСский кондСнсатор)










Для цилиндричСского кондСнсатора (см. рис. 2.3) для Ρ‚ΠΎΡ‡ΠΊΠΈ Π² диэлСктрикС ΠΌΠ΅ΠΆΠ΄Ρƒ элСктродами, находящСйся Π½Π° расстоянии Ρ… ΠΎΡ‚ оси кондСнсатора (/ 1[c.25]











Π’Π°ΠΊΠΈΠΌ ΠΎΠ±Ρ€Π°Π·ΠΎΠΌ, Π² цилиндричСском кондСнсаторС (Π΄Π°ΠΆΠ΅ с ΠΎΠ΄Π½ΠΎΡ€ΠΎΠ΄Π½Ρ‹ΠΌ диэлСктриком) ΠΏΠΎΠ»Π΅ Π½Π΅ΠΎΠ΄Π½ΠΎΡ€ΠΎΠ΄Π½ΠΎ наибольшая Π½Π°ΠΏΡ€ΡΠΆΠ΅Π½Π½ΠΎΡΡ‚ΡŒ ΠΈΠΌΠ΅Π΅Ρ‚ мСсто Π² Ρ‚ΠΎΡ‡ΠΊΠ°Ρ… диэлСктрика, нСпосрСдствСнно ΠΏΡ€ΠΈΠΌΡ‹ΠΊΠ°ΡŽΡ‰ΠΈΡ… ΠΊ Π²Π½ΡƒΡ‚Ρ€Π΅Π½Π½Π΅ΠΌΡƒ элСктроду x = ri)  [c. 25]

Если ΠΆΠ΅ диэлСктрик цилиндричСского кондСнсатора многослойный (ΠΏ слоСв), Ρ‚ΠΎ Π½Π°ΠΏΡ€ΡΠΆΠ΅Π½Π½ΠΎΡΡ‚ΡŒ Π² -ΠΌ слоС Π½Π° расстоянии Ρ… ΠΎΡ‚ оси кондСнсатора Π³ΠΈ Ρ…[c.25]

Емкостный ΠΌΠ΅Ρ‚ΠΎΠ΄ контроля ΠΌΠΎΠΆΠ΅Ρ‚ Π±Ρ‹Ρ‚ΡŒ ΠΊΠ°ΠΊ ΠΊΠΎΠ½Ρ‚Π°ΠΊΡ‚Π½Ρ‹ΠΌ, Ρ‚Π°ΠΊ ΠΈ бСсконтактным. ΠŸΡ€ΠΈ бСсконтактном ΠΌΠ΅Ρ‚ΠΎΠ΄Π΅ ΠΎΠ΄Π½ΠΎΠΉ ΠΈΠ· пластин кондСнсатора слуТит само ΠΊΠΎΠ½Ρ‚Ρ€ΠΎΠ»ΠΈΡ€ΡƒΠ΅ΠΌΠΎΠ΅ ΠΈΠ·Π΄Π΅Π»ΠΈΠ΅ ΠΏΡ€ΠΈ ΠΊΠΎΠ½Ρ‚Π°ΠΊΡ‚Π½ΠΎΠΌ ΠΌΠ΅Ρ‚ΠΎΠ΄Π΅ Смкостный Π΄Π°Ρ‚Ρ‡ΠΈΠΊ прСдставляСт собой плоский ΠΈΠ»ΠΈ цилиндричСский кондСнсатор, ΠΎΠ΄Π½Π° ΠΈΠ· пластин ΠΊΠΎΡ‚ΠΎΡ€ΠΎΠ³ΠΎ связана с ΠΈΠ·ΠΌΠ΅Ρ€ΠΈΡ‚Π΅Π»ΡŒΠ½Ρ‹ΠΌ стСрТнСм. БСсконтактный ΠΌΠ΅Ρ‚ΠΎΠ΄ Π½Π°Ρ…ΠΎΠ΄ΠΈΡ‚ ΠΎΠ³Ρ€Π°Π½ΠΈΡ‡Π΅Π½Π½ΠΎΠ΅ ΠΏΡ€ΠΈΠΌΠ΅Π½Π΅Π½ΠΈΠ΅.  [c.200]

Допустим, Ρ‡Ρ‚ΠΎ Π½Π΅ΠΎΠ±Ρ…ΠΎΠ΄ΠΈΠΌΠΎ произвСсти соСдинСниС ΠΊΠΎΠ»ΠΏΠ°Ρ‡ΠΊΠΎΠ² с сСкциСй цилиндричСского кондСнсатора (рис. 3). Для выполнСния этой Ρ€Π°Π±ΠΎΡ‚Ρ‹ ΠΏΡ€Π΅ΠΆΠ΄Π΅ всСго Π½ΡƒΠΆΠ½ΠΎ ΠΎΠΏΡ€Π΅Π΄Π΅Π»ΠΈΡ‚ΡŒ сборочный состав издСлия, Π½Π΅ΠΎΠ±Ρ…ΠΎΠ΄ΠΈΠΌΡ‹Π΅ Π΄Π΅Ρ‚Π°Π»ΠΈ ΠΈ ΠΌΠ°Ρ‚Π΅Ρ€ΠΈΠ°Π»Ρ‹, ΠΈΡ… количСство, ΠΎΠΏΡ€Π΅Π΄Π΅Π»ΠΈΡ‚ΡŒ, Π±Π°Π·ΠΎΠ²ΡƒΡŽ Π΄Π΅Ρ‚Π°Π»ΡŒ ΠΈΠ»ΠΈ элСмСнт. Анализ Ρ€Π°Π±ΠΎΡ‡Π΅Π³ΠΎ Ρ‡Π΅Ρ€Ρ‚Π΅ΠΆΠ° собираСмого издСлия ΠΏΠΎΠΊΠ°Π·Ρ‹Π²Π°Π΅Ρ‚, Ρ‡Ρ‚ΠΎ Π² качСствС Π±Π°Π·ΠΎΠ²ΠΎΠΉ Π΄Π΅Ρ‚Π°Π»ΠΈ ΠΌΠΎΠΆΠ½ΠΎ ΠΈΡΠΏΠΎΠ»ΡŒΠ·ΠΎΠ²Π°Ρ‚ΡŒ ΡΠ΅ΠΊΡ†ΠΈΡŽ кондСнсатора 1 Π² сборС с припаянными ΠΊ Π½Π΅ΠΉ Π²Ρ‹Π²ΠΎΠ΄Π°ΠΌΠΈ 2 ΠΊΠΎΠ»ΠΏΠ°Ρ‡ΠΊΠΈ 7 Π½Π°Π΄Π΅Π²Π°ΡŽΡ‚ Π½Π° ΡΠ΅ΠΊΡ†ΠΈΡŽ с Π΄Π²ΡƒΡ… сторон, Π° Π·Π°Ρ‚Π΅ΠΌ Π²ΡΡ‚Π°Π²Π»ΡΡŽΡ‚ Π²  [c. 16]



Рис. 3. ΠžΠ±Ρ‰ΠΈΠΉ Π²ΠΈΠ΄ Π΄Π²ΡƒΡ… Π²Π°Ρ€ΠΈΠ°Π½Ρ‚ΠΎΠ² цилиндричСских кондСнсаторов










Рассмотрим схСму тСхнологичСского процСсса сборки сСкции цилиндричСского кондСнсатора с двумя ΠΊΠΎΠ»ΠΏΠ°Ρ‡ΠΊΠ°ΠΌΠΈ (рис. 4). Π—Π° Π½Π°Ρ‡Π°Π»ΠΎ сборки (Π±Π°Π·ΠΎΠ²Ρ‹ΠΉ элСмСнт—  [c.18]








На основании ΠΏΠΎΠ»ΡƒΡ‡Π΅Π½Π½Ρ‹Ρ… Π·Π½Π°Ρ‡Π΅Π½ΠΈΠΉ Qt ΠΌΠΎΠΆΠ½ΠΎ ΡΠ΄Π΅Π»Π°Ρ‚ΡŒ Π²Ρ‹Π²ΠΎΠ΄, Ρ‡Ρ‚ΠΎ с Ρ‚ΠΎΡ‡ΠΊΠΈ зрСния осущСствлСния автоматичСской сборки конструкция Π»Π°ΠΌΠΏΠΎΠ²ΠΎΠΉ ΠΏΠ°Π½Π΅Π»ΠΈ Π±ΠΎΠ»Π΅Π΅ Ρ‚Π΅Ρ…Π½ΠΎΠ»ΠΎΠ³ΠΈΡ‡Π½Π°, Ρ‡Π΅ΠΌ конструкция цилиндричСского кондСнсатора.  [c.29]

ΠšΠΈΠ½Π΅ΠΌΠ°Ρ‚ΠΈΡ‡Π΅ΡΠΊΠ°Ρ схСма автоматичСской установки ΠΏΡ€ΠΈΠΏΠ°ΠΉΠΊΠΈ Π²Ρ‹Π²ΠΎΠ΄ΠΎΠ² ΠΊ сСкциям цилиндричСских кондСнсаторов ΠΏΠΎΠΊΠ°Π·Π°Π½Π° Π½Π° рис. 74. Π’ Π°Π²Ρ‚ΠΎΠΌΠ°Ρ‚Π΅ совмСщСны ΠΎΠΏΠ΅Ρ€Π°Ρ†ΠΈΠΈ изготовлСния Π΄Π²ΡƒΡ… Π²Ρ‹Π²ΠΎΠ΄ΠΎΠ² ΠΈ ΠΏΡ€ΠΈΠΏΠ°ΠΉΠΊΠΈ ΠΈΡ… ΠΊ сСкциям.  [c.212]



Рис. 74. ΠšΠΈΠ½Π΅ΠΌΠ°Ρ‚ΠΈΡ‡Π΅ΡΠΊΠ°Ρ схСма автоматичСской установки ΠΏΡ€ΠΈΠΏΠ°ΠΉΠΊΠΈ Π²Ρ‹Π²ΠΎΠ΄ΠΎΠ² ΠΊ сСкциям цилиндричСских кондСнсаторов.

ЭлСктричСская схСма для управлСния Ρ€Π°Π±ΠΎΡ‚ΠΎΠΉ Π°Π²Ρ‚ΠΎΠΌΠ°Ρ‚Π° мСханичСской сборки цилиндричСских кондСнсаторов прСдставлСна Π½Π° рис. 121.  [c.335]

Π˜ΠΎΠ½ΠΈΠ·Π°Ρ†ΠΈΠΎΠ½Π½Π°Ρ ΠΊΠ°ΠΌΠ΅Ρ€Π° ΠΎΠ±Ρ‹Ρ‡Π½ΠΎ Ρ€Π°Π±ΠΎΡ‚Π°Π΅Ρ‚ Π² Ρ€Π΅ΠΆΠΈΠΌΠ΅ Ρ‚ΠΎΠΊΠ° насыщСния, Π³Π΄Π΅ Π½Π΅Ρ‚ Π³Π°Π·ΠΎΠ²ΠΎΠ³ΠΎ усилСния. Π’ этом случаС число ΠΏΠ°Ρ€ ΠΈΠΎΠ½ΠΎΠ², Π²ΠΎΠ·Π½ΠΈΠΊΠ°ΡŽΡ‰ΠΈΡ… ΠΏΠΎΠ΄ дСйствиСм ΠΏΠΎΠΏΠ°Π΄Π°ΡŽΡ‰Π΅ΠΉ Π² ΠΈΠΎΠ½ΠΈΠ·Π°Ρ†ΠΈΠΎΠ½Π½ΡƒΡŽ ΠΊΠ°ΠΌΠ΅Ρ€Ρƒ заряТСнной частицы, ΠΎΡ‚Π½ΠΎΡΠΈΡ‚Π΅Π»ΡŒΠ½ΠΎ Π½Π΅Π²Π΅Π»ΠΈΠΊΠΎ ΠΈ рСгистрация ΠΎΡ‚Π΄Π΅Π»ΡŒΠ½Ρ‹Ρ…. частиц с ΠΏΠΎΠΌΠΎΡ‰ΡŒΡŽ ΠΈΠΎΠ½ΠΈΠ·Π°Ρ†ΠΈΠΎΠ½Π½ΠΎΠΉ ΠΊΠ°ΠΌΠ΅Ρ€Ρ‹ ΠΏΡ€ΠΈ отсутствии Π³Π°Π·Π±Π²ΠΎΠ³ΠΎ усилСния связана с большими трудностями. Π’ Ρ€Π΅ΠΆΠΈΠΌΠ΅ Π³Π°Π·ΠΎΠ²ΠΎΠ³ΠΎ усилСния ионизационная ΠΊΠ°ΠΌΠ΅Ρ€Π° ΠΌΠΎΠΆΠ΅Ρ‚ Ρ€Π°Π±ΠΎΡ‚Π°Ρ‚ΡŒ Π² качСствС счСтчика ΠΎΡ‚Π΄Π΅Π»ΡŒΠ½Ρ‹Ρ… заряТСнных частиц. ΠŸΠΎΡΡ‚ΠΎΠΌΡƒ ΠΈΠΎΠ½ΠΈΠ·Π°Ρ†ΠΈΠΎΠ½Π½Ρ‹Π΅ ΠΊΠ°ΠΌΠ΅Ρ€Ρ‹ ΠΎΠ±Ρ‹Ρ‡Π½ΠΎ ΠΏΠΎΠ΄Ρ€Π°Π·Π΄Π΅Π»ΡΡŽΡ‚ΡΡ Π½Π° Π΄Π²Π° Π²ΠΈΠ΄Π° счСтно-ΠΈΠΎΠ½ΠΈΠ·Π°Ρ†ΠΈΠΎΠ½Π½Ρ‹Π΅ ΠΊΠ°ΠΌΠ΅Ρ€Ρ‹, ΠΏΡ€Π΅Π΄Π½Π°Π·Π½Π°Ρ‡Π΅Π½Π½Ρ‹Π΅ для рСгистрации прохоТдСния Ρ‡Π΅Ρ€Π΅Π· ΠΊΠ°ΠΌΠ΅Ρ€Ρƒ ΠΎΠ΄Π½ΠΎΠΉ ΠΊΠ°ΠΊΠΎΠΉ-Π»ΠΈΠ±ΠΎ заряТСнной частицы, ΠΈ ΠΈΠ½Ρ‚Π΅Π³Ρ€ΠΈΡ€ΡƒΡŽΡ‰ΠΈΠ΅ ΠΈΠΎΠ½ΠΈΠ·Π°Ρ†ΠΈΠΎΠ½Π½Ρ‹Π΅ ΠΊΠ°ΠΌΠ΅Ρ€Ρ‹, примСняСмыС для измСрСния интСнсивности ΠΏΠΎΡ‚ΠΎΠΊΠ° частиц. Π’ зависимости ΠΎΡ‚ условий Π·Π°Π΄Π°Ρ‡ΠΈ ΠΈΠΎΠ½ΠΈΠ·Π°Ρ†ΠΈΠΎΠ½Π½Ρ‹Π΅ ΠΊΠ°ΠΌΠ΅Ρ€Ρ‹ ΠΏΠΎ Ρ„ΠΎΡ€ΠΌΠ΅ элСктродов ΠΈΠΌΠ΅ΡŽΡ‚ Π²ΠΈΠ΄ плоского, сфСричСского ΠΈΠ»ΠΈ цилиндричСского кондСнсатора. Π Π°Π·ΠΌΠ΅Ρ€Ρ‹ ΠΈΡ… ΠΌΠΎΠ³ΡƒΡ‚ Π±Ρ‹Ρ‚ΡŒ вСсьма Ρ€Π°Π·Π»ΠΈΡ‡Π½Ρ‹ΠΌΠΈ β€” ΠΎΡ‚ Π΄ΠΎΠ»Π΅ΠΉ кубичСских ΠΌΠΈΠ»Π»ΠΈΠΌΠ΅Ρ‚Ρ€ΠΎΠ² Π΄ΠΎ сотСн Π»ΠΈΡ‚Ρ€ΠΎΠ², Π² зависимости ΠΎΡ‚ ΠΈΡ… назначСния.  [c.39]












I β€” Π΄Π»ΠΈΠ½Π° цилиндричСского кондСнсатора Π² см, Ρ‚. Π΅. Π΄Π»ΠΈΠ½Π° элСктрода для ΠΊΠΎΠ½ΠΊΡ€Π΅Ρ‚Π½ΠΎΠΉ конструкции I = onst  [c.113]

Π‘ Ρ†Π΅Π»ΡŒΡŽ ΠΏΡ€ΠΎΠ²Π΅Ρ€ΠΊΠΈ ΠΏΠΎΠ»ΡƒΡ‡Π΅Π½Π½Ρ‹Ρ… Ρ€Π΅ΠΊΠΎΠΌΠ΅Π½Π΄Π°Ρ†ΠΈΠΉ ΠΈ Π²Ρ‹Π²ΠΎΠ΄ΠΎΠ² Π±Ρ‹Π»Π° ΠΏΡ€ΠΎΠ²Π΅Π΄Π΅Π½Π° сСрия экспСримСнтов ΠΏΠΎ ΠΈΠ·ΡƒΡ‡Π΅Π½ΠΈΡŽ Π³Π°Π·ΠΎΡ€Π΅Π³ΡƒΠ»ΠΈΡ€ΡƒΠ΅ΠΌΠΎΠΉ Π’Π’ ΠΎΡ‚ΠΊΡ€Ρ‹Ρ‚ΠΎΠ³ΠΎ Ρ‚ΠΈΠΏΠ°. Π˜ΡΡΠ»Π΅Π΄ΡƒΠ΅ΠΌΠ°Ρ Ρ‚Ρ€ΡƒΠ±Π° ΠΈΠΌΠ΅Π»Π° Π΄Π»ΠΈΠ½Ρƒ 1,5 ΠΌ, внСшний Π΄ΠΈΠ°ΠΌΠ΅Ρ‚Ρ€ 10 ΠΌ ΠΈ состояла ΠΈΠ· испаритСля ΠΈ кондСнсатора. Π˜ΡΠΏΠ°Ρ€ΠΈΡ‚Π΅Π»ΡŒ Π±Ρ‹Π» ΠΈΠ· ΠΌΠ΅Π΄ΠΈ, ΠΈΠΌΠ΅Π» Ρ„ΠΎΡ€ΠΌΡƒ ΠΌΠ΅Π΄Π½ΠΎΠ³ΠΎ ΠΏΠΎΠ»ΠΎΠ³ΠΎ Ρ†ΠΈΠ»ΠΈΠ½Π΄Ρ€Π° Π΄Π»ΠΈΠ½ΠΎΠΉ 500 ΠΌΠΌ, Π½Π° Π²Π½ΡƒΡ‚Ρ€Π΅Π½Π½Π΅ΠΉ повСрхности ΠΊΠΎΡ‚ΠΎΡ€ΠΎΠ³ΠΎ Π±Ρ‹Π»ΠΎ 16 Π°ΠΊΡΠΈΠ°Π»ΡŒΠ½Ρ‹Ρ… ΠΏΡ€ΡΠΌΠΎΡƒΠ³ΠΎΠ»ΡŒΠ½Ρ‹Ρ… ΠΊΠ°Π½Π°Π²ΠΎΠΊ ΡˆΠΈΡ€ΠΈΠ½ΠΎΠΉ 0,4 ΠΌΠΌ ΠΈ Π³Π»ΡƒΠ±ΠΈΠ½ΠΎΠΉ 0,6 ΠΌΠΌ. Выбирался ΠΎΠ½ с ΠΌΠ°Π»Ρ‹ΠΌ тСрмичСским сопротивлСниСм с Ρ†Π΅Π»ΡŒΡŽ получСния высоких Π·Π½Π°Ρ‡Π΅Π½ΠΈΠΉ коэффициСнта Ρ‚Π΅ΠΌΠΏΠ΅Ρ€Π°Ρ‚ΡƒΡ€Π½ΠΎΠΉ Ρ‡ΡƒΠ²ΡΡ‚Π²ΠΈΡ‚Π΅Π»ΡŒΠ½ΠΎΡΡ‚ΠΈ, Π° Ρ‚Π°ΠΊΠΆΠ΅ ΡƒΠΌΠ΅Π½ΡŒΡˆΠ΅Π½ΠΈΡ ΠΏΡƒΠ»ΡŒΡΠ°Ρ†ΠΈΠΉ Ρ‚Π΅ΠΌΠΏΠ΅Ρ€Π°Ρ‚ΡƒΡ€Ρ‹ ΠΈ давлСния. ЦилиндричСский кондСнсатор Π±Ρ‹Π» Π²Ρ‹ΠΏΠΎΠ»Π½Π΅Π½ ΠΈΠ· тСрмостойкого стСкла Π΄Π»ΠΈΠ½ΠΎΠΉ 1 ΠΌ для ΡƒΠΌΠ΅Π½ΡŒΡˆΠ΅Π½ΠΈΡ аксиальной ΡΠΎΡΡ‚Π°Π²Π»ΡΡŽΡ‰Π΅ΠΉ Ρ‚Π΅ΠΏΠ»ΠΎΠ²ΠΎΠ³ΠΎ ΠΏΠΎΡ‚ΠΎΠΊΠ° Π² Π·ΠΎΠ½Π΅ Ρ€Π°Π·Π΄Π΅Π»Π° ΠΏΠ°Ρ€β€”Π³Π°Π· ΠΈ Π²ΠΈΠ·ΡƒΠ°Π»ΠΈΠ·Π°Ρ†ΠΈΠΈ процСссов. ΠšΠΎΠ½Π΄Π΅Π½ΡΠ°Ρ‚ΠΎΡ€ ΠΈΠΌΠ΅Π» Π³ΠΈΠ±ΠΊΠΎΠ΅ соСдинСниС с испаритСлСм ΠΈ ΠΌΠΎΠ³ ΠΈΠ·ΠΌΠ΅Π½ΡΡ‚ΡŒ ΡƒΠ³ΠΎΠ» Π½Π°ΠΊΠ»ΠΎΠ½Π° ΠΎΡ‚ β€”90 Π΄ΠΎ +90Β°. На внСшнСй повСрхности испаритСля ΠΈΠΌΠΈΡ‚ΠΈΡ€ΠΎΠ²Π°Π»ΠΈΡΡŒ Π³Ρ€Π°Π½ΠΈΡ‡Π½Ρ‹Π΅ условия II Ρ€ΠΎΠ΄Π° (Ρ‚Ρ€ΠΈ сСкции омичСского нагрСватСля), Π° Π½Π° внСшнСй повСрхности кондСнсатора— III Ρ€ΠΎΠ΄Π° (сб 10 Π’Ρ‚/(ΠΌ -К)). Поля Ρ‚Π΅ΠΌΠΏΠ΅Ρ€Π°Ρ‚ΡƒΡ€ ΠΈΠ·ΠΌΠ΅Ρ€ΡΠ»ΠΈΡΡŒ Ρ…Ρ€ΠΎΠΌΠ΅Π»ΡŒ-ΠΊΠΎΠΏΠ΅Π»Π΅Π²Ρ‹ΠΌΠΈ Ρ‚Π΅Ρ€ΠΌΠΎΠΏΠ°Ρ€Π°ΠΌΠΈ, Π° Ρ‚Π°ΠΊΠΆΠ΅ ΠΏΠ»Π΅Π½ΠΎΡ‡Π½Ρ‹ΠΌ Ρ‚Π΅Ρ€ΠΌΠΎΠ½ΠΉΠ΄ΠΈΠΊΠ°Ρ‚ΠΎΡ€ΠΎΠΌ Π½Π° Π±Π°Π·Π΅ ΠΆΠΈΠ΄ΠΊΠΈΡ… кристаллов (Π² Π·ΠΎΠ½Π΅ Ρ€Π°Π·Π΄Π΅Π»Π° ΠΏΠ°Ρ€β€”Π³Π°Π·). Π’ качСствС Ρ‚Π΅ΠΏΠ»ΠΎ-нос1 тСля использовался этиловый спирт, Π° нСкондСнси-Ρ€ΡƒΡŽΡ‰Π΅Π³ΠΎΡΡ Π³Π°Π·Π° β€” Π²ΠΎΠ·Π΄ΡƒΡ… ΠΈΠ»ΠΈ Ρ„Ρ€Π΅ΠΎΠ½-11. ΠžΡ‚Π½ΠΎΡˆΠ΅Π½ΠΈΡ молСкулярных вСсов ΠΈΠΌΠ΅Π»ΠΈ значСния /Π‘ΠΌ= 1,324 ΠΈ /Π‘ΠΌ = 0,276 соотвСтствСнно. Π”ΠΈΠ°ΠΌΠ΅Ρ‚Ρ€ ΠΏΠ°Ρ€ΠΎΠ²ΠΎΠ³ΠΎ ΠΊΠ°Π½Π°Π»Π° кондСнсатора Π½Π°ΠΌΠ½ΠΎΠ³ΠΎ ΠΏΡ€Π΅Π²Ρ‹ΡˆΠ°Π» минимальноС ΠΏΠΎΡ€ΠΎΠ³ΠΎΠ²ΠΎΠ΅ Π·Π½Π°Ρ‡Π΅Π½ΠΈΠ΅ da для ΠΏΠ°Ρ€Ρ‹ этанол—фрСон-11. По Ρ€Π΅Π·ΡƒΠ»ΡŒΡ‚Π°Ρ‚Π°ΠΌ экспСримСнта Π±Ρ‹Π»ΠΈ построСны Π³Ρ€Π°Ρ„ΠΈΠΊΠΈ, ΠΏΠΎΠΊΠ°Π·Π°Π½Π½Ρ‹Π΅ Π½Π° рис. 9. РаспрСдСлСниС Ρ‚Π΅ΠΌΠΏΠ΅Ρ€Π°Ρ‚ΡƒΡ€Ρ‹ Π² области ΠΏΠ°Ρ€ΠΎΠ³Π°Π·ΠΎΠ²ΠΎΠ³ΠΎ Ρ„Ρ€ΠΎΠ½Ρ‚Π° соотвСтствовало расчСтам ΠΈ рСкомСндациям. ΠŸΡ€ΠΎΡ‚ΡΠΆΠ΅Π½Π½ΠΎΡΡ‚ΡŒ Π·ΠΎΠ½Ρ‹ Ρ€Π°Π·Π΄Π΅Π»Π° этанол β€” Π²ΠΎΠ·Π΄ΡƒΡ… составила 0,004,Π° Π·ΠΎΠ½Ρ‹ этанол β€” Ρ„Ρ€Π΅ΠΎΠ½-11 β€”0,5 ΠΌ, Ρ‚. Π΅. Π½Π° Π΄Π²Π° порядка большС. АналогичныС Ρ€Π΅Π·ΡƒΠ»ΡŒΡ‚Π°Ρ‚Ρ‹ Π±Ρ‹Π»ΠΈ ΠΏΠΎΠ»ΡƒΡ‡Π΅Π½Ρ‹ ΠΏΡ€ΠΈ ΠΎΡ‚Ρ€ΠΈΡ†Π°Ρ‚Π΅Π»ΡŒΠ½Ρ‹Ρ… ΡƒΠ³Π»Π°Ρ… Π½Π°ΠΊΠ»ΠΎΠ½Π° кондСнсатора (ΠΈΡΠΏΠ°Ρ€ΠΈΡ‚Π΅Π»ΡŒ Π½Π°Π΄ кондСнсатором).  [c.32]

ЧВО Ρ‚Π΅ΠΏΠ»ΠΎΠΏΠ΅Ρ€Π΅Π΄Π°ΡŽΡ‰ΠΈΠ΅ характСристики гладкостСмиых Ρ†ΠΈΠ»ΠΈΠ½Π΄Ρ€ΠΎΠ² Π·Π½Π°Ρ‡ΠΈΡ‚Π΅Π»ΡŒΠ½ΠΎ Ρ…ΡƒΠΆΠ΅ характСристик усСчСнного конуса, Ρ‡Ρ‚ΠΎ ΠΈ слСдовало ΠΎΠΆΠΈΠ΄Π°Ρ‚ΡŒ, Ρ‚Π°ΠΊ ΠΊΠ°ΠΊ Π² коничСском кондСнсаторном участкС Ρ†Π΅Π½Ρ‚Ρ€ΠΎΠ±Π΅ΠΆΠ½Ρ‹Π΅ силы ΡƒΡΠΊΠΎΡ€ΡΡŽΡ‚ Π΄Π²ΠΈΠΆΠ΅Π½ΠΈΠ΅ Тидкости Π² Π½Π°ΠΏΡ€Π°Π²Π»Π΅Π½ΠΈΠΈ испаритСля. Π’ΠΎ Π²Ρ€Π°Ρ‰Π°ΡŽΡ‰Π΅ΠΌΡΡ Ρ†ΠΈΠ»ΠΈΠ½Π΄Ρ€Π΅ Ρ‚Π΅Ρ‡Π΅Π½ΠΈΠ΅ кондСнсата вызываСтся Π³Ρ€Π°Π΄ΠΈΠ΅Π½Ρ‚ΠΎΠΌ гидростатичСского давлСния, ΠΊΠΎΡ‚ΠΎΡ€Ρ‹ΠΉ создаСтся Π² кондСнсаторС Π·Π° счСт измСнСния Ρ‚ΠΎΠ»Ρ‰ΠΈΠ½Ρ‹ ΠΏΠ»Π΅Π½ΠΊΠΈ вдоль Π΅Π³ΠΎ оси. ΠŸΡ€ΠΈ Ρ€Π°Π²Π½Ρ‹Ρ… условиях стСкания Ρ€Π°Π±ΠΎΡ‡Π΅ΠΉ Тидкости Π² ΠΊΠΎΠ½Ρ†Π΅ кондСнсаторного участка Ρ€Π΅Π·ΡƒΠ»ΡŒΡ‚ΠΈΡ€ΡƒΡŽΡ‰Π°Ρ Ρ‚ΠΎΠ»Ρ‰ΠΈΠ½Π° ΠΏΠ»Π΅Π½ΠΊΠΈ Тидкости Π² цилиндричСском кондСнсаторС большС, Ρ‡Π΅ΠΌ Π² коничСском.  [c.131]

Для изготовлСния кондСнсаторных Π²Ρ‚ΡƒΠ»ΠΎΠΊ ΠΏΡ€ΠΈΠΌΠ΅Π½ΡΡŽΡ‚ Ρ‚Π΅ ΠΆΠ΅ ΠΌΠ°Ρ‚Π΅Ρ€ΠΈΠ°Π»Ρ‹ (намоточная Π±ΡƒΠΌΠ°Π³Π° ΠΈ Π»Π°ΠΊ), Ρ‡Ρ‚ΠΎ ΠΈ Π² производствС Π±ΡƒΠΌΠ°ΠΆΠ½ΠΎΠ±Π°ΠΊΠ΅Π»ΠΈΡ‚ΠΎΠ²Ρ‹Ρ… Ρ‚Ρ€ΡƒΠ±ΠΎΠΊ. ВСхнология Π»Π°ΠΊΠΈΡ€ΠΎΠ²ΠΊΠΈ Π±ΡƒΠΌΠ°Π³ΠΈ, Π½Π°ΠΌΠΎΡ‚ΠΊΠΈ издСлия ΠΈ Π΅Π³ΠΎ Ρ‚Π΅ΠΏΠ»ΠΎΠ²ΠΎΠΉ ΠΎΠ±Ρ€Π°Π±ΠΎΡ‚ΠΊΠΈ Ρ‚Π°ΠΊΠΆΠ΅ Π°Π½Π°Π»ΠΎΠ³ΠΈΡ‡Π½Ρ‹ с Ρ‚ΠΎΠΉ лишь Ρ€Π°Π·Π½ΠΈΡ†Π΅ΠΉ, Ρ‡Ρ‚ΠΎ Π² процСссС Π½Π°ΠΌΠΎΡ‚ΠΊΠΈ Π½Π° Π·Π°Π΄Π°Π½Π½Ρ‹Ρ… Π΄ΠΈΠ°ΠΌΠ΅Ρ‚Ρ€Π°Ρ… Π² Ρ‚Π΅Π»ΠΎ Π½Π°ΠΌΠ°Ρ‚Ρ‹Π²Π°Π΅ΠΌΠΎΠΉ Π²Ρ‚ΡƒΠ»ΠΊΠΈ Π·Π°ΠΊΠ»Π°Π΄Ρ‹Π²Π°ΡŽΡ‚ Π°Π»ΡŽΠΌΠΈΠ½ΠΈΠ΅Π²Ρ‹Π΅ ΠΏΡ€ΠΎΠΊΠ»Π°Π΄ΠΊΠΈ, слуТащиС ΠΎΠ±ΠΊΠ»Π°Π΄ΠΊΠ°ΠΌΠΈ цилиндричСского кондСнсатора. ΠšΠΎΠ»ΠΈΡ‡Π΅ΡΡ‚Π²ΠΎ ΠΎΠ±ΠΊΠ»Π°Π΄ΠΎΠΊ ΠΎΠ±Ρ‹Ρ‡Π½ΠΎ составляСт 9β€”11. ΠœΠ΅Π΄Π½Ρ‹Π΅ Ρ‚Ρ€ΡƒΠ±ΠΊΠΈ ΠΈΠ»ΠΈ стСрТни, Π½Π° ΠΊΠΎΡ‚ΠΎΡ€Ρ‹Π΅ Π½Π°ΠΌΠ°Ρ‚Ρ‹Π²Π°ΡŽΡ‚ΡΡ Π²Ρ‚ΡƒΠ»ΠΊΠΈ, послС тСрмичСской ΠΎΠ±Ρ€Π°Π±ΠΎΡ‚ΠΊΠΈ ΠΈΠ·Π΄Π΅Π»ΠΈΠΉ Π½Π΅ ΠΈΠ·Π²Π»Π΅ΠΊΠ°ΡŽΡ‚ΡΡ ΠΈΠ· Π½ΠΈΡ… ΠΈ слуТат ΠΏΡ€ΠΈ эксплуатации  [c.341]

Рассмотрим ΠΊΠΎΠ½ΡΡ‚Ρ€ΡƒΠΊΡ†ΠΈΡŽ цилиндричСского кондСнсатора (Π±ΡƒΠΌΠ°ΠΆΠ½ΠΎΠ³ΠΎ ΠΈ ΠΌΠ΅Ρ‚Π°Π»Π»ΠΎ-Π±ΡƒΠΌΠ°ΠΆΠ½ΠΎΠ³ΠΎ) с Π²ΠΈΡ‚ΠΎΠΉ сСкциСй, ΠΈΠΌΠ΅ΡŽΡ‰Π΅Π³ΠΎ Π΄Π²Π° Ρ‚ΠΎΠ½ΠΊΠΈΡ… осСвых Π²Ρ‹Π²ΠΎΠ΄Π°. Π‘Ρ…Π΅ΠΌΠ° Ρ‚Π°ΠΊΠΎΠ³ΠΎ кондСнсатора Π±Ρ‹Π»Π° ΠΏΠΎΠΊΠ°Π·Π°Π½Π° Π½Π° рис. 3, Π°, Π²Π°Ρ€ΠΈΠ°Π½Ρ‚ II. ΠšΠΎΠ½Π΄Π΅Π½ΡΠ°Ρ‚ΠΎΡ€ состоит ΠΈΠ· Π²ΠΈΠ³ΠΎΠΉ сСкции 1, ΠΊ Ρ‚ΠΎΡ€Ρ†Π°ΠΌ ΠΊΠΎΡ‚ΠΎΡ€ΠΎΠΉ ΠΏΡ€ΠΈΠΏΠ°ΠΈΠ²Π°ΡŽΡ‚ΡΡ Π²Ρ‹Π²ΠΎΠ΄Ρ‹ 2. БСкция с Π²Ρ‹Π²ΠΎΠ΄Π°ΠΌΠΈ ΠΏΠΎΠΌΠ΅Ρ‰Π΅Π½Π° Π² Π°Π»ΡŽΠΌΠΈΠ½ΠΈΠ΅Π²Ρ‹ΠΉ корпус 6 с ΠΊΠΎΠ»ΠΏΠ°Ρ‡ΠΊΠ°ΠΌΠΈ 4, изготовляСмыми ΠΈΠ· ΠΏΠΎΠ»ΠΈΠΌΠ΅Ρ€Π½Ρ‹Ρ… ΠΌΠ°Ρ‚Π΅Ρ€ΠΈΠ°Π»ΠΎΠ². ΠšΠΎΠ»ΠΏΠ°Ρ‡ΠΊΠΈ слуТат для элСктроизоляции сСкции ΠΎΡ‚ корпуса ΠΈ для компСнсации ΠΏΠΎΠ³Ρ€Π΅ΡˆΠ½ΠΎΡΡ‚Π΅ΠΉ изготовлСния сСкции. ВнутрСнняя ΠΏΠΎΠ»ΠΎΡΡ‚ΡŒ кондСнсатора гСрмСтизируСтся Π·Π°Π»ΠΈΠ²ΠΊΠΎΠΉ ΠΊΠΎΠΌΠΏΠ°ΡƒΠ½Π΄ΠΎΠΌ Π½Π° основС эпоксидных смол ΠΈΠ»ΠΈ уплотняСтся Ρ€Π΅Π·ΠΈΠ½ΠΎΠ²Ρ‹ΠΌΠΈ шайбами (см. рис. 3, Π±). Π’ ΠΏΠΎΠ΄ΠΎΠ±Π½Ρ‹.ΠΊ  [c.171]


ЭлСктричСская Π΅ΠΌΠΊΠΎΡΡ‚ΡŒ: ΠΎΠΏΡ€Π΅Π΄Π΅Π»Π΅Π½ΠΈΠ΅, Ρ„ΠΎΡ€ΠΌΡƒΠ»Ρ‹, Π΅Π΄ΠΈΠ½ΠΈΡ†Ρ‹ измСрСния

Π’ элСктротСхникС часто встрСчаСтся понятиС ёмкости. ΠŸΡ€ΠΈ этом Ρ€Π΅Ρ‡ΡŒ ΠΈΠ΄Ρ‘Ρ‚ Π½Π΅ ΠΎ Π²Π΅Π΄Ρ€Π΅ ΠΈΠ»ΠΈ Π΄Ρ€ΡƒΠ³ΠΎΠΌ сосудС, Π° ΠΎΠ± элСктричСской ёмкости ΠΏΡ€ΠΎΠ²ΠΎΠ΄Π½ΠΈΠΊΠ°, аккумулятора ΠΈ кондСнсатора. ΠŸΡƒΡ‚Π°Ρ‚ΡŒ эти понятия нСльзя. Π’ этой ΡΡ‚Π°Ρ‚ΡŒΠ΅ ΠΌΡ‹ разбСрСмся, Ρ‡Ρ‚ΠΎ Ρ‚Π°ΠΊΠΎΠ΅ элСктричСская Ρ‘ΠΌΠΊΠΎΡΡ‚ΡŒ, ΠΎΡ‚ Ρ‡Π΅Π³ΠΎ ΠΎΠ½Π° зависит ΠΈ Π² ΠΊΠ°ΠΊΠΈΡ… Π΅Π΄ΠΈΠ½ΠΈΡ†Π°Ρ… измСряСтся.

ΠžΠΏΡ€Π΅Π΄Π΅Π»Π΅Π½ΠΈΠ΅

Для ΠΏΡ€ΠΎΠ²ΠΎΠ΄Π½ΠΈΠΊΠΎΠ² элСктричСской Ρ‘ΠΌΠΊΠΎΡΡ‚ΡŒΡŽ называСтся Π²Π΅Π»ΠΈΡ‡ΠΈΠ½Π°, которая Ρ…Π°Ρ€Π°ΠΊΡ‚Π΅Ρ€ΠΈΠ·ΡƒΠ΅Ρ‚ ΡΠΏΠΎΡΠΎΠ±Π½ΠΎΡΡ‚ΡŒ Ρ‚Π΅Π»Π° Π½Π°ΠΊΠ°ΠΏΠ»ΠΈΠ²Π°Ρ‚ΡŒ элСктричСский заряд. Π­Ρ‚ΠΎ ΠΈ Π΅ΡΡ‚ΡŒ Π΅Ρ‘ физичСский смысл. ΠžΠ±ΠΎΠ·Π½Π°Ρ‡Π°Π΅Ρ‚ΡΡ латинской Π±ΡƒΠΊΠ²ΠΎΠΉ C. Она Ρ€Π°Π²Π½Π° ΠΎΡ‚Π½ΠΎΡˆΠ΅Π½ΠΈΡŽ заряда ΠΊ ΠΏΠΎΡ‚Π΅Π½Ρ†ΠΈΠ°Π»Ρƒ, Ссли это Π·Π°ΠΏΠΈΡΠ°Ρ‚ΡŒ Π² Π²ΠΈΠ΄Π΅ Ρ„ΠΎΡ€ΠΌΡƒΠ»Ρ‹, Ρ‚ΠΎ получаСтся ΡΠ»Π΅Π΄ΡƒΡŽΡ‰Π΅Π΅:

C=q/Π€

Π­Π»Π΅ΠΊΡ‚Ρ€ΠΎΠ΅ΠΌΠΊΠΎΡΡ‚ΡŒ любого ΠΏΡ€Π΅Π΄ΠΌΠ΅Ρ‚Π° зависит ΠΎΡ‚ Π΅Π³ΠΎ Ρ„ΠΎΡ€ΠΌΡ‹ ΠΈ гСомСтричСских Ρ€Π°Π·ΠΌΠ΅Ρ€ΠΎΠ². Если Ρ€Π°ΡΡΠΌΠΎΡ‚Ρ€Π΅Ρ‚ΡŒ ΠΏΡ€ΠΎΠ²ΠΎΠ΄Π½ΠΈΠΊ Π² Ρ„ΠΎΡ€ΠΌΠ΅ ΡˆΠ°Ρ€Π°, Π² качСствС ΠΏΡ€ΠΈΠΌΠ΅Ρ€Π°, Ρ‚ΠΎ Ρ„ΠΎΡ€ΠΌΡƒΠ»Π° для расчСта Π΅Ρ‘ Π²Π΅Π»ΠΈΡ‡ΠΈΠ½Ρ‹ Π±ΡƒΠ΄Π΅Ρ‚ ΠΈΠΌΠ΅Ρ‚ΡŒ Π²ΠΈΠ΄:

Π­Ρ‚Π° Ρ„ΠΎΡ€ΠΌΡƒΠ»Π° справСдлива для ΡƒΠ΅Π΄ΠΈΠ½Π΅Π½Π½ΠΎΠ³ΠΎ ΠΏΡ€ΠΎΠ²ΠΎΠ΄Π½ΠΈΠΊΠ°. Если Ρ€Π°ΡΠΏΠΎΠ»ΠΎΠΆΠΈΡ‚ΡŒ рядом Π΄Π²Π° ΠΏΡ€ΠΎΠ²ΠΎΠ΄Π½ΠΈΠΊΠ° ΠΈ Ρ€Π°Π·Π΄Π΅Π»ΠΈΡ‚ΡŒ ΠΈΡ… диэлСктриком, Ρ‚ΠΎΠ³Π΄Π° получится кондСнсатор. Об этом Π½Π΅ΠΌΠ½ΠΎΠ³ΠΎ ΠΏΠΎΠ·ΠΆΠ΅, сСйчас Π΄Π°Π²Π°ΠΉΡ‚Π΅ разбСрСмся, Π² Ρ‡Π΅ΠΌ измСряСтся ΡΠ»Π΅ΠΊΡ‚Ρ€ΠΎΠ΅ΠΌΠΊΠΎΡΡ‚ΡŒ. (-12) Π€

ΠšΠΎΠ½Π΄Π΅Π½ΡΠ°Ρ‚ΠΎΡ€Ρ‹

ΠšΠΎΠ½Π΄Π΅Π½ΡΠ°Ρ‚ΠΎΡ€ β€” это Π΄Π²Π΅ пластины ΠΈΠ· проводящСго ΠΌΠ°Ρ‚Π΅Ρ€ΠΈΠ°Π»Π°, располоТСнныС Π΄Ρ€ΡƒΠ³ Π½Π°ΠΏΡ€ΠΎΡ‚ΠΈΠ² Π΄Ρ€ΡƒΠ³Π°, ΠΌΠ΅ΠΆΠ΄Ρƒ ΠΊΠΎΡ‚ΠΎΡ€Ρ‹ΠΌ находится слой диэлСктрика. Π’ заряТСнном состоянии ΠΎΠ±ΠΊΠ»Π°Π΄ΠΊΠΈ ΠΈΠΌΠ΅ΡŽΡ‚ Ρ€Π°Π·Π½Ρ‹Π΅ ΠΏΠΎΡ‚Π΅Π½Ρ†ΠΈΠ°Π»Ρ‹: ΠΎΠ΄Π½Π° ΠΈΠ· Π½ΠΈΡ… Π±ΡƒΠ΄Π΅Ρ‚ ΠΏΠΎΠ»ΠΎΠΆΠΈΡ‚Π΅Π»ΡŒΠ½ΠΎΠΉ, Π° вторая ΠΎΡ‚Ρ€ΠΈΡ†Π°Ρ‚Π΅Π»ΡŒΠ½ΠΎΠΉ. Π­Π»Π΅ΠΊΡ‚Ρ€ΠΎΠ΅ΠΌΠΊΠΎΡΡ‚ΡŒ кондСнсатора зависит ΠΎΡ‚ Π²Π΅Π»ΠΈΡ‡ΠΈΠ½Ρ‹ заряда Π½Π° Π΅Π³ΠΎ ΠΎΠ±ΠΊΠ»Π°Π΄ΠΊΠ°Ρ… ΠΈ разности ΠΏΠΎΡ‚Π΅Π½Ρ†ΠΈΠ°Π»ΠΎΠ², напряТСния ΠΌΠ΅ΠΆΠ΄Ρƒ Π½ΠΈΠΌΠΈ. ΠœΠ΅ΠΆΠ΄Ρƒ пластинами Π²ΠΎΠ·Π½ΠΈΠΊΠ°Π΅Ρ‚ элСктростатичСскоС ΠΏΠΎΠ»Π΅, ΠΊΠΎΡ‚ΠΎΡ€ΠΎΠ΅ ΡƒΠ΄Π΅Ρ€ΠΆΠΈΠ²Π°Π΅Ρ‚ заряды Π½Π° ΠΎΠ±ΠΊΠ»Π°Π΄ΠΊΠ°Ρ…. Π€ΠΎΡ€ΠΌΡƒΠ»Π° элСктричСской Смкости кондСнсатора Π² ΠΎΠ±Ρ‰Π΅ΠΌ случаС:

C=q/U

Если ΡΠΊΠ°Π·Π°Ρ‚ΡŒ простыми словами, Ρ‚ΠΎ Π΅ΠΌΠΊΠΎΡΡ‚ΡŒ кондСнсатора зависит ΠΎΡ‚ ΠΏΠ»ΠΎΡ‰Π°Π΄ΠΈ пластин ΠΈ расстояния ΠΌΠ΅ΠΆΠ΄Ρƒ Π½ΠΈΠΌΠΈ, Π° Ρ‚Π°ΠΊΠΆΠ΅ ΠΎΡ‚Π½ΠΎΡΠΈΡ‚Π΅Π»ΡŒΠ½ΠΎΠΉ диэлСктричСской проницаСмости ΠΌΠ°Ρ‚Π΅Ρ€ΠΈΠ°Π»Π°, располоТСнного ΠΌΠ΅ΠΆΠ΄Ρƒ Π½ΠΈΠΌΠΈ. Π˜Ρ… Ρ€Π°Π·Π»ΠΈΡ‡Π°ΡŽΡ‚ ΠΏΠΎ ΠΈΡΠΏΠΎΠ»ΡŒΠ·ΡƒΠ΅ΠΌΠΎΠΌΡƒ диэлСктрику:

  • кСрамичСскиС;
  • ΠΏΠ»Ρ‘Π½ΠΎΡ‡Π½Ρ‹Π΅;
  • ΡΠ»ΡŽΠ΄ΡΠ½Ρ‹Π΅;
  • ΠΌΠ΅Ρ‚Π°Π»Π»ΠΎΠ±ΡƒΠΌΠ°ΠΆΠ½Ρ‹Π΅;
  • элСктролитичСскиС;
  • Ρ‚Π°Π½Ρ‚Π°Π»ΠΎΠ²Ρ‹Π΅ ΠΈ ΠΏΡ€.

По Ρ„ΠΎΡ€ΠΌΠ΅ ΠΎΠ±ΠΊΠ»Π°Π΄ΠΎΠΊ:

  • плоскиС;
  • цилиндричСскиС;
  • сфСричСскиС ΠΈ ΠΏΡ€.

Π’Π°ΠΊ ΠΊΠ°ΠΊ Ρ„ΠΎΡ€ΠΌΡƒΠ»Π° ΠΏΠ»ΠΎΡ‰Π°Π΄ΠΈ Ρ„ΠΈΠ³ΡƒΡ€Ρ‹ зависит ΠΎΡ‚ Π΅Ρ‘ Ρ„ΠΎΡ€ΠΌΡ‹, Ρ‚ΠΎ ΠΈ Ρ„ΠΎΡ€ΠΌΡƒΠ»Π° ёмкости Π±ΡƒΠ΄Π΅Ρ‚ Ρ€Π°Π·Π½ΠΎΠΉ для ΠΊΠ°ΠΆΠ΄ΠΎΠ³ΠΎ случая.

Для плоского кондСнсатора:

Для Π΄Π²ΡƒΡ… концСнтричСских сфСр с ΠΎΠ±Ρ‰ΠΈΠΌ Ρ†Π΅Π½Ρ‚Ρ€ΠΎΠΌ:

Для цилиндричСского кондСнсатора:

Как ΠΈ Ρƒ Π΄Ρ€ΡƒΠ³ΠΈΡ… элСмСнтов элСктричСской Ρ†Π΅ΠΏΠΈ ΠΈ Π² этом случаС Π΅ΡΡ‚ΡŒ Π΄Π²Π° основных способа соСдинСния кондСнсаторов: ΠΏΠ°Ρ€Π°Π»Π»Π΅Π»ΡŒΠ½ΠΎΠ΅ ΠΈ ΠΏΠΎΡΠ»Π΅Π΄ΠΎΠ²Π°Ρ‚Π΅Π»ΡŒΠ½ΠΎΠ΅.

ΠžΡ‚ этого зависит итоговая элСктричСская Π΅ΠΌΠΊΠΎΡΡ‚ΡŒ ΠΏΠΎΠ»ΡƒΡ‡Π΅Π½Π½ΠΎΠΉ Ρ†Π΅ΠΏΠΈ. Расчёты ёмкости Π½Π΅ΡΠΊΠΎΠ»ΡŒΠΊΠΈΡ… кондСнсаторов Π½Π°ΠΏΠΎΠΌΠΈΠ½Π°ΡŽΡ‚ расчёты сопротивлСния рСзисторов Π² Ρ€Π°Π·Π½ΠΎΠΌ Π²ΠΊΠ»ΡŽΡ‡Π΅Π½ΠΈΠΈ, Ρ‚ΠΎΠ»ΡŒΠΊΠΎ Ρ„ΠΎΡ€ΠΌΡƒΠ»Ρ‹ для способов соСдинСния располоТСны Π½Π°ΠΎΠ±ΠΎΡ€ΠΎΡ‚, Ρ‚ΠΎ Π΅ΡΡ‚ΡŒ:

  1. ΠŸΡ€ΠΈ ΠΏΠ°Ρ€Π°Π»Π»Π΅Π»ΡŒΠ½ΠΎΠΌ соСдинСнии общая ΡΠ»Π΅ΠΊΡ‚Ρ€ΠΎΠ΅ΠΌΠΊΠΎΡΡ‚ΡŒ Ρ†Π΅ΠΏΠΈ являСтся суммой СмкостСй ΠΊΠ°ΠΆΠ΄ΠΎΠ³ΠΎ ΠΈΠ· элСмСнтов. ΠšΠ°ΠΆΠ΄Ρ‹ΠΉ ΡΠ»Π΅Π΄ΡƒΡŽΡ‰ΠΈΠΉ ΠΏΠΎΠ΄ΠΊΠ»ΡŽΡ‡Π΅Π½Π½Ρ‹ΠΉ ΡƒΠ²Π΅Π»ΠΈΡ‡ΠΈΠ²Π°Π΅Ρ‚ ΠΈΡ‚ΠΎΠ³ΠΎΠ²ΡƒΡŽ Π΅ΠΌΠΊΠΎΡΡ‚ΡŒ

CΠΎΠ±Ρ‰=C1+C2+C3

  1. ΠŸΡ€ΠΈ ΠΏΠΎΡΠ»Π΅Π΄ΠΎΠ²Π°Ρ‚Π΅Π»ΡŒΠ½ΠΎΠΌ ΠΏΠΎΠ΄ΠΊΠ»ΡŽΡ‡Π΅Π½ΠΈΠΈ ΡΠ»Π΅ΠΊΡ‚Ρ€ΠΎΠ΅ΠΌΠΊΠΎΡΡ‚ΡŒ Ρ†Π΅ΠΏΠΈ сниТаСтся, ΠΏΠΎΠ΄ΠΎΠ±Π½ΠΎ сниТСниС сопротивлСния Π² Ρ†Π΅ΠΏΠΈ ΠΏΠ°Ρ€Π°Π»Π»Π΅Π»ΡŒΠ½ΠΎ Π²ΠΊΠ»ΡŽΡ‡Ρ‘Π½Π½Ρ‹Ρ… рСзисторов. Π’ΠΎ Π΅ΡΡ‚ΡŒ:

CΠΎΠ±Ρ‰=(1/Π‘1)+ (1/Π‘2)+ (1/Π‘3)

Π’Π°ΠΆΠ½ΠΎ! Π’ ΠΏΠ°Ρ€Π°Π»Π»Π΅Π»ΡŒΠ½ΠΎΠΉ схСмС соСдинСния напряТСния Π½Π° ΠΎΠ±ΠΊΠ»Π°Π΄ΠΊΠ°Ρ… ΠΊΠ°ΠΆΠ΄ΠΎΠ³ΠΎ элСмСнта ΠΎΠ΄ΠΈΠ½Π°ΠΊΠΎΠ²Ρ‹. Π­Ρ‚ΠΎ ΠΈΡΠΏΠΎΠ»ΡŒΠ·ΡƒΡŽΡ‚ для получСния Π±ΠΎΠ»ΡŒΡˆΠΈΡ… Π·Π½Π°Ρ‡Π΅Π½ΠΈΠΉ элСктроСмкости. Π’ ΠΏΠΎΡΠ»Π΅Π΄ΠΎΠ²Π°Ρ‚Π΅Π»ΡŒΠ½ΠΎΠΌ Π²ΠΊΠ»ΡŽΡ‡Π΅Π½ΠΈΠΈ Π΄Π²ΡƒΡ… элСмСнтов напряТСния Π½Π° ΠΎΠ±ΠΊΠ»Π°Π΄ΠΊΠ°Ρ… ΠΊΠ°ΠΆΠ΄ΠΎΠ³ΠΎ ΠΈΠ· кондСнсаторов ΡΠΎΡΡ‚Π°Π²Π»ΡΡŽΡ‚ ΠΏΠΎ ΠΏΠΎΠ»ΠΎΠ²ΠΈΠ½Π΅ ΠΎΠ±Ρ‰Π΅Π³ΠΎ напряТСния. Для Ρ‚Ρ€Ρ‘Ρ… – Ρ‚Ρ€Π΅Ρ‚ΠΈ ΠΈ Ρ‚Π°ΠΊ Π΄Π°Π»Π΅Π΅.

Аккумуляторы ΠΈ ΡΠ»Π΅ΠΊΡ‚Ρ€ΠΎΠ΅ΠΌΠΊΠΎΡΡ‚ΡŒ

ΠžΡΠ½ΠΎΠ²Π½Ρ‹ΠΌΠΈ характСристиками аккумуляторных Π±Π°Ρ‚Π°Ρ€Π΅ΠΉ являСтся:

  • НоминальноС напряТСниС.
  • Π•ΠΌΠΊΠΎΡΡ‚ΡŒ.
  • ΠœΠ°ΠΊΡΠΈΠΌΠ°Π»ΡŒΠ½Ρ‹ΠΉ Ρ‚ΠΎΠΊ разряда.

Π’ Π΄Π°Π½Π½ΠΎΠΌ случаС для опрСдСлСния количСствСнной характСристики Π²Ρ€Π΅ΠΌΠ΅Π½ΠΈ Ρ€Π°Π±ΠΎΡ‚Ρ‹ ΠΈΠ»ΠΈ, говоря простым языком, Ρ‡Ρ‚ΠΎΠ±Ρ‹ Ρ€Π°ΡΡΡ‡ΠΈΡ‚Π°Ρ‚ΡŒ, Π½Π° ΠΊΠ°ΠΊΠΎΠ΅ врСмя Ρ€Π°Π±ΠΎΡ‚Ρ‹ ΠΏΡ€ΠΈΠ±ΠΎΡ€Π° Ρ…Π²Π°Ρ‚ΠΈΡ‚ аккумулятора, ΠΈΡΠΏΠΎΠ»ΡŒΠ·ΡƒΡŽΡ‚ Π²Π΅Π»ΠΈΡ‡ΠΈΠ½Ρƒ ёмкости.

Π’ аккумуляторных батарСях для описания элСктричСской ёмкости ΠΈΡΠΏΠΎΠ»ΡŒΠ·ΡƒΡŽΡ‚ ΡΠ»Π΅Π΄ΡƒΡŽΡ‰ΠΈΠ΅ размСрности:

  • А*Ρ‡ β€” Π°ΠΌΠΏΠ΅Ρ€-часы для Π±ΠΎΠ»ΡŒΡˆΠΈΡ… аккумуляторов, Π½Π°ΠΏΡ€ΠΈΠΌΠ΅Ρ€ Π°Π²Ρ‚ΠΎΠΌΠΎΠ±ΠΈΠ»ΡŒΠ½Ρ‹Ρ….
  • мА*Ρ‡ β€” ΠΌΠΈΠ»ΠΈΠ°ΠΌΠΏΠ΅Ρ€-часы, для аккумуляторов для носимых устройств, Π½Π°ΠΏΡ€ΠΈΠΌΠ΅Ρ€ смартфонов, ΠΊΠ²Π°Π΄Ρ€ΠΎΠΊΠΎΠΏΠ΅Ρ‚Ρ€ΠΎΠ² ΠΈ элСктронных сигарСт.
  • Π’Ρ‚*часы β€” Π²Π°Ρ‚Ρ‚-часы.

Π­Ρ‚ΠΈ характСристики ΠΏΠΎΠ·Π²ΠΎΠ»ΡΡŽΡ‚ ΠΎΠΏΡ€Π΅Π΄Π΅Π»ΠΈΡ‚ΡŒ, сколько Π²Ρ€Π΅ΠΌΠ΅Π½ΠΈ Ρ€Π°Π±ΠΎΡ‚Ρ‹ Π²Ρ‹Π΄Π΅Ρ€ΠΆΠΈΡ‚ аккумулятор ΠΏΡ€ΠΈ ΠΊΠΎΠ½ΠΊΡ€Π΅Ρ‚Π½ΠΎΠΉ Π½Π°Π³Ρ€ΡƒΠ·ΠΊΠ΅. Для опрСдСлСния ΡΠ»Π΅ΠΊΡ‚Ρ€ΠΈΡ‡Π΅ΡΠΊΡƒΡŽ Π΅ΠΌΠΊΠΎΡΡ‚ΡŒ аккумулятора ΠΈΠ·ΠΌΠ΅Ρ€ΡΡŽΡ‚ Π² ΠΊΡƒΠ»ΠΎΠ½Π°Ρ… (Кл). Π’ свою ΠΎΡ‡Π΅Ρ€Π΅Π΄ΡŒ ΠΊΡƒΠ»ΠΎΠ½ Ρ€Π°Π²Π΅Π½ количСству элСктричСства, ΠΏΠ΅Ρ€Π΅Π΄Π°Π½Π½ΠΎΠΌΡƒ аккумулятору ΠΏΡ€ΠΈ силС Ρ‚ΠΎΠΊΠ° 1А Π·Π° 1с. Π’ΠΎΠ³Π΄Π° Ссли пСрСвСсти Π² часы, Ρ‚ΠΎ ΠΏΡ€ΠΈ Ρ‚ΠΎΠΊΠ΅ Π² 1А Π·Π° 1 час пСрСдаСтся 3600 Кл.

Одним ΠΈΠ· способов измСрСния Смкости аккумулятора являСтся Π΅Π³ΠΎ разряд Π·Π°Π²Π΅Π΄ΠΎΠΌΠΎ извСстным Ρ‚ΠΎΠΊΠΎΠΌ, ΠΏΡ€ΠΈ этом Π²Ρ‹ Π΄ΠΎΠ»ΠΆΠ½Ρ‹ Π·Π°ΠΌΠ΅Ρ€ΠΈΡ‚ΡŒ врСмя разряда. Допустим, Ссли аккумулятор разрядился Π΄ΠΎ минимального уровня напряТСния Π·Π° 10 часов Ρ‚ΠΎΠΊΠΎΠΌ Π² 5А – Π·Π½Π°Ρ‡ΠΈΡ‚ Π΅Π³ΠΎ Π΅ΠΌΠΊΠΎΡΡ‚ΡŒ 50 А*Ρ‡

Π­Π»Π΅ΠΊΡ‚Ρ€ΠΎΠ΅ΠΌΠΊΠΎΡΡ‚ΡŒ – это ваТная Π²Π΅Π»ΠΈΡ‡ΠΈΠ½Π° Π² элСктроникС ΠΈ элСктротСхникС. На ΠΏΡ€Π°ΠΊΡ‚ΠΈΠΊΠ΅ кондСнсаторы ΠΏΡ€ΠΈΠΌΠ΅Π½ΡΡŽΡ‚ΡΡ практичСски Π² ΠΊΠ°ΠΆΠ΄ΠΎΠΉ схСмС элСктронного устройства. НапримСр, Π² Π±Π»ΠΎΠΊΠ°Ρ… питания – для сглаТивания ΠΏΡƒΠ»ΡŒΡΠ°Ρ†ΠΈΠΉ, ΡƒΠΌΠ΅Π½ΡŒΡˆΠ΅Π½ΠΈΡ влияния Π²Ρ‹ΡΠΎΠΊΠΎΠ²ΠΎΠ»ΡŒΡ‚Π½Ρ‹Ρ… всплСсков Π½Π° силовыС ΠΊΠ»ΡŽΡ‡ΠΈ. Π’ΠΎ Π²Ρ€Π΅ΠΌΡΠ·Π°Π΄Π°ΡŽΡ‰ΠΈΡ… цСпях Ρ€Π°Π·Π»ΠΈΡ‡Π½Ρ‹Ρ… схСм, Π° Ρ‚Π°ΠΊΠΆΠ΅ Π² ШИМ-ΠΊΠΎΠ½Ρ‚Ρ€ΠΎΠ»Π»Π΅Ρ€Π°Ρ… для Ρ‚ΠΎΠ³ΠΎ, Ρ‡Ρ‚ΠΎΠ±Ρ‹ Π·Π°Π΄Π°Ρ‚ΡŒ Ρ€Π°Π±ΠΎΡ‡ΡƒΡŽ частоту. Аккумуляторы Ρ‚Π°ΠΊΠΆΠ΅ ΠΏΡ€ΠΈΠΌΠ΅Π½ΡΡŽΡ‚ΡΡ повсСмСстно. Π’ΠΎΠΎΠ±Ρ‰Π΅ Π·Π°Π΄Π°Ρ‡ΠΈ накапливания энСргии ΠΈ сдвига Ρ„Π°Π· Π²ΡΡ‚Ρ€Π΅Ρ‡Π°ΡŽΡ‚ΡΡ ΠΎΡ‡Π΅Π½ΡŒ часто.

Π‘ΠΎΠ»Π΅Π΅ ΠΏΠΎΠ΄Ρ€ΠΎΠ±Π½ΠΎ ΠΈΠ·ΡƒΡ‡ΠΈΡ‚ΡŒ вопрос ΠΏΠΎΠΌΠΎΠΆΠ΅Ρ‚ прСдоставлСнноС Π²ΠΈΠ΄Π΅ΠΎ:

ΠšΡ€Π°Ρ‚ΠΊΠΎ объяснСниС ΠΈΠ·Π»ΠΎΠΆΠ΅Π½ΠΎ Π² этом Π²ΠΈΠ΄Π΅ΠΎ ΡƒΡ€ΠΎΠΊΠ΅:

Π’Π΅ΠΏΠ΅Ρ€ΡŒ Π²Ρ‹ Π·Π½Π°Π΅Ρ‚Π΅, Ρ‡Ρ‚ΠΎ Ρ‚Π°ΠΊΠΎΠ΅ элСктричСская Π΅ΠΌΠΊΠΎΡΡ‚ΡŒ, Π² ΠΊΠ°ΠΊΠΈΡ… Π΅Π΄ΠΈΠ½ΠΈΡ†Π°Ρ… происходит Π΅Π΅ ΠΈΠ·ΠΌΠ΅Ρ€Π΅Π½ΠΈΠ΅ ΠΈ ΠΎΡ‚ Ρ‡Π΅Π³ΠΎ зависит данная Π²Π΅Π»ΠΈΡ‡ΠΈΠ½Π°. НадССмся, прСдоставлСнная информация Π±Ρ‹Π»Π° для вас ΠΏΠΎΠ»Π΅Π·Π½ΠΎΠΉ ΠΈ понятной!

ΠœΠ°Ρ‚Π΅Ρ€ΠΈΠ°Π»Ρ‹ ΠΏΠΎ Ρ‚Π΅ΠΌΠ΅:

ВычислСниС Смкости кондСнсатора. Наука Ρ‚Π΅Ρ…Π½ΠΈΠΊΠ° Ρ‚Π΅Ρ…Π½ΠΎΠ»ΠΎΠ³ΠΈΠΈ


Плоский кондСнсатор – это физичСскоС ΡƒΠΏΡ€ΠΎΡ‰Π΅Π½ΠΈΠ΅, взявшСС Π½Π°Ρ‡Π°Π»ΠΎ ΠΈΠ· Ρ€Π°Π½Π½ΠΈΡ… исслСдований элСктричСства, ΠΏΡ€Π΅Π΄ΡΡ‚Π°Π²Π»ΡΡŽΡ‰Π΅Π΅ собой ΠΊΠΎΠ½ΡΡ‚Ρ€ΡƒΠΊΡ†ΠΈΡŽ, Π³Π΄Π΅ ΠΎΠ±ΠΊΠ»Π°Π΄ΠΊΠΈ ΠΈΠΌΠ΅ΡŽΡ‚ Ρ„ΠΎΡ€ΠΌΡƒ плоскостСй ΠΈ Π² ΠΊΠ°ΠΆΠ΄ΠΎΠΉ Ρ‚ΠΎΡ‡ΠΊΠ΅ ΠΏΠ°Ρ€Π°Π»Π»Π΅Π»ΡŒΠ½Ρ‹.

Π€ΠΎΡ€ΠΌΡƒΠ»Ρ‹

МногиС ΠΈΡ‰ΡƒΡ‚ Ρ„ΠΎΡ€ΠΌΡƒΠ»Ρ‹, ΠΎΠΏΠΈΡΡ‹Π²Π°ΡŽΡ‰ΠΈΠ΅ Ρ‘ΠΌΠΊΠΎΡΡ‚ΡŒ плоского кондСнсатора. Если это Ρ‚Π°ΠΊ, Ρ‚ΠΎ Π½Π΅ Ρ‡ΠΈΡ‚Π°ΠΉΡ‚Π΅ Π½ΠΈΠΆΠ΅ Π»ΡŽΠ±ΠΎΠΏΡ‹Ρ‚Π½Ρ‹Π΅ ΠΈ малоизвСстныС Ρ„Π°ΠΊΡ‚Ρ‹, ΠΏΠΎΡ‚ΠΎΠΌΡƒ Ρ‡Ρ‚ΠΎ сухиС матСматичСскиС Π·Π½Π°ΠΊΠΈ, ΠΊΠΎΠ½Π΅Ρ‡Π½ΠΎ ΠΆΠ΅, Π²Π°ΠΆΠ½Π΅Π΅.

ΠŸΠ΅Ρ€Π²Ρ‹ΠΌ ΠΎΠΏΡ€Π΅Π΄Π΅Π»ΠΈΠ» Ρ‘ΠΌΠΊΠΎΡΡ‚ΡŒ плоского кондСнсатора Π’ΠΎΠ»ΡŒΡ‚Π°. Π’ Π΅Π³ΠΎ распоряТСнии Π΅Ρ‰Ρ‘ Π½Π΅ Π±Ρ‹Π»ΠΎ Ρ‚Π°ΠΊΠΎΠΉ Π²Π΅Π»ΠΈΡ‡ΠΈΠ½Ρ‹, ΠΊΠ°ΠΊ Ρ€Π°Π·Π½ΠΈΡ†Π° ΠΏΠΎΡ‚Π΅Π½Ρ†ΠΈΠ°Π»ΠΎΠ², имСнуСмая напряТСниСм, Π½ΠΎ ΠΈΠ½Ρ‚ΡƒΠΈΡ‚ΠΈΠ²Π½ΠΎ ΠΎΠ½ ΡΠΎΠ²Π΅Ρ€ΡˆΠ΅Π½Π½ΠΎ ΠΏΡ€Π°Π²ΠΈΠ»ΡŒΠ½ΠΎ объяснил ΡΡƒΡ‚ΡŒ явлСния. Π§Ρ‚ΠΎ касаСтся количСства зарядов, Ρ‚ΠΎ ΠΎΠ½ Ρ‚Ρ€Π°ΠΊΡ‚ΠΎΠ²Π°Π» Π΅Ρ‘, ΠΊΠ°ΠΊ объСм элСктричСского Ρ„Π»ΡŽΠΈΠ΄Π° атмосфСры – Π½Π΅ совсСм ΠΏΡ€Π°Π²ΠΈΠ»ΡŒΠ½ΠΎ, Π½ΠΎ Π² ΠΊΠΎΠ½Π΅Ρ‡Π½ΠΎΠΌ ΠΈΡ‚ΠΎΠ³Π΅ ΠΏΠΎΡ…ΠΎΠΆΠ΅ Π½Π° ΠΏΡ€Π°Π²Π΄Ρƒ. Богласно этому ΠΌΠΈΡ€ΠΎΠ²ΠΎΠ·Π·Ρ€Π΅Π½ΠΈΡŽ Ρ‘ΠΌΠΊΠΎΡΡ‚ΡŒ плоского кондСнсатора ΠΌΠΎΠΆΠ΅Ρ‚ Π±Ρ‹Ρ‚ΡŒ Π½Π°ΠΉΠ΄Π΅Π½Π°, ΠΊΠ°ΠΊ ΠΎΡ‚Π½ΠΎΡˆΠ΅Π½ΠΈΠ΅ ΠΎΠ±ΡŠΡ‘ΠΌΠ° Π½Π°ΠΊΠΎΠΏΠ»Π΅Π½Π½ΠΎΠ³ΠΎ элСктричСского Ρ„Π»ΡŽΠΈΠ΄Π° ΠΊ Ρ€Π°Π·Π½ΠΈΡ†Π΅ атмосфСрных ΠΏΠΎΡ‚Π΅Π½Ρ†ΠΈΠ°Π»ΠΎΠ², Ρ‚ΠΎ Π΅ΡΡ‚ΡŒ:

Π­Ρ‚Π° Ρ„ΠΎΡ€ΠΌΡƒΠ»Π° ΠΏΡ€ΠΈΠΌΠ΅Π½ΠΈΠΌΠ° ΠΊ Π»ΡŽΠ±ΠΎΠΌΡƒ кондСнсатору, Π²Π½Π΅ зависимости ΠΎΡ‚ Π΅Π³ΠΎ конструкции. Π’ΠΎ Π΅ΡΡ‚ΡŒ, являСтся ΡƒΠ½ΠΈΠ²Π΅Ρ€ΡΠ°Π»ΡŒΠ½ΠΎΠΉ. Π‘ΠΏΠ΅Ρ†ΠΈΠ°Π»ΡŒΠ½ΠΎ для плоских кондСнсаторов имССтся Ρ„ΠΎΡ€ΠΌΡƒΠ»Π° ёмкости, выраТСнная Ρ‡Π΅Ρ€Π΅Π· свойства ΠΌΠ°Ρ‚Π΅Ρ€ΠΈΠ°Π»Π° диэлСктрика ΠΈ гСомСтричСскиС Ρ€Π°Π·ΠΌΠ΅Ρ€Ρ‹:

Π’ этой Ρ„ΠΎΡ€ΠΌΡƒΠ»Π΅ Ρ‡Π΅Ρ€Π΅Π· S ΠΎΠ±ΠΎΠ·Π½Π°Ρ‡Π΅Π½Π° ΠΏΠ»ΠΎΡ‰Π°Π΄ΡŒ ΠΎΠ±ΠΊΠ»Π°Π΄ΠΎΠΊ, вычисляСмая Ρ‡Π΅Ρ€Π΅Π· ΠΏΡ€ΠΎΠΈΠ·Π²Π΅Π΄Π΅Π½ΠΈΠ΅ сторон, Π° d – ΠΏΠΎΠΊΠ°Π·Ρ‹Π²Π°Π΅Ρ‚ расстояниС ΠΌΠ΅ΠΆΠ΄Ρƒ ΠΎΠ±ΠΊΠ»Π°Π΄ΠΊΠ°ΠΌΠΈ. ΠŸΡ€ΠΎΡ‡ΠΈΠ΅ символы – элСктричСская постоянная (8,854 ΠΏΠ€/ΠΌ) ΠΈ диэлСктричСская ΠΏΡ€ΠΎΠ½ΠΈΡ†Π°Π΅ΠΌΠΎΡΡ‚ΡŒ ΠΌΠ°Ρ‚Π΅Ρ€ΠΈΠ°Π»Π° диэлСктрика, Π΄Π° простит Π’ΠΎΡ‚ ΡΡ‚ΠΎΠ»ΡŒ ΠΎΡ‚ΠΊΡ€ΠΎΠ²Π΅Π½Π½Ρ‹Π΅ Ρ‚Π°Π²Ρ‚ΠΎΠ»ΠΎΠ³ΠΈΠΈ. ЭлСктролитичСскиС кондСнсаторы ΠΎΠ±Π»Π°Π΄Π°ΡŽΡ‚ ΡΡ‚ΠΎΠ»ΡŒ большой Ρ‘ΠΌΠΊΠΎΡΡ‚ΡŒΡŽ ΠΏΠΎ Ρ‚ΠΎΠΉ ΠΏΡ€ΠΈΡ‡ΠΈΠ½Π΅, Ρ‡Ρ‚ΠΎ проводящий раствор ΠΎΡ‚Π΄Π΅Π»Π΅Π½ ΠΎΡ‚ ΠΌΠ΅Ρ‚Π°Π»Π»Π° ΠΎΡ‡Π΅Π½ΡŒ Ρ‚ΠΎΠ½ΠΊΠΈΠΌ слоСм оксида. Π‘Π»Π΅Π΄ΠΎΠ²Π°Ρ‚Π΅Π»ΡŒΠ½ΠΎ, d Π² этом случаСт Π±ΡƒΠ΄Π΅Ρ‚ ΠΌΠΈΠ½ΠΈΠΌΠ°Π»ΡŒΠ½Ρ‹ΠΌ. ЕдинствСнный минус Π² Ρ‚ΠΎΠΌ, Ρ‡Ρ‚ΠΎ элСктролитичСскиС кондСнсаторы полярныС, ΠΈΡ… нСльзя ΠΏΠΎΠ΄ΠΊΠ»ΡŽΡ‡Π°Ρ‚ΡŒ Π² Ρ†Π΅ΠΏΠΈ ΠΏΠ΅Ρ€Π΅ΠΌΠ΅Π½Π½ΠΎΠ³ΠΎ Ρ‚ΠΎΠΊΠ°. Π‘ этой Ρ†Π΅Π»ΡŒΡŽ Π½Π° ΠΊΠ°ΠΆΠ΄ΠΎΠΌ Π°Π½ΠΎΠ΄ ΠΈΠ»ΠΈ ΠΊΠ°Ρ‚ΠΎΠ΄ ΠΎΠ±ΠΎΠ·Π½Π°Ρ‡Π΅Π½Ρ‹ Π·Π½Π°Ρ‡ΠΊΠ°ΠΌΠΈ плюса ΠΈΠ»ΠΈ минуса.

ПлоскиС кондСнсаторы сСгодня Ρ€Π΅Π΄ΠΊΠΎ Π²ΡΡ‚Ρ€Π΅Ρ‡Π°ΡŽΡ‚ΡΡ, ΠΈ это прСимущСствСнно ΠΏΠ»Ρ‘Π½ΠΎΡ‡Π½Ρ‹Π΅ микроскопичСскиС Ρ‚Π΅Ρ…Π½ΠΎΠ»ΠΎΠ³ΠΈΠΈ, Π³Π΄Π΅ Ρ‚Π°ΠΊΠΎΠΉ Ρ€ΠΎΠ΄ повСрхностСй являСтся Π΄ΠΎΠΌΠΈΠ½ΠΈΡ€ΡƒΡŽΡ‰ΠΈΠΌ. ВсС пассивныС ΠΈ Π°ΠΊΡ‚ΠΈΠ²Π½Ρ‹Π΅ элСмСнты ΠΎΠ±Ρ€Π°Π·ΡƒΡŽΡ‚ΡΡ Ρ‡Π΅Ρ€Π΅Π· Ρ‚Ρ€Π°Ρ„Π°Ρ€Π΅Ρ‚. И, ΡΠ»Π΅Π΄ΠΎΠ²Π°Ρ‚Π΅Π»ΡŒΠ½ΠΎ, ΠΈΠΌΠ΅ΡŽΡ‚ Π²ΠΈΠ΄ ΠΏΠ»Ρ‘Π½ΠΎΠΊ. ПлоскиС индуктивности, рСзисторы ΠΈ кондСнсаторы наносятся Π² Π²ΠΈΠ΄Π΅ токопроводящих паст.

ΠžΡ‚ ΠΌΠ°Ρ‚Π΅Ρ€ΠΈΠ°Π»Π° диэлСктрика Ρ‘ΠΌΠΊΠΎΡΡ‚ΡŒ зависит ΠΏΠΎ Ρ‚ΠΎΠΉ ΠΏΡ€ΠΈΡ‡ΠΈΠ½Π΅, Ρ‡Ρ‚ΠΎ Ρƒ ΠΊΠ°ΠΆΠ΄ΠΎΠ³ΠΎ ΠΈΠ· Π½ΠΈΡ… структура своя. БчитаСтся, Ρ‡Ρ‚ΠΎ Π°ΠΌΠΎΡ€Ρ„Π½ΠΎΠ΅ вСщСство состоит ΠΈΠ· Π½Π΅ΠΎΡ€ΠΈΠ΅Π½Ρ‚ΠΈΡ€ΠΎΠ²Π°Π½Π½Ρ‹Ρ… Π΄ΠΈΠΏΠΎΠ»Π΅ΠΉ, ΡƒΠΏΡ€ΡƒΠ³ΠΎ ΡƒΠΊΡ€Π΅ΠΏΠ»Ρ‘Π½Π½Ρ‹Ρ… Π½Π° своих мСстах. ΠŸΡ€ΠΈ ΠΏΡ€ΠΈΠ»ΠΎΠΆΠ΅Π½ΠΈΠΈ внСшнСго элСктричСского поля ΠΎΠ½ΠΈ ΠΎΠ±Ρ€Π°Ρ‚ΠΈΠΌΠΎ ΠΎΡ€ΠΈΠ΅Π½Ρ‚ΠΈΡ€ΡƒΡŽΡ‚ΡΡ вдоль силовых Π»ΠΈΠ½ΠΈΠΉ, ослабляя Π½Π°ΠΏΡ€ΡΠΆΡ‘Π½Π½ΠΎΡΡ‚ΡŒ. Π’ Ρ€Π΅Π·ΡƒΠ»ΡŒΡ‚Π°Ρ‚Π΅ заряд ΠΏΡ€ΠΎΠ΄ΠΎΠ»ΠΆΠ°Π΅Ρ‚ Π½Π°ΠΊΠ°ΠΏΠ»ΠΈΠ²Π°Ρ‚ΡŒΡΡ, ΠΏΠΎΠΊΠ° этот процСсс Π½Π΅ прСкратится. По ΠΌΠ΅Ρ€Π΅ Π²Ρ‹Ρ…ΠΎΠ΄Π° энСргии ΠΈΠ· ΠΎΠ±ΠΊΠ»Π°Π΄ΠΎΠΊ Π΄ΠΈΠΏΠΎΠ»ΠΈ Π²ΠΎΠ·Π²Ρ€Π°Ρ‰Π°ΡŽΡ‚ΡΡ Π½Π° свои мСста, дСлая Π²ΠΎΠ·ΠΌΠΎΠΆΠ½Ρ‹ΠΌ ΡΠ»Π΅Π΄ΡƒΡŽΡ‰ΠΈΠΉ Ρ€Π°Π±ΠΎΡ‡ΠΈΠΉ Ρ†ΠΈΠΊΠ». Π’Π°ΠΊ Ρ„ΡƒΠ½ΠΊΡ†ΠΈΠΎΠ½ΠΈΡ€ΡƒΠ΅Ρ‚ плоский элСктричСский кондСнсатор.

Из истории

Π˜ΡΡ‚ΠΎΡ€ΠΈΡ‡Π΅ΡΠΊΠΈ ΠΏΠ΅Ρ€Π²Ρ‹ΠΌ Π½Π°Ρ‡Π°Π» ΠΈΡΡΠ»Π΅Π΄ΠΎΠ²Π°Ρ‚ΡŒ Π½Π°ΠΊΠΎΠΏΠ»Π΅Π½ΠΈΠ΅ заряда Π²Π΅Π»ΠΈΠΊΠΈΠΉ АлСссандро Π’ΠΎΠ»ΡŒΡ‚Π°. Π’ Π΄ΠΎΠΊΠ»Π°Π΄Π΅ ΠšΠΎΡ€ΠΎΠ»Π΅Π²ΡΠΊΠΎΠΌΡƒ Π½Π°ΡƒΡ‡Π½ΠΎΠΌΡƒ общСству Π·Π° 1782 Π³ΠΎΠ΄ ΠΎΠ½ Π²ΠΏΠ΅Ρ€Π²Ρ‹Π΅ ΠΎΠ·Π²ΡƒΡ‡ΠΈΠ» слово кондСнсатор. Π’ ΠΏΠΎΠ½ΠΈΠΌΠ°Π½ΠΈΠΈ Π’ΠΎΠ»ΡŒΡ‚Ρ‹ элСктрофорус, ΠΏΡ€Π΅Π΄ΡΡ‚Π°Π²Π»ΡΡŽΡ‰ΠΈΠΉ собой Π΄Π²Π΅ ΠΏΠ°Ρ€Π°Π»Π»Π΅Π»ΡŒΠ½Ρ‹Π΅ ΠΎΠ±ΠΊΠ»Π°Π΄ΠΊΠΈ, Π²Ρ‹ΠΊΠ°Ρ‡ΠΈΠ²Π°Π» ΠΈΠ· эфира элСктричСский Ρ„Π»ΡŽΠΈΠ΄.

Π’ Ρ‚ΠΎ врСмя всС познания сводились ΠΊ Ρ‚ΠΎΠΌΡƒ, Ρ‡Ρ‚ΠΎ ΡƒΡ‡Ρ‘Π½Ρ‹Π΅ Π΄ΡƒΠΌΠ°Π»ΠΈ, Π±ΡƒΠ΄Ρ‚ΠΎ атмосфСра Π—Π΅ΠΌΠ»ΠΈ содСрТит Π² сСбС Π½Π΅Ρ‡Ρ‚ΠΎ, Ρ‡Ρ‚ΠΎ Π½Π΅ ΠΌΠΎΠΆΠ΅Ρ‚ Π±Ρ‹Ρ‚ΡŒ ΠΎΠΏΡ€Π΅Π΄Π΅Π»Π΅Π½ΠΎ ΠΏΡ€ΠΈΠ±ΠΎΡ€Π°ΠΌΠΈ. БущСствовали Ρ‚ΠΎΠ»ΡŒΠΊΠΎ ΠΏΡ€ΠΎΡΡ‚Π΅ΠΉΡˆΠΈΠ΅ элСктроскопы, способныС ΠΎΠΏΡ€Π΅Π΄Π΅Π»ΠΈΡ‚ΡŒ Π·Π½Π°ΠΊ заряда ΠΈ Π΅Π³ΠΎ Π½Π°Π»ΠΈΡ‡ΠΈΠ΅, Π½ΠΎ Π½Π΅ дававшиС прСдставлСния ΠΎ количСствС. Π£Ρ‡Ρ‘Π½Ρ‹Π΅ просто Π½Π°Ρ‚ΠΈΡ€Π°Π»ΠΈ ΠΌΠ΅Ρ…ΠΎΠΌ ΠΏΠΎΠ²Π΅Ρ€Ρ…Π½ΠΎΡΡ‚ΡŒ Ρ‚Π΅Π»Π° ΠΈ подносили Π΅Π³ΠΎ для исслСдования Π² ΠΎΠ±Π»Π°ΡΡ‚ΡŒ влияния ΠΏΡ€ΠΈΠ±ΠΎΡ€Π°. Π•Ρ‰Ρ‘ Π“ΠΈΠ»ΡŒΠ±Π΅Ρ€Ρ‚ ΠΏΠΎΠΊΠ°Π·Π°Π», Ρ‡Ρ‚ΠΎ элСктричСскиС ΠΈ ΠΌΠ°Π³Π½ΠΈΡ‚Π½Ρ‹Π΅ взаимодСйствия ΠΎΡΠ»Π°Π±Π΅Π²Π°ΡŽΡ‚ с расстояниСм. ΠŸΠΎΡΡ‚ΠΎΠΌΡƒ ΡƒΡ‡Ρ‘Π½Ρ‹Π΅ ΠΏΡ€ΠΈΠΌΠ΅Ρ€Π½ΠΎ Π·Π½Π°Π»ΠΈ, Ρ‡Ρ‚ΠΎ Π½ΡƒΠΆΠ½ΠΎ Π΄Π΅Π»Π°Ρ‚ΡŒ, Π½ΠΎ исслСдования Π½Π΅ ΠΏΡ€ΠΎΠ΄Π²ΠΈΠ³Π°Π»ΠΈΡΡŒ Π½ΠΈ Π½Π° ΠΉΠΎΡ‚Ρƒ.

Π“ΠΈΠΏΠΎΡ‚Π΅Π·Π° ΠΎΠ± атмосфСрном элСктричСствС высказана Π‘Π΅Π½Π΄ΠΆΠ°ΠΌΠΈΠ½ΠΎΠΌ Π€Ρ€Π°Π½ΠΊΠ»ΠΈΠ½ΠΎΠΌ. Он Π°ΠΊΡ‚ΠΈΠ²Π½ΠΎ исслСдовал ΠΌΠΎΠ»Π½ΠΈΠΈ ΠΈ ΠΏΡ€ΠΈΡˆΡ‘Π» ΠΊ Π²Ρ‹Π²ΠΎΠ΄Ρƒ, Ρ‡Ρ‚ΠΎ это проявлСния всС Ρ‚ΠΎΠΉ ΠΆΠ΅ Π΅Π΄ΠΈΠ½ΠΎΠΉ силы. Запуская Π²ΠΎΠ·Π΄ΡƒΡˆΠ½ΠΎΠ³ΠΎ змСя Π² Π½Π΅Π±ΠΎ, ΠΎΠ½ соСдинял Π΅Π³ΠΎ ΡˆΡ‘Π»ΠΊΠΎΠ²ΠΎΠΉ Π½ΠΈΡ‚ΡŒΡŽ с Π·Π΅ΠΌΠ»Ρ‘ΠΉ ΠΈ наблюдал Π΄ΡƒΠ³ΠΎΠ²ΠΎΠΉ разряд. Π­Ρ‚ΠΎ Π±Ρ‹Π»ΠΈ достаточно опасныС ΠΎΠΏΡ‹Ρ‚Ρ‹, ΠΈ Π‘Π΅Π½Π΄ΠΆΠ°ΠΌΠΈΠ½ ΠΌΠ½ΠΎΠ³ΠΎ Ρ€Π°Π· рисковал своСй Тизнью Ρ€Π°Π΄ΠΈ развития Π½Π°ΡƒΠΊΠΈ. О Ρ‚ΠΎΠΌ, Ρ‡Ρ‚ΠΎ ΡˆΡ‘Π»ΠΊΠΎΠ²Π°Ρ Π½ΠΈΡ‚ΡŒ ΠΏΡ€ΠΎΠ²ΠΎΠ΄ΠΈΡ‚ статичСский заряд, Π±Ρ‹Π»ΠΎ извСстно ΠΎΡ‚ Π‘Ρ‚ΠΈΠ²Π΅Π½Π° ГрСя, ΠΏΠ΅Ρ€Π²Ρ‹ΠΌ ΡΠΎΠ±Ρ€Π°Π²ΡˆΠ΅Π³ΠΎ Π² 1732 Π³ΠΎΠ΄Ρƒ ΡΠ»Π΅ΠΊΡ‚Ρ€ΠΈΡ‡Π΅ΡΠΊΡƒΡŽ Ρ†Π΅ΠΏΡŒ.

Π£ΠΆΠ΅ Ρ‡Π΅Ρ€Π΅Π· 20 Π»Π΅Ρ‚ (1752 Π³ΠΎΠ΄) Π‘Π΅Π½Π΄ΠΆΠ°ΠΌΠΈΠ½ Π€Ρ€Π°Π½ΠΊΠ»ΠΈΠ½ ΠΏΡ€Π΅Π΄Π»ΠΎΠΆΠΈΠ» ΠΊΠΎΠ½ΡΡ‚Ρ€ΡƒΠΊΡ†ΠΈΡŽ ΠΏΠ΅Ρ€Π²ΠΎΠ³ΠΎ Π³Ρ€ΠΎΠΌΠΎΠΎΡ‚Π²ΠΎΠ΄Π°, ΠΎΡΡƒΡ‰Π΅ΡΡ‚Π²Π»ΡΠ²ΡˆΠ΅Π³ΠΎ ΠΌΠΎΠ»Π½ΠΈΠ΅Π·Π°Ρ‰ΠΈΡ‚Ρ‹ Π±Π»ΠΈΠ·Π»Π΅ΠΆΠ°Ρ‰ΠΈΡ… построСк. Волько Π²Π΄ΡƒΠΌΠ°Ρ‚ΡŒΡΡ! – Π΄ΠΎ этого всякий ΠΌΠΎΠ³ ΠΎΠΆΠΈΠ΄Π°Ρ‚ΡŒ Ρ‚ΠΎΠ³ΠΎ, Ρ‡Ρ‚ΠΎ Π΅Π³ΠΎ Π΄ΠΎΠΌ сгорит ΠΎΡ‚ случайного ΡƒΠ΄Π°Ρ€Π°. ИмСнно Π‘Π΅Π½Π΄ΠΆΠ°ΠΌΠΈΠ½ Π€Ρ€Π°Π½ΠΊΠ»ΠΈΠ½ ΠΏΡ€Π΅Π΄Π»ΠΎΠΆΠΈΠ» ΠΎΠ΄ΠΈΠ½ ΠΈΠ· Π²ΠΈΠ΄ΠΎΠ² заряда Π½Π°Π·Ρ‹Π²Π°Ρ‚ΡŒ ΠΏΠΎΠ»ΠΎΠΆΠΈΡ‚Π΅Π»ΡŒΠ½Ρ‹ΠΌ (стСклянный), Π° Π΄Ρ€ΡƒΠ³ΠΎΠΉ ΠΎΡ‚Ρ€ΠΈΡ†Π°Ρ‚Π΅Π»ΡŒΠ½Ρ‹ΠΌ (смоляной). Π’Π°ΠΊ Ρ„ΠΈΠ·ΠΈΠΊΠΈ Π±Ρ‹Π»ΠΈ Π²Π²Π΅Π΄Π΅Π½Ρ‹ Π² Π·Π°Π±Π»ΡƒΠΆΠ΄Π΅Π½ΠΈΠ΅ ΠΎΡ‚Π½ΠΎΡΠΈΡ‚Π΅Π»ΡŒΠ½ΠΎ истинного направлСния двиТСния элСктронов. Но ΠΊΠ°ΠΊ ΠΎΠ½ΠΈ ΠΌΠΎΠ³Π»ΠΈ Π΄ΡƒΠΌΠ°Ρ‚ΡŒ ΠΈΠ½Π°Ρ‡Π΅, ΠΊΠΎΠ³Π΄Π° Π² 1802 Π³ΠΎΠ΄Ρƒ Π½Π° ΠΏΡ€ΠΈΠΌΠ΅Ρ€Π΅ ΠΎΠΏΡ‹Ρ‚ΠΎΠ² нашСго соотСчСствСнника ΠŸΠ΅Ρ‚Ρ€ΠΎΠ²Π° ΡƒΠ²ΠΈΠ΄Π΅Π»ΠΈ, Ρ‡Ρ‚ΠΎ Π½Π° Π°Π½ΠΎΠ΄Π΅ образуСтся ямка? Π‘Π»Π΅Π΄ΠΎΠ²Π°Ρ‚Π΅Π»ΡŒΠ½ΠΎ, ΠΏΠΎΠ»ΠΎΠΆΠΈΡ‚Π΅Π»ΡŒΠ½Ρ‹Π΅ частицы пСрСносили заряд Π½Π° ΠΊΠ°Ρ‚ΠΎΠ΄, Π²ΠΎΡ‚ Ρ‚ΠΎΠ»ΡŒΠΊΠΎ это Π±Ρ‹Π»ΠΈ ΠΈΠΎΠ½Ρ‹ Π²ΠΎΠ·Π΄ΡƒΡˆΠ½ΠΎΠΉ ΠΏΠ»Π°Π·ΠΌΡ‹.

К Π½Π°Ρ‡Π°Π»Ρƒ исслСдования Π’ΠΎΠ»ΡŒΡ‚ΠΎΠΉ элСктричСских явлСний, Ρ‚Π°ΠΊΠΈΠΌ ΠΎΠ±Ρ€Π°Π·ΠΎΠΌ, Π±Ρ‹Π»ΠΈ ΡƒΠΆΠ΅ извСстны статичСскиС заряды ΠΈ Ρ„Π°ΠΊΡ‚ наличия Ρƒ Π½ΠΈΡ… Π΄Π²ΡƒΡ… Π·Π½Π°ΠΊΠΎΠ², ΠΊΡ€ΠΎΠΌΠ΅ Ρ‚ΠΎΠ³ΠΎ люди ΡƒΠΏΠΎΡ€Π½ΠΎ считали, Ρ‡Ρ‚ΠΎ вСсь Β«Ρ„Π»ΡŽΠΈΠ΄Β» бСрётся ΠΈΠ· Π²ΠΎΠ·Π΄ΡƒΡ…Π°. На эту ΠΌΡ‹ΡΠ»ΡŒ ΠΈΡ… Π½Π°Ρ‚ΠΎΠ»ΠΊΠ½ΡƒΠ»ΠΈ ΠΎΠΏΡ‹Ρ‚Ρ‹ с Π½Π°Ρ‚ΠΈΡ€Π°Π½ΠΈΠ΅ΠΌ янтаря ΡˆΠ΅Ρ€ΡΡ‚ΡŒΡŽ, ΠΊΠΎΡ‚ΠΎΡ€Ρ‹Π΅ Π½Π΅ ΠΌΠΎΠ³Π»ΠΈ Π±Ρ‹Ρ‚ΡŒ ΠΏΡ€ΠΎΠ²Π΅Π΄Π΅Π½Ρ‹ ΠΏΠΎΠ΄ Π²ΠΎΠ΄ΠΎΠΉ. Π‘Π»Π΅Π΄ΠΎΠ²Π°Ρ‚Π΅Π»ΡŒΠ½ΠΎ, Π»ΠΎΠ³ΠΈΡ‡Π½Ρ‹ΠΌ Π±Ρ‹Π»ΠΎ ΠΏΡ€Π΅Π΄ΠΏΠΎΠ»ΠΎΠΆΠΈΡ‚ΡŒ, Ρ‡Ρ‚ΠΎ элСктричСство ΠΌΠΎΠΆΠ΅Ρ‚ ΠΏΡ€ΠΎΠΈΡΡ…ΠΎΠ΄ΠΈΡ‚ΡŒ Ρ‚ΠΎΠ»ΡŒΠΊΠΎ ΠΈΠ· атмосфСры Π—Π΅ΠΌΠ»ΠΈ, Ρ‡Ρ‚ΠΎ, ΠΊΠΎΠ½Π΅Ρ‡Π½ΠΎ ΠΆΠ΅, ΡΠΎΠ²Π΅Ρ€ΡˆΠ΅Π½Π½ΠΎ Π½Π΅Π²Π΅Ρ€Π½ΠΎ. Π’ частности, ΠΌΠ½ΠΎΠ³ΠΈΠ΅ растворы, исслСдованныС Π₯Π°ΠΌΠΏΡ„Ρ€ΠΈ Дэви, ΠΌΠΎΠ³ΡƒΡ‚ ΠΏΡ€ΠΎΠ²ΠΎΠ΄ΠΈΡ‚ΡŒ элСктричСский Ρ‚ΠΎΠΊ.

ΠŸΡ€ΠΈΡ‡ΠΈΠ½Π°, ΡΠ»Π΅Π΄ΠΎΠ²Π°Ρ‚Π΅Π»ΡŒΠ½ΠΎ, Π±Ρ‹Π»Π° Π² Π΄Ρ€ΡƒΠ³ΠΎΠΌ – ΠΏΡ€ΠΈ Π½Π°Ρ‚ΠΈΡ€Π°Π½ΠΈΠΈ янтаря ΠΏΠΎΠ΄ Π²ΠΎΠ΄ΠΎΠΉ силы трСния сниТались Π² дСсятки ΠΈ сотни Ρ€Π°Π·, Π° заряд рассСивался ΠΏΠΎ всСму ΠΎΠ±ΡŠΡ‘ΠΌΡƒ Тидкости. Π‘Π»Π΅Π΄ΠΎΠ²Π°Ρ‚Π΅Π»ΡŒΠ½ΠΎ, этот процСсс Π±Ρ‹Π» всСго лишь нСэффСктивным. Но сСгодня ΠΊΠ°ΠΆΠ΄Ρ‹ΠΉ Π΄ΠΎΠ±Ρ‹Ρ‚Ρ‡ΠΈΠΊ Π·Π½Π°Π΅Ρ‚, Ρ‡Ρ‚ΠΎ Π½Π΅Ρ„Ρ‚ΡŒ прСкрасно элСктризуСтся Ρ‚Ρ€Π΅Π½ΠΈΠ΅ΠΌ ΠΎ Ρ‚Ρ€ΡƒΠ±Ρ‹ ΠΈ Π±Π΅Π· Π²ΠΎΠ·Π΄ΡƒΡ…Π°. Π‘Π»Π΅Π΄ΠΎΠ²Π°Ρ‚Π΅Π»ΡŒΠ½ΠΎ, атмосфСра для Β«Ρ„Π»ΡŽΠΈΠ΄Π°Β» Π½Π΅ являСтся ΠΎΠ±ΡΠ·Π°Ρ‚Π΅Π»ΡŒΠ½Ρ‹ΠΌ ΠΊΠΎΠΌΠΏΠΎΠ½Π΅Π½Ρ‚ΠΎΠΌ.

Π‘Π°ΠΌΡ‹ΠΉ большой Π² ΠΌΠΈΡ€Π΅ плоский кондСнсатор

Π‘Ρ‚ΠΎΠ»ΡŒ систСматизированныС, Π½ΠΎ Π² ΠΊΠΎΡ€Π½Π΅ Π½Π΅Π²Π΅Ρ€Π½Ρ‹Π΅ толкования всС-Ρ‚Π°ΠΊΠΈ Π½Π΅ смогли ΠΎΡΡ‚Π°Π½ΠΎΠ²ΠΈΡ‚ΡŒ Π’ΠΎΠ»ΡŒΡ‚Ρƒ Π½Π° Π΅Π³ΠΎ ΠΈΡΡΠ»Π΅Π΄ΠΎΠ²Π°Ρ‚Π΅Π»ΡŒΡΠΊΠΎΠΌ ΠΏΡƒΡ‚ΠΈ. Он ΡƒΠΏΠΎΡ€Π½ΠΎ ΠΈΠ·ΡƒΡ‡Π°Π» элСктрофорус, ΠΊΠ°ΠΊ ΠΎΠ΄ΠΈΠ½ ΠΈΠ· самых ΡΠΎΠ²Π΅Ρ€ΡˆΠ΅Π½Π½Ρ‹Ρ… Π³Π΅Π½Π΅Ρ€Π°Ρ‚ΠΎΡ€ΠΎΠ², ΡΡƒΡ‰Π΅ΡΡ‚Π²ΠΎΠ²Π°Π²ΡˆΠΈΡ… Π² Ρ‚ΠΎ врСмя. Π’Ρ‚ΠΎΡ€Ρ‹ΠΌ Π±Ρ‹Π» сСрный ΡˆΠ°Ρ€ ΠžΡ‚Ρ‚ΠΎ Ρ„ΠΎΠ½ Π“Π΅Ρ€ΠΈΠΊΠ΅, ΠΈΠ·ΠΎΠ±Ρ€Π΅Ρ‚Ρ‘Π½Π½Ρ‹ΠΉ Π±ΠΎΠ»Π΅Π΅ Ρ‡Π΅ΠΌ Π·Π° Π²Π΅ΠΊ Π΄ΠΎ этого (1663 Π³ΠΎΠ΄). Π‘ Ρ‚Π΅Ρ… самых ΠΏΠΎΡ€ Π΅Π³ΠΎ конструкция ΠΌΠ°Π»ΠΎ мСнялась, Π½ΠΎ послС ΠΎΡ‚ΠΊΡ€Ρ‹Ρ‚ΠΈΠΉ Π‘Ρ‚ΠΈΠ²Π΅Π½Π° ГрСя заряд Π½Π°Ρ‡Π°Π»ΠΈ ΡΠ½ΠΈΠΌΠ°Ρ‚ΡŒ ΠΏΡ€ΠΈ ΠΏΠΎΠΌΠΎΡ‰ΠΈ ΠΏΡ€ΠΎΠ²ΠΎΠ΄Π½ΠΈΠΊΠΎΠ². Π’ частности, Π² для этого слуТат мСталличСскиС Π³Ρ€Π΅Π±Ρ‘Π½ΠΊΠΈ-Π½Π΅ΠΉΡ‚Ρ€Π°Π»ΠΈΠ·Π°Ρ‚ΠΎΡ€Ρ‹.

Π”ΠΎΠ»Π³ΠΎΠ΅ врСмя ΡƒΡ‡Ρ‘Π½Ρ‹Π΅ Ρ…ΠΎΠ΄ΠΈΠ»ΠΈ Π²ΠΎΠΊΡ€ΡƒΠ³ Π΄Π° ΠΎΠΊΠΎΠ»ΠΎ. ЭлСктрофорная машина 1880 Π³ΠΎΠ΄Π° ΠΌΠΎΠΆΠ΅Ρ‚ ΡΡ‡ΠΈΡ‚Π°Ρ‚ΡŒΡΡ ΠΏΠ΅Ρ€Π²Ρ‹ΠΌ ΠΌΠΎΡ‰Π½Ρ‹ΠΌ Π³Π΅Π½Π΅Ρ€Π°Ρ‚ΠΎΡ€ΠΎΠΌ разряда, позволявшим ΠΏΠΎΠ»ΡƒΡ‡ΠΈΡ‚ΡŒ Π΄ΡƒΠ³Ρƒ, Π½ΠΎ своСй настоящСй силы элСктроны достигли Π² Π³Π΅Π½Π΅Ρ€Π°Ρ‚ΠΎΡ€Π΅ Π’Π°Π½ Π΄Π΅ Π“Ρ€Π°Π°Ρ„Π° (1929 Π³ΠΎΠ΄), Π³Π΄Π΅ Ρ€Π°Π·Π½ΠΈΡ†Π° ΠΏΠΎΡ‚Π΅Π½Ρ†ΠΈΠ°Π»ΠΎΠ² составила Π΅Π΄ΠΈΠ½ΠΈΡ†Ρ‹ ΠΌΠ΅Π³Π°Π²ΠΎΠ»ΡŒΡ‚Π°. Для сравнСния Π³Ρ€ΠΎΠ·ΠΎΠ²ΠΎΠ΅ ΠΎΠ±Π»Π°ΠΊΠΎ, согласно Π΄Π°Π½Π½Ρ‹ΠΌ Π’ΠΈΠΊΠΈΠΏΠ΅Π΄ΠΈΠΈ, ΠΌΠΎΠΆΠ΅Ρ‚ ΠΈΠΌΠ΅Ρ‚ΡŒ ΠΏΠΎΡ‚Π΅Π½Ρ†ΠΈΠ°Π» ΠΎΡ‚Π½ΠΎΡΠΈΡ‚Π΅Π»ΡŒΠ½ΠΎ Π—Π΅ΠΌΠ»ΠΈ Π² Π΅Π΄ΠΈΠ½ΠΈΡ†Ρ‹ Π³ΠΈΠ³Π°Π²ΠΎΠ»ΡŒΡ‚ (Π½Π° Ρ‚Ρ€ΠΈ порядка большС, Ρ‡Π΅ΠΌ Π² чСловСчСской машинС).

Буммируя сказанноС, ΠΌΠΎΠΆΠ½ΠΎ с ΠΎΠΏΡ€Π΅Π΄Π΅Π»Ρ‘Π½Π½ΠΎΠΉ Π΄ΠΎΠ»Π΅ΠΉ увСрСнности ΡΠΊΠ°Π·Π°Ρ‚ΡŒ, Ρ‡Ρ‚ΠΎ ΠΏΡ€ΠΈΡ€ΠΎΠ΄Π½Ρ‹Π΅ процСссы ΠΈΡΠΏΠΎΠ»ΡŒΠ·ΡƒΡŽΡ‚ Π² качСствС ΠΏΡ€ΠΈΠ½Ρ†ΠΈΠΏΠ° своСго дСйствия ΡΠ»Π΅ΠΊΡ‚Ρ€ΠΈΠ·Π°Ρ†ΠΈΡŽ Ρ‚Ρ€Π΅Π½ΠΈΠ΅ΠΌ, влияниСм ΠΈ Π½Π΅ΠΊΠΎΡ‚ΠΎΡ€Ρ‹Π΅ Π΄Ρ€ΡƒΠ³ΠΈΠ΅ Π΅Ρ‘ Π²ΠΈΠ΄Ρ‹, Π° ΠΌΠΎΡ‰Π½Ρ‹ΠΉ Ρ†ΠΈΠΊΠ»ΠΎΠ½ являСтся самым большим ΠΈΠ· извСстных Π½Π°ΠΌ плоских кондСнсаторов. Молния ΠΏΠΎΠΊΠ°Π·Ρ‹Π²Π°Π΅Ρ‚, Ρ‡Ρ‚ΠΎ Π±Ρ‹Π²Π°Π΅Ρ‚, ΠΊΠΎΠ³Π΄Π° диэлСктрик (атмосфСра) Π½Π΅ Π²Ρ‹Π΄Π΅Ρ€ΠΆΠΈΠ²Π°Π΅Ρ‚ ΠΏΡ€ΠΈΠ»ΠΎΠΆΠ΅Π½Π½ΠΎΠΉ Ρ€Π°Π·Π½ΠΈΡ†Ρ‹ ΠΏΠΎΡ‚Π΅Π½Ρ†ΠΈΠ°Π»ΠΎΠ² ΠΈ пробиваСтся. Π’ точности Ρ‚ΠΎ ΠΆΠ΅ самоС происходит Π² любом плоском кондСнсаторС, созданном Ρ‡Π΅Π»ΠΎΠ²Π΅ΠΊΠΎΠΌ, Ссли Π²ΠΎΠ»ΡŒΡ‚Π°ΠΆ оказываСтся для Π½Π΅Π³ΠΎ Π½Π΅ΠΏΠΎΠΌΠ΅Ρ€Π½Ρ‹ΠΌ. ΠŸΡ€ΠΎΠ±ΠΎΠΉ Ρ‚Π²Ρ‘Ρ€Π΄ΠΎΠ³ΠΎ диэлСктрика Π½Π΅ΠΎΠ±Ρ€Π°Ρ‚ΠΈΠΌ, Π° Π²ΠΎΠ·Π½ΠΈΠΊΠ°ΡŽΡ‰Π°Ρ элСктричСская Π΄ΡƒΠ³Π° часто слуТит ΠΏΡ€ΠΈΡ‡ΠΈΠ½ΠΎΠΉ расплавлСния ΠΎΠ±ΠΊΠ»Π°Π΄ΠΎΠΊ ΠΈ Π²Ρ‹Ρ…ΠΎΠ΄Π° издСлия ΠΈΠ· строя.

ЭлСктрофорус

Π˜Ρ‚Π°ΠΊ, Π’ΠΎΠ»ΡŒΡ‚Π° взялся Π·Π° исслСдованиС ΠΌΠΎΠ΄Π΅Π»ΠΈ ΠΏΡ€ΠΈΡ€ΠΎΠ΄Π½Ρ‹Ρ… процСссов. ΠŸΠ΅Ρ€Π²Ρ‹ΠΉ элСктрофорус появился Π² 1762 Π³ΠΎΠ΄Ρƒ сконструированный Π™ΠΎΡ…Π°Π½ΠΎΠΌ ΠšΠ°Ρ€Π»ΠΎΠΌ Π’ΠΈΠ»ΡŒΠΊΠ΅. По-настоящСму популярным ΠΏΡ€ΠΈΠ±ΠΎΡ€ становится послС Π΄ΠΎΠΊΠ»Π°Π΄ΠΎΠ² Π’ΠΎΠ»ΡŒΡ‚Ρ‹ ΠšΠΎΡ€ΠΎΠ»Π΅Π²ΡΠΊΠΎΠΌΡƒ Π½Π°ΡƒΡ‡Π½ΠΎΠΌΡƒ общСству (сСрСдина 70-Ρ… Π³ΠΎΠ΄ΠΎΠ² XVIII Π²Π΅ΠΊΠ°). Π’ΠΎΠ»ΡŒΡ‚Π° ΠΆΠ΅ ΠΈ Π΄Π°Π» ΠΏΡ€ΠΈΠ±ΠΎΡ€Ρƒ Π΅Π³ΠΎ Π½Ρ‹Π½Π΅ΡˆΠ½Π΅Π΅ Π½Π°Π·Π²Π°Π½ΠΈΠ΅.

ЭлСктрофорус способСн Π½Π°ΠΊΠ°ΠΏΠ»ΠΈΠ²Π°Ρ‚ΡŒ элСктростатичСский заряд, ΠΎΠ±Ρ€Π°Π·ΠΎΠ²Π°Π½Π½Ρ‹ΠΉ Ρ‚Ρ€Π΅Π½ΠΈΠ΅ΠΌ Ρ€Π΅Π·ΠΈΠ½Ρ‹ куском ΡˆΠ΅Ρ€ΡΡ‚ΠΈ. Он состоит ΠΈΠ· Π΄Π²ΡƒΡ… плоских, ΠΏΠ°Ρ€Π°Π»Π»Π΅Π»ΡŒΠ½Ρ‹Ρ… Π΄Ρ€ΡƒΠ³ Π΄Ρ€ΡƒΠ³Ρƒ ΠΎΠ±ΠΊΠ»Π°Π΄ΠΎΠΊ:

  • НиТняя прСдставляСт собой Ρ‚ΠΎΠ½ΠΊΠΈΠΉ кусок Ρ€Π΅Π·ΠΈΠ½Ρ‹. Π’ΠΎΠ»Ρ‰ΠΈΠ½Π° Π΅Π³ΠΎ выбираСтся ΠΈΠ· сообраТСний эффСктивности устройства. Если Π²Ρ‹Π±Ρ€Π°Ρ‚ΡŒ кусок Π±ΠΎΠ»Π΅Π΅ солидный, Ρ‚ΠΎ Π·Π½Π°Ρ‡ΠΈΡ‚Π΅Π»ΡŒΠ½Π°Ρ Ρ‡Π°ΡΡ‚ΡŒ энСргии Π±ΡƒΠ΄Π΅Ρ‚ Π½Π°ΠΊΠ°ΠΏΠ»ΠΈΠ²Π°Ρ‚ΡŒΡΡ Π²Π½ΡƒΡ‚Ρ€ΠΈ диэлСктрика Π½Π° ΠΎΡ€ΠΈΠ΅Π½Ρ‚Π°Ρ†ΠΈΡŽ Π΅Π³ΠΎ ΠΌΠΎΠ»Π΅ΠΊΡƒΠ». Π§Ρ‚ΠΎ ΠΈ Π½Π°Π±Π»ΡŽΠ΄Π°Π΅Ρ‚ΡΡ Π² соврСмСнном плоском кондСнсаторС, ΠΊΡƒΠ΄Π° диэлСктрик помСщаСтся для увСличСния элСктроёмкости.
  • ВСрхняя пластина ΠΈΠ· Ρ‚ΠΎΠ½ΠΊΠΎΠΉ стали кладётся свСрху, ΠΊΠΎΠ³Π΄Π° заряд ΡƒΠΆΠ΅ Π½Π°ΠΊΠΎΠΏΠ»Π΅Π½ Ρ‚Ρ€Π΅Π½ΠΈΠ΅ΠΌ. Π—Π° счёт влияния Π½Π° Π²Π΅Ρ€Ρ…Π½Π΅ΠΉ повСрхности образуСтся ΠΈΠ·Π±Ρ‹Ρ‚ΠΎΠΊ ΠΎΡ‚Ρ€ΠΈΡ†Π°Ρ‚Π΅Π»ΡŒΠ½ΠΎΠ³ΠΎ заряда, ΠΈ ΠΎΠ½ Π΄ΠΎΠ»ΠΆΠ΅Π½ Π±Ρ‹Ρ‚ΡŒ снят Π½Π° Π·Π°Π·Π΅ΠΌΠ»ΠΈΡ‚Π΅Π»ΡŒ, Ρ‡Ρ‚ΠΎΠ±Ρ‹ ΠΏΡ€ΠΈ расстыковкС Π΄Π²ΡƒΡ… ΠΎΠ±ΠΊΠ»Π°Π΄ΠΎΠΊ Π½Π΅ ΠΏΡ€ΠΎΠΈΠ·ΠΎΡˆΠ»ΠΎ Π²Π·Π°ΠΈΠΌΠ½ΠΎΠΉ компСнсации.

ΠŸΡ€ΠΈΠ½Ρ†ΠΈΠΏ дСйствия этого плоского кондСнсатора Π΄ΠΎΠ»ΠΆΠ΅Π½ Π±Ρ‹Ρ‚ΡŒ ΡƒΠΆΠ΅ понятСн. ΠžΠΏΠ΅Ρ€Π°Ρ‚ΠΎΡ€ Ρ‚Ρ€Ρ‘Ρ‚ Ρ€Π΅Π·ΠΈΠ½Ρƒ ΡˆΠ΅Ρ€ΡΡ‚ΡŒΡŽ, оставляя Π½Π° Π½Π΅ΠΉ ΠΎΡ‚Ρ€ΠΈΡ†Π°Ρ‚Π΅Π»ΡŒΠ½Ρ‹ΠΉ заряд. Π—Π°Ρ‚Π΅ΠΌ свСрху кладётся кусок ΠΌΠ΅Ρ‚Π°Π»Π»Π°. Из-Π·Π° Π·Π½Π°Ρ‡ΠΈΡ‚Π΅Π»ΡŒΠ½ΠΎΠΉ ΡˆΠ΅Ρ€ΠΎΡ…ΠΎΠ²Π°Ρ‚ΠΎΡΡ‚ΠΈ повСрхностСй ΠΎΠ½ΠΈ Π½Π΅ ΡΠΎΠΏΡ€ΠΈΠΊΠ°ΡΠ°ΡŽΡ‚ΡΡ, Π½ΠΎ находятся Π½Π° Π½Π΅ΠΊΠΎΡ‚ΠΎΡ€ΠΎΠΌ расстоянии Π΄Ρ€ΡƒΠ³ ΠΎΡ‚ Π΄Ρ€ΡƒΠ³Π°. Π’ Ρ€Π΅Π·ΡƒΠ»ΡŒΡ‚Π°Ρ‚Π΅ ΠΌΠ΅Ρ‚Π°Π»Π» элСктризуСтся влияниСм. Π­Π»Π΅ΠΊΡ‚Ρ€ΠΎΠ½Ρ‹ ΠΎΡ‚Ρ‚Π°Π»ΠΊΠΈΠ²Π°ΡŽΡ‚ΡΡ повСрхностным зарядом Ρ€Π΅Π·ΠΈΠ½Ρ‹ ΠΈ уходят Π½Π° внСшнюю ΠΏΠ»ΠΎΡΠΊΠΎΡΡ‚ΡŒ, Π³Π΄Π΅ ΠΎΠΏΠ΅Ρ€Π°Ρ‚ΠΎΡ€ ΠΈΡ… снимаСт Ρ‡Π΅Ρ€Π΅Π· Π·Π°Π·Π΅ΠΌΠ»ΠΈΡ‚Π΅Π»ΡŒ Π»Ρ‘Π³ΠΊΠΈΠΌ ΠΊΡ€Π°Ρ‚ΠΊΠΎΠ²Ρ€Π΅ΠΌΠ΅Π½Π½Ρ‹ΠΌ прикосновСниСм.

Низ мСталличСской ΠΎΠ±ΠΊΠ»Π°Π΄ΠΊΠΈ остаётся заряТСнным ΠΏΠΎΠ»ΠΎΠΆΠΈΡ‚Π΅Π»ΡŒΠ½ΠΎ. ΠŸΡ€ΠΈ расстыковкС Π΄Π²ΡƒΡ… повСрхностСй этот эффСкт сохраняСтся, ΠΏΠΎΡ‚ΠΎΠΌΡƒ Ρ‡Ρ‚ΠΎ Π² ΠΌΠ°Ρ‚Π΅Ρ€ΠΈΠ°Π»Π΅ Π½Π°Π±Π»ΡŽΠ΄Π°Π΅Ρ‚ΡΡ Π΄Π΅Ρ„ΠΈΡ†ΠΈΡ‚ элСктронов. И ΠΌΠΎΠΆΠ½ΠΎ Π½Π°Π±Π»ΡŽΠ΄Π°Ρ‚ΡŒ искру, Ссли Π΄ΠΎΡ‚Ρ€ΠΎΠ½ΡƒΡ‚ΡŒΡΡ Π΄ΠΎ мСталличСской ΠΎΠ±ΠΊΠ»Π°Π΄ΠΊΠΈ. Π­Ρ‚ΠΎΡ‚ ΠΎΠΏΡ‹Ρ‚ ΠΌΠΎΠΆΠ½ΠΎ Π½Π° ΠΎΠ΄Π½ΠΎΠΌ ΠΈ Ρ‚ΠΎΠΌ ΠΆΠ΅ зарядС Ρ€Π΅Π·ΠΈΠ½Ρ‹ ΠΏΡ€ΠΎΠ΄Π΅Π»Ρ‹Π²Π°Ρ‚ΡŒ сотни Ρ€Π°Π·, ΠΏΠΎΡΠΊΠΎΠ»ΡŒΠΊΡƒ Π΅Ρ‘ повСрхностноС статичСскоС сопротивлСниС вСсьма Π²Π΅Π»ΠΈΠΊΠΎ. Π­Ρ‚ΠΎ Π½Π΅ Π΄Π°Ρ‘Ρ‚ заряду Ρ€Π°ΡΡ‚Π΅ΠΊΠ°Ρ‚ΡŒΡΡ. ДСмонстрируя этот ΠΎΠΏΡ‹Ρ‚, Π’ΠΎΠ»ΡŒΡ‚Π° ΠΏΡ€ΠΈΠ²Π»Ρ‘ΠΊ Π²Π½ΠΈΠΌΠ°Π½ΠΈΠ΅ всСго Π½Π°ΡƒΡ‡Π½ΠΎΠ³ΠΎ ΠΌΠΈΡ€Π°, Π½ΠΎ исслСдования Π½ΠΈΠΊΠ°ΠΊ Π½Π΅ двигались Π²ΠΏΠ΅Ρ€Ρ‘Π΄, Ссли Π½Π΅ ΡΡ‡ΠΈΡ‚Π°Ρ‚ΡŒ ΠΎΡ‚ΠΊΡ€Ρ‹Ρ‚ΠΈΠΉ Шарля ΠšΡƒΠ»ΠΎΠ½Π°.

Π’ 1800 Π³ΠΎΠ΄Ρƒ сам АлСссандро Π΄Π°Ρ‘Ρ‚ Ρ‚ΠΎΠ»Ρ‡ΠΎΠΊ Ρ€Π°Π·Π²ΠΈΡ‚ΠΈΡŽ изысканий Π² области элСктричСства, изобрСтя свой Π·Π½Π°ΠΌΠ΅Π½ΠΈΡ‚Ρ‹ΠΉ Π³Π°Π»ΡŒΠ²Π°Π½ΠΈΡ‡Π΅ΡΠΊΠΈΠΉ источник питания.

ΠšΠΎΠ½ΡΡ‚Ρ€ΡƒΠΊΡ†ΠΈΡ плоского кондСнсатора

ЭлСктрофорус ΠΏΠΎ сути прСдставляСт собой ΠΏΠ΅Ρ€Π²Ρ‹ΠΉ ΠΈΠ· ΠΊΠΎΠ³Π΄Π°-Π»ΠΈΠ±ΠΎ сконструированных плоских кондСнсаторов. Π•Π³ΠΎ ΠΎΠ±ΠΊΠ»Π°Π΄ΠΊΠΈ способны Ρ…Ρ€Π°Π½ΠΈΡ‚ΡŒ Ρ‚ΠΎΠ»ΡŒΠΊΠΎ статичСский заряд, ΠΏΠΎΡ‚ΠΎΠΌΡƒ Ρ‡Ρ‚ΠΎ ΠΈΠ½Π°Ρ‡Π΅ Π½Π°ΡΠ»Π΅ΠΊΡ‚Ρ€ΠΈΠ·ΠΎΠ²Π°Ρ‚ΡŒ Ρ€Π΅Π·ΠΈΠ½Ρƒ Π½Π΅Π²ΠΎΠ·ΠΌΠΎΠΆΠ½ΠΎ. ΠŸΠΎΠ²Π΅Ρ€Ρ…Π½ΠΎΡΡ‚ΡŒ ΠΎΡ‡Π΅Π½ΡŒ Π΄ΠΎΠ»Π³ΠΎ Ρ…Ρ€Π°Π½ΠΈΡ‚ элСктроны. Π’ΠΎΠ»ΡŒΡ‚Π° Π΄Π°ΠΆΠ΅ ΠΏΡ€Π΅Π΄Π»Π°Π³Π°Π» ΡΠ½ΠΈΠΌΠ°Ρ‚ΡŒ ΠΈΡ… ΠΏΠ»Π°ΠΌΠ΅Π½Π΅ΠΌ свСчи Ρ‡Π΅Ρ€Π΅Π· ΠΈΠΎΠ½ΠΈΠ·ΠΈΡ€ΠΎΠ²Π°Π½Π½Ρ‹ΠΉ Π²ΠΎΠ·Π΄ΡƒΡ… ΠΈΠ»ΠΈ ΡƒΠ»ΡŒΡ‚Ρ€Π°Ρ„ΠΈΠΎΠ»Π΅Ρ‚ΠΎΠ²Ρ‹ΠΌ ΠΈΠ·Π»ΡƒΡ‡Π΅Π½ΠΈΠ΅ΠΌ Π‘ΠΎΠ»Π½Ρ†Π°. БСгодня ΠΊΠ°ΠΆΠ΄Ρ‹ΠΉ школьник Π·Π½Π°Π΅Ρ‚, Ρ‡Ρ‚ΠΎ Ρ‚ΠΎ ΠΆΠ΅ самоС ΠΌΠΎΠΆΠ½ΠΎ ΠΏΡ€ΠΎΠ΄Π΅Π»Π°Ρ‚ΡŒ ΠΈ Π²ΠΎΠ΄ΠΎΠΉ. ΠŸΡ€Π°Π²Π΄Π°, элСктрофорус Π½ΡƒΠΆΠ½ΠΎ Π±ΡƒΠ΄Π΅Ρ‚ послС этого Π²Ρ‹ΡΡƒΡˆΠΈΡ‚ΡŒ.

Π’ соврСмСнном ΠΌΠΈΡ€Π΅ Π½ΠΈΠΆΠ½Π΅ΠΉ ΠΎΠ±ΠΊΠ»Π°Π΄ΠΊΠΎΠΉ ΠΌΠΎΠΆΠ΅Ρ‚ ΡΠ»ΡƒΠΆΠΈΡ‚ΡŒ Ρ‚Π΅Ρ„Π»ΠΎΠ½ΠΎΠ²ΠΎΠ΅ ΠΏΠΎΠΊΡ€Ρ‹Ρ‚ΠΈΠ΅ ΠΈΠ»ΠΈ пластик. Они Ρ‚ΠΎΠΆΠ΅ Ρ…ΠΎΡ€ΠΎΡˆΠΎ Π½Π°Π±ΠΈΡ€Π°ΡŽΡ‚ статичСский заряд. ДиэлСктриком здСсь слуТит Π²ΠΎΠ·Π΄ΡƒΡ…. Π§Ρ‚ΠΎΠ±Ρ‹ ΠΏΠ΅Ρ€Π΅ΠΉΡ‚ΠΈ ΠΊ конструкции соврСмСнного кондСнсатора, Π½ΡƒΠΆΠ½ΠΎ ΠΎΠ±Π΅ ΠΎΠ±ΠΊΠ»Π°Π΄ΠΊΠΈ ΡΠ΄Π΅Π»Π°Ρ‚ΡŒ мСталличСскими. Π’ΠΎΠ³Π΄Π° ΠΏΡ€ΠΈ Π²ΠΎΠ·Π½ΠΈΠΊΠ½ΠΎΠ²Π΅Π½ΠΈΠΈ Π½Π° ΠΎΠ΄Π½ΠΎΠΉ ΠΈΠ· Π½ΠΈΡ… заряда влияниСм элСктризация распространится Π½Π° Π²Ρ‚ΠΎΡ€ΡƒΡŽ, ΠΈ Ссли Π΄Ρ€ΡƒΠ³ΠΎΠΉ ΠΊΠΎΠ½Ρ‚Π°ΠΊΡ‚ Π·Π°Π·Π΅ΠΌΠ»Ρ‘Π½, накоплСнная энСргия ΠΌΠΎΠΆΠ΅Ρ‚ Ρ…Ρ€Π°Π½ΠΈΡ‚ΡŒΡΡ ΠΊΠ°ΠΊΠΎΠ΅-Ρ‚ΠΎ врСмя.

Запас элСктронов Π½Π°ΠΏΡ€ΡΠΌΡƒΡŽ зависит ΠΎΡ‚ ΠΌΠ°Ρ‚Π΅Ρ€ΠΈΠ°Π»Π° диэлСктриков. Π’Π°ΠΊ Π½Π°ΠΏΡ€ΠΈΠΌΠ΅Ρ€, срСди соврСмСнных кондСнсаторов Π²ΡΡ‚Ρ€Π΅Ρ‡Π°ΡŽΡ‚ΡΡ:

  1. Π‘Π»ΡŽΠ΄ΡΠ½Ρ‹Π΅.
  2. Π’ΠΎΠ·Π΄ΡƒΡˆΠ½Ρ‹Π΅.
  3. ЭлСктролитичСскиС (оксидныС).
  4. ΠšΠ΅Ρ€Π°ΠΌΠΈΡ‡Π΅ΡΠΊΠΈΠ΅.

Π’ эти названия ΠΊΠ°ΠΊ Ρ€Π°Π· ΠΈ Π·Π°Π»ΠΎΠΆΠ΅Π½ ΠΌΠ°Ρ‚Π΅Ρ€ΠΈΠ°Π» диэлСктрика. ΠžΡ‚ Π΅Π³ΠΎ состава зависит Π½Π°ΠΏΡ€ΡΠΌΡƒΡŽ Ρ‘ΠΌΠΊΠΎΡΡ‚ΡŒ, которая ΠΌΠΎΠΆΠ΅Ρ‚ Π±Ρ‹Ρ‚ΡŒ ΡƒΠ²Π΅Π»ΠΈΡ‡Π΅Π½Π° Π²ΠΎ ΠΌΠ½ΠΎΠ³ΠΎ Ρ€Π°Π·. Роль диэлСктриков объяснялась Π²Ρ‹ΡˆΠ΅, Π² частности ΠΈΡ… ΠΏΠ°Ρ€Π°ΠΌΠ΅Ρ‚Ρ€Ρ‹ ΠΎΠΏΡ€Π΅Π΄Π΅Π»ΡΡŽΡ‚ΡΡ нСпосрСдствСнно строСниСм вСщСства. Однако ΠΌΠ½ΠΎΠ³ΠΈΠ΅ ΠΌΠ°Ρ‚Π΅Ρ€ΠΈΠ°Π»Ρ‹, ΠΎΠ±Π»Π°Π΄Π°ΡŽΡ‰ΠΈΠ΅ высокими характСристиками, ΠΈΡΠΏΠΎΠ»ΡŒΠ·ΠΎΠ²Π°Ρ‚ΡŒ Π½Π΅ удаётся ΠΏΠΎ ΠΏΡ€ΠΈΡ‡ΠΈΠ½Π΅ ΠΈΡ… нСпригодности. Π’Π°ΠΊ Π½Π°ΠΏΡ€ΠΈΠΌΠ΅Ρ€, Π²ΠΎΠ΄Π° ΠΎΠ±Π»Π°Π΄Π°Π΅Ρ‚ высокой диэлСктричСской ΠΏΡ€ΠΎΠ½ΠΈΡ†Π°Π΅ΠΌΠΎΡΡ‚ΡŒΡŽ.

Π₯арактСристика плоского кондСнсатора, ΠΌΠ΅Ρ€Π° Π΅Π³ΠΎ способности Π½Π°ΠΊΠ°ΠΏΠ»ΠΈΠ²Π°Ρ‚ΡŒ элСктричСский заряд.

Π’Π°ΠΊ ΠΊΠ°ΠΊ ΠΏΠΎΠ»Π΅ сосрСдоточСно Π²Π½ΡƒΡ‚Ρ€ΠΈ кондСнсатора, Ρ‚ΠΎ Π»ΠΈΠ½ΠΈΠΈ напряТСнности Π½Π°Ρ‡ΠΈΠ½Π°ΡŽΡ‚ΡΡ Π½Π° ΠΎΠ΄Π½ΠΎΠΉ ΠΎΠ±ΠΊΠ»Π°Π΄ΠΊΠ΅ ΠΈ ΠΊΠΎΠ½Ρ‡Π°ΡŽΡ‚ΡΡ Π½Π° Π΄Ρ€ΡƒΠ³ΠΎΠΉ, поэтому свободныС заряды, ΠΊΠΎΡ‚ΠΎΡ€Ρ‹Π΅ Π²ΠΎΠ·Π½ΠΈΠΊΠ°ΡŽΡ‚ Π½Π° Ρ€Π°Π·Π½Ρ‹Ρ… ΠΎΠ±ΠΊΠ»Π°Π΄ΠΊΠ°Ρ…, Ρ€Π°Π²Π½Ρ‹ ΠΏΠΎ ΠΌΠΎΠ΄ΡƒΠ»ΡŽ ΠΈ ΠΏΡ€ΠΎΡ‚ΠΈΠ²ΠΎΠΏΠΎΠ»ΠΎΠΆΠ½Ρ‹ ΠΏΠΎ Π·Π½Π°ΠΊΡƒ. Под Π΅ΠΌΠΊΠΎΡΡ‚ΡŒΡŽ кондСнсатора понимаСтся физичСская Π²Π΅Π»ΠΈΡ‡ΠΈΠ½Π°, равная ΠΎΡ‚Π½ΠΎΡˆΠ΅Π½ΠΈΡŽ заряда Q, Π½Π°ΠΊΠΎΠΏΠ»Π΅Π½Π½ΠΎΠ³ΠΎ Π² кондСнсаторС, ΠΊ разности ΠΏΠΎΡ‚Π΅Π½Ρ†ΠΈΠ°Π»ΠΎΠ² (Ο†1 — Ο†2) ΠΌΠ΅ΠΆΠ΄Ρƒ Π΅Π³ΠΎ ΠΎΠ±ΠΊΠ»Π°Π΄ΠΊΠ°ΠΌΠΈ

ΠŸΡ€ΠΈ Π½Π΅Π±ΠΎΠ»ΡŒΡˆΠΈΡ… Ρ€Π°Π·ΠΌΠ΅Ρ€Π°Ρ… кондСнсатор отличаСтся Π·Π½Π°Ρ‡ΠΈΡ‚Π΅Π»ΡŒΠ½ΠΎΠΉ Π΅ΠΌΠΊΠΎΡΡ‚ΡŒΡŽ, Π½Π΅ зависящСй ΠΎΡ‚ наличия Π²Π±Π»ΠΈΠ·ΠΈ Π½Π΅Π³ΠΎ Π΄Ρ€ΡƒΠ³ΠΈΡ… зарядов ΠΈΠ»ΠΈ ΠΏΡ€ΠΎΠ²ΠΎΠ΄Π½ΠΈΠΊΠΎΠ². Обкладкам кондСнсатора ΡΠΎΠΎΠ±Ρ‰Π°ΡŽΡ‚ ΠΎΠ΄ΠΈΠ½Π°ΠΊΠΎΠ²Ρ‹Π΅ ΠΏΠΎ ΠΌΠΎΠ΄ΡƒΠ»ΡŽ, Π½ΠΎ ΠΏΡ€ΠΎΡ‚ΠΈΠ²ΠΎΠΏΠΎΠ»ΠΎΠΆΠ½Ρ‹Π΅ ΠΏΠΎ Π·Π½Π°ΠΊΡƒ заряды, Ρ‡Ρ‚ΠΎ способствуСт накоплСнию зарядов, Ρ‚Π°ΠΊ ΠΊΠ°ΠΊ Ρ€Π°Π·Π½ΠΎΠΈΠΌΠ΅Π½Π½Ρ‹Π΅ заряды ΠΏΡ€ΠΈΡ‚ΡΠ³ΠΈΠ²Π°ΡŽΡ‚ΡΡ ΠΈ поэтому Ρ€Π°ΡΠΏΠΎΠ»Π°Π³Π°ΡŽΡ‚ΡΡ Π½Π° Π²Π½ΡƒΡ‚Ρ€Π΅Π½Π½ΠΈΡ… повСрхностях пластин.

Под зарядом кондСнсатора ΠΏΠΎΠ½ΠΈΠΌΠ°ΡŽΡ‚ заряд ΠΎΠ΄Π½ΠΎΠΉ пластины.

Π’Π°ΠΊ ΠΆΠ΅ Π΅ΡΡ‚ΡŒ:

ЭнСргия кондСнсатора:

ΠΠΌΠΊΠΎΡΡ‚ΡŒ цилиндричСского кондСнсатора:

Π•ΠΌΠΊΠΎΡΡ‚ΡŒ сфСричСского кондСнсатора:

Π’ Ρ„ΠΎΡ€ΠΌΡƒΠ»Π΅ ΠΌΡ‹ использовали:

ЭлСктричСская Ρ‘ΠΌΠΊΠΎΡΡ‚ΡŒ (Ρ‘ΠΌΠΊΠΎΡΡ‚ΡŒ кондСнсатора)

ΠžΡ‚Π½ΠΎΡΠΈΡ‚Π΅Π»ΡŒΠ½Π°Ρ диэлСктричСская ΠΏΡ€ΠΎΠ½ΠΈΡ†Π°Π΅ΠΌΠΎΡΡ‚ΡŒ

ЭлСктричСская постоянная

Плоский кондСнсатор состоит ΠΈΠ· Π΄Π²ΡƒΡ…
ΠΏΠ°Ρ€Π°Π»Π»Π΅Π»ΡŒΠ½Ρ‹Ρ… пластин, Ρ€Π°Π·Π΄Π΅Π»Ρ‘Π½Π½Ρ‹Ρ…
нСбольшим Π·Π°Π·ΠΎΡ€ΠΎΠΌ ΡˆΠΈΡ€ΠΈΠ½ΠΎΠΉ
,
Π·Π°ΠΏΠΎΠ»Π½Π΅Π½Π½Ρ‹ΠΌ ΠΎΠ΄Π½ΠΎΡ€ΠΎΠ΄Π½Ρ‹ΠΌ диэлСктриком.

Нам извСстно, Ρ‡Ρ‚ΠΎ ΠΏΠΎΠ»Π΅ ΠΌΠ΅ΠΆΠ΄Ρƒ двумя
Ρ€Π°Π·Π½ΠΎΠΈΠΌΡ‘Π½Π½ΠΎ заряТСнными пластинами с
ΠΎΠ΄ΠΈΠ½Π°ΠΊΠΎΠ²ΠΎΠΉ ΠΏΠΎ Π²Π΅Π»ΠΈΡ‡ΠΈΠ½Π΅ повСрхностной
ΠΏΠ»ΠΎΡ‚Π½ΠΎΡΡ‚ΡŒΡŽ
Ρ€Π°Π²Π½ΠΎ,
Π³Π΄Π΅,S– ΠΏΠ»ΠΎΡ‰Π°Π΄ΡŒ ΠΊΠ°ΠΆΠ΄ΠΎΠΉ пластины.
НапряТСниС ΠΌΠ΅ΠΆΠ΄Ρƒ ΠΎΠ±ΠΊΠ»Π°Π΄ΠΊΠ°ΠΌΠΈ:

Π˜ΡΠΏΠΎΠ»ΡŒΠ·ΡƒΡ
ΠΎΠΏΡ€Π΅Π΄Π΅Π»Π΅Π½ΠΈΠ΅ Смкости кондСнсатора,
ΠΏΠΎΠ»ΡƒΡ‡Π°Π΅ΠΌ:

ΠžΡ‚ΠΌΠ΅Ρ‚ΠΈΠΌ, Ρ‡Ρ‚ΠΎ получСнная Ρ„ΠΎΡ€ΠΌΡƒΠ»Π° являСтся
ΠΏΡ€ΠΈΠ±Π»ΠΈΠΆΠ΅Π½Π½ΠΎΠΉ, Ρ‚Π°ΠΊ ΠΊΠ°ΠΊ Π²Ρ‹Π²Π΅Π΄Π΅Π½Π° Π±Π΅Π· ΡƒΡ‡Π΅Ρ‚Π°
искаТСния поля Ρƒ ΠΊΡ€Π°Π΅Π² пластин. РасчСт
ΠΏΠΎ этой Ρ„ΠΎΡ€ΠΌΡƒΠ»Π΅ Π΄Π°Π΅Ρ‚ Π·Π°Π²Ρ‹ΡˆΠ΅Π½Π½ΠΎΠ΅ Π·Π½Π°Ρ‡Π΅Π½ΠΈΠ΅
ёмкости ΠΈ Ρ‚Π΅ΠΌ Ρ‚ΠΎΡ‡Π½Π΅Π΅, Ρ‡Π΅ΠΌ мСньшС Π·Π°Π·ΠΎΡ€
ΠΏΠΎ ΡΡ€Π°Π²Π½Π΅Π½ΠΈΡŽ с Π»ΠΈΠ½Π΅ΠΉΠ½Ρ‹ΠΌΠΈ Ρ€Π°Π·ΠΌΠ΅Ρ€Π°ΠΌΠΈ
пластин.

ΠΠΌΠΊΠΎΡΡ‚ΡŒ сфСричСского кондСнсатора.

БфСричСский кондСнсатор прСдставляСт
собой систСму Π΄Π²ΡƒΡ… концСнтричСских
сфСр с радиусами
ΠΈ.
ЭлСктричСскоС ΠΏΠΎΠ»Π΅ ΠΌΠ΅ΠΆΠ΄Ρƒ ΠΎΠ±ΠΊΠ»Π°Π΄ΠΊΠ°ΠΌΠΈ
сфСричСского кондСнсатора согласно
Ρ‚Π΅ΠΎΡ€Π΅ΠΌΠ΅ Гаусса опрСдСляСтся зарядом
Π²Π½ΡƒΡ‚Ρ€Π΅Π½Π½Π΅ΠΉ сфСры. НапряТСниС ΠΌΠ΅ΠΆΠ΄Ρƒ
ΠΎΠ±ΠΊΠ»Π°Π΄ΠΊΠ°ΠΌΠΈ Ρ€Π°Π²Π½ΠΎ:

.

Для
ёмкости сфСричСского кондСнсатора
ΠΏΠΎΠ»ΡƒΡ‡Π°Π΅ΠΌ:

Π­Ρ‚ΠΎ
Ρ„ΠΎΡ€ΠΌΡƒΠ»Π° точная.

Если
,
получСнная Ρ„ΠΎΡ€ΠΌΡƒΠ»Π° ΠΏΠ΅Ρ€Π΅Ρ…ΠΎΠ΄ΠΈΡ‚ Π² Π²Ρ‹Ρ€Π°ΠΆΠ΅Π½ΠΈΠ΅
для ёмкости плоского кондСнсатора.

ΠΠΌΠΊΠΎΡΡ‚ΡŒ
цилиндричСского кондСнсатора.

ЦилиндричСский кондСнсатор составляСт
систСму Π΄Π²ΡƒΡ… ΠΊΠΎΠ°ΠΊΡΠΈΠ°Π»ΡŒΠ½Ρ‹Ρ… Ρ†ΠΈΠ»ΠΈΠ½Π΄Ρ€ΠΎΠ² с
радиусами
ΠΈ,
Π΄Π»ΠΈΠ½ΠΎΠΉ.

РассуТдая
Π°Π½Π°Π»ΠΎΠ³ΠΈΡ‡Π½ΠΎ Π²Ρ‹Π²ΠΎΠ΄Ρƒ ёмкости сфСричСского
кондСнсатора, ΠΏΠΎΠ»ΡƒΡ‡Π°Π΅ΠΌ:

..

ΠŸΠΎΠ»ΡƒΡ‡Π΅Π½Π½Π°Ρ Ρ„ΠΎΡ€ΠΌΡƒΠ»Π° являСтся ΠΏΡ€ΠΈΠ±Π»ΠΈΠΆΠ΅Π½Π½ΠΎΠΉ
ΠΈ ΠΏΡ€ΠΈ ΠΌΠ°Π»ΠΎΠΌ Π·Π°Π·ΠΎΡ€Π΅
ΠΏΠ΅Ρ€Π΅Ρ…ΠΎΠ΄ΠΈΡ‚ Π² Ρ„ΠΎΡ€ΠΌΡƒΠ»Ρƒ Смкости плоского
кондСнсатора.

Π‘ΠΎΠ΅Π΄ΠΈΠ½Π΅Π½ΠΈΠ΅
кондСнсаторов.

Π’ ΠΏΡ€Π°ΠΊΡ‚ΠΈΠΊΠ΅ для получСния Π½Π΅ΠΎΠ±Ρ…ΠΎΠ΄ΠΈΠΌΡ‹Ρ…
Π·Π½Π°Ρ‡Π΅Π½ΠΈΠΉ Смкости ΠΈΡΠΏΠΎΠ»ΡŒΠ·ΡƒΡŽΡ‚ соСдинСния
кондСнсаторов: Π°) ΠΏΠΎΡΠ»Π΅Π΄ΠΎΠ²Π°Ρ‚Π΅Π»ΡŒΠ½ΠΎΠ΅, Π±)
ΠΏΠ°Ρ€Π°Π»Π»Π΅Π»ΡŒΠ½ΠΎΠ΅, Π²) смСшанноС (см. рисунок).

ΠΠΌΠΊΠΎΡΡ‚ΡŒ
ΠΏΠΎΡΠ»Π΅Π΄ΠΎΠ²Π°Ρ‚Π΅Π»ΡŒΠ½ΠΎΠ³ΠΎ соСдинСния
кондСнсаторов.

Заряды ΠΏΠΎΡΠ»Π΅Π΄ΠΎΠ²Π°Ρ‚Π΅Π»ΡŒΠ½ΠΎ соСдинСнных
кондСнсаторов Ρ€Π°Π²Π½Ρ‹
,
Π° напряТСниС Π½Π° Π±Π°Ρ‚Π°Ρ€Π΅Π΅.
Из опрСдСлСния Смкости слСдуСт:

Если
,
Ρ‚ΠΎ(Ρ‘ΠΌΠΊΠΎΡΡ‚ΡŒ ΠΏΠΎΡΠ»Π΅Π΄ΠΎΠ²Π°Ρ‚Π΅Π»ΡŒΠ½ΠΎΠ³ΠΎ соСдинСния
мСньшС наимСньшСй ёмкости Π² ΠΏΠΎΡΠ»Π΅Π΄ΠΎΠ²Π°Ρ‚Π΅Π»ΡŒΠ½ΠΎΠΌ
соСдинСнии).

Для
ΠΏΠΎΡΠ»Π΅Π΄ΠΎΠ²Π°Ρ‚Π΅Π»ΡŒΠ½ΠΎ соСдинСнных кондСнсаторов
Π΅ΠΌΠΊΠΎΡΡ‚ΡŒ вычисляСтся ΠΏΠΎ Ρ„ΠΎΡ€ΠΌΡƒΠ»Π΅:

ΠΠΌΠΊΠΎΡΡ‚ΡŒ
ΠΏΠ°Ρ€Π°Π»Π»Π΅Π»ΡŒΠ½ΠΎΠ³ΠΎ соСдинСния кондСнсаторов.

Заряд
Π±Π°Ρ‚Π°Ρ€Π΅ΠΈ Ρ€Π°Π²Π΅Π½ суммС зарядов:

Π°
напряТСниС
.
По ΠΎΠΏΡ€Π΅Π΄Π΅Π»Π΅Π½ΠΈΡŽ Смкости ΠΏΠΎΠ»ΡƒΡ‡Π°Π΅ΠΌ:

Для
ΠΏΠ°Ρ€Π°Π»Π»Π΅Π»ΡŒΠ½ΠΎ соСдинСнных кондСнсаторов:.

Π’
случаС ΠΎΠ΄ΠΈΠ½Π°ΠΊΠΎΠ²Ρ‹Ρ… кондСнсаторов:
.

ΠžΡ†Π΅Π½ΠΈΡ‚ΡŒ
Π΅ΠΌΠΊΠΎΡΡ‚ΡŒ Π±Π°Ρ‚Π°Ρ€Π΅ΠΈ (см. рисунок)
.

Π˜ΡΠΏΠΎΠ»ΡŒΠ·ΡƒΡ
свойство бСсконСчности ΠΌΠΎΠΆΠ½ΠΎ ΠΏΡ€Π΅Π΄ΡΡ‚Π°Π²ΠΈΡ‚ΡŒ
Ρ†Π΅ΠΏΡŒ Π² Π²ΠΈΠ΄Π΅ соСдинСния (см. рисунок).

Для
расчСта ёмкости Π±Π°Ρ‚Π°Ρ€Π΅ΠΈ ΠΏΠΎΠ»ΡƒΡ‡Π°Π΅ΠΌ:

ΠžΡ‚ΠΊΡƒΠ΄Π°:
,
Ρ‚Π°ΠΊ ΠΊΠ°ΠΊ,
Ρ‚ΠΎ.

ЛСкция 7.

ДиэлСктрики
Π² элСктричСском ΠΏΠΎΠ»Π΅.

ДиэлСктриками (изоляторами) Π½Π°Π·Ρ‹Π²Π°ΡŽΡ‚
вСщСства, Π½Π΅ проводящиС постоянного
элСктричСского Ρ‚ΠΎΠΊΠ°. Π­Ρ‚ΠΎ ΠΎΠ·Π½Π°Ρ‡Π°Π΅Ρ‚, Ρ‡Ρ‚ΠΎ
Π² диэлСктриках ΠΎΡ‚ΡΡƒΡ‚ΡΡ‚Π²ΡƒΡŽΡ‚ «свободныС»
заряды, способныС ΠΏΠ΅Ρ€Π΅ΠΌΠ΅Ρ‰Π°Ρ‚ΡŒΡΡ Π½Π°
Π·Π½Π°Ρ‡ΠΈΡ‚Π΅Π»ΡŒΠ½Ρ‹Π΅ расстояния.

ДиэлСктрики состоят Π»ΠΈΠ±ΠΎ ΠΈΠ· Π½Π΅ΠΉΡ‚Ρ€Π°Π»ΡŒΠ½Ρ‹Ρ…
ΠΌΠΎΠ»Π΅ΠΊΡƒΠ», Π»ΠΈΠ±ΠΎ ΠΈΠ· ΠΈΠΎΠ½ΠΎΠ², находящихся Π²
ΡƒΠ·Π»Π°Ρ… кристалличСской Ρ€Π΅ΡˆΠ΅Ρ‚ΠΊΠΈ. Π‘Π°ΠΌΠΈ ΠΆΠ΅
ΠΌΠΎΠ»Π΅ΠΊΡƒΠ»Ρ‹ ΠΌΠΎΠ³ΡƒΡ‚ Π±Ρ‹Ρ‚ΡŒ полярными
инСполярными.
ΠŸΠΎΠ»ΡΡ€Π½Ρ‹Π΅ ΠΌΠΎΠ»Π΅ΠΊΡƒΠ»Ρ‹
ΠΎΠ±Π»Π°Π΄Π°ΡŽΡ‚ Π΄ΠΈΠΏΠΎΠ»ΡŒΠ½Ρ‹ΠΌ ΠΌΠΎΠΌΠ΅Π½Ρ‚ΠΎΠΌ, Ρƒ нСполярных
ΠΌΠΎΠ»Π΅ΠΊΡƒΠ» Π΄ΠΈΠΏΠΎΠ»ΡŒΠ½Ρ‹ΠΉ ΠΌΠΎΠΌΠ΅Π½Ρ‚ Ρ€Π°Π²Π΅Π½ Π½ΡƒΠ»ΡŽ.

ΠŸΠΎΠ»ΡΡ€ΠΈΠ·Π°Ρ†ΠΈΡ.

Π’ элСктричСском ΠΏΠΎΠ»Π΅ диэлСктрики
ΠΏΠΎΠ»ΡΡ€ΠΈΠ·ΡƒΡŽΡ‚ΡΡ. Π­Ρ‚ΠΎ явлСниС связано с
появлСниСм Π² объСмС ΠΈ Π½Π° повСрхности
диэлСктрика «связанных
Β» зарядов.
ΠŸΡ€ΠΈ этом ΠΊΠΎΠ½Π΅Ρ‡Π½Ρ‹ΠΉ объСм диэлСктрика
ΠΏΡ€ΠΈΠΎΠ±Ρ€Π΅Ρ‚Π°Π΅Ρ‚ Π΄ΠΈΠΏΠΎΠ»ΡŒΠ½Ρ‹ΠΉ ΠΌΠΎΠΌΠ΅Π½Ρ‚. ΠœΠ΅Ρ…Π°Π½ΠΈΠ·ΠΌ
поляризации связан с ΠΊΠΎΠ½ΠΊΡ€Π΅Ρ‚Π½Ρ‹ΠΌ строСниСм
диэлСктрика. Если диэлСктрик состоит
ΠΈΠ· нСполярных ΠΌΠΎΠ»Π΅ΠΊΡƒΠ», Ρ‚ΠΎ Π² ΠΏΡ€Π΅Π΄Π΅Π»Π°Ρ…
ΠΊΠ°ΠΆΠ΄ΠΎΠΉ ΠΌΠΎΠ»Π΅ΠΊΡƒΠ»Ρ‹ происходит смСщСниС
зарядов – ΠΏΠΎΠ»ΠΎΠΆΠΈΡ‚Π΅Π»ΡŒΠ½Ρ‹Ρ… ΠΏΠΎ полю,
ΠΎΡ‚Ρ€ΠΈΡ†Π°Ρ‚Π΅Π»ΡŒΠ½Ρ‹Ρ… ΠΏΡ€ΠΎΡ‚ΠΈΠ² поля, Ρ‚.Π΅. ΠΌΠΎΠ»Π΅ΠΊΡƒΠ»Ρ‹,
ΠΏΡ€ΠΈΠΎΠ±Ρ€Π΅Ρ‚Π°ΡŽΡ‚ Π΄ΠΈΠΏΠΎΠ»ΡŒΠ½Ρ‹ΠΉ ΠΌΠΎΠΌΠ΅Π½Ρ‚. Π£
диэлСктрика с полярными ΠΌΠΎΠ»Π΅ΠΊΡƒΠ»Π°ΠΌΠΈ Π²
отсутствии внСшнСго элСктричСского
поля ΠΈΡ… Π΄ΠΈΠΏΠΎΠ»ΡŒΠ½Ρ‹Π΅ ΠΌΠΎΠΌΠ΅Π½Ρ‚Ρ‹ ΠΎΡ€ΠΈΠ΅Π½Ρ‚ΠΈΡ€ΠΎΠ²Π°Π½Ρ‹
Ρ…Π°ΠΎΡ‚ΠΈΡ‡Π½ΠΎ.

Под дСйствиСм элСктричСского поля
Π΄ΠΈΠΏΠΎΠ»ΠΈ ΠΎΡ€ΠΈΠ΅Π½Ρ‚ΠΈΡ€ΡƒΡŽΡ‚ΡΡ прСимущСствСнно
Π² Π½Π°ΠΏΡ€Π°Π²Π»Π΅Π½ΠΈΠΈ поля. Рассмотрим ΠΏΠΎΠ΄Ρ€ΠΎΠ±Π½Π΅Π΅
этот ΠΌΠ΅Ρ…Π°Π½ΠΈΠ·ΠΌ (см. рисунок). ΠŸΠ°Ρ€Π° сил
исоздаСт
Π²Ρ€Π°Ρ‰Π°Ρ‚Π΅Π»ΡŒΠ½Ρ‹ΠΉ ΠΌΠΎΠΌΠ΅Π½Ρ‚ Ρ€Π°Π²Π½Ρ‹ΠΉ,
Π³Π΄Π΅-
Π΄ΠΈΠΏΠΎΠ»ΡŒΠ½Ρ‹ΠΉ ΠΌΠΎΠΌΠ΅Π½Ρ‚ ΠΌΠΎΠ»Π΅ΠΊΡƒΠ»Ρ‹. Π­Ρ‚ΠΎΡ‚ ΠΌΠΎΠΌΠ΅Π½Ρ‚
стрСмится ΠΎΡ€ΠΈΠ΅Π½Ρ‚ΠΈΡ€ΠΎΠ²Π°Ρ‚ΡŒ диполь вдоль
поля. Π’ ΠΈΠΎΠ½Π½Ρ‹Ρ… кристаллах ΠΏΠΎΠ΄ дСйствиСм
элСктричСского поля всС ΠΏΠΎΠ»ΠΎΠΆΠΈΡ‚Π΅Π»ΡŒΠ½Ρ‹Π΅
ΠΈΠΎΠ½Ρ‹ ΡΠΌΠ΅Ρ‰Π°ΡŽΡ‚ΡΡ ΠΏΠΎ полю, ΠΎΡ‚Ρ€ΠΈΡ†Π°Ρ‚Π΅Π»ΡŒΠ½Ρ‹Π΅
– ΠΏΡ€ΠΎΡ‚ΠΈΠ² поля. ΠžΡ‚ΠΌΠ΅Ρ‚ΠΈΠΌ, Ρ‡Ρ‚ΠΎ смСщСниС
зарядов ΠΎΡ‡Π΅Π½ΡŒ ΠΌΠ°Π»Ρ‹ Π΄Π°ΠΆΠ΅ ΠΏΠΎ ΡΡ€Π°Π²Π½Π΅Π½ΠΈΡŽ с
Ρ€Π°Π·ΠΌΠ΅Ρ€Π°ΠΌΠΈ ΠΌΠΎΠ»Π΅ΠΊΡƒΠ». Π­Ρ‚ΠΎ связано с Ρ‚Π΅ΠΌ,
Ρ‡Ρ‚ΠΎ Π½Π°ΠΏΡ€ΡΠΆΠ΅Π½Π½ΠΎΡΡ‚ΡŒ внСшнСго элСктричСского
поля ΠΎΠ±Ρ‹Ρ‡Π½ΠΎ ΠΌΠ½ΠΎΠ³ΠΎ мСньшС напряТСнности
Π²Π½ΡƒΡ‚Ρ€Π΅Π½Π½ΠΈΡ… элСктричСских ΠΏΠΎΠ»Π΅ΠΉ Π²
ΠΌΠΎΠ»Π΅ΠΊΡƒΠ»Π°Ρ….

ΠžΡ‚ΠΌΠ΅Ρ‚ΠΈΠΌ, Ρ‡Ρ‚ΠΎ ΡΡƒΡ‰Π΅ΡΡ‚Π²ΡƒΡŽΡ‚ диэлСктрики,
поляризованныС Π΄Π°ΠΆΠ΅ ΠΏΡ€ΠΈ отсутствии
внСшнСго поля (элСктрСты, сСгнСтоэлСктрики).
ΠœΡ‹ остановимся Π½Π° рассмотрСнии Ρ‚ΠΎΠ»ΡŒΠΊΠΎ
ΠΎΠ΄Π½ΠΎΡ€ΠΎΠ΄Π½Ρ‹Ρ… диэлСктриков, Π² ΠΊΠΎΡ‚ΠΎΡ€Ρ‹Ρ…
отсутствуСт остаточная поляризация, Π°
ΠΎΠ±ΡŠΠ΅ΠΌΠ½Ρ‹ΠΉ ΠΈ «связанный» заряд всСгда
Ρ€Π°Π²Π΅Π½ Π½ΡƒΠ»ΡŽ
.

ΠšΠžΠΠ”Π•ΠΠ‘ΠΠ’ΠžΠ 
— ΠΎΠ·Π½Π°Ρ‡Π°Π΅Ρ‚ Π½Π°ΠΊΠΎΠΏΠΈΡ‚Π΅Π»ΡŒ. Π’ Ρ€Π°Π΄ΠΈΠΎ ΠΈ элСктронной
Π°ΠΏΠΏΠ°Ρ€Π°Ρ‚ΡƒΡ€Π΅ кондСнсатор являСтся Π½Π°ΠΊΠΎΠΏΠΈΡ‚Π΅Π»Π΅ΠΌ элСктричСских
зарядов. ΠŸΡ€ΠΎΡΡ‚Π΅ΠΉΡˆΠΈΠΉ кондСнсатор состоит ΠΈΠ· Π΄Π²ΡƒΡ… мСталличСских
пластинок Ρ€Π°Π·Π΄Π΅Π»Π΅Π½Π½Ρ‹Ρ… слоСм диэлСктрика. ДиэлСктрик — это
ΠΌΠ°Ρ‚Π΅Ρ€ΠΈΠ°Π» ΠΊΠΎΡ‚ΠΎΡ€Ρ‹ΠΉ Π½Π΅ ΠΏΡ€ΠΎΠ²ΠΎΠ΄ΠΈΡ‚ элСктричСского Ρ‚ΠΎΠΊΠ° ΠΈ ΠΎΠ±Π»Π°Π΄Π°Π΅Ρ‚
ΠΎΠΏΡ€Π΅Π΄Π΅Π»Π΅Π½Π½Ρ‹ΠΌΠΈ свойствами ΠΎ ΠΊΠΎΡ‚ΠΎΡ€Ρ‹Ρ… ΠΏΠΎΠ³ΠΎΠ²ΠΎΡ€ΠΈΠΌ Ρ‡ΡƒΡ‚ΡŒ ΠΏΠΎΠ·ΠΆΠ΅.

Π’Π°ΠΊ ΠΊΠ°ΠΊ кондСнсатор являСтся Π½Π°ΠΊΠΎΠΏΠΈΡ‚Π΅Π»Π΅ΠΌ, Ρ‚ΠΎ ΠΎΠ½ Π΄ΠΎΠ»ΠΆΠ΅Π½
ΠΎΠ±Π»Π°Π΄Π°Ρ‚ΡŒ ΠΎΠΏΡ€Π΅Π΄Π΅Π»Π΅Π½Π½ΠΎΠΉ Π΅ΠΌΠΊΠΎΡΡ‚ΡŒΡŽ (объСмом для накоплСния
зарядов). На Π΅ΠΌΠΊΠΎΡΡ‚ΡŒ кондСнсатора Π²Π»ΠΈΡΡŽΡ‚ ΠΏΠ»ΠΎΡ‰Π°Π΄ΡŒ пластин
(Π΅Ρ‰Π΅ ΠΈΡ… Π½Π°Π·Ρ‹Π²Π°ΡŽΡ‚ «ΠΎΠ±ΠΊΠ»Π°Π΄ΠΊΠ°ΠΌΠΈ»), расстояниС ΠΌΠ΅ΠΆΠ΄Ρƒ ΠΎΠ±ΠΊΠ»Π°Π΄ΠΊΠ°ΠΌΠΈ
ΠΈ качСство диэлСктрика. К Ρ…ΠΎΡ€ΠΎΡˆΠΈΠΌ диэлСктрикам
относятся Π²Π°ΠΊΡƒΡƒΠΌ, эбонит, Ρ„Π°Ρ€Ρ„ΠΎΡ€, слюда, полиэтилСн,
тСкстолит ΠΈ ΠΌΠ½ΠΎΠ³ΠΎ Π΄Ρ€ΡƒΠ³ΠΈΡ… синтСтичСских ΠΌΠ°Ρ‚Π΅Ρ€ΠΈΠ°Π»ΠΎΠ².
На рисункС ΠΈΠ·ΠΎΠ±Ρ€Π°ΠΆΠ΅Π½ ΠΏΡ€ΠΎΡΡ‚Π΅ΠΉΡˆΠΈΠΉ кондСнсатор
с двумя ΠΏΠ°Ρ€Π°Π»Π»Π΅Π»ΡŒΠ½Ρ‹ΠΌΠΈ ΠΎΠ±ΠΊΠ»Π°Π΄ΠΊΠ°ΠΌΠΈ ΠΏΠ»ΠΎΡ‰Π°Π΄ΡŒΡŽ S
(S = m * n), ΠΊΠΎΡ‚ΠΎΡ€Ρ‹Π΅ находятся Π² Π²Π°ΠΊΡƒΡƒΠΌΠ΅ Π½Π° расстоянии d Π΄Ρ€ΡƒΠ³
ΠΎΡ‚ Π΄Ρ€ΡƒΠ³Π°.

Если ΠΌΠ΅ΠΆΠ΄Ρƒ Π²Π΅Ρ€Ρ…Π½Π΅ΠΉ ΠΈ Π½ΠΈΠΆΠ½Π΅ΠΉ ΠΎΠ±ΠΊΠ»Π°Π΄ΠΊΠ°ΠΌΠΈ кондСнсатора
ΠΏΡ€ΠΈΠ»ΠΎΠΆΠΈΡ‚ΡŒ напряТСниС Uab, Ρ‚ΠΎ Π½Π° Π²Π΅Ρ€Ρ…Π½Π΅ΠΉ ΠΈ Π½ΠΈΠΆΠ½Π΅ΠΉ ΠΎΠ±ΠΊΠ»Π°Π΄ΠΊΠ°Ρ…
кондСнсатора накопятся ΠΎΠ΄ΠΈΠ½Π°ΠΊΠΎΠ²Ρ‹Π΅ ΠΏΠΎΠ»ΠΎΠΆΠΈΡ‚Π΅Π»ΡŒΠ½Ρ‹ΠΉ +q ΠΈ
ΠΎΡ‚Ρ€ΠΈΡ†Π°Ρ‚Π΅Π»ΡŒΠ½Ρ‹ΠΉ -q заряды, ΠΊΠΎΡ‚ΠΎΡ€Ρ‹Π΅ Π½Π°Π·Ρ‹Π²Π°ΡŽΡ‚ свободными. ΠœΠ΅ΠΆΠ΄Ρƒ
ΠΎΠ±ΠΊΠ»Π°Π΄ΠΊΠ°ΠΌΠΈ Π²ΠΎΠ·Π½ΠΈΠΊΠ°Π΅Ρ‚ элСктричСскоС ΠΏΠΎΠ»Π΅ ΠΎΠ±ΠΎΠ·Π½Π°Ρ‡Π΅Π½Π½ΠΎΠ΅ Π½Π° рисункС
Π±ΡƒΠΊΠ²ΠΎΠΉ Π•.
Π•ΠΌΠΊΠΎΡΡ‚ΡŒ нашСго кондСнсатора (обозначаСтся Π±ΡƒΠΊΠ²ΠΎΠΉ Π‘) Π±ΡƒΠ΄Π΅Ρ‚:
Π‘ = Eo*S/d, Π³Π΄Π΅ Π•ΠΎ — элСктричСская постоянная (для Π²Π°ΠΊΡƒΡƒΠΌΠ°)
Π•ΠΎ=8,854 * 10 -12 Π€/ΠΌ (Π€Π°Ρ€Π°Π΄ Π½Π° ΠΌΠ΅Ρ‚Ρ€).
Если ΠΌΠ΅ΠΆΠ΄Ρƒ ΠΎΠ±ΠΊΠ»Π°Π΄ΠΊΠ°ΠΌΠΈ ΠΏΠΎΠΌΠ΅ΡΡ‚ΠΈΡ‚ΡŒ диэлСктрик,

Ρ‚ΠΎ Ρ‘ΠΌΠΊΠΎΡΡ‚ΡŒ кондСнсатора
Π±ΡƒΠ΄Π΅Ρ‚: Π‘ = Er * Eo *S / d. Π’ Ρ„ΠΎΡ€ΠΌΡƒΠ»Π΅ расчСта ёмкости добавилась
Π²Π΅Π»ΠΈΡ‡ΠΈΠ½Π° Er — ΠΎΡ‚Π½ΠΎΡΠΈΡ‚Π΅Π»ΡŒΠ½Π°Ρ диэлСктричСская ΠΏΡ€ΠΎΠ½ΠΈΡ†Π°Π΅ΠΌΠΎΡΡ‚ΡŒ
Π²Π²Π΅Π΄Ρ‘Π½Π½ΠΎΠ³ΠΎ диэлСктрика.
Из Ρ„ΠΎΡ€ΠΌΡƒΠ»Ρ‹ слСдуСт, Ρ‡Ρ‚ΠΎ Π΅ΠΌΠΊΠΎΡΡ‚ΡŒ кондСнсатора увСличиваСтся Π½Π°
Π²Π΅Π»ΠΈΡ‡ΠΈΠ½Ρƒ Er проницаСмости диэлСктрика. Π˜Ρ‚Π°ΠΊ, Ρ‡Π΅ΠΌ большС ΠΏΠ»ΠΎΡ‰Π°Π΄ΡŒ
S пластин кондСнсатора, большС Π·Π½Π°Ρ‡Π΅Π½ΠΈΠ΅ Er ΠΈ мСньшС расстояниС
d ΠΌΠ΅ΠΆΠ΄Ρƒ пластинами, Ρ‚Π΅ΠΌ большС Π΅ΠΌΠΊΠΎΡΡ‚ΡŒ кондСнсатора.
Основной Π΅Π΄ΠΈΠ½ΠΈΡ†Π΅ΠΉ Смкости Π² систСмС Π΅Π΄ΠΈΠ½ΠΈΡ† БИ являСтся Ρ„Π°Ρ€Π°Π΄
(Π€). Π•ΠΌΠΊΠΎΡΡ‚ΡŒ 1Π€ ΠΎΡ‡Π΅Π½ΡŒ Π²Π΅Π»ΠΈΠΊΠ°. Π’ элСктротСхникС ΠΎΠ±Ρ‹Ρ‡Π½ΠΎ
ΠΈΡΠΏΠΎΠ»ΡŒΠ·ΡƒΡŽΡ‚ Π΄ΠΎΠ»ΡŒΠ½Ρ‹Π΅ Π΅Π΄ΠΈΠ½ΠΈΡ†Ρ‹ Смкости:
ΠΌΠΈΠΊΡ€ΠΎΡ„Π°Ρ€Π°Π΄Π° (ΠΌΠΊΠ€), 1ΠΌΠΊΠ€ = 1*10 -6 Π€,
Π½Π°Π½ΠΎΡ„Π°Ρ€Π°Π΄Π° (Π½Π€), 1Π½Π€ = 1*10 -9 Π€, ΠΈ
ΠΏΠΈΠΊΠΎΡ„Π°Ρ€Π°Π΄Π° (ΠΏΠ€), 1ΠΏΠ€ = 1*10 -12 Π€.

ΠŸΡ€ΠΈ Π²Ρ‹Π±ΠΎΡ€Π΅ диэлСктрика для кондСнсаторов, ΠΊΡ€ΠΎΠΌΠ΅ ΠΎΡ‚Π½ΠΎΡΠΈΡ‚Π΅Π»ΡŒΠ½ΠΎΠΉ
диэлСктричСской проницаСмости диэлСктрика, ΡƒΡ‡ΠΈΡ‚Ρ‹Π²Π°ΡŽΡ‚ Π΅Ρ‰Π΅ Π΄Π²Π°
Π²Π°ΠΆΠ½Ρ‹Ρ… ΠΏΠ°Ρ€Π°ΠΌΠ΅Ρ‚Ρ€Π°:
1) Π­Π»Π΅ΠΊΡ‚Ρ€ΠΈΡ‡Π΅ΡΠΊΡƒΡŽ ΠΏΡ€ΠΎΡ‡Π½ΠΎΡΡ‚ΡŒ — ΠΏΡ€ΠΎΡ‡Π½ΠΎΡΡ‚ΡŒ диэлСктрика ΠΏΡ€ΠΈ ΠΏΠΎΠ΄Π°Ρ‡Π΅
Π½Π° ΠΏΡ€ΠΎΠΊΠ»Π°Π΄ΠΊΠΈ кондСнсатора высокого напряТСния. ΠŸΡ€ΠΈ Π½ΠΈΠ·ΠΊΠΎΠΉ
элСктричСской прочности ΠΌΠΎΠΆΠ΅Ρ‚ ΠΏΡ€ΠΎΠΈΠ·ΠΎΠΉΡ‚ΠΈ элСктричСский ΠΏΡ€ΠΎΠ±ΠΎΠΉ,
ΠΈ диэлСктрик станСт ΠΏΡ€ΠΎΠ²ΠΎΠ΄Π½ΠΈΠΊΠΎΠΌ элСктричСского Ρ‚ΠΎΠΊΠ°;
2) УдСльноС объСмноС сопротивлСниС — элСктричСскоС
сопротивлСниС диэлСктрика постоянному Ρ‚ΠΎΠΊΡƒ. Π§Π΅ΠΌ большС ΡƒΠ΄Π΅Π»ΡŒΠ½ΠΎΠ΅
сопротивлСниС диэлСктрика, Ρ‚Π΅ΠΌ мСньшС ΡƒΡ‚Π΅Ρ‡ΠΊΠ° Π½Π°ΠΊΠΎΠΏΠ»Π΅Π½Π½Ρ‹Ρ…
зарядов Π² кондСнсаторС.

ΠšΠžΠΠ”Π•ΠΠ‘ΠΠ’ΠžΠ  Π’ Π¦Π•ΠŸΠ˜ ΠŸΠžΠ‘Π’ΠžΠ―ΠΠΠžΠ“Πž ВОКА.

На Π³Ρ€Π°Ρ„ΠΈΠΊΠ΅ Π½Π°ΠΊΠΎΠΏΠ»Π΅Π½ΠΈΠ΅ заряда кондСнсатором выглядит ΠΊΠ°ΠΊ
ΠΏΠΎΠΊΠ°Π·Π°Π½ΠΎ Π½Π° рисункС 1.

ВрСмя заряда кондСнсатора зависит ΠΎΡ‚
ёмкости кондСнсатора (ΠΏΡ€ΠΈ ΠΎΠ΄ΠΈΠ½Π°ΠΊΠΎΠ²ΠΎΠΌ ΠΏΡ€ΠΈΠ»ΠΎΠΆΠ΅Π½Π½ΠΎΠΌ напряТСнии).
Π§Π΅ΠΌ большС Ρ‘ΠΌΠΊΠΎΡΡ‚ΡŒ кондСнсатора, Ρ‚Π΅ΠΌ большС врСмя заряда.
Аналогичная ΠΊΠ°Ρ€Ρ‚ΠΈΠ½Π° (Рис. 2) Π½Π°Π±Π»ΡŽΠ΄Π°Π΅Ρ‚ΡΡ ΠΏΡ€ΠΈ разрядкС
кондСнсатора Π½Π° сопротивлСниС. ΠŸΡ€ΠΈ ΠΎΠ΄ΠΈΠ½Π°ΠΊΠΎΠ²ΠΎΠΌ сопротивлСнии
врСмя разряда большС Ρƒ кондСнсатора с большСй Ρ‘ΠΌΠΊΠΎΡΡ‚ΡŒΡŽ.

ΠšΠžΠΠ”Π•ΠΠ‘ΠΠ’ΠžΠ  Π’ Π¦Π•ΠŸΠ˜ ΠŸΠ•Π Π•ΠœΠ•ΠΠΠžΠ“Πž ВОКА.

Если напряТСниС ΠΏΡ€ΠΈΠ»ΠΎΠΆΠ΅Π½Π½ΠΎΠ΅ ΠΊ Смкостному элСмСнту, Π±ΡƒΠ΄Π΅Ρ‚
ΠΈΠ·ΠΌΠ΅Π½ΡΡ‚ΡŒΡΡ ΠΏΠΎ Π°ΠΌΠΏΠ»ΠΈΡ‚ΡƒΠ΄Π΅ (ΠΏΠ΅Ρ€Π΅ΠΌΠ΅Π½Π½ΠΎΠ΅ напряТСниС),Ρ‚ΠΎ Π±ΡƒΠ΄Π΅Ρ‚
ΠΈΠ·ΠΌΠ΅Π½ΡΡ‚ΡŒΡΡ ΠΈ заряд кондСнсатора, Ρ‚ΠΎ Π΅ΡΡ‚ΡŒ Π² Смкостном элСмСнтС
появится Ρ‚ΠΎΠΊ.

Π’ΠΎΠΊ Ic проходящий Ρ‡Π΅Ρ€Π΅Π· кондСнсатор зависит ΠΎΡ‚
частоты f ΠΏΡ€ΠΈΠ»ΠΎΠΆΠ΅Π½Π½ΠΎΠ³ΠΎ ΠΏΠ΅Ρ€Π΅ΠΌΠ΅Π½Π½ΠΎΠ³ΠΎ напряТСния ΠΈ ёмкости Π‘
кондСнсатора. Если для постоянного Ρ‚ΠΎΠΊΠ° сопротивлСниС
кондСнсатора ΠΌΠΎΠΆΠ½ΠΎ ΡΡ‡ΠΈΡ‚Π°Ρ‚ΡŒ Ρ€Π°Π²Π½Ρ‹ΠΌ бСсконСчности, Ρ‚ΠΎ для
ΠΏΠ΅Ρ€Π΅ΠΌΠ΅Π½Π½ΠΎΠ³ΠΎ Ρ‚ΠΎΠΊΠ° кондСнсатор ΠΎΠ±Π»Π°Π΄Π°Π΅Ρ‚ ΠΎΠΏΡ€Π΅Π΄Π΅Π»Ρ‘Π½Π½Ρ‹ΠΌ
сопротивлСниСм. Π‘ΠΎΠΏΡ€ΠΎΡ‚ΠΈΠ²Π»Π΅Π½ΠΈΠ΅ кондСнсатора ΠΏΠ΅Ρ€Π΅ΠΌΠ΅Π½Π½ΠΎΠΌΡƒ Ρ‚ΠΎΠΊΡƒ
Rc рассчитываСтся ΠΏΠΎ Ρ„ΠΎΡ€ΠΌΡƒΠ»Π΅ ΠΏΠΎΠΊΠ°Π·Π°Π½Π½ΠΎΠΉ Π½Π° рисункС.
Π’ Ρ„ΠΎΡ€ΠΌΡƒΠ»Π΅ расчСта Смкостного сопротивлСния ΠΏΠ΅Ρ€Π΅ΠΌΠ΅Π½Π½ΠΎΠΌΡƒ Ρ‚ΠΎΠΊΡƒ
частота выраТаСтся Π² Π³Π΅Ρ€Ρ†Π°Ρ…, Π° Π΅ΠΌΠΊΠΎΡΡ‚ΡŒ кондСнсатора Π²
Ρ„Π°Ρ€Π°Π΄Π°Ρ…. Из Ρ„ΠΎΡ€ΠΌΡƒΠ»Ρ‹ Π²ΠΈΠ΄Π½ΠΎ, Ρ‡Ρ‚ΠΎ с ΡƒΠ²Π΅Π»ΠΈΡ‡Π΅Π½ΠΈΠ΅ΠΌ частоты f ΠΏΡ€ΠΈ
Π½Π΅ΠΈΠ·ΠΌΠ΅Π½Π½ΠΎΠΉ Смкости кондСнсатора сопротивлСниС Rc сниТаСтся,
Π°Π½Π°Π»ΠΎΠ³ΠΈΡ‡Π½ΠΎ с ΡƒΠ²Π΅Π»ΠΈΡ‡Π΅Π½ΠΈΠ΅ΠΌ Смкости кондСнсатора ΠΏΡ€ΠΈ Π½Π΅ΠΈΠ·ΠΌΠ΅Π½Π½ΠΎΠΉ
частотС сопротивлСниС Rc Ρ‚Π°ΠΊ ΠΆΠ΅ сниТаСтся.
ΠšΠΎΠ½Π΄Π΅Π½ΡΠ°Ρ‚ΠΎΡ€Ρ‹, Ρ‚Π°ΠΊ ΠΆΠ΅ ΠΊΠ°ΠΊ ΠΈ рСзисторы, для получСния Π·Π°Π΄Π°Π½Π½ΠΎΠΉ
Смкости Π‘ΠΎ ΠΌΠΎΠΆΠ½ΠΎ Π²ΠΊΠ»ΡŽΡ‡Π°Ρ‚ΡŒ ΠΏΠ°Ρ€Π°Π»Π»Π΅Π»ΡŒΠ½ΠΎ ΠΈ ΠΏΠΎΡΠ»Π΅Π΄ΠΎΠ²Π°Ρ‚Π΅Π»ΡŒΠ½ΠΎ.
Π€ΠΎΡ€ΠΌΡƒΠ»Ρ‹ расчСта Ρ€Π΅Π·ΡƒΠ»ΡŒΡ‚ΠΈΡ€ΡƒΡŽΡ‰Π΅ΠΉ Смкости ΠΏΠΎΠΊΠ°Π·Π°Π½Ρ‹ Π½Π° рисункС.

КОНБВРУКЦИЯ, ΠŸΠΠ ΠΠœΠ•Π’Π Π« И ВИПЫ ΠšΠžΠΠ”Π•ΠΠ‘ΠΠ’ΠžΠ ΠžΠ’.

ΠŸΡ€Π΅Π΄ΠΏΠΎΠ»ΠΎΠΆΠΈΠΌ, Ρ‡Ρ‚ΠΎ ΠΌΡ‹ конструируСм кондСнсатор ΠΈ ΠΏΠΎΠΏΡ€ΠΎΠ±ΡƒΠ΅ΠΌ,
ΡƒΠΆΠ΅ обладая ΠΎΠΏΡ€Π΅Π΄Π΅Π»Π΅Π½Π½Ρ‹ΠΌΠΈ знаниями, Ρ€Π°ΡΡΡ‡ΠΈΡ‚Π°Ρ‚ΡŒ Π΅ΠΌΠΊΠΎΡΡ‚ΡŒ
кондСнсатора. Как извСстно, Π΅ΠΌΠΊΠΎΡΡ‚ΡŒ кондСнсатора зависит ΠΎΡ‚
ΠΏΠ»ΠΎΡ‰Π°Π΄ΠΈ ΠΎΠ±ΠΊΠ»Π°Π΄ΠΎΠΊ S, расстояния ΠΌΠ΅ΠΆΠ΄Ρƒ ΠΎΠ±ΠΊΠ»Π°Π΄ΠΊΠ°ΠΌΠΈ d ΠΈ
диэлСктричСской проницаСмости примСняСмого диэлСктрика Er.
Обкладки кондСнсатора ΠΈΠ·Π³ΠΎΡ‚Π°Π²Π»ΠΈΠ²Π°ΡŽΡ‚ΡΡ ΠΈΠ· ΠΌΠ΅Ρ‚Π°Π»Π»ΠΎΠ² с Ρ…ΠΎΡ€ΠΎΡˆΠ΅ΠΉ
элСктричСской ΠΏΡ€ΠΎΠ²ΠΎΠ΄ΠΈΠΌΠΎΡΡ‚ΡŒΡŽ — алюминий, мСдь, сСрСбро, Π·ΠΎΠ»ΠΎΡ‚ΠΎ.
Π•ΠΌΠΊΠΎΡΡ‚ΡŒ кондСнсатора Π½Π΅ зависит ΠΎΡ‚ Ρ‚ΠΎΠ»Ρ‰ΠΈΠ½Ρ‹ ΠΎΠ±ΠΊΠ»Π°Π΄ΠΎΠΊ, поэтому
Ρ‡Π΅ΠΌ Ρ‚ΠΎΠ½ΡŒΡˆΠ΅ ΠΎΠ±ΠΊΠ»Π°Π΄ΠΊΠΈ кондСнсатора, Ρ‚Π΅ΠΌ Π»ΡƒΡ‡ΡˆΠ΅ — экономим ΠΌΠ΅Ρ‚Π°Π»Π»
ΠΈ ΡƒΠΌΠ΅Π½ΡŒΡˆΠ°Π΅ΠΌ гСомСтричСский ΠΎΠ±ΡŠΡ‘ΠΌ кондСнсатора.

РасстояниС d Π½Π΅ Π΄ΠΎΠ»ΠΆΠ½ΠΎ Π±Ρ‹Ρ‚ΡŒ слишком ΠΌΠ°Π»Ρ‹ΠΌ, Π²ΠΎ ΠΈΠ·Π±Π΅ΠΆΠ°Π½ΠΈΠ΅
элСктричСского пробоя диэлСктрика.
Π’Ρ‹Π±Π΅Ρ€Π΅ΠΌ Π² качСствС диэлСктрика Π½Π°ΠΈΠ±ΠΎΠ»Π΅Π΅ распространСнный
ΠΌΠ°Ρ‚Π΅Ρ€ΠΈΠ°Π» — гСтинакс с Er Ρ€Π°Π²Π½ΠΎΠΉ 6 … 8. ΠŸΡ€ΠΈΠΌΠ΅ΠΌ Er для
нашСго кондСнсатора Ρ€Π°Π²Π½ΠΎΠΉ 7.

ΠŸΠ»ΠΎΡ‰Π°Π΄ΡŒ S вычисляСтся для ΠΎΠ΄Π½ΠΎΠΉ ΠΎΠ±ΠΊΠ»Π°Π΄ΠΊΠΈ кондСнсатора ΠΏΡ€ΠΈ условии,
Ρ‡Ρ‚ΠΎ Π»ΠΈΠ½Π΅ΠΉΠ½Ρ‹Π΅ Ρ€Π°Π·ΠΌΠ΅Ρ€Ρ‹ ΠΎΠ±ΠΊΠ»Π°Π΄ΠΎΠΊ ΠΎΠ΄ΠΈΠ½Π°ΠΊΠΎΠ²Ρ‹.
Если ΠΎΠ΄Π½Π° ΠΈΠ· ΠΎΠ±ΠΊΠ»Π°Π΄ΠΎΠΊ ΠΈΠΌΠ΅Π΅Ρ‚ мСньшиС Π΄Π»ΠΈΠ½Ρƒ ΠΈΠ»ΠΈ ΡˆΠΈΡ€ΠΈΠ½Ρƒ Ρ‚ΠΎ ΠΏΠ»ΠΎΡ‰Π°Π΄ΡŒ
вычисляСтся для мСньшСй ΠΎΠ±ΠΊΠ»Π°Π΄ΠΊΠΈ.
ВсС Ρ€Π°Π·ΠΌΠ΅Ρ€Ρ‹ — Π΄Π»ΠΈΠ½Π° ΠΈ ΡˆΠΈΡ€ΠΈΠ½Π° ΠΎΠ±ΠΊΠ»Π°Π΄ΠΎΠΊ ΠΈ расстояниС ΠΌΠ΅ΠΆΠ΄Ρƒ Π½ΠΈΠΌΠΈ
Π΄ΠΎΠ»ΠΆΠ½Ρ‹ Π±Ρ‹Ρ‚ΡŒ Π²Ρ‹Ρ€Π°ΠΆΠ΅Π½Ρ‹ Π² ΠΌΠ΅Ρ‚Ρ€Π°Ρ…. ΠŸΡ€ΠΈΠΌΠ΅ΠΌ Ρ€Π°Π·ΠΌΠ΅Ρ€Ρ‹ Ρ‚Π°ΠΊΠΈΠ΅, ΠΊΠ°ΠΊΠΈΠ΅
ΠΏΠΎΠΊΠ°Π·Π°Π½Ρ‹ Π½Π° рисункС. ΠŸΠΎΠ΄ΡΡ‚Π°Π²ΠΈΠΌ Π² Ρ„ΠΎΡ€ΠΌΡƒΠ»Ρƒ расчСта Смкости
кондСнсатора наши Π΄Π°Π½Π½Ρ‹Π΅: C = Er * Eo * S / d;
C = 7 * 8.854*10 -12 * 0.0025 / 0.001= 0.000000000155Π€ (Ρ„Π°Ρ€Π°Π΄Ρ‹).
Π’ΠΎΠ·Π²Π΅Π΄Π΅ΠΌ ΠΏΠΎΠ»ΡƒΡ‡Π΅Π½Π½Ρ‹ΠΉ Ρ€Π΅Π·ΡƒΠ»ΡŒΡ‚Π°Ρ‚ Π² 12 ΡΡ‚Π΅ΠΏΠ΅Π½ΡŒ Ρ‡Ρ‚ΠΎΠ±Ρ‹ ΠΏΠΎΠ»ΡƒΡ‡ΠΈΡ‚ΡŒ
Π·Π½Π°Ρ‡Π΅Π½ΠΈΠ΅ Смкости Π² ΠΏΠΈΠΊΠΎΡ„Π°Ρ€Π°Π΄Π°Ρ…:
C = 0.000000000155 12 = 155ΠΏΠ€.
ΠŸΠΎΠ»ΡƒΡ‡Π΅Π½Π½Π°Ρ Π½Π°ΠΌΠΈ Ρ‘ΠΌΠΊΠΎΡΡ‚ΡŒ кондСнсатора 155ΠΏΡ„ ΠΎΡ‡Π΅Π½ΡŒ ΠΌΠ°Π»Π°, ΠΎΠ±Ρ‹Ρ‡Π½ΠΎ
Ρ‚Π°ΠΊΠΈΠ΅ ёмкости ΠΈΡΠΏΠΎΠ»ΡŒΠ·ΡƒΡŽΡ‚ΡΡ Π² Π°ΠΏΠΏΠ°Ρ€Π°Ρ‚ΡƒΡ€Π΅ Ρ€Π°Π±ΠΎΡ‚Π°ΡŽΡ‰Π΅ΠΉ Π½Π° высоких
частотах ΠΏΠ΅Ρ€Π΅ΠΌΠ΅Π½Π½ΠΎΠ³ΠΎ Ρ‚ΠΎΠΊΠ° порядка 1 — 600 ΠœΠ“Ρ† (ΠΌΠ΅Π³Π°Π³Π΅Ρ€Ρ†).
ΠŸΡ€Π΅Π΄ΡΡ‚Π°Π²ΡŒΡ‚Π΅ сСбС, Ρ‡Ρ‚ΠΎ ΠΌΡ‹ Ρ€Π°Π·Ρ€Π°Π±Π°Ρ‚Ρ‹Π²Π°Π΅ΠΌ ΠΌΠΈΠ½ΠΈΠ°Ρ‚ΡŽΡ€Π½Ρ‹ΠΉ ΠΊΠ°Ρ€ΠΌΠ°Π½Π½Ρ‹ΠΉ
Ρ€Π°Π΄ΠΈΠΎΠΏΡ€ΠΈΠ΅ΠΌΠ½ΠΈΠΊ Π² ΠΊΠΎΡ‚ΠΎΡ€ΠΎΠΌ трСбуСтся порядка 30 Ρ‚Π°ΠΊΠΈΡ… кондСнсаторов.

Если ΠΌΡ‹ установим Π² схСму 30 Ρ€Π°Π·Ρ€Π°Π±ΠΎΡ‚Π°Π½Π½Ρ‹Ρ… Π½Π°ΠΌΠΈ кондСнсаторов,
Π½Π΅ считая Π΄Ρ€ΡƒΠ³ΠΈΡ… Π½Π΅ΠΎΠ±Ρ…ΠΎΠ΄ΠΈΠΌΡ‹Ρ… Ρ€Π°Π΄ΠΈΠΎΠ΄Π΅Ρ‚Π°Π»Π΅ΠΉ, Ρ‚ΠΎ наш Ρ€Π°Π΄ΠΈΠΎΠΏΡ€ΠΈΠ΅ΠΌΠ½ΠΈΠΊ
Π½ΠΈΠΊΠ°ΠΊ Π½Π΅ получится ΠΌΠΈΠ½ΠΈΠ°Ρ‚ΡŽΡ€Π½Ρ‹ΠΌ. ВсС Π΄Π΅Π»ΠΎ Π² Ρ‚ΠΎΠΌ, Ρ‡Ρ‚ΠΎ ΠΎΠ±ΡŠΡ‘ΠΌ
Ρ‚ΠΎΠ»ΡŒΠΊΠΎ Π½Π°ΡˆΠΈΡ… кондСнсаторов получится Ρ‚Π°ΠΊΠΈΠΌ, Ρ‡Ρ‚ΠΎ Π΅Π³ΠΎ Π½ΠΈΠΊΠ°ΠΊ
нСльзя Π±ΡƒΠ΄Π΅Ρ‚ Π½Π°Π·Π²Π°Ρ‚ΡŒ ΠΏΡ€ΠΈΠ΅ΠΌΠ»Π΅ΠΌΡ‹ΠΌ.
ОбъСм ΠΎΠ΄Π½ΠΎΠ³ΠΎ кондСнсатора Vc Ρ€Π°Π²Π΅Π½ Vc = 5см * 5см * 0,1см
Vc = 2,5см Π² ΠΊΡƒΠ±Π΅. Π’ΠΎΠ³Π΄Π° объСм 30 кондСнсаторов Π±ΡƒΠ΄Π΅Ρ‚ Ρ€Π°Π²Π΅Π½:
V = 30 * 2,5 = 75см Π² ΠΊΡƒΠ±Π΅.
Π§Ρ‚ΠΎ Π΄Π΅Π»Π°Ρ‚ΡŒ, ΠΊΠ°ΠΊ Π±Ρ‹Ρ‚ΡŒ, ΠΊΠ°ΠΊ ΡƒΠΌΠ΅Π½ΡŒΡˆΠΈΡ‚ΡŒ гСомСтричСский объСм
кондСнсатора для примСнСния Π² ΠΌΠΈΠ½ΠΈΠ°Ρ‚ΡŽΡ€Π½ΠΎΠΉ Ρ€Π°Π΄ΠΈΠΎΠ°ΠΏΠΏΠ°Ρ€Π°Ρ‚ΡƒΡ€Π΅?
Для Ρ€Π΅ΡˆΠ΅Π½ΠΈΡ этой ΠΏΡ€ΠΎΠ±Π»Π΅ΠΌΡ‹ максимально ΡƒΠΌΠ΅Π½ΡŒΡˆΠ°ΡŽΡ‚ расстояниС
ΠΌΠ΅ΠΆΠ΄Ρƒ ΠΎΠ±ΠΊΠ»Π°Π΄ΠΊΠ°ΠΌΠΈ, Ρ‚ΠΎΠ³Π΄Π° увСличиваСтся Π΅ΠΌΠΊΠΎΡΡ‚ΡŒ ΠΈ ΡƒΠΌΠ΅Π½ΡŒΡˆΠ°Π΅Ρ‚ΡΡ
гСомСтричСский объСм кондСнсатора. Но расстояниС ΡƒΠΌΠ΅Π½ΡŒΡˆΠ°ΡŽΡ‚
Π΄ΠΎ ΠΎΠΏΡ€Π΅Π΄Π΅Π»Π΅Π½Π½Ρ‹Ρ… ΠΏΡ€Π΅Π΄Π΅Π»ΠΎΠ² ΠΈΠ½Π°Ρ‡Π΅ кондСнсатор Π±ΡƒΠ΄Π΅Ρ‚ ΠΏΡ€ΠΎΠ±ΠΈΠ²Π°Ρ‚ΡŒΡΡ
Π΄Π°ΠΆΠ΅ ΠΏΡ€ΠΈ Π½ΠΈΠ·ΠΊΠΎΠΌ напряТСнии ΠΏΠΎΠ΄Π°Π²Π°Π΅ΠΌΠΎΠΌ Π½Π° кондСнсатор. Π’ связи
с этим Π½Π° ΠΊΠ°ΠΆΠ΄ΠΎΠΌ кондСнсаторС указываСтся напряТСниС ΠΊΠΎΡ‚ΠΎΡ€ΠΎΠ΅
ΠΎΠ½ ΠΌΠΎΠΆΠ΅Ρ‚ Π²Ρ‹Π΄Π΅Ρ€ΠΆΠ°Ρ‚ΡŒ.

Для ΡƒΠΌΠ΅Π½ΡŒΡˆΠ΅Π½ΠΈΡ ΠΏΠ»ΠΎΡ‰Π°Π΄ΠΈ ΠΎΠ±ΠΊΠ»Π°Π΄ΠΎΠΊ кондСнсатор Π΄Π΅Π»Π°ΡŽΡ‚ многослойным
состоящим ΠΊΠ°ΠΊ Π±Ρ‹ ΠΈΠ· Π½Π΅ΡΠΊΠΎΠ»ΡŒΠΊΠΈΡ… ΠΏΠ°Ρ€Π°Π»Π»Π΅Π»ΡŒΠ½ΠΎ Π²ΠΊΠ»ΡŽΡ‡Π΅Π½Π½Ρ‹Ρ… кондСнсаторов
(вспомнитС Ρ„ΠΎΡ€ΠΌΡƒΠ»Ρƒ ΠΏΠ°Ρ€Π°Π»Π»Π΅Π»ΡŒΠ½ΠΎΠ³ΠΎ Π²ΠΊΠ»ΡŽΡ‡Π΅Π½ΠΈΡ кондСнсаторов).
Π’ качСствС диэлСктрика Π² ΠΌΠΈΠ½ΠΈΠ°Ρ‚ΡŽΡ€Π½Ρ‹Ρ… кондСнсаторах ΠΈΡΠΏΠΎΠ»ΡŒΠ·ΡƒΡŽΡ‚
Ρ‚ΠΎΠ½ΠΊΠΈΠ΅ ΠΏΠ»Π΅Π½ΠΊΠΈ ΠΈΠ· синтСтичСских ΠΌΠ°Ρ‚Π΅Ρ€ΠΈΠ°Π»ΠΎΠ², Π° Π² качСствС ΠΎΠ±ΠΊΠ»Π°Π΄ΠΎΠΊ
ΠΌΠ΅Ρ‚Π°Π»Π»ΠΈΡ‡Π΅ΡΠΊΡƒΡŽ Ρ„ΠΎΠ»ΡŒΠ³Ρƒ, Ρ‡Π°Ρ‰Π΅ всСго ΠΈΠ· алюминия.

На корпусС кондСнсатора, ΠΎΠ±Ρ‹Ρ‡Π½ΠΎ, указываСтся Π΅Π³ΠΎ Ρ‚ΠΈΠΏ, Π΅ΠΌΠΊΠΎΡΡ‚ΡŒ ΠΈ
Ρ€Π°Π±ΠΎΡ‡Π΅Π΅ напряТСниС. ΠžΡΡ‚Π°Π»ΡŒΠ½Ρ‹Π΅ ΠΏΠ°Ρ€Π°ΠΌΠ΅Ρ‚Ρ€Ρ‹ кондСнсатора
ΠΎΠΏΡ€Π΅Π΄Π΅Π»ΡΡŽΡ‚ΡΡ ΠΈΠ· справочников. Π•ΠΌΠΊΠΎΡΡ‚ΡŒ кондСнсатора указываСтся
Π½Π΅ Ρ‚Π°ΠΊ, ΠΊΠ°ΠΊ Π½Π° элСктричСских схСмах. НапримСр Π΅ΠΌΠΊΠΎΡΡ‚ΡŒ 2,2ΠΏΠ€
обозначаСтся 2П2, Π΅ΠΌΠΊΠΎΡΡ‚ΡŒ 1500 ΠΏΠ€ — 1Н5, Π΅ΠΌΠΊΠΎΡΡ‚ΡŒ 0,1 ΠΌΠΊΠ€ — М1,
Π΅ΠΌΠΊΠΎΡΡ‚ΡŒ 2,2 ΠΌΠΊΠ€ — 2М2, Π΅ΠΌΠΊΠΎΡΡ‚ΡŒ 10 ΠΌΠΊΠ€ — 10М.
Π£ ΠΎΠ±Ρ‹Ρ‡Π½Ρ‹Ρ… кондСнсаторов КМ, ΠšΠ”, ΠœΠ‘Πœ ΠΈ Ρ‚Π°ΠΊ Π΄Π°Π»Π΅Π΅ Ρ‚Ρ€ΡƒΠ΄Π½ΠΎ ΠΏΠΎΠ»ΡƒΡ‡ΠΈΡ‚ΡŒ
Π±ΠΎΠ»ΡŒΡˆΡƒΡŽ Ρ‘ΠΌΠΊΠΎΡΡ‚ΡŒ ΠΏΡ€ΠΈ ΠΌΠ°Π»Ρ‹Ρ… Π³Π°Π±Π°Ρ€ΠΈΡ‚Π°Ρ… поэтому Π±Ρ‹Π»ΠΈ Ρ€Π°Π·Ρ€Π°Π±ΠΎΡ‚Π°Π½Ρ‹
Ρ‚Π°ΠΊ Π½Π°Π·Ρ‹Π²Π°Π΅ΠΌΡ‹Π΅ элСктролитичСскиС кондСнсаторы Ρƒ ΠΊΠΎΡ‚ΠΎΡ€Ρ‹Ρ… Π²
качСствС диэлСктрика ΠΈΡΠΏΠΎΠ»ΡŒΠ·ΡƒΠ΅Ρ‚ΡΡ ΡΠΏΠ΅Ρ†ΠΈΠ°Π»ΡŒΠ½Π°Ρ элСктролитичСская
ΠΆΠΈΠ΄ΠΊΠΎΡΡ‚ΡŒ с ΠΎΡ‡Π΅Π½ΡŒ большим Er. ΠΠΌΠΊΠΎΡΡ‚ΡŒ Ρ‚Π°ΠΊΠΈΡ… кондСнсаторов ΠΌΠΎΠΆΠ΅Ρ‚
Π΄ΠΎΡΡ‚ΠΈΠ³Π°Ρ‚ΡŒ сотСн тысяч ΠΌΠΈΠΊΡ€ΠΎΡ„Π°Ρ€Π°Π΄. К нСдостатку Ρ‚Π°ΠΊΠΈΡ… кондСнсаторов
слСдуСт отнСсти Π½ΠΈΠ·ΠΊΠΎΠ΅ Ρ€Π°Π±ΠΎΡ‡Π΅Π΅ напряТСниС (Π΄ΠΎ 500V) ΠΈ ΠΎΠ±ΡΠ·Π°Ρ‚Π΅Π»ΡŒΠ½ΠΎΠ΅
соблюдСниС полярности ΠΏΡ€ΠΈ Π²ΠΊΠ»ΡŽΡ‡Π΅Π½ΠΈΠΈ Π² схСму.
Для настройки ΠΈ подстройки Π½Π΅ΠΊΠΎΡ‚ΠΎΡ€Ρ‹Ρ… Ρ‚ΠΈΠΏΠΎΠ² Ρ€Π°Π΄ΠΈΠΎΠ°ΠΏΠΏΠ°Ρ€Π°Ρ‚ΡƒΡ€Ρ‹,
Π½Π°ΠΏΡ€ΠΈΠΌΠ΅Ρ€ Ρ€Π°Π΄ΠΈΠΎΠΏΡ€ΠΈΠ΅ΠΌΠ½ΠΈΠΊ ΠΈΠ»ΠΈ Ρ‚Π΅Π»Π΅Π²ΠΈΠ·ΠΎΡ€, ΠΏΡ€ΠΈΠΌΠ΅Π½ΡΡŽΡ‚ ΡΠΏΠ΅Ρ†ΠΈΠ°Π»ΡŒΠ½Ρ‹Π΅ кондСнсаторы
с измСняСмой Ρ‘ΠΌΠΊΠΎΡΡ‚ΡŒΡŽ.

Π’ зависимости ΠΎΡ‚ назначСния Ρ‚Π°ΠΊΠΈΠ΅ кондСнсаторы Π½Π°Π·Ρ‹Π²Π°ΡŽΡ‚ «ΠΏΠΎΠ΄ΡΡ‚Ρ€ΠΎΠ΅Ρ‡Π½Ρ‹Π΅»
ΠΈ «ΠΊΠΎΠ½Π΄Π΅Π½ΡΠ°Ρ‚ΠΎΡ€Ρ‹ ΠΏΠ΅Ρ€Π΅ΠΌΠ΅Π½Π½ΠΎΠΉ Смкости».
Π•ΠΌΠΊΠΎΡΡ‚ΡŒ ΠΏΠ΅Ρ€Π΅ΠΌΠ΅Π½Π½Ρ‹Ρ… ΠΈ подстроСчных кондСнсаторов измСняСтся
мСханичСским способом, ΠΏΡƒΡ‚Π΅ΠΌ измСнСния расстояния ΠΌΠ΅ΠΆΠ΄Ρƒ
ΠΎΠ±ΠΊΠ»Π°Π΄ΠΊΠ°ΠΌΠΈ ΠΈΠ»ΠΈ измСнСния ΠΏΠ»ΠΎΡ‰Π°Π΄ΠΈ пластин. Π’ качСствС
диэлСктрика Π² Ρ‚Π°ΠΊΠΈΡ… кондСнсаторах ΠΈΡΠΏΠΎΠ»ΡŒΠ·ΡƒΠ΅Ρ‚ΡΡ Π²ΠΎΠ·Π΄ΡƒΡ… ΠΈΠ»ΠΈ
Ρ„Π°Ρ€Ρ„ΠΎΡ€.
Π’ Π·Π°ΠΊΠ»ΡŽΡ‡Π΅Π½ΠΈΠ΅ слСдуСт ΠΎΡ‚ΠΌΠ΅Ρ‚ΠΈΡ‚ΡŒ, Ρ‡Ρ‚ΠΎ Π² настоящСС врСмя, Π² связи
с Π±ΡƒΡ€Π½Ρ‹ΠΌ Ρ€Π°Π·Π²ΠΈΡ‚ΠΈΠ΅ΠΌ радиоэлСктроники подстроСчныС ΠΈ ΠΏΠ΅Ρ€Π΅ΠΌΠ΅Π½Π½Ρ‹Π΅
кондСнсаторы практичСски Π½Π΅ ΠΏΡ€ΠΈΠΌΠ΅Π½ΡΡŽΡ‚ΡΡ. Π˜Ρ… с успСхом Π·Π°ΠΌΠ΅Π½ΡΡŽΡ‚
ΡΠΏΠ΅Ρ†ΠΈΠ°Π»ΡŒΠ½Ρ‹Π΅ Ρ„ΠΈΠ»ΡŒΡ‚Ρ€Ρ‹ ΠΈ ΠΏΠΎΠ»ΡƒΠΏΡ€ΠΎΠ²ΠΎΠ΄Π½ΠΈΠΊΠΎΠ²Ρ‹Π΅ ΠΏΡ€ΠΈΠ±ΠΎΡ€Ρ‹ ΠΊΠΎΡ‚ΠΎΡ€Ρ‹Π΅ Π½Π΅
Ρ‚Ρ€Π΅Π±ΡƒΡŽΡ‚ мСханичСского измСнСния ΠΏΠ°Ρ€Π°ΠΌΠ΅Ρ‚Ρ€ΠΎΠ².

Π€ΠΎΡ€ΠΌΡƒΠ»Π° Смкости кондСнсатора Ρ‡Π΅Ρ€Π΅Π· напряТСниС. Зарядка кондСнсатора ΠΎΡ‚ источника постоянной эдс

По Π½Π°Π·Π½Π°Ρ‡Π΅Π½ΠΈΡŽ кондСнсатор ΠΌΠΎΠΆΠ½ΠΎ ΡΡ€Π°Π²Π½ΠΈΡ‚ΡŒ с Π±Π°Ρ‚Π°Ρ€Π΅ΠΉΠΊΠΎΠΉ. Но имССтся ΠΏΡ€ΠΈΠ½Ρ†ΠΈΠΏΠΈΠ°Π»ΡŒΠ½ΠΎΠ΅ ΠΎΡ‚Π»ΠΈΡ‡ΠΈΠ΅ Π² Ρ€Π°Π±ΠΎΡ‚Π΅ Π΄Π°Π½Π½Ρ‹Ρ… элСмСнтов. Π‘ΡƒΡ‰Π΅ΡΡ‚Π²ΡƒΡŽΡ‚ отличия Π² ΠΏΡ€Π΅Π΄Π΅Π»ΡŒΠ½ΠΎΠΉ Смкости ΠΈ скорости зарядки кондСнсатора ΠΈ Π±Π°Ρ‚Π°Ρ€Π΅ΠΉΠΊΠΈ.

Π€ΠΎΡ€ΠΌΡƒΠ»Π° заряда кондСнсатора

Π³Π΄Π΅ q – Π²Π΅Π»ΠΈΡ‡ΠΈΠ½Π° заряда ΠΎΠ΄Π½ΠΎΠΉ ΠΈΠ· ΠΎΠ±ΠΊΠ»Π°Π΄ΠΎΠΊ кондСнсатора, Π° – Ρ€Π°Π·Π½ΠΎΡΡ‚ΡŒ ΠΏΠΎΡ‚Π΅Π½Ρ†ΠΈΠ°Π»ΠΎΠ² ΠΌΠ΅ΠΆΠ΄Ρƒ Π΅Π³ΠΎ ΠΎΠ±ΠΊΠ»Π°Π΄ΠΊΠ°ΠΌΠΈ.

Π­Π»Π΅ΠΊΡ‚Ρ€ΠΎΠ΅ΠΌΠΊΠΎΡΡ‚ΡŒ кондСнсатора — это Π²Π΅Π»ΠΈΡ‡ΠΈΠ½Π°, которая зависит Ρ‚ΠΎ Ρ€Π°Π·ΠΌΠ΅Ρ€ΠΎΠ² ΠΈ устройства кондСнсатора.

Заряд Π½Π° пластинах плоского кондСнсатора Ρ€Π°Π²Π΅Π½:

Π³Π΄Π΅ – элСктричСская постоянная; – ΠΏΠ»ΠΎΡ‰Π°Π΄ΡŒ ΠΊΠ°ΠΆΠ΄ΠΎΠΉ (ΠΈΠ»ΠΈ наимСньшСй) пластины;
– расстояниС ΠΌΠ΅ΠΆΠ΄Ρƒ пластинами; – диэлСктричСская ΠΏΡ€ΠΎΠ½ΠΈΡ†Π°Π΅ΠΌΠΎΡΡ‚ΡŒ диэлСктрика, ΠΊΠΎΡ‚ΠΎΡ€Ρ‹ΠΉ находится ΠΌΠ΅ΠΆΠ΄Ρƒ пластинами кондСнсатора.

Заряд Π½Π° ΠΎΠ±ΠΊΠ»Π°Π΄ΠΊΠ°Ρ… цилиндричСского кондСнсатора вычисляСтся ΠΏΡ€ΠΈ ΠΏΠΎΠΌΠΎΡ‰ΠΈ Ρ„ΠΎΡ€ΠΌΡƒΠ»Ρ‹:

Π³Π΄Π΅ l – высота Ρ†ΠΈΠ»ΠΈΠ½Π΄Ρ€ΠΎΠ²; – радиус внСшнСй ΠΎΠ±ΠΊΠ»Π°Π΄ΠΊΠΈ;
– радиус Π²Π½ΡƒΡ‚Ρ€Π΅Π½Π½Π΅ΠΉ ΠΎΠ±ΠΊΠ»Π°Π΄ΠΊΠΈ.

Заряд Π½Π° ΠΎΠ±ΠΊΠ»Π°Π΄ΠΊΠ°Ρ… сфСричСского кондСнсатора Π½Π°ΠΉΠ΄Π΅ΠΌ ΠΊΠ°ΠΊ:

Заряд кондСнсатора связан с энСргиСй поля (W) Π²Π½ΡƒΡ‚Ρ€ΠΈ Π½Π΅Π³ΠΎ:

Из Ρ„ΠΎΡ€ΠΌΡƒΠ»Ρ‹ (6) слСдуСт, Ρ‡Ρ‚ΠΎ заряд ΠΌΠΎΠΆΠ½ΠΎ Π²Ρ‹Ρ€Π°Π·ΠΈΡ‚ΡŒ ΠΊΠ°ΠΊ:

Рассмотрим ΠΏΠΎΡΠ»Π΅Π΄ΠΎΠ²Π°Ρ‚Π΅Π»ΡŒΠ½ΠΎΠ΅ соСдинСниС ΠΈΠ· N кондСнсаторов (рис. 1).

Π—Π΄Π΅ΡΡŒ (рис.1) ΠΏΠΎΠ»ΠΎΠΆΠΈΡ‚Π΅Π»ΡŒΠ½Π°Ρ ΠΎΠ±ΠΊΠ»Π°Π΄ΠΊΠ° ΠΎΠ΄Π½ΠΎΠ³ΠΎ кондСнсатора соСдиняСтся с ΠΎΡ‚Ρ€ΠΈΡ†Π°Ρ‚Π΅Π»ΡŒΠ½ΠΎΠΉ ΠΎΠ±ΠΊΠ»Π°Π΄ΠΊΠΎΠΉ ΡΠ»Π΅Π΄ΡƒΡŽΡ‰Π΅Π³ΠΎ кондСнсатора. ΠŸΡ€ΠΈ Ρ‚Π°ΠΊΠΎΠΌ соСдинСнии, ΠΎΠ±ΠΊΠ»Π°Π΄ΠΊΠΈ сосСдних кондСнсаторов ΡΠΎΠ·Π΄Π°ΡŽΡ‚ Π΅Π΄ΠΈΠ½Ρ‹ΠΉ ΠΏΡ€ΠΎΠ²ΠΎΠ΄Π½ΠΈΠΊ. Π£ всСх кондСнсаторов, соСдинСнных ΠΏΠΎΡΠ»Π΅Π΄ΠΎΠ²Π°Ρ‚Π΅Π»ΡŒΠ½ΠΎ Π½Π° ΠΎΠ±ΠΊΠ»Π°Π΄ΠΊΠ°Ρ… ΠΈΠΌΠ΅ΡŽΡ‚ΡΡ Ρ€Π°Π²Π½Ρ‹Π΅ ΠΏΠΎ Π²Π΅Π»ΠΈΡ‡ΠΈΠ½Π΅ заряды.

ΠŸΡ€ΠΈ ΠΏΠ°Ρ€Π°Π»Π»Π΅Π»ΡŒΠ½ΠΎΠΌ соСдинСнии кондСнсаторов (рис.2), ΡΠΎΠ΅Π΄ΠΈΠ½ΡΡŽΡ‚ ΠΎΠ±ΠΊΠ»Π°Π΄ΠΊΠΈ, ΠΈΠΌΠ΅ΡŽΡ‰ΠΈΠ΅ заряды ΠΎΠ΄Π½ΠΎΠ³ΠΎ Π·Π½Π°ΠΊΠ°. Π‘ΡƒΠΌΠΌΠ°Ρ€Π½Ρ‹ΠΉ заряд соСдинСния (q) Ρ€Π°Π²Π΅Π½ суммС зарядов кондСнсаторов.

ΠŸΡ€ΠΈΠΌΠ΅Ρ€Ρ‹ Ρ€Π΅ΡˆΠ΅Π½ΠΈΡ Π·Π°Π΄Π°Ρ‡ ΠΏΠΎ Ρ‚Π΅ΠΌΠ΅ «Заряд кондСнсатора»

ru.solverbook.com

Π€ΠΎΡ€ΠΌΡƒΠ»Π° Смкости кондСнсатора, Π‘

Если q – Π²Π΅Π»ΠΈΡ‡ΠΈΠ½Π° заряда ΠΎΠ΄Π½ΠΎΠΉ ΠΈΠ· ΠΎΠ±ΠΊΠ»Π°Π΄ΠΎΠΊ кондСнсатора, Π° – Ρ€Π°Π·Π½ΠΎΡΡ‚ΡŒ ΠΏΠΎΡ‚Π΅Π½Ρ†ΠΈΠ°Π»ΠΎΠ² ΠΌΠ΅ΠΆΠ΄Ρƒ Π΅Π³ΠΎ ΠΎΠ±ΠΊΠ»Π°Π΄ΠΊΠ°ΠΌΠΈ, Ρ‚ΠΎ Π²Π΅Π»ΠΈΡ‡ΠΈΠ½Π° C, равная:

называСтся Π΅ΠΌΠΊΠΎΡΡ‚ΡŒΡŽ кондСнсатора. Π­Ρ‚ΠΎ постоянная Π²Π΅Π»ΠΈΡ‡ΠΈΠ½Π°, которая зависит Ρ‚ΠΎ Ρ€Π°Π·ΠΌΠ΅Ρ€ΠΎΠ² ΠΈ устройства кондСнсатора.

Рассмотрим Π΄Π²Π° ΠΎΠ΄ΠΈΠ½Π°ΠΊΠΎΠ²Ρ‹Ρ… кондСнсатора, Ρ€Π°Π·Π½ΠΈΡ†Π° ΠΌΠ΅ΠΆΠ΄Ρƒ ΠΊΠΎΡ‚ΠΎΡ€Ρ‹ΠΌ Π·Π°ΠΊΠ»ΡŽΡ‡Π°Π΅Ρ‚ΡΡ Ρ‚ΠΎΠ»ΡŒΠΊΠΎ Π² Ρ‚ΠΎΠΌ, Ρ‡Ρ‚ΠΎ ΠΌΠ΅ΠΆΠ΄Ρƒ ΠΎΠ±ΠΊΠ»Π°Π΄ΠΊΠ°ΠΌΠΈ ΠΎΠ΄Π½ΠΎΠ³ΠΎ Π²Π°ΠΊΡƒΡƒΠΌ (ΠΈΠ»ΠΈ часто говорят Π²ΠΎΠ·Π΄ΡƒΡ…), ΠΌΠ΅ΠΆΠ΄Ρƒ ΠΎΠ±ΠΊΠ»Π°Π΄ΠΊΠ°ΠΌΠΈ Π΄Ρ€ΡƒΠ³ΠΎΠ³ΠΎ находится диэлСктрик. Π’ Ρ‚Π°ΠΊΠΎΠΌ случаС ΠΏΡ€ΠΈ Ρ€Π°Π²Π½Ρ‹Ρ… зарядах Π½Π° кондСнсаторах Ρ€Π°Π·Π½ΠΎΡΡ‚ΡŒ ΠΏΠΎΡ‚Π΅Π½Ρ†ΠΈΠ°Π»ΠΎΠ² Π²ΠΎΠ·Π΄ΡƒΡˆΠ½ΠΎΠ³ΠΎ кондСнсатора Π±ΡƒΠ΄Π΅Ρ‚ Π² Ρ€Π°Π· мСньшС, Ρ‡Π΅ΠΌ ΠΌΠ΅ΠΆΠ΄Ρƒ ΠΎΠ±ΠΊΠ»Π°Π΄ΠΊΠ°ΠΌΠΈ Π²Ρ‚ΠΎΡ€ΠΎΠ³ΠΎ. Π—Π½Π°Ρ‡ΠΈΡ‚ Π΅ΠΌΠΊΠΎΡΡ‚ΡŒ кондСнсатора с диэлСктриком (C) Π² Ρ€Π°Π· большС, Ρ‡Π΅ΠΌ Π²ΠΎΠ·Π΄ΡƒΡˆΠ½ΠΎΠ³ΠΎ ():

Π³Π΄Π΅ – диэлСктричСская ΠΏΡ€ΠΎΠ½ΠΈΡ†Π°Π΅ΠΌΠΎΡΡ‚ΡŒ диэлСктрика.

Π—Π° Π΅Π΄ΠΈΠ½ΠΈΡ†Ρƒ Смкости кондСнсатора ΠΏΡ€ΠΈΠ½ΠΈΠΌΠ°ΡŽΡ‚ Π΅ΠΌΠΊΠΎΡΡ‚ΡŒ Ρ‚Π°ΠΊΠΎΠ³ΠΎ кондСнсатора, ΠΊΠΎΡ‚ΠΎΡ€Ρ‹ΠΉ Π΅Π΄ΠΈΠ½ΠΈΡ‡Π½Ρ‹ΠΌ зарядом (1 Кл) заряТаСтся Π΄ΠΎ разности ΠΏΠΎΡ‚Π΅Π½Ρ†ΠΈΠ°Π»ΠΎΠ², Ρ€Π°Π²Π½ΠΎΠΉ ΠΎΠ΄Π½ΠΎΠΌΡƒ Π²ΠΎΠ»ΡŒΡ‚Ρƒ (Π² БИ). Π•Π΄ΠΈΠ½ΠΈΡ†Π΅ΠΉ Смкости кондСнсатора (ΠΊΠ°ΠΊ ΠΈ любой эклСктичСской Смкости) Π² ΠΌΠ΅ΠΆΠ΄ΡƒΠ½Π°Ρ€ΠΎΠ΄Π½ΠΎΠΉ систСмС Π΅Π΄ΠΈΠ½ΠΈΡ† (БИ) слуТит Ρ„Π°Ρ€Π°Π΄ (Π€).

Π€ΠΎΡ€ΠΌΡƒΠ»Π° элСктричСской Смкости плоского кондСнсатора

ПолС ΠΌΠ΅ΠΆΠ΄Ρƒ ΠΎΠ±ΠΊΠ»Π°Π΄ΠΊΠ°ΠΌΠΈ плоского кондСнсатора ΠΎΠ±Ρ‹Ρ‡Π½ΠΎ ΡΡ‡ΠΈΡ‚Π°ΡŽΡ‚ ΠΎΠ΄Π½ΠΎΡ€ΠΎΠ΄Π½Ρ‹ΠΌ. Π•Π³ΠΎ ΠΎΠ΄Π½ΠΎΡ€ΠΎΠ΄Π½ΠΎΡΡ‚ΡŒ Π½Π°Ρ€ΡƒΡˆΠ°Π΅Ρ‚ΡΡ Ρ‚ΠΎΠ»ΡŒΠΊΠΎ ΠΎΠΊΠΎΠ»ΠΎ ΠΊΡ€Π°Π΅Π². ΠŸΡ€ΠΈ вычислСнии Смкости плоского кондСнсатора этими ΠΊΡ€Π°Π΅Π²Ρ‹ΠΌΠΈ эффСктами часто ΠΏΡ€Π΅Π½Π΅Π±Ρ€Π΅Π³Π°ΡŽΡ‚. Π­Ρ‚ΠΎ слСдуСт Π΄Π΅Π»Π°Ρ‚ΡŒ, Ссли расстояниС ΠΌΠ΅ΠΆΠ΄Ρƒ пластинами ΠΌΠ°Π»ΠΎ Π² сравнСнии с ΠΈΡ… Π»ΠΈΠ½Π΅ΠΉΠ½Ρ‹ΠΌΠΈ Ρ€Π°Π·ΠΌΠ΅Ρ€Π°ΠΌΠΈ. Для расчСта Смкости плоского кондСнсатора ΠΏΡ€ΠΈΠΌΠ΅Π½ΡΡŽΡ‚ Ρ„ΠΎΡ€ΠΌΡƒΠ»Ρƒ:

ЭлСктричСская Π΅ΠΌΠΊΠΎΡΡ‚ΡŒ плоского кондСнсатора, ΠΊΠΎΡ‚ΠΎΡ€Ρ‹ΠΉ содСрТит N слоСв диэлСктрика Ρ‚ΠΎΠ»Ρ‰ΠΈΠ½Π° ΠΊΠ°ΠΆΠ΄ΠΎΠ³ΠΎ , ΡΠΎΠΎΡ‚Π²Π΅Ρ‚ΡΡ‚Π²ΡƒΡŽΡ‰Π°Ρ диэлСктричСская ΠΏΡ€ΠΎΠ½ΠΈΡ†Π°Π΅ΠΌΠΎΡΡ‚ΡŒ i-Π³ΠΎ слоя , Ρ€Π°Π²Π½Π°:

Π€ΠΎΡ€ΠΌΡƒΠ»Π° элСктричСской Смкости цилиндричСского кондСнсатора

ЦилиндричСский кондСнсатор прСдставляСтся собой Π΄Π²Π΅ соосных (ΠΊΠΎΠ°ΠΊΡΠΈΠ°Π»ΡŒΠ½Ρ‹Ρ…) цилиндричСскиС проводящиС повСрхности, Ρ€Π°Π·Π½ΠΎΠ³ΠΎ радиуса, пространство ΠΌΠ΅ΠΆΠ΄Ρƒ ΠΊΠΎΡ‚ΠΎΡ€Ρ‹ΠΌΠΈ заполняСт диэлСктрик. ЭлСктричСская Π΅ΠΌΠΊΠΎΡΡ‚ΡŒ цилиндричСского кондСнсатора вычисляСтся ΠΊΠ°ΠΊ:

Π€ΠΎΡ€ΠΌΡƒΠ»Π° элСктричСской Смкости сфСричСского кондСнсатора

БфСричСским кондСнсатором Π½Π°Π·Ρ‹Π²Π°ΡŽΡ‚ кондСнсатор, ΠΎΠ±ΠΊΠ»Π°Π΄ΠΊΠ°ΠΌΠΈ ΠΊΠΎΡ‚ΠΎΡ€ΠΎΠ³ΠΎ ΡΠ²Π»ΡΡŽΡ‚ΡΡ Π΄Π²Π΅ концСнтричСскиС сфСричСскиС проводящиС повСрхности, пространство ΠΌΠ΅ΠΆΠ΄Ρƒ Π½ΠΈΠΌΠΈ Π·Π°ΠΏΠΎΠ»Π½Π΅Π½ΠΎ диэлСктриком. Π•ΠΌΠΊΠΎΡΡ‚ΡŒ Ρ‚Π°ΠΊΠΎΠ³ΠΎ кондСнсатора находят ΠΊΠ°ΠΊ:

Π³Π΄Π΅ – радиусы ΠΎΠ±ΠΊΠ»Π°Π΄ΠΎΠΊ кондСнсатора.

ΠŸΡ€ΠΈΠΌΠ΅Ρ€Ρ‹ Ρ€Π΅ΡˆΠ΅Π½ΠΈΡ Π·Π°Π΄Π°Ρ‡ ΠΏΠΎ Ρ‚Π΅ΠΌΠ΅ Β«Π•ΠΌΠΊΠΎΡΡ‚ΡŒ кондСнсатора»

ru.solverbook.com

ΠΠΌΠΊΠΎΡΡ‚ΡŒ кондСнсатора — ВсС Ρ„ΠΎΡ€ΠΌΡƒΠ»Ρ‹

ЭлСктричСская Ρ‘ΠΌΠΊΠΎΡΡ‚ΡŒ — характСристика ΠΏΡ€ΠΎΠ²ΠΎΠ΄Π½ΠΈΠΊΠ° (кондСнсатора), ΠΌΠ΅Ρ€Π° Π΅Π³ΠΎ способности Π½Π°ΠΊΠ°ΠΏΠ»ΠΈΠ²Π°Ρ‚ΡŒ элСктричСский заряд.

ΠšΠΎΠ½Π΄Π΅Π½ΡΠ°Ρ‚ΠΎΡ€ состоит ΠΈΠ· Π΄Π²ΡƒΡ… ΠΏΡ€ΠΎΠ²ΠΎΠ΄Π½ΠΈΠΊΠΎΠ² (ΠΎΠ±ΠΊΠ»Π°Π΄ΠΎΠΊ), ΠΊΠΎΡ‚ΠΎΡ€Ρ‹Π΅ Ρ€Π°Π·Π΄Π΅Π»Π΅Π½Ρ‹ диэлСктриком. На Π΅ΠΌΠΊΠΎΡΡ‚ΡŒ кондСнсатора Π½Π΅ Π΄ΠΎΠ»ΠΆΠ½Ρ‹ Π²Π»ΠΈΡΡ‚ΡŒ ΠΎΠΊΡ€ΡƒΠΆΠ°ΡŽΡ‰ΠΈΠ΅ Ρ‚Π΅Π»Π°, поэтому ΠΏΡ€ΠΎΠ²ΠΎΠ΄Π½ΠΈΠΊΠ°ΠΌ ΠΏΡ€ΠΈΠ΄Π°ΡŽΡ‚ Ρ‚Π°ΠΊΡƒΡŽ Ρ„ΠΎΡ€ΠΌΡƒ, Ρ‡Ρ‚ΠΎΠ±Ρ‹ ΠΏΠΎΠ»Π΅, ΠΊΠΎΡ‚ΠΎΡ€ΠΎΠ΅ создаСтся Π½Π°ΠΊΠ°ΠΏΠ»ΠΈΠ²Π°Π΅ΠΌΡ‹ΠΌΠΈ зарядами, Π±Ρ‹Π»ΠΎ сосрСдоточСно Π² ΡƒΠ·ΠΊΠΎΠΌ Π·Π°Π·ΠΎΡ€Π΅ ΠΌΠ΅ΠΆΠ΄Ρƒ ΠΎΠ±ΠΊΠ»Π°Π΄ΠΊΠ°ΠΌΠΈ кондСнсатора. Π­Ρ‚ΠΎΠΌΡƒ ΡƒΡΠ»ΠΎΠ²ΠΈΡŽ ΡƒΠ΄ΠΎΠ²Π»Π΅Ρ‚Π²ΠΎΡ€ΡΡŽΡ‚: 1) Π΄Π²Π΅ плоскиС пластины; 2) Π΄Π²Π΅ концСнтричСскиС сфСры; 3) Π΄Π²Π° ΠΊΠΎΠ°ΠΊΡΠΈΠ°Π»ΡŒΠ½Ρ‹Ρ… Ρ†ΠΈΠ»ΠΈΠ½Π΄Ρ€Π°. ΠŸΠΎΡΡ‚ΠΎΠΌΡƒ Π² зависимости ΠΎΡ‚ Ρ„ΠΎΡ€ΠΌΡ‹ ΠΎΠ±ΠΊΠ»Π°Π΄ΠΎΠΊ кондСнсаторы дСлятся Π½Π° плоскиС, сфСричСскиС ΠΈ цилиндричСскиС.

Π’Π°ΠΊ ΠΊΠ°ΠΊ ΠΏΠΎΠ»Π΅ сосрСдоточСно Π²Π½ΡƒΡ‚Ρ€ΠΈ кондСнсатора, Ρ‚ΠΎ Π»ΠΈΠ½ΠΈΠΈ напряТСнности Π½Π°Ρ‡ΠΈΠ½Π°ΡŽΡ‚ΡΡ Π½Π° ΠΎΠ΄Π½ΠΎΠΉ ΠΎΠ±ΠΊΠ»Π°Π΄ΠΊΠ΅ ΠΈ ΠΊΠΎΠ½Ρ‡Π°ΡŽΡ‚ΡΡ Π½Π° Π΄Ρ€ΡƒΠ³ΠΎΠΉ, поэтому свободныС заряды, ΠΊΠΎΡ‚ΠΎΡ€Ρ‹Π΅ Π²ΠΎΠ·Π½ΠΈΠΊΠ°ΡŽΡ‚ Π½Π° Ρ€Π°Π·Π½Ρ‹Ρ… ΠΎΠ±ΠΊΠ»Π°Π΄ΠΊΠ°Ρ…, Ρ€Π°Π²Π½Ρ‹ ΠΏΠΎ ΠΌΠΎΠ΄ΡƒΠ»ΡŽ ΠΈ ΠΏΡ€ΠΎΡ‚ΠΈΠ²ΠΎΠΏΠΎΠ»ΠΎΠΆΠ½Ρ‹ ΠΏΠΎ Π·Π½Π°ΠΊΡƒ. Под Π΅ΠΌΠΊΠΎΡΡ‚ΡŒΡŽ кондСнсатора понимаСтся физичСская Π²Π΅Π»ΠΈΡ‡ΠΈΠ½Π°, равная ΠΎΡ‚Π½ΠΎΡˆΠ΅Π½ΠΈΡŽ заряда Q, Π½Π°ΠΊΠΎΠΏΠ»Π΅Π½Π½ΠΎΠ³ΠΎ Π² кондСнсаторС, ΠΊ разности ΠΏΠΎΡ‚Π΅Π½Ρ†ΠΈΠ°Π»ΠΎΠ² (Ο†1 — Ο†2) ΠΌΠ΅ΠΆΠ΄Ρƒ Π΅Π³ΠΎ ΠΎΠ±ΠΊΠ»Π°Π΄ΠΊΠ°ΠΌΠΈ

Для получСния Π±ΠΎΠ»ΡŒΡˆΠΈΡ… ёмкостСй кондСнсаторы ΡΠΎΠ΅Π΄ΠΈΠ½ΡΡŽΡ‚ ΠΏΠ°Ρ€Π°Π»Π»Π΅Π»ΡŒΠ½ΠΎ. ΠŸΡ€ΠΈ этом напряТСниС ΠΌΠ΅ΠΆΠ΄Ρƒ ΠΎΠ±ΠΊΠ»Π°Π΄ΠΊΠ°ΠΌΠΈ всСх кондСнсаторов ΠΎΠ΄ΠΈΠ½Π°ΠΊΠΎΠ²ΠΎ. ΠžΠ±Ρ‰Π°Ρ Ρ‘ΠΌΠΊΠΎΡΡ‚ΡŒ Π±Π°Ρ‚Π°Ρ€Π΅ΠΈ ΠΏΠ°Ρ€Π°Π»Π»Π΅Π»ΡŒΠ½ΠΎ соСдинённых кондСнсаторов Ρ€Π°Π²Π½Π° суммС ёмкостСй всСх кондСнсаторов, входящих Π² Π±Π°Ρ‚Π°Ρ€Π΅ΡŽ.

ΠšΠΎΠ½Π΄Π΅Π½ΡΠ°Ρ‚ΠΎΡ€Ρ‹ ΠΌΠΎΠΆΠ½ΠΎ ΠΊΠ»Π°ΡΡΠΈΡ„ΠΈΡ†ΠΈΡ€ΠΎΠ²Π°Ρ‚ΡŒ ΠΏΠΎ ΡΠ»Π΅Π΄ΡƒΡŽΡ‰ΠΈΠΌ ΠΏΡ€ΠΈΠ·Π½Π°ΠΊΠ°ΠΌ ΠΈ свойствам:

1) ΠΏΠΎ Π½Π°Π·Π½Π°Ρ‡Π΅Π½ΠΈΡŽ — кондСнсаторы постоянной ΠΈ ΠΏΠ΅Ρ€Π΅ΠΌΠ΅Π½Π½ΠΎΠΉ Смкости;

2) ΠΏΠΎ Ρ„ΠΎΡ€ΠΌΠ΅ ΠΎΠ±ΠΊΠ»Π°Π΄ΠΎΠΊ Ρ€Π°Π·Π»ΠΈΡ‡Π°ΡŽΡ‚ кондСнсаторы плоскиС, сфСричСскиС, цилиндричСскиС ΠΈ Π΄Ρ€.;

3) ΠΏΠΎ Ρ‚ΠΈΠΏΡƒ диэлСктрика — Π²ΠΎΠ·Π΄ΡƒΡˆΠ½Ρ‹Π΅, Π±ΡƒΠΌΠ°ΠΆΠ½Ρ‹Π΅, ΡΠ»ΡŽΠ΄ΡΠ½Ρ‹Π΅, кСрамичСскиС, элСктролитичСскиС ΠΈ Ρ‚.Π΄.

Π’Π°ΠΊ ΠΆΠ΅ Π΅ΡΡ‚ΡŒ:

ЭнСргия кондСнсатора:

ΠΠΌΠΊΠΎΡΡ‚ΡŒ цилиндричСского кондСнсатора:

ΠΠΌΠΊΠΎΡΡ‚ΡŒ плоского кондСнсатора:

Π•ΠΌΠΊΠΎΡΡ‚ΡŒ сфСричСского кондСнсатора:

Π’ Ρ„ΠΎΡ€ΠΌΡƒΠ»Π΅ ΠΌΡ‹ использовали:

ЭлСктричСская Ρ‘ΠΌΠΊΠΎΡΡ‚ΡŒ (Ρ‘ΠΌΠΊΠΎΡΡ‚ΡŒ кондСнсатора)

ΠŸΠΎΡ‚Π΅Π½Ρ†ΠΈΠ°Π» ΠΏΡ€ΠΎΠ²ΠΎΠ΄Π½ΠΈΠΊΠ° (НапряТСниС)

ΠŸΠΎΡ‚Π΅Π½Ρ†ΠΈΠ°Π»

ΠžΡ‚Π½ΠΎΡΠΈΡ‚Π΅Π»ΡŒΠ½Π°Ρ диэлСктричСская ΠΏΡ€ΠΎΠ½ΠΈΡ†Π°Π΅ΠΌΠΎΡΡ‚ΡŒ

ЭлСктричСская постоянная

ΠŸΠ»ΠΎΡ‰Π°Π΄ΡŒ ΠΎΠ΄Π½ΠΎΠΉ ΠΎΠ±ΠΊΠ»Π°Π΄ΠΊΠΈ

РасстояниС ΠΌΠ΅ΠΆΠ΄Ρƒ ΠΎΠ±ΠΊΠ»Π°Π΄ΠΊΠ°ΠΌΠΈ

xn--b1agsdjmeuf9e. xn--p1ai

Заряд кондСнсатора, тСория ΠΈ ΠΏΡ€ΠΈΠΌΠ΅Ρ€Ρ‹ Π·Π°Π΄Π°Ρ‡

ΠžΠΏΡ€Π΅Π΄Π΅Π»Π΅Π½ΠΈΠ΅ ΠΈ заряд кондСнсатора

Π’ΠΎΠ·ΠΌΠΎΠΆΠ½ΠΎΡΡ‚ΡŒ кондСнсатора Π½Π°ΠΊΠΎΠΏΠΈΡ‚ΡŒ элСктричСский заряд зависит ΠΎΡ‚ Π³Π»Π°Π²Π½ΠΎΠΉ характСристики кондСнсатора – Смкости (C).

По своСму Π½Π°Π·Π½Π°Ρ‡Π΅Π½ΠΈΡŽ кондСнсатор ΠΌΠΎΠΆΠ½ΠΎ ΡƒΠΏΠΎΠ΄ΠΎΠ±ΠΈΡ‚ΡŒ Π±Π°Ρ‚Π°Ρ€Π΅ΠΉΠΊΠ΅. Однако сущСствуСт ΠΏΡ€ΠΈΠ½Ρ†ΠΈΠΏΠΈΠ°Π»ΡŒΠ½ΠΎΠ΅ ΠΎΡ‚Π»ΠΈΡ‡ΠΈΠ΅ Π² ΠΏΡ€ΠΈΠ½Ρ†ΠΈΠΏΠ°Ρ… Ρ€Π°Π±ΠΎΡ‚Ρ‹ этих элСмСнтов. ΠžΡ‚Π»ΠΈΡ‡Π°ΡŽΡ‚ΡΡ, Ρ‚Π°ΠΊΠΆΠ΅ ΠΌΠ°ΠΊΡΠΈΠΌΠ°Π»ΡŒΠ½Ρ‹Π΅ Смкости ΠΈ скорости зарядки ΠΈ разряда кондСнсатора ΠΈ Π±Π°Ρ‚Π°Ρ€Π΅ΠΉΠΊΠΈ.

Если ΠΊ кондСнсатору ΠΏΡ€ΠΈΡΠΎΠ΅Π΄ΠΈΠ½ΠΈΡ‚ΡŒ источник напряТСния (рис.1), Ρ‚ΠΎ Π½Π° ΠΎΠ΄Π½ΠΎΠΉ ΠΈΠ· пластин кондСнсатора станут Π½Π°ΠΊΠ°ΠΏΠ»ΠΈΠ²Π°Ρ‚ΡŒΡΡ ΠΎΡ‚Ρ€ΠΈΡ†Π°Ρ‚Π΅Π»ΡŒΠ½Ρ‹Π΅ заряды (элСктроны), Π½Π° Π΄Ρ€ΡƒΠ³ΠΎΠΉ ΠΏΠΎΠ»ΠΎΠΆΠΈΡ‚Π΅Π»ΡŒΠ½Ρ‹Π΅ частицы (ΠΏΠΎΠ»ΠΎΠΆΠΈΡ‚Π΅Π»ΡŒΠ½Ρ‹Π΅ ΠΈΠΎΠ½Ρ‹). ΠœΠ΅ΠΆΠ΄Ρƒ ΠΎΠ±ΠΊΠ»Π°Π΄ΠΊΠ°ΠΌΠΈ кондСнсатора находится диэлСктрик, вслСдствиС этого, заряды Π½Π΅ ΠΌΠΎΠ³ΡƒΡ‚ ΠΏΠ΅Ρ€Π΅Π±Ρ€Π°Ρ‚ΡŒΡΡ Π½Π° ΠΏΡ€ΠΎΡ‚ΠΈΠ²ΠΎΠΏΠΎΠ»ΠΎΠΆΠ½ΡƒΡŽ пластину. Однако Π·Π°ΠΌΠ΅Ρ‚ΠΈΠΌ, Ρ‡Ρ‚ΠΎ элСктроны Π΄Π²ΠΈΠ³Π°ΡŽΡ‚ΡΡ ΠΎΡ‚ источника Ρ‚ΠΎΠΊΠ° Π΄ΠΎ пластины кондСнсатора.

ΠŸΡ€ΠΈ ΠΏΠ΅Ρ€Π²ΠΎΠ½Π°Ρ‡Π°Π»ΡŒΠ½ΠΎΠΌ соСдинСнии кондСнсатора ΠΈ источника Ρ‚ΠΎΠΊΠ° Π½Π° ΠΎΠ±ΠΊΠ»Π°Π΄ΠΊΠ°Ρ… кондСнсатора ΠΌΠ½ΠΎΠ³ΠΎ свободного мСста. Π­Ρ‚ΠΎ ΠΎΠ·Π½Π°Ρ‡Π°Π΅Ρ‚, Ρ‡Ρ‚ΠΎ сопротивлСниС Ρ‚ΠΎΠΊΡƒ этот ΠΌΠΎΠΌΠ΅Π½Ρ‚ Π²Ρ€Π΅ΠΌΠ΅Π½ΠΈ минимально, сам Ρ‚ΠΎΠΊ максималСн. Π’ Ρ…ΠΎΠ΄Π΅ зарядки кондСнсатора сила Ρ‚ΠΎΠΊΠ° Π² Ρ†Π΅ΠΏΠΈ постСпСнно ΠΏΠ°Π΄Π°Π΅Ρ‚, Π΄ΠΎ Ρ‚ΠΎΠ³ΠΎ ΠΌΠΎΠΌΠ΅Π½Ρ‚Π° ΠΏΠΎΠΊΠ° Π½Π΅ закончится свободноС мСсто Π½Π° ΠΎΠ±ΠΊΠ»Π°Π΄ΠΊΠ°Ρ…. ΠŸΡ€ΠΈ ΠΏΠΎΠ»Π½ΠΎΠΉ зарядкС кондСнсатора Ρ‚ΠΎΠΊ Π² Ρ†Π΅ΠΏΠΈ прСкратится.

ВрСмя, ΠΊΠΎΡ‚ΠΎΡ€ΠΎΠ΅ затрачиваСтся Π½Π° зарядку кондСнсатора ΠΎΡ‚ Π½ΡƒΠ»Π΅Π²ΠΎΠ³ΠΎ заряда (максимального Ρ‚ΠΎΠΊΠ°) Π΄ΠΎ ΠΏΠΎΠ»Π½ΠΎΡΡ‚ΡŒΡŽ заряТСнного кондСнсатора (минимальная ΠΈΠ»ΠΈ нулСвая сила Ρ‚ΠΎΠΊΠ°) Π½Π°Π·Ρ‹Π²Π°ΡŽΡ‚ ΠΏΠ΅Ρ€Π΅Ρ…ΠΎΠ΄Π½Ρ‹ΠΌ ΠΏΠ΅Ρ€ΠΈΠΎΠ΄ΠΎΠΌ заряда кондСнсатора. На ΠΏΡ€Π°ΠΊΡ‚ΠΈΠΊΠ΅ процСсс зарядки кондСнсатора ΡΡ‡ΠΈΡ‚Π°ΡŽΡ‚ Π·Π°ΠΊΠΎΠ½Ρ‡Π΅Π½Π½Ρ‹ΠΌ, Ссли сила Ρ‚ΠΎΠΊΠ° ΡƒΠΌΠ΅Π½ΡŒΡˆΠΈΠ»Π°ΡΡŒ Π΄ΠΎ 1% ΠΎΡ‚ Π½Π°Ρ‡Π°Π»ΡŒΠ½ΠΎΠΉ Π²Π΅Π»ΠΈΡ‡ΠΈΠ½Ρ‹.

Π’Π΅Π»ΠΈΡ‡ΠΈΠ½Π° заряда кондСнсатора (q) связана с Π΅Π³ΠΎ Π΅ΠΌΠΊΠΎΡΡ‚ΡŒΡŽ (C) ΠΈ Ρ€Π°Π·Π½ΠΎΡΡ‚ΡŒΡŽ ΠΏΠΎΡ‚Π΅Π½Ρ†ΠΈΠ°Π»ΠΎΠ² (U) ΠΌΠ΅ΠΆΠ΄Ρƒ Π΅Π³ΠΎ ΠΎΠ±ΠΊΠ»Π°Π΄ΠΊΠ°ΠΌΠΈ ΠΊΠ°ΠΊ:

ΠŸΡ€ΠΈΠΌΠ΅Ρ€Ρ‹ Ρ€Π΅ΡˆΠ΅Π½ΠΈΡ Π·Π°Π΄Π°Ρ‡

ru.solverbook.com

Π€ΠΎΡ€ΠΌΡƒΠ»Π° элСктроСмкости кондСнсатора

Обкладки Π΄ΠΎΠ»ΠΆΠ½Ρ‹ ΠΈΠΌΠ΅Ρ‚ΡŒ Ρ‚Π°ΠΊΡƒΡŽ Ρ„ΠΎΡ€ΠΌΡƒ ΠΈ Π±Ρ‹Ρ‚ΡŒ располоТСны Ρ‚Π°ΠΊ ΠΎΡ‚Π½ΠΎΡΠΈΡ‚Π΅Π»ΡŒΠ½ΠΎ Π΄Ρ€ΡƒΠ³ Π΄Ρ€ΡƒΠ³Π°, Ρ‡Ρ‚ΠΎ ΠΏΠΎΠ»Π΅, ΠΊΠΎΡ‚ΠΎΡ€ΠΎΠ΅ создаСтся Π΄Π°Π½Π½ΠΎΠΉ систСмой, Π±Ρ‹Π»ΠΎ максимально сосрСдоточСно Π² ΠΎΠ³Ρ€Π°Π½ΠΈΡ‡Π΅Π½Π½ΠΎΠΉ области пространства, ΠΌΠ΅ΠΆΠ΄Ρƒ ΠΎΠ±ΠΊΠ»Π°Π΄ΠΊΠ°ΠΌΠΈ.

НазначСниС кондСнсатора Π² Ρ‚ΠΎΠΌ, Ρ‡Ρ‚ΠΎΠ±Ρ‹ Π½Π°ΠΊΠ°ΠΏΠ»ΠΈΠ²Π°Ρ‚ΡŒ ΠΈ ΠΎΡ‚Π΄Π°Π²Π°Ρ‚ΡŒ Π² элСктричСской Ρ†Π΅ΠΏΠΈ заряд.

Основной характСристикой кондСнсатора являСтся элСктричСская Π΅ΠΌΠΊΠΎΡΡ‚ΡŒ (C). ЭлСктричСская Π΅ΠΌΠΊΠΎΡΡ‚ΡŒ кондСнсатора – это взаимная Π΅ΠΌΠΊΠΎΡΡ‚ΡŒ ΠΏΡ€ΠΈΠ½Π°Π΄Π»Π΅ΠΆΠ°Ρ‰ΠΈΡ… Π΅ΠΌΡƒ ΠΎΠ±ΠΊΠ»Π°Π΄ΠΎΠΊ:

q – Π²Π΅Π»ΠΈΡ‡ΠΈΠ½Π° заряда Π½Π° ΠΎΠ±ΠΊΠ»Π°Π΄ΠΊΠ΅; – Ρ€Π°Π·Π½ΠΎΡΡ‚ΡŒ ΠΏΠΎΡ‚Π΅Π½Ρ†ΠΈΠ°Π»ΠΎΠ² ΠΌΠ΅ΠΆΠ΄Ρƒ ΠΎΠ±ΠΊΠ»Π°Π΄ΠΊΠ°ΠΌΠΈ.

ЭлСктричСская Ρ‘ΠΌΠΊΠΎΡΡ‚ΡŒ кондСнсатора зависит ΠΎΡ‚ диэлСктричСской проницаСмости диэлСктрика, ΠΊΠΎΡ‚ΠΎΡ€Ρ‹ΠΉ заполняСт пространство ΠΌΠ΅ΠΆΠ΄Ρƒ Π΅Π³ΠΎ ΠΎΠ±ΠΊΠ»Π°Π΄ΠΊΠ°ΠΌΠΈ. Если пространство ΠΌΠ΅ΠΆΠ΄Ρƒ ΠΎΠ±ΠΊΠ»Π°Π΄ΠΊΠ°ΠΌΠΈ ΠΎΠ΄Π½ΠΎΠ³ΠΎ кондСнсатора Π·Π°ΠΏΠΎΠ»Π½Π΅Π½ΠΎ диэлСктриком с ΠΏΡ€ΠΎΠ½ΠΈΡ†Π°Π΅ΠΌΠΎΡΡ‚ΡŒΡŽ Ρ€Π°Π²Π½ΠΎΠΉ , Π° Ρƒ Π²Ρ‚ΠΎΡ€ΠΎΠ³ΠΎ кондСнсатора Π²ΠΎΠ·Π΄ΡƒΡ… ΠΌΠ΅ΠΆΠ΄Ρƒ пластинами, Ρ‚ΠΎ Π΅ΠΌΠΊΠΎΡΡ‚ΡŒ кондСнсатора с диэлСктриком (C) Π² Ρ€Π°Π· большС, Ρ‡Π΅ΠΌ Π΅ΠΌΠΊΠΎΡΡ‚ΡŒ Π²ΠΎΠ·Π΄ΡƒΡˆΠ½ΠΎΠ³ΠΎ кондСнсатора ():

Π€ΠΎΡ€ΠΌΡƒΠ»Π° элСктроСмкости основных Ρ‚ΠΈΠΏΠΎΠ² кондСнсаторов

ΠŸΡ€ΠΈ расчСтС элСктроСмкости плоского кондСнсатора Π½Π°Ρ€ΡƒΡˆΠ΅Π½ΠΈΠ΅ΠΌ однородности поля ΠΎΠΊΠΎΠ»ΠΎ ΠΊΡ€Π°Ρ‘Π² ΠΎΠ±ΠΊΠ»Π°Π΄ΠΎΠΊ ΠΎΠ±Ρ‹Ρ‡Π½ΠΎ ΠΏΡ€Π΅Π½Π΅Π±Ρ€Π΅Π³Π°ΡŽΡ‚. Π­Ρ‚ΠΎ становится Π²ΠΎΠ·ΠΌΠΎΠΆΠ½Ρ‹ΠΌ, Ссли расстояниС ΠΌΠ΅ΠΆΠ΄Ρƒ пластинами сущСствСнно мСньшС, Ρ‡Π΅ΠΌ Π»ΠΈΠ½Π΅ΠΉΠ½Ρ‹Π΅ Ρ€Π°Π·ΠΌΠ΅Ρ€Ρ‹ ΠΎΠ±ΠΊΠ»Π°Π΄ΠΎΠΊ. Π’ Ρ‚Π°ΠΊΠΎΠΌ случаС ΡΠ»Π΅ΠΊΡ‚Ρ€ΠΈΡ‡Π΅ΡΠΊΡƒΡŽ Π΅ΠΌΠΊΠΎΡΡ‚ΡŒ плоского кондСнсатора Π²Ρ‹Ρ‡ΠΈΡΠ»ΡΡŽΡ‚ ΠΏΡ€ΠΈ ΠΏΠΎΠΌΠΎΡ‰ΠΈ Ρ„ΠΎΡ€ΠΌΡƒΠ»Ρ‹:

Π³Π΄Π΅ – элСктричСская постоянная; S – ΠΏΠ»ΠΎΡ‰Π°Π΄ΡŒ ΠΊΠ°ΠΆΠ΄ΠΎΠΉ (ΠΈΠ»ΠΈ наимСньшСй) пластины; d – расстояниС ΠΌΠ΅ΠΆΠ΄Ρƒ пластинами.

Если плоский кондСнсатор ΠΌΠ΅ΠΆΠ΄Ρƒ ΠΎΠ±ΠΊΠ»Π°Π΄ΠΊΠ°ΠΌΠΈ ΠΈΠΌΠ΅Π΅Ρ‚ N слоСв диэлСктрика, ΠΏΡ€ΠΈ этом Ρ‚ΠΎΠ»Ρ‰ΠΈΠ½Π° ΠΊΠ°ΠΆΠ΄ΠΎΠ³ΠΎ слоя Ρ€Π°Π²Π½Π° , Π° диэлСктричСская ΠΏΡ€ΠΎΠ½ΠΈΡ†Π°Π΅ΠΌΠΎΡΡ‚ΡŒ , Ρ‚ΠΎ Π΅Π³ΠΎ ΡΠ»Π΅ΠΊΡ‚Ρ€ΠΈΡ‡Π΅ΡΠΊΡƒΡŽ Π΅ΠΌΠΊΠΎΡΡ‚ΡŒ Ρ€Π°ΡΡΡ‡ΠΈΡ‚Ρ‹Π²Π°ΡŽΡ‚ ΠΏΡ€ΠΈ ΠΏΠΎΠΌΠΎΡ‰ΠΈ Ρ„ΠΎΡ€ΠΌΡƒΠ»Ρ‹:

ЦилиндричСский кондСнсатор ΡΠΎΡΡ‚Π°Π²Π»ΡΡŽΡ‚ Π΄Π²Π΅ соосных (ΠΊΠΎΠ°ΠΊΡΠΈΠ°Π»ΡŒΠ½Ρ‹Ρ…) цилиндричСскиС проводящиС повСрхности, Ρ€Π°Π·Π½ΠΎΠ³ΠΎ радиуса, пространство ΠΌΠ΅ΠΆΠ΄Ρƒ ΠΊΠΎΡ‚ΠΎΡ€Ρ‹ΠΌΠΈ Π·Π°ΠΏΠΎΠ»Π½Π΅Π½ΠΎ диэлСктриком. ΠŸΡ€ΠΈ этом Π΅ΠΌΠΊΠΎΡΡ‚ΡŒ цилиндричСского кондСнсатора находят ΠΊΠ°ΠΊ:

Π³Π΄Π΅ l – высота Ρ†ΠΈΠ»ΠΈΠ½Π΄Ρ€ΠΎΠ²; – радиус внСшнСй ΠΎΠ±ΠΊΠ»Π°Π΄ΠΊΠΈ; – радиус Π²Π½ΡƒΡ‚Ρ€Π΅Π½Π½Π΅ΠΉ ΠΎΠ±ΠΊΠ»Π°Π΄ΠΊΠΈ.

Π£ сфСричСского кондСнсатора ΠΎΠ±ΠΊΠ»Π°Π΄ΠΊΠ°ΠΌΠΈ слуТат Π΄Π²Π΅ концСнтричСскиС сфСричСскиС проводящиС повСрхности, пространство ΠΎΠ±ΠΊΠ»Π°Π΄ΠΊΠ°ΠΌΠΈ заполняСт диэлСктрик. Π•ΠΌΠΊΠΎΡΡ‚ΡŒ сфСричСского кондСнсатора Π²Ρ‹Ρ‡ΠΈΡΠ»ΡΡŽΡ‚ ΠΊΠ°ΠΊ:

Π³Π΄Π΅ – радиусы ΠΎΠ±ΠΊΠ»Π°Π΄ΠΎΠΊ кондСнсатора. Если , Ρ‚ΠΎ ΠΌΠΎΠΆΠ½ΠΎ ΡΡ‡ΠΈΡ‚Π°Ρ‚ΡŒ, Ρ‡Ρ‚ΠΎ , Ρ‚ΠΎΠ³Π΄Π°, ΠΌΡ‹ ΠΈΠΌΠ΅Π΅ΠΌ:

Ρ‚Π°ΠΊ ΠΊΠ°ΠΊ – ΠΏΠ»ΠΎΡ‰Π°Π΄ΡŒ повСрхности сфСры, ΠΈ Ссли ΠΎΠ±ΠΎΠ·Π½Π°Ρ‡ΠΈΡ‚ΡŒ , Ρ‚ΠΎ ΠΏΠΎΠ»ΡƒΡ‡ΠΈΠΌ Ρ„ΠΎΡ€ΠΌΡƒΠ»Ρƒ для Смкости плоского кондСнсатора (3). Если расстояниС ΠΌΠ΅ΠΆΠ΄Ρƒ ΠΎΠ±ΠΊΠ»Π°Π΄ΠΊΠ°ΠΌΠΈ сфСричСского ΠΈ цилиндричСского кондСнсаторов ΠΌΠ°Π»Ρ‹ (Π² сравнСнии с ΠΈΡ… радиусами), Ρ‚ΠΎ Π² ΠΏΡ€ΠΈΠ±Π»ΠΈΠΆΠ΅Π½Π½Ρ‹Ρ… расчСтах ΠΈΡΠΏΠΎΠ»ΡŒΠ·ΡƒΡŽΡ‚ Ρ„ΠΎΡ€ΠΌΡƒΠ»Ρƒ Смкости для плоского кондСнсатора.

Π­Π»Π΅ΠΊΡ‚Ρ€ΠΈΡ‡Π΅ΡΠΊΡƒΡŽ Π΅ΠΌΠΊΠΎΡΡ‚ΡŒ для Π»ΠΈΠ½ΠΈΠΈ ΠΈΠ· Π΄Π²ΡƒΡ… ΠΏΡ€ΠΎΠ²ΠΎΠ΄ΠΎΠ² находят ΠΊΠ°ΠΊ:

Π³Π΄Π΅ d – расстояниС ΠΌΠ΅ΠΆΠ΄Ρƒ осями ΠΏΡ€ΠΎΠ²ΠΎΠ΄ΠΎΠ²; R – радиус ΠΏΡ€ΠΎΠ²ΠΎΠ΄ΠΎΠ²; l – Π΄Π»ΠΈΠ½Π° Π»ΠΈΠ½ΠΈΠΈ.

Π€ΠΎΡ€ΠΌΡƒΠ»Ρ‹ для вычислСния элСктричСской Смкости соСдинСний кондСнсаторов

Если кондСнсаторы соСдинСны ΠΏΠ°Ρ€Π°Π»Π»Π΅Π»ΡŒΠ½ΠΎ, Ρ‚ΠΎ суммарная Π΅ΠΌΠΊΠΎΡΡ‚ΡŒ Π±Π°Ρ‚Π°Ρ€Π΅ΠΈ (C) находится ΠΊΠ°ΠΊ сумма СмкостСй ΠΎΡ‚Π΄Π΅Π»ΡŒΠ½Ρ‹Ρ… кондСнсаторов ():

ΠŸΡ€ΠΈ ΠΏΠΎΡΠ»Π΅Π΄ΠΎΠ²Π°Ρ‚Π΅Π»ΡŒΠ½ΠΎΠΌ соСдинСнии кондСнсаторов Π΅ΠΌΠΊΠΎΡΡ‚ΡŒ Π±Π°Ρ‚Π°Ρ€Π΅ΠΈ Π²Ρ‹Ρ‡ΠΈΡΠ»ΡΡŽΡ‚ ΠΊΠ°ΠΊ:

Если ΠΏΠΎΡΠ»Π΅Π΄ΠΎΠ²Π°Ρ‚Π΅Π»ΡŒΠ½ΠΎ соСдинСны N кондСнсаторов, с Смкостями Ρ‚ΠΎ Π΅ΠΌΠΊΠΎΡΡ‚ΡŒ Π±Π°Ρ‚Π°Ρ€Π΅ΠΈ Π½Π°ΠΉΠ΄Π΅ΠΌ ΠΊΠ°ΠΊ:

Π‘ΠΎΠΏΡ€ΠΎΡ‚ΠΈΠ²Π»Π΅Π½ΠΈΠ΅ кондСнсатора

Если кондСнсатор Π²ΠΊΠ»ΡŽΡ‡Π΅Π½ Π² Ρ†Π΅ΠΏΡŒ с постоянного Ρ‚ΠΎΠΊΠ°, Ρ‚ΠΎ сопротивлСниС кондСнсатора ΠΌΠΎΠΆΠ½ΠΎ ΡΡ‡ΠΈΡ‚Π°Ρ‚ΡŒ бСсконСчно большим.

ΠŸΡ€ΠΈ Π²ΠΊΠ»ΡŽΡ‡Π΅Π½ΠΈΠΈ кондСнсатора Π² Ρ†Π΅ΠΏΡŒ ΠΏΠ΅Ρ€Π΅ΠΌΠ΅Π½Π½ΠΎΠ³ΠΎ Ρ‚ΠΎΠΊΠ°, Π΅Π³ΠΎ сопротивлСниС носит Π½Π°Π·Π²Π°Π½ΠΈΠ΅ Смкостного, ΠΈ Π²Ρ‹Ρ‡ΠΈΡΠ»ΡΡŽΡ‚ Π΅Π³ΠΎ с ΠΏΠΎΠΌΠΎΡ‰ΡŒΡŽ Ρ„ΠΎΡ€ΠΌΡƒΠ»Ρ‹:

Π³Π΄Π΅ – частота ΠΏΠ΅Ρ€Π΅ΠΌΠ΅Π½Π½ΠΎΠ³ΠΎ Ρ‚ΠΎΠΊΠ°; – угловая частота Ρ‚ΠΎΠΊΠ°; C – Π΅ΠΌΠΊΠΎΡΡ‚ΡŒ кондСнсатора.

ЭнСргия поля кондСнсатора

ЭлСктричСскоС ΠΏΠΎΠ»Π΅ Π»ΠΎΠΊΠ°Π»ΠΈΠ·ΠΎΠ²Π°Π½Π½ΠΎΠ΅ ΠΌΠ΅ΠΆΠ΄Ρƒ пластинами кондСнсатора ΠΎΠ±Π»Π°Π΄Π°Π΅Ρ‚ энСргиСй, ΠΊΠΎΡ‚ΠΎΡ€ΡƒΡŽ ΠΌΠΎΠΆΠ½ΠΎ Π²Ρ‹Ρ‡ΠΈΡΠ»ΠΈΡ‚ΡŒ ΠΏΡ€ΠΈ ΠΏΠΎΠΌΠΎΡ‰ΠΈ Ρ„ΠΎΡ€ΠΌΡƒΠ»Ρ‹:

Π³Π΄Π΅ –энСргия поля кондСнсатора; q – заряд кондСнсатора; C – Π΅ΠΌΠΊΠΎΡΡ‚ΡŒ кондСнсатора; – Ρ€Π°Π·Π½ΠΎΡΡ‚ΡŒ ΠΏΠΎΡ‚Π΅Π½Ρ†ΠΈΠ°Π»ΠΎΠ² ΠΌΠ΅ΠΆΠ΄Ρƒ ΠΎΠ±ΠΊΠ»Π°Π΄ΠΊΠ°ΠΌΠΈ кондСнсатора.

ЭнСргия поля плоского кондСнсатора:

ΠŸΡ€ΠΈΠΌΠ΅Ρ€Ρ‹ Ρ€Π΅ΡˆΠ΅Π½ΠΈΡ Π·Π°Π΄Π°Ρ‡ ΠΏΠΎ Ρ‚Π΅ΠΌΠ΅ Β«Π­Π»Π΅ΠΊΡ‚Ρ€ΠΎΠ΅ΠΌΠΊΠΎΡΡ‚ΡŒ кондСнсатора»

ru.solverbook.com

Как Π½Π°ΠΉΡ‚ΠΈ заряд кондСнсатора 🚩 ΠΊΠ°ΠΊ ΠΎΠΏΡ€Π΅Π΄Π΅Π»ΠΈΡ‚ΡŒ Π²Π΅Π»ΠΈΡ‡ΠΈΠ½Ρƒ заряда 🚩 ЕстСствСнныС Π½Π°ΡƒΠΊΠΈ

Π’ ΠΎΠ±Ρ‹Ρ‡Π½ΠΎΠΌ (Π±Π΅Π· ΠΏΠ»Π°Π³ΠΈΠ½ΠΎΠ² ΠΈ ΠΌΠΎΠ΄ΠΎΠ²) Π²Π°Ρ€ΠΈΠ°Π½Ρ‚Π΅ Minecraft Ρ‚Π°ΠΊΠΎΠ³ΠΎ понятия, ΠΊΠ°ΠΊ кондСнсатор, Π½Π΅ сущСствуСт. Π’Π΅Ρ€Π½Π΅Π΅, устройство, Π²Ρ‹ΠΏΠΎΠ»Π½ΡΡŽΡ‰Π΅Π΅ Π΅Π³ΠΎ Ρ„ΡƒΠ½ΠΊΡ†ΠΈΠΈ, имССтся, Π½ΠΎ Π½Π°Π·Π²Π°Π½ΠΈΠ΅ Ρƒ Π½Π΅Π³ΠΎ ΡΠΎΠ²Π΅Ρ€ΡˆΠ΅Π½Π½ΠΎ Π΄Ρ€ΡƒΠ³ΠΎΠ΅ — ΠΊΠΎΠΌΠΏΠ°Ρ€Π°Ρ‚ΠΎΡ€. НСкоторая ΠΏΡƒΡ‚Π°Π½ΠΈΡ†Π° Π² этом ΠΏΠ»Π°Π½Π΅ ΠΏΡ€ΠΎΠΈΠ·ΠΎΡˆΠ»Π° Π΅Ρ‰Π΅ Π² ΠΏΠ΅Ρ€ΠΈΠΎΠ΄ Ρ€Π°Π·Ρ€Π°Π±ΠΎΡ‚ΠΊΠΈ Ρ‚Π°ΠΊΠΎΠ³ΠΎ ΠΏΡ€ΠΈΠ±ΠΎΡ€Π°. Π‘ΠΏΠ΅Ρ€Π²Π° — Π² ноябрС 2012-Π³ΠΎ — прСдставитСли Mojang (ΠΊΠΎΠΌΠΏΠ°Π½ΠΈΠΈ-создатСля ΠΈΠ³Ρ€Ρ‹) объявили ΠΎ скором появлСнии Π² Π³Π΅ΠΉΠΌΠΏΠ»Π΅Π΅ кондСнсатора. Однако Ρ‡Π΅Ρ€Π΅Π· мСсяц ΠΎΠ½ΠΈ Π²Ρ‹ΡΠΊΠ°Π·Π°Π»ΠΈΡΡŒ ΡƒΠΆΠ΅ ΠΎ Ρ‚ΠΎΠΌ, Ρ‡Ρ‚ΠΎ ΠΊΠ°ΠΊ Ρ‚Π°ΠΊΠΎΠ²ΠΎΠ³ΠΎ этого ΠΏΡ€ΠΈΠ±ΠΎΡ€Π° Π½Π΅ Π±ΡƒΠ΄Π΅Ρ‚, Π° вмСсто Π½Π΅Π³ΠΎ Π² ΠΈΠ³Ρ€Π΅ Π±ΡƒΠ΄Π΅Ρ‚ ΠΊΠΎΠΌΠΏΠ°Ρ€Π°Ρ‚ΠΎΡ€.

ПодобноС устройство сущСствуСт для ΠΏΡ€ΠΎΠ²Π΅Ρ€ΠΊΠΈ заполнСнности располоТСнных ΠΏΠΎΠ·Π°Π΄ΠΈ Π½Π΅Π³ΠΎ ΠΊΠΎΠ½Ρ‚Π΅ΠΉΠ½Π΅Ρ€ΠΎΠ². Π’Π°ΠΊΠΎΠ²Ρ‹ΠΌΠΈ ΠΌΠΎΠ³ΡƒΡ‚ Π±Ρ‹Ρ‚ΡŒ сундуки (Π² Ρ‚ΠΎΠΌ числС Π² Π²ΠΈΠ΄Π΅ Π»ΠΎΠ²ΡƒΡˆΠ΅ΠΊ), Π²Π°Ρ€ΠΎΡ‡Π½Ρ‹Π΅ стойки, Ρ€Π°Π·Π΄Π°Ρ‚Ρ‡ΠΈΠΊΠΈ, выбрасыватСли, ΠΏΠ΅Ρ‡ΠΈ, Π·Π°Π³Ρ€ΡƒΠ·ΠΎΡ‡Π½Ρ‹Π΅ Π²ΠΎΡ€ΠΎΠ½ΠΊΠΈ ΠΈ Ρ‚.ΠΏ.

Помимо этого, Π΅Π³ΠΎ часто ΠΈΡΠΏΠΎΠ»ΡŒΠ·ΡƒΡŽΡ‚ для сравнСния Π΄Π²ΡƒΡ… сигналов рСдстоуна ΠΌΠ΅ΠΆΠ΄Ρƒ собою — ΠΎΠ½ Π²Ρ‹Π΄Π°Π΅Ρ‚ Ρ€Π΅Π·ΡƒΠ»ΡŒΡ‚Π°Ρ‚ Π² соотвСтствии с Ρ‚Π΅ΠΌ, ΠΊΠ°ΠΊ Π±Ρ‹Π»ΠΎ Π·Π°ΠΏΡ€ΠΎΠ³Ρ€Π°ΠΌΠΌΠΈΡ€ΠΎΠ²Π°Π½ΠΎ Π² Π΄Π°Π½Π½ΠΎΠΉ Ρ†Π΅ΠΏΠΈ, ΠΈ с Ρ‚Π΅ΠΌ, ΠΊΠ°ΠΊΠΎΠΉ Ρ€Π΅ΠΆΠΈΠΌ Π²Ρ‹Π±Ρ€Π°Π½ для самого ΠΌΠ΅Ρ…Π°Π½ΠΈΠ·ΠΌΠ°. Π’ частности, ΠΊΠΎΠΌΠΏΠ°Ρ€Π°Ρ‚ΠΎΡ€ ΠΌΠΎΠΆΠ΅Ρ‚ Ρ€Π°Π·Ρ€Π΅ΡˆΠΈΡ‚ΡŒ Π·Π°ΠΆΠΈΠ³Π°Π½ΠΈΠ΅ Ρ„Π°ΠΊΠ΅Π»Π°, Ссли ΠΏΠ΅Ρ€Π²Ρ‹ΠΉ сигнал большС Π»ΠΈΠ±ΠΎ Ρ€Π°Π²Π΅Π½ Π΄Ρ€ΡƒΠ³ΠΎΠΌΡƒ.

Π’Π°ΠΊΠΆΠ΅ ΠΏΠΎΡ€ΠΎΠΉ кондСнсатор-ΠΊΠΎΠΌΠΏΠ°Ρ€Π°Ρ‚ΠΎΡ€ ΡƒΡΡ‚Π°Π½Π°Π²Π»ΠΈΠ²Π°ΡŽΡ‚ рядом с ΠΏΡ€ΠΎΠΈΠ³Ρ€Ρ‹Π²Π°Ρ‚Π΅Π»Π΅ΠΌ, ΠΏΠΎΠ΄ΠΊΠ»ΡŽΡ‡Π°Ρ Π΅Π³ΠΎ Π²Ρ…ΠΎΠ΄ΠΎΠΌ ΠΊ послСднСму. Когда Π² звуковоспроизводящСм устройствС проигрываСтся какая-Π»ΠΈΠ±ΠΎ пластинка, Π²Ρ‹ΡˆΠ΅ΡƒΠΏΠΎΠΌΡΠ½ΡƒΡ‚Ρ‹ΠΉ ΠΏΡ€ΠΈΠ±ΠΎΡ€ Π±ΡƒΠ΄Π΅Ρ‚ Π²Ρ‹Π΄Π°Π²Π°Ρ‚ΡŒ сигнал, Ρ€Π°Π²Π½Ρ‹ΠΉ ΠΏΠΎ силС порядковому Π½ΠΎΠΌΠ΅Ρ€Ρƒ диска.

Π‘ΠΊΡ€Π°Ρ„Ρ‚ΠΈΡ‚ΡŒ Ρ‚Π°ΠΊΠΎΠΉ ΠΊΠΎΠΌΠΏΠ°Ρ€Π°Ρ‚ΠΎΡ€ нСслоТно, Ссли имССтся достаточно Ρ‚Ρ€ΡƒΠ΄Π½ΠΎ Π΄ΠΎΠ±Ρ‹Π²Π°Π΅ΠΌΡ‹ΠΉ рСсурс — адский ΠΊΠ²Π°Ρ€Ρ†. Π•Π³ΠΎ Π½Π°Π΄ΠΎ ΠΏΠΎΡΡ‚Π°Π²ΠΈΡ‚ΡŒ Π² Ρ†Π΅Π½Ρ‚Ρ€Π°Π»ΡŒΠ½Ρ‹ΠΉ слот вСрстака, Π½Π°Π΄ Π½ΠΈΠΌ ΠΈ ΠΏΠΎ Π±ΠΎΠΊΠ°ΠΌ ΠΎΡ‚ Π½Π΅Π³ΠΎ ΡƒΡΡ‚Π°Π½ΠΎΠ²ΠΈΡ‚ΡŒ Ρ‚Ρ€ΠΈ красных Ρ„Π°ΠΊΠ΅Π»Π°, Π° Π² Π½ΠΈΠΆΠ½Π΅ΠΌ ряду — Ρ‚Π°ΠΊΠΎΠ΅ ΠΆΠ΅ количСство ΠΊΠ°ΠΌΠ΅Π½Π½Ρ‹Ρ… Π±Π»ΠΎΠΊΠΎΠ².

Π’ большом количСствС ΠΌΠΎΠ΄ΠΎΠ² ΠΏΠΎΠΏΠ°Π΄Π°ΡŽΡ‚ΡΡ кондСнсаторы, ΠΈΠΌΠ΅ΡŽΡ‰ΠΈΠ΅ самоС Ρ€Π°Π·Π½ΠΎΠ΅ ΠΏΡ€Π΅Π΄Π½Π°Π·Π½Π°Ρ‡Π΅Π½ΠΈΠ΅. К ΠΏΡ€ΠΈΠΌΠ΅Ρ€Ρƒ, Π² Galacticraft, Π³Π΄Π΅ Ρƒ Π³Π΅ΠΉΠΌΠ΅Ρ€ΠΎΠ² Π΅ΡΡ‚ΡŒ Π²ΠΎΠ·ΠΌΠΎΠΆΠ½ΠΎΡΡ‚ΡŒ ΡΠ»Π΅Ρ‚Π°Ρ‚ΡŒ Π½Π° ΠΌΠ½ΠΎΠ³ΠΈΠ΅ ΠΏΠ»Π°Π½Π΅Ρ‚Ρ‹ для ознакомлСния с Ρ‚Π°ΠΌΠΎΡˆΠ½ΠΈΠΌΠΈ рСалиями, появляСтся Ρ€Π΅Ρ†Π΅ΠΏΡ‚ ΠΊΡ€Π°Ρ„Ρ‚Π° кислородного кондСнсатора. Он слуТит для создания ΠΌΠ΅Ρ…Π°Π½ΠΈΠ·ΠΌΠΎΠ² Π²Ρ€ΠΎΠ΄Π΅ ΠΊΠΎΠ»Π»Π΅ΠΊΡ‚ΠΎΡ€Π° ΠΈ накопитСля Π³Π°Π·Π° для дыхания, Π° Ρ‚Π°ΠΊΠΆΠ΅ Ρ€Π°ΠΌΠΊΠΈ Π²ΠΎΠ·Π΄ΡƒΡˆΠ½ΠΎΠ³ΠΎ шлюза. Для Π΅Π³ΠΎ изготовлСния Ρ‡Π΅Ρ‚Ρ‹Ρ€Π΅ ΡΡ‚Π°Π»ΡŒΠ½Ρ‹Ρ… пластины Ρ€Π°Π·ΠΌΠ΅Ρ‰Π°ΡŽΡ‚ΡΡ ΠΏΠΎ ΡƒΠ³Π»Π°ΠΌ вСрстака, Π² Ρ†Π΅Π½Ρ‚Ρ€Π΅ — оловянная канистра, Π° ΠΏΠΎΠ΄ нСю — Π²ΠΎΠ·Π΄ΡƒΡ…ΠΎΠ²ΠΎΠ΄. ΠžΡΡ‚Π°Π»ΡŒΠ½Ρ‹Π΅ Ρ‚Ρ€ΠΈ ячСйки Π·Π°Π½ΠΈΠΌΠ°ΡŽΡ‚ пластины ΠΈΠ· ΠΎΠ»ΠΎΠ²Π°.

Π’ JurassiCraft сущСствуСт кондСнсатор ΠΏΠΎΡ‚ΠΎΠΊΠ° — Π½Π΅ΠΊΠΈΠΉ Ρ‚Π΅Π»Π΅ΠΏΠΎΡ€Ρ‚, ΠΏΠΎΠ·Π²ΠΎΠ»ΡΡŽΡ‰ΠΈΠΉ ΠΏΠ΅Ρ€Π΅ΠΌΠ΅ΡΡ‚ΠΈΡ‚ΡŒΡΡ Π² ΡƒΠ΄ΠΈΠ²ΠΈΡ‚Π΅Π»ΡŒΠ½Ρ‹ΠΉ ΠΈΠ³Ρ€ΠΎΠ²ΠΎΠΉ ΠΌΠΈΡ€, ΠΊΠΈΡˆΠ°Ρ‰ΠΈΠΉ Π΄ΠΈΠ½ΠΎΠ·Π°Π²Ρ€Π°ΠΌΠΈ. Для создания Ρ‚Π°ΠΊΠΎΠ³ΠΎ ΠΏΡ€ΠΈΠ±ΠΎΡ€Π° Π½ΡƒΠΆΠ½ΠΎ ΠΏΠΎΠΌΠ΅ΡΡ‚ΠΈΡ‚ΡŒ Π² Π΄Π²Π° ΠΊΡ€Π°ΠΉΠ½ΠΈΡ… Π²Π΅Ρ€Ρ‚ΠΈΠΊΠ°Π»ΡŒΠ½Ρ‹Ρ… ряда ΡˆΠ΅ΡΡ‚ΡŒ ΠΆΠ΅Π»Π΅Π·Π½Ρ‹Ρ… слитков, Π° Π² срСдний — Π΄Π²Π° Π°Π»ΠΌΠ°Π·Π° ΠΈ ΠΌΠ΅ΠΆΠ΄Ρƒ Π½ΠΈΠΌΠΈ Π΅Π΄ΠΈΠ½ΠΈΡ†Ρƒ ΠΏΡ‹Π»ΠΈ рСдстоуна. Π”Π°Π±Ρ‹ устройство Π·Π°Ρ€Π°Π±ΠΎΡ‚Π°Π»ΠΎ, Π½Π°Π΄ΠΎ ΠΏΠΎΡΡ‚Π°Π²ΠΈΡ‚ΡŒ Π΅Π³ΠΎ Π½Π° свинью Π»ΠΈΠ±ΠΎ Π²Π°Π³ΠΎΠ½Π΅Ρ‚ΠΊΡƒ, Π° Π·Π°Ρ‚Π΅ΠΌ Ρ‰Π΅Π»ΠΊΠ½ΡƒΡ‚ΡŒ ΠΏΠΎ Π½Π΅ΠΌΡƒ ΠΏΡ€Π°Π²ΠΎΠΉ клавишСй ΠΌΡ‹ΡˆΠΈ, быстро Π·Π°ΠΏΡ€Ρ‹Π³Π½ΡƒΠ² Ρ‚ΡƒΠ΄Π°. ΠŸΡ€ΠΈ этом трСбуСтся ΠΏΠΎΠ΄Π΄Π΅Ρ€ΠΆΠ°Π½ΠΈΠ΅ высокой скорости устройства.

Π‘ ΠΌΠΎΠ΄ΠΎΠΌ Industrial Craft2 Ρƒ ΠΈΠ³Ρ€ΠΎΠΊΠ° появляСтся Π²ΠΎΠ·ΠΌΠΎΠΆΠ½ΠΎΡΡ‚ΡŒ ΡΠΎΠ·Π΄Π°Π²Π°Ρ‚ΡŒ ΠΊΠ°ΠΊ ΠΌΠΈΠ½ΠΈΠΌΡƒΠΌ Π΄Π²Π° Π²ΠΈΠ΄Π° Ρ‚Π΅ΠΏΠ»ΠΎΠ²Ρ‹Ρ… кондСнсаторов — красный ΠΈ Π»Π°Π·ΡƒΡ€ΠΈΡ‚ΠΎΠ²Ρ‹ΠΉ. Они слуТат ΠΈΡΠΊΠ»ΡŽΡ‡ΠΈΡ‚Π΅Π»ΡŒΠ½ΠΎ для охлаТдСния ядСрного Ρ€Π΅Π°ΠΊΡ‚ΠΎΡ€Π° ΠΈ для накоплСния Π΅Π³ΠΎ энСргии ΠΈ Ρ…ΠΎΡ€ΠΎΡˆΠΈ для цикличСских сооруТСний Ρ‚Π°ΠΊΠΎΠ³ΠΎ Ρ‚ΠΈΠΏΠ°. ΠžΡΡ‚ΡƒΠΆΠ°ΡŽΡ‚ΡΡ ΠΎΠ½ΠΈ сами, соотвСтствСнно, красной ΠΏΡ‹Π»ΡŒΡŽ ΠΈΠ»ΠΈ Π»Π°Π·ΡƒΡ€ΠΈΡ‚ΠΎΠΌ.

ΠšΡ€Π°ΡΠ½Ρ‹ΠΉ тСплокондСнсатор дСлаСтся ΠΈΠ· сСми Π΅Π΄ΠΈΠ½ΠΈΡ† ΠΏΡ‹Π»ΠΈ рСдстоуна — ΠΈΡ… Π½Π°Π΄ΠΎ ΡƒΡΡ‚Π°Π½ΠΎΠ²ΠΈΡ‚ΡŒ Π² Π²ΠΈΠ΄Π΅ Π±ΡƒΠΊΠ²Ρ‹ П ΠΈ Ρ€Π°ΡΡΡ‚Π°Π²ΠΈΡ‚ΡŒ ΠΏΠΎΠ΄ Π½ΠΈΠΌΠΈ Ρ‚Π΅ΠΏΠ»ΠΎΠΎΡ‚Π²ΠΎΠ΄ ΠΈ Ρ‚Π΅ΠΏΠ»ΠΎΠΎΠ±ΠΌΠ΅Π½Π½ΠΈΠΊ. ΠšΡ€Π°Ρ„Ρ‚ΠΈΠ½Π³ ΠΆΠ΅ Π»Π°Π·ΡƒΡ€ΠΈΡ‚ΠΎΠ²ΠΎΠ³ΠΎ устройства Ρ‡ΡƒΡ‚ΡŒ послоТнСС. Для Π΅Π³ΠΎ создания Ρ‡Π΅Ρ‚Ρ‹Ρ€Π΅ Π΅Π΄ΠΈΠ½ΠΈΡ†Ρ‹ ΠΏΡ‹Π»ΠΈ рСдстоуна Ρ€Π°ΡΡΡ‚Π°Π²Π»ΡΡŽΡ‚ΡΡ ΠΏΠΎ ΡƒΠ³Π»Π°ΠΌ станка, Π² Ρ†Π΅Π½Ρ‚Ρ€ ΠΏΠΎΠΉΠ΄Π΅Ρ‚ Π±Π»ΠΎΠΊ Π»Π°Π·ΡƒΡ€ΠΈΡ‚Π°, ΠΏΠΎ Π±ΠΎΠΊΠ°ΠΌ ΠΎΡ‚ Π½Π΅Π³ΠΎ — Π΄Π²Π° красных Ρ‚Π΅ΠΏΠ»ΠΎΠ²Ρ‹Ρ… кондСнсатора, свСрху — Ρ‚Π΅ΠΏΠ»ΠΎΠΎΡ‚Π²ΠΎΠ΄ Ρ€Π΅Π°ΠΊΡ‚ΠΎΡ€Π°, Π° снизу — Π΅Π³ΠΎ ΠΆΠ΅ Ρ‚Π΅ΠΏΠ»ΠΎΠΎΠ±ΠΌΠ΅Π½Π½ΠΈΠΊ.

Π’ ThaumCraft, Π³Π΄Π΅ сдСлан Π°ΠΊΡ†Π΅Π½Ρ‚ Π½Π° настоящСм чародСйствС, кондСнсаторы Ρ‚ΠΎΠΆΠ΅ ΠΈΡΠΏΠΎΠ»ΡŒΠ·ΡƒΡŽΡ‚ΡΡ. НапримСр, ΠΎΠ΄ΠΈΠ½ ΠΈΠ· Π½ΠΈΡ… — кристалличСский — сущСствуСт для аккумуляции ΠΈ ΠΎΡ‚Π΄Π°Ρ‡ΠΈ ΠΌΠ°Π³ΠΈΠΈ. ΠŸΡ€ΠΈΡ‡Π΅ΠΌ, Ρ‡Ρ‚ΠΎ интСрСсно, ΡΠΎΠ·Π΄Π°Π²Π°Ρ‚ΡŒ Π΅Π³ΠΎ ΠΈ ΠΌΠ½ΠΎΠ³ΠΈΠ΅ Π΄Ρ€ΡƒΠ³ΠΈΠ΅ Π²Π΅Ρ‰ΠΈ Ρ€Π°Π·Ρ€Π΅ΡˆΠ΅Π½ΠΎ лишь послС изучСния особого элСмСнта гСймплСя — исслСдования, ΠΏΡ€ΠΎΠ²ΠΎΠ΄ΠΈΠΌΠΎΠ³ΠΎ Π·Π° ΡΠΏΠ΅Ρ†ΠΈΠ°Π»ΡŒΠ½Ρ‹ΠΌ столом ΠΈ с ΠΎΠΏΡ€Π΅Π΄Π΅Π»Π΅Π½Π½Ρ‹ΠΌΠΈ ΠΏΡ€ΠΈΠ±ΠΎΡ€Π°ΠΌΠΈ.

ДСлаСтся Ρ‚Π°ΠΊΠΎΠΉ кондСнсатор ΠΈΠ· восьми тусклых осколков, Π² Ρ†Π΅Π½Ρ‚Ρ€ ΠΊΠΎΡ‚ΠΎΡ€Ρ‹Ρ… Π½Π° вСрстакС помСщаСтся мистичСский дСрСвянный Π±Π»ΠΎΠΊ. К соТалСнию, ΠΏΠΎΠ΄ΠΎΠ±Π½Ρ‹ΠΉ ΠΏΡ€ΠΈΠ±ΠΎΡ€ — Ρ€Π°Π²Π½ΠΎ ΠΊΠ°ΠΊ ΠΈ Π΅Π³ΠΎ ΡΠΎΡΡ‚Π°Π²Π»ΡΡŽΡ‰ΠΈΠ΅ — просущСствовал лишь Π΄ΠΎ ThaumCraft 3, Π° Π² Ρ‡Π΅Ρ‚Π²Π΅Ρ€Ρ‚ΠΎΠΉ вСрсии ΠΌΠΎΠ΄Π° Π±Ρ‹Π» ΡƒΠΏΡ€Π°Π·Π΄Π½Π΅Π½.

www.kakprosto.ru

Π‘ΠΎΠ΅Π΄ΠΈΠ½Π΅Π½ΠΈΠ΅ кондСнсаторов: Ρ„ΠΎΡ€ΠΌΡƒΠ»Ρ‹

Π‘ΠΎΠ΄Π΅Ρ€ΠΆΠ°Π½ΠΈΠ΅:

  1. ΠŸΠΎΡΠ»Π΅Π΄ΠΎΠ²Π°Ρ‚Π΅Π»ΡŒΠ½ΠΎΠ΅ соСдинСниС
  2. Онлайн ΠΊΠ°Π»ΡŒΠΊΡƒΠ»ΡΡ‚ΠΎΡ€
  3. БмСшанноС соСдинСниС
  4. ΠŸΠ°Ρ€Π°Π»Π»Π΅Π»ΡŒΠ½ΠΎΠ΅ соСдинСниС
  5. Π’ΠΈΠ΄Π΅ΠΎ

Π’ элСктронных ΠΈ радиотСхничСских схСмах ΡˆΠΈΡ€ΠΎΠΊΠΎΠ΅ распространСниС ΠΏΠΎΠ»ΡƒΡ‡ΠΈΠ»ΠΎ ΠΏΠ°Ρ€Π°Π»Π»Π΅Π»ΡŒΠ½ΠΎΠ΅ ΠΈ ΠΏΠΎΡΠ»Π΅Π΄ΠΎΠ²Π°Ρ‚Π΅Π»ΡŒΠ½ΠΎΠ΅ соСдинСниС кондСнсаторов. Π’ ΠΏΠ΅Ρ€Π²ΠΎΠΌ случаС соСдинСниС осущСствляСтся Π±Π΅Π· ΠΊΠ°ΠΊΠΈΡ…-Π»ΠΈΠ±ΠΎ ΠΎΠ±Ρ‰ΠΈΡ… ΡƒΠ·Π»ΠΎΠ², Π° Π²ΠΎ Π²Ρ‚ΠΎΡ€ΠΎΠΌ Π²Π°Ρ€ΠΈΠ°Π½Ρ‚Π΅ всС элСмСнты ΠΎΠ±ΡŠΠ΅Π΄ΠΈΠ½ΡΡŽΡ‚ΡΡ Π² Π΄Π²Π° ΡƒΠ·Π»Π° ΠΈ Π½Π΅ связаны с Π΄Ρ€ΡƒΠ³ΠΈΠΌΠΈ ΡƒΠ·Π»Π°ΠΌΠΈ, Ссли это Π·Π°Ρ€Π°Π½Π΅Π΅ Π½Π΅ прСдусмотрСно схСмой.

ΠŸΠΎΡΠ»Π΅Π΄ΠΎΠ²Π°Ρ‚Π΅Π»ΡŒΠ½ΠΎΠ΅ соСдинСниС

ΠŸΡ€ΠΈ ΠΏΠΎΡΠ»Π΅Π΄ΠΎΠ²Π°Ρ‚Π΅Π»ΡŒΠ½ΠΎΠΌ соСдинСнии Π΄Π²Π° ΠΈ Π±ΠΎΠ»Π΅Π΅ кондСнсаторов ΡΠΎΠ΅Π΄ΠΈΠ½ΡΡŽΡ‚ΡΡ Π² ΠΎΠ±Ρ‰ΡƒΡŽ Ρ†Π΅ΠΏΡŒ Ρ‚Π°ΠΊΠΈΠΌ ΠΎΠ±Ρ€Π°Π·ΠΎΠΌ, Ρ‡Ρ‚ΠΎ ΠΊΠ°ΠΆΠ΄Ρ‹ΠΉ ΠΏΡ€Π΅Π΄Ρ‹Π΄ΡƒΡ‰ΠΈΠΉ кондСнсатор соСдиняСтся с ΠΏΠΎΡΠ»Π΅Π΄ΡƒΡŽΡ‰ΠΈΠΌ лишь Π² ΠΎΠ΄Π½ΠΎΠΉ ΠΎΠ±Ρ‰Π΅ΠΉ Ρ‚ΠΎΡ‡ΠΊΠ΅. Π’ΠΎΠΊ (i), ΠΎΡΡƒΡ‰Π΅ΡΡ‚Π²Π»ΡΡŽΡ‰ΠΈΠΉ зарядку ΠΏΠΎΡΠ»Π΅Π΄ΠΎΠ²Π°Ρ‚Π΅Π»ΡŒΠ½ΠΎΠΉ Ρ†Π΅ΠΏΠΈ кондСнсаторов Π±ΡƒΠ΄Π΅Ρ‚ ΠΈΠΌΠ΅Ρ‚ΡŒ ΠΎΠ΄ΠΈΠ½Π°ΠΊΠΎΠ²ΠΎΠ΅ Π·Π½Π°Ρ‡Π΅Π½ΠΈΠ΅ для ΠΊΠ°ΠΆΠ΄ΠΎΠ³ΠΎ элСмСнта, ΠΏΠΎΡΠΊΠΎΠ»ΡŒΠΊΡƒ ΠΎΠ½ ΠΏΡ€ΠΎΡ…ΠΎΠ΄ΠΈΡ‚ Ρ‚ΠΎΠ»ΡŒΠΊΠΎ ΠΏΠΎ СдинствСнно Π²ΠΎΠ·ΠΌΠΎΠΆΠ½ΠΎΠΌΡƒ ΠΏΡƒΡ‚ΠΈ. Π­Ρ‚ΠΎ ΠΏΠΎΠ»ΠΎΠΆΠ΅Π½ΠΈΠ΅ подтвСрТдаСтся Ρ„ΠΎΡ€ΠΌΡƒΠ»ΠΎΠΉ: i = ic1 = ic2 = ic3 = ic4.

Π’ связи с ΠΎΠ΄ΠΈΠ½Π°ΠΊΠΎΠ²Ρ‹ΠΌ Π·Π½Π°Ρ‡Π΅Π½ΠΈΠ΅ΠΌ Ρ‚ΠΎΠΊΠ°, ΠΏΡ€ΠΎΡ‚Π΅ΠΊΠ°ΡŽΡ‰Π΅Π³ΠΎ Ρ‡Π΅Ρ€Π΅Π· кондСнсаторы с ΠΏΠΎΡΠ»Π΅Π΄ΠΎΠ²Π°Ρ‚Π΅Π»ΡŒΠ½Ρ‹ΠΌ соСдинСниСм, Π²Π΅Π»ΠΈΡ‡ΠΈΠ½Π° заряда, Π½Π°ΠΊΠΎΠΏΠ»Π΅Π½Π½ΠΎΠ³ΠΎ ΠΊΠ°ΠΆΠ΄Ρ‹ΠΌ ΠΈΠ· Π½ΠΈΡ…, Π±ΡƒΠ΄Π΅Ρ‚ ΠΎΠ΄ΠΈΠ½Π°ΠΊΠΎΠ²ΠΎΠΉ, нСзависимо ΠΎΡ‚ Смкости. Π’Π°ΠΊΠΎΠ΅ становится Π²ΠΎΠ·ΠΌΠΎΠΆΠ½Ρ‹ΠΌ, ΠΏΠΎΡΠΊΠΎΠ»ΡŒΠΊΡƒ заряд, приходящий с ΠΎΠ±ΠΊΠ»Π°Π΄ΠΊΠΈ ΠΏΡ€Π΅Π΄Ρ‹Π΄ΡƒΡ‰Π΅Π³ΠΎ кондСнсатора, накапливаСтся Π½Π° ΠΎΠ±ΠΊΠ»Π°Π΄ΠΊΠ΅ ΠΏΠΎΡΠ»Π΅Π΄ΡƒΡŽΡ‰Π΅Π³ΠΎ элСмСнта Ρ†Π΅ΠΏΠΈ. ΠŸΠΎΡΡ‚ΠΎΠΌΡƒ Π²Π΅Π»ΠΈΡ‡ΠΈΠ½Π° заряда Ρƒ ΠΏΠΎΡΠ»Π΅Π΄ΠΎΠ²Π°Ρ‚Π΅Π»ΡŒΠ½ΠΎ соСдинСнных кондСнсаторов Π±ΡƒΠ΄Π΅Ρ‚ Π²Ρ‹Π³Π»ΡΠ΄Π΅Ρ‚ΡŒ ΡΠ»Π΅Π΄ΡƒΡŽΡ‰ΠΈΠΌ ΠΎΠ±Ρ€Π°Π·ΠΎΠΌ: QΠΎΠ±Ρ‰= Q1 = Q2 = Q3.

Если Ρ€Π°ΡΡΠΌΠΎΡ‚Ρ€Π΅Ρ‚ΡŒ Ρ‚Ρ€ΠΈ кондСнсатора Π‘1, Π‘2 ΠΈ Π‘3, соСдинСнныС Π² ΠΏΠΎΡΠ»Π΅Π΄ΠΎΠ²Π°Ρ‚Π΅Π»ΡŒΠ½ΡƒΡŽ Ρ†Π΅ΠΏΡŒ, Ρ‚ΠΎ выясняСтся, Ρ‡Ρ‚ΠΎ срСдний кондСнсатор Π‘2 ΠΏΡ€ΠΈ постоянном Ρ‚ΠΎΠΊΠ΅ оказываСтся элСктричСски ΠΈΠ·ΠΎΠ»ΠΈΡ€ΠΎΠ²Π°Π½Π½Ρ‹ΠΌ ΠΎΡ‚ ΠΎΠ±Ρ‰Π΅ΠΉ Ρ†Π΅ΠΏΠΈ. Π’ ΠΊΠΎΠ½Π΅Ρ‡Π½ΠΎΠΌ ΠΈΡ‚ΠΎΠ³Π΅ Π²Π΅Π»ΠΈΡ‡ΠΈΠ½Π° эффСктивной ΠΏΠ»ΠΎΡ‰Π°Π΄ΠΈ ΠΎΠ±ΠΊΠ»Π°Π΄ΠΎΠΊ Π±ΡƒΠ΄Π΅Ρ‚ ΡƒΠΌΠ΅Π½ΡŒΡˆΠ΅Π½Π° Π΄ΠΎ ΠΏΠ»ΠΎΡ‰Π°Π΄ΠΈ ΠΎΠ±ΠΊΠ»Π°Π΄ΠΎΠΊ кондСнсатора с самыми ΠΌΠΈΠ½ΠΈΠΌΠ°Π»ΡŒΠ½Ρ‹ΠΌΠΈ Ρ€Π°Π·ΠΌΠ΅Ρ€Π°ΠΌΠΈ. ПолноС Π·Π°ΠΏΠΎΠ»Π½Π΅Π½ΠΈΠ΅ ΠΎΠ±ΠΊΠ»Π°Π΄ΠΎΠΊ элСктричСским зарядом, Π΄Π΅Π»Π°Π΅Ρ‚ Π½Π΅Π²ΠΎΠ·ΠΌΠΎΠΆΠ½Ρ‹ΠΌ дальнСйшСС ΠΏΡ€ΠΎΡ…ΠΎΠΆΠ΄Π΅Π½ΠΈΠ΅ ΠΏΠΎ Π½Π΅ΠΌΡƒ Ρ‚ΠΎΠΊΠ°. Π’ Ρ€Π΅Π·ΡƒΠ»ΡŒΡ‚Π°Ρ‚Π΅, Π΄Π²ΠΈΠΆΠ΅Π½ΠΈΠ΅ Ρ‚ΠΎΠΊΠ° прСкращаСтся Π²ΠΎ всСй Ρ†Π΅ΠΏΠΈ, соотвСтствСнно прСкращаСтся ΠΈ зарядка всСх ΠΎΡΡ‚Π°Π»ΡŒΠ½Ρ‹Ρ… кондСнсаторов.

ΠžΠ±Ρ‰Π΅Π΅ расстояниС ΠΌΠ΅ΠΆΠ΄Ρƒ ΠΎΠ±ΠΊΠ»Π°Π΄ΠΊΠ°ΠΌΠΈ ΠΏΡ€ΠΈ ΠΏΠΎΡΠ»Π΅Π΄ΠΎΠ²Π°Ρ‚Π΅Π»ΡŒΠ½ΠΎΠΌ соСдинСнии прСдставляСт собой сумму расстояний ΠΌΠ΅ΠΆΠ΄Ρƒ ΠΎΠ±ΠΊΠ»Π°Π΄ΠΊΠ°ΠΌΠΈ ΠΊΠ°ΠΆΠ΄ΠΎΠ³ΠΎ элСмСнта. Π’ Ρ€Π΅Π·ΡƒΠ»ΡŒΡ‚Π°Ρ‚Π΅ соСдинСния Π² ΠΏΠΎΡΠ»Π΅Π΄ΠΎΠ²Π°Ρ‚Π΅Π»ΡŒΠ½ΡƒΡŽ Ρ†Π΅ΠΏΡŒ, формируСтся Π΅Π΄ΠΈΠ½Ρ‹ΠΉ большой кондСнсатор, ΠΏΠ»ΠΎΡ‰Π°Π΄ΡŒ ΠΎΠ±ΠΊΠ»Π°Π΄ΠΎΠΊ ΠΊΠΎΡ‚ΠΎΡ€ΠΎΠ³ΠΎ соотвСтствуСт ΠΎΠ±ΠΊΠ»Π°Π΄ΠΊΠ°ΠΌ элСмСнта с минимальной Π΅ΠΌΠΊΠΎΡΡ‚ΡŒΡŽ. РасстояниС ΠΌΠ΅ΠΆΠ΄Ρƒ ΠΎΠ±ΠΊΠ»Π°Π΄ΠΊΠ°ΠΌΠΈ оказываСтся Ρ€Π°Π²Π½Ρ‹ΠΌ суммС всСх расстояний, ΠΈΠΌΠ΅ΡŽΡ‰ΠΈΡ…ΡΡ Π² Ρ†Π΅ΠΏΠΈ.

ПадСниС напряТСния Π½Π° ΠΊΠ°ΠΆΠ΄Ρ‹ΠΉ кондСнсатор Π±ΡƒΠ΄Π΅Ρ‚ Ρ€Π°Π·Π½Ρ‹ΠΌ, Π² зависимости ΠΎΡ‚ Смкости. Π”Π°Π½Π½ΠΎΠ΅ ΠΏΠΎΠ»ΠΎΠΆΠ΅Π½ΠΈΠ΅ опрСдСляСтся Ρ„ΠΎΡ€ΠΌΡƒΠ»ΠΎΠΉ: Π‘ = Q/V, Π² ΠΊΠΎΡ‚ΠΎΡ€ΠΎΠΉ Π΅ΠΌΠΊΠΎΡΡ‚ΡŒ ΠΎΠ±Ρ€Π°Ρ‚Π½ΠΎ ΠΏΡ€ΠΎΠΏΠΎΡ€Ρ†ΠΈΠΎΠ½Π°Π»ΡŒΠ½Π° Π½Π°ΠΏΡ€ΡΠΆΠ΅Π½ΠΈΡŽ. Π’Π°ΠΊΠΈΠΌ ΠΎΠ±Ρ€Π°Π·ΠΎΠΌ, с ΡƒΠΌΠ΅Π½ΡŒΡˆΠ΅Π½ΠΈΠ΅ΠΌ Смкости кондСнсатора Π½Π° Π½Π΅Π³ΠΎ ΠΏΠ°Π΄Π°Π΅Ρ‚ Π±ΠΎΠ»Π΅Π΅ высокоС напряТСниС. Буммарная Π΅ΠΌΠΊΠΎΡΡ‚ΡŒ всСх кондСнсаторов вычисляСтся ΠΏΠΎ Ρ„ΠΎΡ€ΠΌΡƒΠ»Π΅: 1/CΠΎΠ±Ρ‰ = 1/C1 + 1/C2 + 1/C3.

Главная ΠΎΡΠΎΠ±Π΅Π½Π½ΠΎΡΡ‚ΡŒ Ρ‚Π°ΠΊΠΎΠΉ схСмы Π·Π°ΠΊΠ»ΡŽΡ‡Π°Π΅Ρ‚ΡΡ Π² ΠΏΡ€ΠΎΡ…ΠΎΠΆΠ΄Π΅Π½ΠΈΠΈ элСктричСской энСргии Ρ‚ΠΎΠ»ΡŒΠΊΠΎ Π² ΠΎΠ΄Π½ΠΎΠΌ Π½Π°ΠΏΡ€Π°Π²Π»Π΅Π½ΠΈΠΈ. ΠŸΠΎΡΡ‚ΠΎΠΌΡƒ Π² ΠΊΠ°ΠΆΠ΄ΠΎΠΌ кондСнсаторС Π·Π½Π°Ρ‡Π΅Π½ΠΈΠ΅ Ρ‚ΠΎΠΊΠ° Π±ΡƒΠ΄Π΅Ρ‚ ΠΎΠ΄ΠΈΠ½Π°ΠΊΠΎΠ²Ρ‹ΠΌ. ΠšΠ°ΠΆΠ΄Ρ‹ΠΉ Π½Π°ΠΊΠΎΠΏΠΈΡ‚Π΅Π»ΡŒ Π² ΠΏΠΎΡΠ»Π΅Π΄ΠΎΠ²Π°Ρ‚Π΅Π»ΡŒΠ½ΠΎΠΉ Ρ†Π΅ΠΏΠΈ Π½Π°ΠΊΠ°ΠΏΠ»ΠΈΠ²Π°Π΅Ρ‚ Ρ€Π°Π²Π½ΠΎΠ΅ количСство энСргии, нСзависимо ΠΎΡ‚ Смкости. Π’ΠΎ Π΅ΡΡ‚ΡŒ Π΅ΠΌΠΊΠΎΡΡ‚ΡŒ ΠΌΠΎΠΆΠ΅Ρ‚ Π²ΠΎΡΠΏΡ€ΠΎΠΈΠ·Π²ΠΎΠ΄ΠΈΡ‚ΡŒΡΡ Π·Π° счСт энСргии, ΠΏΡ€ΠΈΡΡƒΡ‚ΡΡ‚Π²ΡƒΡŽΡ‰Π΅ΠΉ Π² сосСднСм Π½Π°ΠΊΠΎΠΏΠΈΡ‚Π΅Π»Π΅.

Онлайн ΠΊΠ°Π»ΡŒΠΊΡƒΠ»ΡΡ‚ΠΎΡ€, для расчСта Смкости кондСнсаторов соСдинСнных ΠΏΠΎΡΠ»Π΅Π΄ΠΎΠ²Π°Ρ‚Π΅Π»ΡŒΠ½ΠΎ Π² элСктричСской Ρ†Π΅ΠΏΠΈ.

БмСшанноС соСдинСниС

ΠŸΠ°Ρ€Π°Π»Π»Π΅Π»ΡŒΠ½ΠΎΠ΅ соСдинСниС кондСнсаторов

ΠŸΠ°Ρ€Π°Π»Π»Π΅Π»ΡŒΠ½Ρ‹ΠΌ считаСтся Ρ‚Π°ΠΊΠΎΠ΅ соСдинСниС, ΠΏΡ€ΠΈ ΠΊΠΎΡ‚ΠΎΡ€ΠΎΠΌ кондСнсаторы ΡΠΎΠ΅Π΄ΠΈΠ½ΡΡŽΡ‚ΡΡ ΠΌΠ΅ΠΆΠ΄Ρƒ собой двумя ΠΊΠΎΠ½Ρ‚Π°ΠΊΡ‚Π°ΠΌΠΈ. Π’Π°ΠΊΠΈΠΌ ΠΎΠ±Ρ€Π°Π·ΠΎΠΌ Π² ΠΎΠ΄Π½ΠΎΠΉ Ρ‚ΠΎΡ‡ΠΊΠ΅ ΠΌΠΎΠΆΠ΅Ρ‚ ΡΠΎΠ΅Π΄ΠΈΠ½ΡΡ‚ΡŒΡΡ сразу нСсколько элСмСнтов.

Π”Π°Π½Π½Ρ‹ΠΉ Π²ΠΈΠ΄ соСдинСния позволяСт ΡΡ„ΠΎΡ€ΠΌΠΈΡ€ΠΎΠ²Π°Ρ‚ΡŒ Π΅Π΄ΠΈΠ½Ρ‹ΠΉ кондСнсатор с большими Ρ€Π°Π·ΠΌΠ΅Ρ€Π°ΠΌΠΈ, ΠΏΠ»ΠΎΡ‰Π°Π΄ΡŒ ΠΎΠ±ΠΊΠ»Π°Π΄ΠΎΠΊ ΠΊΠΎΡ‚ΠΎΡ€ΠΎΠ³ΠΎ Π±ΡƒΠ΄Π΅Ρ‚ Ρ€Π°Π²Π½Π° суммС ΠΏΠ»ΠΎΡ‰Π°Π΄Π΅ΠΉ ΠΎΠ±ΠΊΠ»Π°Π΄ΠΎΠΊ ΠΊΠ°ΠΆΠ΄ΠΎΠ³ΠΎ, ΠΎΡ‚Π΄Π΅Π»ΡŒΠ½ΠΎ взятого кондСнсатора. Π’ связи с Ρ‚Π΅ΠΌ, Ρ‡Ρ‚ΠΎ Π΅ΠΌΠΊΠΎΡΡ‚ΡŒ кондСнсаторов находится Π² прямой ΠΏΡ€ΠΎΠΏΠΎΡ€Ρ†ΠΈΠΎΠ½Π°Π»ΡŒΠ½ΠΎΠΉ зависимости с ΠΏΠ»ΠΎΡ‰Π°Π΄ΡŒΡŽ ΠΎΠ±ΠΊΠ»Π°Π΄ΠΎΠΊ, общая Π΅ΠΌΠΊΠΎΡΡ‚ΡŒ ΡΠΎΡΡ‚Π°Π²ΠΈΡ‚ΡŒ суммарноС количСство всСх СмкостСй кондСнсаторов, соСдинСнных ΠΏΠ°Ρ€Π°Π»Π»Π΅Π»ΡŒΠ½ΠΎ. Π’ΠΎ Π΅ΡΡ‚ΡŒ, Π‘ΠΎΠ±Ρ‰ = Π‘1 + Π‘2 + Π‘3.

ΠŸΠΎΡΠΊΠΎΠ»ΡŒΠΊΡƒ Ρ€Π°Π·Π½ΠΎΡΡ‚ΡŒ ΠΏΠΎΡ‚Π΅Π½Ρ†ΠΈΠ°Π»ΠΎΠ² Π²ΠΎΠ·Π½ΠΈΠΊΠ°Π΅Ρ‚ лишь Π² Π΄Π²ΡƒΡ… Ρ‚ΠΎΡ‡ΠΊΠ°Ρ…, Ρ‚ΠΎ Π½Π° всС кондСнсаторы, соСдинСнныС ΠΏΠ°Ρ€Π°Π»Π»Π΅Π»ΡŒΠ½ΠΎ, Π±ΡƒΠ΄Π΅Ρ‚ ΠΏΠ°Π΄Π°Ρ‚ΡŒ ΠΎΠ΄ΠΈΠ½Π°ΠΊΠΎΠ²ΠΎΠ΅ напряТСниС. Π‘ΠΈΠ»Π° Ρ‚ΠΎΠΊΠ° Π² ΠΊΠ°ΠΆΠ΄ΠΎΠΌ ΠΈΠ· Π½ΠΈΡ… Π±ΡƒΠ΄Π΅Ρ‚ ΠΎΡ‚Π»ΠΈΡ‡Π°Ρ‚ΡŒΡΡ, Π² зависимости ΠΎΡ‚ Смкости ΠΈ значСния напряТСния. Π’Π°ΠΊΠΈΠΌ ΠΎΠ±Ρ€Π°Π·ΠΎΠΌ, ΠΏΠΎΡΠ»Π΅Π΄ΠΎΠ²Π°Ρ‚Π΅Π»ΡŒΠ½ΠΎΠ΅ ΠΈ ΠΏΠ°Ρ€Π°Π»Π»Π΅Π»ΡŒΠ½ΠΎΠ΅ соСдинСниС, примСняСмоС Π² Ρ€Π°Π·Π»ΠΈΡ‡Π½Ρ‹Ρ… схСмах, позволяСт Π²Ρ‹ΠΏΠΎΠ»Π½ΡΡ‚ΡŒ Ρ€Π΅Π³ΡƒΠ»ΠΈΡ€ΠΎΠ²ΠΊΡƒ Ρ€Π°Π·Π»ΠΈΡ‡Π½Ρ‹Ρ… ΠΏΠ°Ρ€Π°ΠΌΠ΅Ρ‚Ρ€ΠΎΠ² Π½Π° Ρ‚Π΅Ρ… ΠΈΠ»ΠΈ ΠΈΠ½Ρ‹Ρ… участках. Π—Π° счСт этого ΠΏΠΎΠ»ΡƒΡ‡Π°ΡŽΡ‚ΡΡ Π½Π΅ΠΎΠ±Ρ…ΠΎΠ΄ΠΈΠΌΡ‹Π΅ Ρ€Π΅Π·ΡƒΠ»ΡŒΡ‚Π°Ρ‚Ρ‹ Ρ€Π°Π±ΠΎΡ‚Ρ‹ всСй систСмы Π² Ρ†Π΅Π»ΠΎΠΌ.

electric-220.ru


Π’ΠΎ всСх элСктронных устройствах ΠΈΡΠΏΠΎΠ»ΡŒΠ·ΡƒΡŽΡ‚ΡΡ кондСнсаторы. ΠŸΡ€ΠΈ ΠΈΡ… конструировании ΠΈΠ»ΠΈ ΠΈΠ·Π³ΠΎΡ‚ΠΎΠ²Π»Π΅Π½ΠΈΠΈ своими Ρ€ΡƒΠΊΠ°ΠΌΠΈ ΠΏΠ°Ρ€Π°ΠΌΠ΅Ρ‚Ρ€Ρ‹ устройств Ρ€Π°ΡΡΡ‡ΠΈΡ‚Ρ‹Π²Π°ΡŽΡ‚ΡΡ ΠΏΠΎ ΡΠΏΠ΅Ρ†ΠΈΠ°Π»ΡŒΠ½Ρ‹ΠΌ Ρ„ΠΎΡ€ΠΌΡƒΠ»Π°ΠΌ.

Расчёт кондСнсаторов

Один ΠΈΠ· Π³Π»Π°Π²Π½Ρ‹Ρ… ΠΏΠ°Ρ€Π°ΠΌΠ΅Ρ‚Ρ€ΠΎΠ² Ρ‚Π°ΠΊΠΈΡ… устройств – Ρ‘ΠΌΠΊΠΎΡΡ‚ΡŒ. Π Π°ΡΡΡ‡ΠΈΡ‚Π°Ρ‚ΡŒ Π΅Ρ‘ ΠΌΠΎΠΆΠ½ΠΎ ΠΏΠΎ ΡΠ»Π΅Π΄ΡƒΡŽΡ‰Π΅ΠΉ Ρ„ΠΎΡ€ΠΌΡƒΠ»Π΅:

  • C – Ρ‘ΠΌΠΊΠΎΡΡ‚ΡŒ,
  • q – заряд ΠΎΠ΄Π½ΠΎΠΉ ΠΈΠ· ΠΎΠ±ΠΊΠ»Π°Π΄ΠΎΠΊ элСмСнта,
  • U – Ρ€Π°Π·Π½ΠΎΡΡ‚ΡŒ ΠΏΠΎΡ‚Π΅Π½Ρ†ΠΈΠ°Π»ΠΎΠ² ΠΌΠ΅ΠΆΠ΄Ρƒ ΠΎΠ±ΠΊΠ»Π°Π΄ΠΊΠ°ΠΌΠΈ.

Π’ элСктротСхникС вмСсто понятия Β«Ρ€Π°Π·Π½ΠΎΡΡ‚ΡŒ ΠΏΠΎΡ‚Π΅Π½Ρ†ΠΈΠ°Π»ΠΎΠ² ΠΌΠ΅ΠΆΠ΄Ρƒ ΠΎΠ±ΠΊΠ»Π°Π΄ΠΊΠ°ΠΌΠΈΒ» ΠΈΡΠΏΠΎΠ»ΡŒΠ·ΡƒΠ΅Ρ‚ΡΡ «напряТСниС Π½Π° кондСнсаторС».

ΠΠΌΠΊΠΎΡΡ‚ΡŒ элСмСнта Π½Π΅ зависит ΠΎΡ‚ конструкции ΠΈ Ρ€Π°Π·ΠΌΠ΅Ρ€ΠΎΠ² устройства, Π° Ρ‚ΠΎΠ»ΡŒΠΊΠΎ ΠΎΡ‚ напряТСния Π½Π° Π½Ρ‘ΠΌ ΠΈ заряда ΠΎΠ±ΠΊΠ»Π°Π΄ΠΎΠΊ. Но эти ΠΏΠ°Ρ€Π°ΠΌΠ΅Ρ‚Ρ€Ρ‹ ΠΌΠΎΠ³ΡƒΡ‚ ΠΈΠ·ΠΌΠ΅Π½ΡΡ‚ΡŒΡΡ Π² зависимости ΠΎΡ‚ расстояния ΠΌΠ΅ΠΆΠ΄Ρƒ Π½ΠΈΠΌΠΈ ΠΈ ΠΌΠ°Ρ‚Π΅Ρ€ΠΈΠ°Π»Π° диэлСктрика. Π­Ρ‚ΠΎ учитываСтся Π² Ρ„ΠΎΡ€ΠΌΡƒΠ»Π΅:

Π‘=Co*Ξ΅, Π³Π΄Π΅:

  • Π‘ – Ρ€Π΅Π°Π»ΡŒΠ½Π°Ρ Ρ‘ΠΌΠΊΠΎΡΡ‚ΡŒ,
  • Π‘ΠΎ – идСальная, ΠΏΡ€ΠΈ условии, Ρ‡Ρ‚ΠΎ ΠΌΠ΅ΠΆΠ΄Ρƒ пластинами Π²Π°ΠΊΡƒΡƒΠΌ ΠΈΠ»ΠΈ Π²ΠΎΠ·Π΄ΡƒΡ…,
  • Ξ΅ – диэлСктричСская ΠΏΡ€ΠΎΠ½ΠΈΡ†Π°Π΅ΠΌΠΎΡΡ‚ΡŒ ΠΌΠ°Ρ‚Π΅Ρ€ΠΈΠ°Π»Π° ΠΌΠ΅ΠΆΠ΄Ρƒ Π½ΠΈΠΌΠΈ.

НапримСр, Ссли Π² качСствС диэлСктрика ΠΈΡΠΏΠΎΠ»ΡŒΠ·ΡƒΠ΅Ρ‚ΡΡ слюда, «Ρ» ΠΊΠΎΡ‚ΠΎΡ€ΠΎΠΉ 6, Ρ‚ΠΎ Ρ‘ΠΌΠΊΠΎΡΡ‚ΡŒ Ρ‚Π°ΠΊΠΎΠ³ΠΎ устройства Π² 6 Ρ€Π°Π· большС, Ρ‡Π΅ΠΌ Π²ΠΎΠ·Π΄ΡƒΡˆΠ½ΠΎΠ³ΠΎ, Π° ΠΏΡ€ΠΈ ΠΈΠ·ΠΌΠ΅Π½Π΅Π½ΠΈΠΈ количСства диэлСктрика ΠΌΠ΅Π½ΡΡŽΡ‚ΡΡ ΠΏΠ°Ρ€Π°ΠΌΠ΅Ρ‚Ρ€Ρ‹ конструкции. На этом ΠΏΡ€ΠΈΠ½Ρ†ΠΈΠΏΠ΅ основана Ρ€Π°Π±ΠΎΡ‚Π° ёмкостного Π΄Π°Ρ‚Ρ‡ΠΈΠΊΠ° полоТСния.

Π•Π΄ΠΈΠ½ΠΈΡ†Π΅ΠΉ ёмкости Π² систСмС БИ являСтся 1 Ρ„Π°Ρ€Π°Π΄ (F). Π­Ρ‚ΠΎ большая Π²Π΅Π»ΠΈΡ‡ΠΈΠ½Π°, поэтому Ρ‡Π°Ρ‰Π΅ ΠΏΡ€ΠΈΠΌΠ΅Π½ΡΡŽΡ‚ΡΡ ΠΌΠΈΠΊΡ€ΠΎΡ„Π°Ρ€Π°Π΄Ρ‹ (1000000mkF=1F) ΠΈ ΠΏΠΈΠΊΠΎΡ„Π°Ρ€Π°Π΄Ρ‹ (1000000pF=1mkF).

РасчСт плоской конструкции

  • Ξ΅ – диэлСктричСская ΠΏΡ€ΠΎΠ½ΠΈΡ†Π°Π΅ΠΌΠΎΡΡ‚ΡŒ ΠΈΠ·ΠΎΠ»ΠΈΡ€ΡƒΡŽΡ‰Π΅Π³ΠΎ ΠΌΠ°Ρ‚Π΅Ρ€ΠΈΠ°Π»Π°,
  • d – расстояниС ΠΌΠ΅ΠΆΠ΄Ρƒ пластинами.

РасчСт конструкции цилиндричСской Ρ„ΠΎΡ€ΠΌΡ‹

ЦилиндричСский кондСнсатор – это Π΄Π²Π΅ соосныС Ρ‚Ρ€ΡƒΠ±ΠΊΠΈ Ρ€Π°Π·Π»ΠΈΡ‡Π½ΠΎΠ³ΠΎ Π΄ΠΈΠ°ΠΌΠ΅Ρ‚Ρ€Π°, вставлСнныС Π΄Ρ€ΡƒΠ³ Π² Π΄Ρ€ΡƒΠ³Π°. ΠœΠ΅ΠΆΠ΄Ρƒ Π½ΠΈΠΌΠΈ находится диэлСктрик. ΠŸΡ€ΠΈ радиусС Ρ†ΠΈΠ»ΠΈΠ½Π΄Ρ€ΠΎΠ², Π±Π»ΠΈΠ·ΠΊΠΎΠΌ Π΄Ρ€ΡƒΠ³ ΠΊ Π΄Ρ€ΡƒΠ³Ρƒ ΠΈ Π½Π°ΠΌΠ½ΠΎΠ³ΠΎ большСм, Ρ‡Π΅ΠΌ расстояниС ΠΌΠ΅ΠΆΠ΄Ρƒ Π½ΠΈΠΌΠΈ, цилиндричСской Ρ„ΠΎΡ€ΠΌΠΎΠΉ ΠΌΠΎΠΆΠ½ΠΎ ΠΏΡ€Π΅Π½Π΅Π±Ρ€Π΅Ρ‡ΡŒ ΠΈ свСсти расчёт ΠΊ Ρ„ΠΎΡ€ΠΌΡƒΠ»Π΅, Π°Π½Π°Π»ΠΎΠ³ΠΈΡ‡Π½ΠΎΠΉ Ρ‚ΠΎΠΉ, ΠΏΠΎ ΠΊΠΎΡ‚ΠΎΡ€ΠΎΠΉ рассчитываСтся плоский кондСнсатор.

Π’Ρ‹Ρ‡ΠΈΡΠ»ΡΡŽΡ‚ΡΡ ΠΏΠ°Ρ€Π°ΠΌΠ΅Ρ‚Ρ€Ρ‹ Ρ‚Π°ΠΊΠΎΠ³ΠΎ устройства ΠΏΠΎ Ρ„ΠΎΡ€ΠΌΡƒΠ»Π΅:

C=(2Ο€*l*R*Ξ΅)/d, Π³Π΄Π΅:

  • l – Π΄Π»ΠΈΠ½Π° устройства,
  • R – радиус Ρ†ΠΈΠ»ΠΈΠ½Π΄Ρ€Π°,
  • Ξ΅ – диэлСктричСская ΠΏΡ€ΠΎΠ½ΠΈΡ†Π°Π΅ΠΌΠΎΡΡ‚ΡŒ изолятора,
  • d – Π΅Π³ΠΎ Ρ‚ΠΎΠ»Ρ‰ΠΈΠ½Π°.

Расчёт сфСричСской конструкции

Π•ΡΡ‚ΡŒ устройства, ΠΎΠ±ΠΊΠ»Π°Π΄ΠΊΠΈ ΠΊΠΎΡ‚ΠΎΡ€Ρ‹Ρ… ΠΏΡ€Π΅Π΄ΡΡ‚Π°Π²Π»ΡΡŽΡ‚ собой Π΄Π²Π° ΡˆΠ°Ρ€Π°, Π²Π»ΠΎΠΆΠ΅Π½Π½Ρ‹Π΅ Π΄Ρ€ΡƒΠ³ Π² Π΄Ρ€ΡƒΠ³Π°. Π€ΠΎΡ€ΠΌΡƒΠ»Π° ёмкости Ρ‚Π°ΠΊΠΎΠ³ΠΎ ΠΏΡ€ΠΈΠ±ΠΎΡ€Π°:

C=(4Ο€*l*R1*R2*Ξ΅)/(R2-R1), Π³Π΄Π΅:

  • R1 – радиус Π²Π½ΡƒΡ‚Ρ€Π΅Π½Π½Π΅ΠΉ сфСры,
  • R2 – радиус внСшнСй сфСры,
  • Ξ΅ – диэлСктричСская ΠΏΡ€ΠΎΠ½ΠΈΡ†Π°Π΅ΠΌΠΎΡΡ‚ΡŒ.

ΠΠΌΠΊΠΎΡΡ‚ΡŒ ΠΎΠ΄ΠΈΠ½ΠΎΡ‡Π½ΠΎΠ³ΠΎ ΠΏΡ€ΠΎΠ²ΠΎΠ΄Π½ΠΈΠΊΠ°

ΠšΡ€ΠΎΠΌΠ΅ кондСнсаторов, ΡΠΏΠΎΡΠΎΠ±Π½ΠΎΡΡ‚ΡŒΡŽ Π½Π°ΠΊΠ°ΠΏΠ»ΠΈΠ²Π°Ρ‚ΡŒ заряд ΠΎΠ±Π»Π°Π΄Π°ΡŽΡ‚ ΠΎΡ‚Π΄Π΅Π»ΡŒΠ½Ρ‹Π΅ ΠΏΡ€ΠΎΠ²ΠΎΠ΄Π½ΠΈΠΊΠΈ. ΠžΠ΄ΠΈΠ½ΠΎΡ‡Π½Ρ‹ΠΌ ΠΏΡ€ΠΎΠ²ΠΎΠ΄Π½ΠΈΠΊΠΎΠΌ считаСтся Ρ‚Π°ΠΊΠΎΠΉ ΠΏΡ€ΠΎΠ²ΠΎΠ΄Π½ΠΈΠΊ, ΠΊΠΎΡ‚ΠΎΡ€Ρ‹ΠΉ бСсконСчно Π΄Π°Π»Ρ‘ΠΊ ΠΎΡ‚ Π΄Ρ€ΡƒΠ³ΠΈΡ… ΠΏΡ€ΠΎΠ²ΠΎΠ΄Π½ΠΈΠΊΠΎΠ². ΠŸΠ°Ρ€Π°ΠΌΠ΅Ρ‚Ρ€Ρ‹ заряТСнного элСмСнта рассчитываСтся ΠΏΠΎ Ρ„ΠΎΡ€ΠΌΡƒΠ»Π΅:

  • Q – заряд,
  • Ο† – ΠΏΠΎΡ‚Π΅Π½Ρ†ΠΈΠ°Π» ΠΏΡ€ΠΎΠ²ΠΎΠ΄Π½ΠΈΠΊΠ°.

ΠžΠ±ΡŠΡ‘ΠΌ заряда опрСдСляСтся Ρ€Π°Π·ΠΌΠ΅Ρ€ΠΎΠΌ ΠΈ Ρ„ΠΎΡ€ΠΌΠΎΠΉ устройства, Π° Ρ‚Π°ΠΊΠΆΠ΅ ΠΎΠΊΡ€ΡƒΠΆΠ°ΡŽΡ‰Π΅ΠΉ срСдой. ΠœΠ°Ρ‚Π΅Ρ€ΠΈΠ°Π» ΠΏΡ€ΠΈΠ±ΠΎΡ€Π° значСния Π½Π΅ ΠΈΠΌΠ΅Π΅Ρ‚.

Бпособы соСдинСния элСмСнтов

НС всСгда Π΅ΡΡ‚ΡŒ Π² Π½Π°Π»ΠΈΡ‡ΠΈΠΈ элСмСнты с Π½Π΅ΠΎΠ±Ρ…ΠΎΠ΄ΠΈΠΌΡ‹ΠΌΠΈ ΠΏΠ°Ρ€Π°ΠΌΠ΅Ρ‚Ρ€Π°ΠΌΠΈ. ΠŸΡ€ΠΈΡ…ΠΎΠ΄ΠΈΡ‚ΡΡ ΡΠΎΠ΅Π΄ΠΈΠ½ΡΡ‚ΡŒ ΠΈΡ… Ρ€Π°Π·Π»ΠΈΡ‡Π½Ρ‹ΠΌΠΈ способами.

ΠŸΠ°Ρ€Π°Π»Π»Π΅Π»ΡŒΠ½ΠΎΠ΅ соСдинСниС

Π­Ρ‚ΠΎ Ρ‚Π°ΠΊΠΎΠ΅ соСдинСниС Π΄Π΅Ρ‚Π°Π»Π΅ΠΉ, ΠΏΡ€ΠΈ ΠΊΠΎΡ‚ΠΎΡ€ΠΎΠΌ ΠΊ ΠΎΠ΄Π½ΠΎΠΉ ΠΊΠ»Π΅ΠΌΠΌΠ΅ ΠΈΠ»ΠΈ ΠΊΠΎΠ½Ρ‚Π°ΠΊΡ‚Ρƒ ΠΏΡ€ΠΈΡΠΎΠ΅Π΄ΠΈΠ½ΡΡŽΡ‚ΡΡ ΠΏΠ΅Ρ€Π²Ρ‹Π΅ ΠΎΠ±ΠΊΠ»Π°Π΄ΠΊΠΈ ΠΊΠ°ΠΆΠ΄ΠΎΠ³ΠΎ кондСнсатора. ΠŸΡ€ΠΈ этом Π²Ρ‚ΠΎΡ€Ρ‹Π΅ ΠΎΠ±ΠΊΠ»Π°Π΄ΠΊΠΈ ΠΏΡ€ΠΈΡΠΎΠ΅Π΄ΠΈΠ½ΡΡŽΡ‚ΡΡ ΠΊ Π΄Ρ€ΡƒΠ³ΠΎΠΉ ΠΊΠ»Π΅ΠΌΠΌΠ΅.

ΠŸΡ€ΠΈ Ρ‚Π°ΠΊΠΎΠΌ соСдинСнии напряТСниС Π½Π° ΠΊΠΎΠ½Ρ‚Π°ΠΊΡ‚Π°Ρ… всСх элСмСнтов Π±ΡƒΠ΄Π΅Ρ‚ ΠΎΠ΄ΠΈΠ½Π°ΠΊΠΎΠ²Ρ‹ΠΌ. Заряд ΠΊΠ°ΠΆΠ΄ΠΎΠ³ΠΎ ΠΈΠ· Π½ΠΈΡ… происходит нСзависимо ΠΎΡ‚ ΠΎΡΡ‚Π°Π»ΡŒΠ½Ρ‹Ρ…, поэтому общая Ρ‘ΠΌΠΊΠΎΡΡ‚ΡŒ Ρ€Π°Π²Π½Π° суммС всСх Π²Π΅Π»ΠΈΡ‡ΠΈΠ½. Π•Ρ‘ находят ΠΏΠΎ Ρ„ΠΎΡ€ΠΌΡƒΠ»Π΅:

Π³Π΄Π΅ C1-Cn – ΠΏΠ°Ρ€Π°ΠΌΠ΅Ρ‚Ρ€Ρ‹ Π΄Π΅Ρ‚Π°Π»Π΅ΠΉ, ΡƒΡ‡Π°ΡΡ‚Π²ΡƒΡŽΡ‰ΠΈΡ… Π² ΠΏΠ°Ρ€Π°Π»Π»Π΅Π»ΡŒΠ½ΠΎΠΌ соСдинСнии.

Π’Π°ΠΆΠ½ΠΎ!
ΠšΠΎΠ½Π΄Π΅Π½ΡΠ°Ρ‚ΠΎΡ€Ρ‹ ΠΈΠΌΠ΅ΡŽΡ‚ ΠΏΡ€Π΅Π΄Π΅Π»ΡŒΠ½ΠΎΠ΅ допустимоС напряТСниС, ΠΏΡ€Π΅Π²Ρ‹ΡˆΠ΅Π½ΠΈΠ΅ ΠΊΠΎΡ‚ΠΎΡ€ΠΎΠ³ΠΎ ΠΏΡ€ΠΈΠ²Π΅Π΄Ρ‘Ρ‚ ΠΊ Π²Ρ‹Ρ…ΠΎΠ΄Ρƒ элСмСнта ΠΈΠ· строя. ΠŸΡ€ΠΈ ΠΏΠ°Ρ€Π°Π»Π»Π΅Π»ΡŒΠ½ΠΎΠΌ соСдинСнии устройств с Ρ€Π°Π·Π»ΠΈΡ‡Π½Ρ‹ΠΌ допустимым напряТСниСм этот ΠΏΠ°Ρ€Π°ΠΌΠ΅Ρ‚Ρ€ ΠΏΠΎΠ»ΡƒΡ‡ΠΈΠ²ΡˆΠ΅ΠΉΡΡ сборки Ρ€Π°Π²Π΅Π½ элСмСнту с наимСньшим Π·Π½Π°Ρ‡Π΅Π½ΠΈΠ΅ΠΌ.

ΠŸΠΎΡΠ»Π΅Π΄ΠΎΠ²Π°Ρ‚Π΅Π»ΡŒΠ½ΠΎΠ΅ соСдинСниС

Π­Ρ‚ΠΎ Ρ‚Π°ΠΊΠΎΠ΅ соСдинСниС, ΠΏΡ€ΠΈ ΠΊΠΎΡ‚ΠΎΡ€ΠΎΠΌ ΠΊ ΠΊΠ»Π΅ΠΌΠΌΠ΅ присоСдиняСтся Ρ‚ΠΎΠ»ΡŒΠΊΠΎ ΠΎΠ΄Π½Π° пластина ΠΏΠ΅Ρ€Π²ΠΎΠ³ΠΎ элСмСнта. Вторая пластина присоСдиняСтся ΠΊ ΠΏΠ΅Ρ€Π²ΠΎΠΉ пластинС Π²Ρ‚ΠΎΡ€ΠΎΠ³ΠΎ элСмСнта, вторая пластина Π²Ρ‚ΠΎΡ€ΠΎΠ³ΠΎ – ΠΊ ΠΏΠ΅Ρ€Π²ΠΎΠΉ пластинС Ρ‚Ρ€Π΅Ρ‚ΡŒΠ΅Π³ΠΎ ΠΈ Ρ‚Π°ΠΊ Π΄Π°Π»Π΅Π΅. Ко Π²Ρ‚ΠΎΡ€ΠΎΠΉ ΠΊΠ»Π΅ΠΌΠΌΠ΅ присоСдиняСтся Ρ‚ΠΎΠ»ΡŒΠΊΠΎ вторая ΠΎΠ±ΠΊΠ»Π°Π΄ΠΊΠ° послСднСго элСмСнта.

ΠŸΡ€ΠΈ Ρ‚Π°ΠΊΠΎΠΌ соСдинСнии заряд Π½Π° ΠΎΠ±ΠΊΠ»Π°Π΄ΠΊΠ°Ρ… кондСнсатора Π² ΠΊΠ°ΠΆΠ΄ΠΎΠΌ ΠΏΡ€ΠΈΠ±ΠΎΡ€Π΅ Π±ΡƒΠ΄Π΅Ρ‚ Ρ€Π°Π²Π΅Π½ ΠΎΡΡ‚Π°Π»ΡŒΠ½Ρ‹ΠΌ, ΠΎΠ΄Π½Π°ΠΊΠΎ напряТСниС Π½Π° Π½ΠΈΡ… Π±ΡƒΠ΄Π΅Ρ‚ Ρ€Π°Π·Π½Ρ‹ΠΌ: для зарядки устройств большСй ёмкости Ρ‚Π΅ΠΌ ΠΆΠ΅ зарядом трСбуСтся мСньшая Ρ€Π°Π·Π½ΠΎΡΡ‚ΡŒ ΠΏΠΎΡ‚Π΅Π½Ρ†ΠΈΠ°Π»ΠΎΠ². ΠŸΠΎΡΡ‚ΠΎΠΌΡƒ вся Ρ†Π΅ΠΏΠΎΡ‡ΠΊΠ° прСдставляСт собой ΠΎΠ΄Π½Ρƒ ΠΊΠΎΠ½ΡΡ‚Ρ€ΡƒΠΊΡ†ΠΈΡŽ, Ρ€Π°Π·Π½ΠΎΡΡ‚ΡŒ ΠΏΠΎΡ‚Π΅Π½Ρ†ΠΈΠ°Π»ΠΎΠ² ΠΊΠΎΡ‚ΠΎΡ€ΠΎΠΉ Ρ€Π°Π²Π½Π° суммС напряТСний Π½Π° всСх элСмСнтах, Π° заряд кондСнсатора Ρ€Π°Π²Π΅Π½ суммС зарядов.

ΠŸΠΎΡΠ»Π΅Π΄ΠΎΠ²Π°Ρ‚Π΅Π»ΡŒΠ½ΠΎΠ΅ соСдинСниС ΡƒΠ²Π΅Π»ΠΈΡ‡ΠΈΠ²Π°Π΅Ρ‚ допустимоС напряТСниС ΠΈ ΡƒΠΌΠ΅Π½ΡŒΡˆΠ°Π΅Ρ‚ ΠΎΠ±Ρ‰ΡƒΡŽ Ρ‘ΠΌΠΊΠΎΡΡ‚ΡŒ, которая мСньшС самого мСньшСго элСмСнта.

Π Π°ΡΡΡ‡ΠΈΡ‚Ρ‹Π²Π°ΡŽΡ‚ΡΡ эти ΠΏΠ°Ρ€Π°ΠΌΠ΅Ρ‚Ρ€Ρ‹ ΡΠ»Π΅Π΄ΡƒΡŽΡ‰ΠΈΠΌ ΠΎΠ±Ρ€Π°Π·ΠΎΠΌ:

  • ДопустимоС напряТСниС:

UΠΎΠ±Ρ‰=U1+U2+U3+…Un, Π³Π΄Π΅ U1-Un – напряТСниС Π½Π° кондСнсаторС;

  • ΠžΠ±Ρ‰Π°Ρ Ρ‘ΠΌΠΊΠΎΡΡ‚ΡŒ:

1/Π‘ΠΎΠ±Ρ‰=1/Π‘1+1/Π‘2+1/Π‘3+…1/Π‘n, Π³Π΄Π΅ Π‘1-Π‘n – ΠΏΠ°Ρ€Π°ΠΌΠ΅Ρ‚Ρ€Ρ‹ ΠΊΠ°ΠΆΠ΄ΠΎΠ³ΠΎ устройства.

Π˜Π½Ρ‚Π΅Ρ€Π΅ΡΠ½ΠΎ.
Если Π² Ρ†Π΅ΠΏΠΈ Ρ‚ΠΎΠ»ΡŒΠΊΠΎ Π΄Π²Π° элСмСнта, Ρ‚ΠΎ ΠΌΠΎΠΆΠ½ΠΎ Π²ΠΎΡΠΏΠΎΠ»ΡŒΠ·ΠΎΠ²Π°Ρ‚ΡŒΡΡ ΡƒΠΏΡ€ΠΎΡ‰Ρ‘Π½Π½ΠΎΠΉ Ρ„ΠΎΡ€ΠΌΡƒΠ»ΠΎΠΉ: Π‘ΠΎΠ±Ρ‰=(Π‘1*Π‘2)/(Π‘1+Π‘2).

БмСшанноС соСдинСниС

Π­Ρ‚ΠΎ Ρ‚Π°ΠΊΠΎΠ΅ соСдинСниС, Π² ΠΊΠΎΡ‚ΠΎΡ€ΠΎΠΌ Π΅ΡΡ‚ΡŒ Π΄Π΅Ρ‚Π°Π»ΠΈ, соСдинённыС ΠΏΠΎΡΠ»Π΅Π΄ΠΎΠ²Π°Ρ‚Π΅Π»ΡŒΠ½ΠΎ, ΠΈ Π΅ΡΡ‚ΡŒ соСдинённыС ΠΏΠ°Ρ€Π°Π»Π»Π΅Π»ΡŒΠ½ΠΎ. ΠŸΠ°Ρ€Π°ΠΌΠ΅Ρ‚Ρ€Ρ‹ всСй Ρ†Π΅ΠΏΠΈ рассчитываСтся Π² ΡΠ»Π΅Π΄ΡƒΡŽΡ‰Π΅ΠΉ ΠΏΠΎΡΠ»Π΅Π΄ΠΎΠ²Π°Ρ‚Π΅Π»ΡŒΠ½ΠΎΡΡ‚ΠΈ:

  1. ΠΎΠΏΡ€Π΅Π΄Π΅Π»ΡΡŽΡ‚ΡΡ Π³Ρ€ΡƒΠΏΠΏΡ‹ элСмСнтов, соСдинённыС ΠΏΠ°Ρ€Π°Π»Π»Π΅Π»ΡŒΠ½ΠΎ;
  2. для ΠΊΠ°ΠΆΠ΄ΠΎΠΉ Π³Ρ€ΡƒΠΏΠΏΡ‹ Π² ΠΎΡ‚Π΄Π΅Π»ΡŒΠ½ΠΎΡΡ‚ΠΈ рассчитываСтся эквивалСнтныС значСния;
  3. рядом с ΠΊΠ°ΠΆΠ΄ΠΎΠΉ Π³Ρ€ΡƒΠΏΠΏΠΎΠΉ ΠΏΠ°Ρ€Π°Π»Π»Π΅Π»ΡŒΠ½ΠΎ соСдинённых Π΄Π΅Ρ‚Π°Π»Π΅ΠΉ ΠΏΠΈΡˆΡƒΡ‚ΡΡ ΠΏΠΎΠ»ΡƒΡ‡ΠΈΠ²ΡˆΠΈΠ΅ΡΡ Π²Π΅Π»ΠΈΡ‡ΠΈΠ½Ρ‹;
  4. ΠΏΠΎΠ»ΡƒΡ‡ΠΈΠ²ΡˆΠ°ΡΡΡ схСма эквивалСнтна ΠΏΠΎΡΠ»Π΅Π΄ΠΎΠ²Π°Ρ‚Π΅Π»ΡŒΠ½ΠΎΠΉ схСмС ΠΈ рассчитываСтся ΠΏΠΎ ΡΠΎΠΎΡ‚Π²Π΅Ρ‚ΡΡ‚Π²ΡƒΡŽΡ‰ΠΈΠΌ Ρ„ΠΎΡ€ΠΌΡƒΠ»Π°ΠΌ.

Π—Π½Π°Π½ΠΈΠ΅ Ρ„ΠΎΡ€ΠΌΡƒΠ», ΠΏΠΎ ΠΊΠΎΡ‚ΠΎΡ€Ρ‹ΠΌ ΠΌΠΎΠΆΠ½ΠΎ Π½Π°ΠΉΡ‚ΠΈ Π΅ΠΌΠΊΠΎΡΡ‚ΡŒ ΠΏΡ€ΠΈ ΠΈΠ·Π³ΠΎΡ‚ΠΎΠ²Π»Π΅Π½ΠΈΠΈ кондСнсаторов ΠΈΠ»ΠΈ ΠΈΡ… соСдинСнии Π½Π΅ΠΎΠ±Ρ…ΠΎΠ΄ΠΈΠΌΠΎ ΠΏΡ€ΠΈ конструировании элСктронных схСм.

Π’ΠΈΠ΄Π΅ΠΎ

Π‘ΠΎΠ΄Π΅Ρ€ΠΆΠ°Π½ΠΈΠ΅:


Одним ΠΈΠ· Π²Π°ΠΆΠ½Ρ‹Ρ… элСмСнтов элСктричСской Ρ†Π΅ΠΏΠΈ являСтся кондСнсатор, Ρ„ΠΎΡ€ΠΌΡƒΠ»Ρ‹ для ΠΊΠΎΡ‚ΠΎΡ€ΠΎΠ³ΠΎ ΠΏΠΎΠ·Π²ΠΎΠ»ΡΡŽΡ‚ Ρ€Π°ΡΡΡ‡ΠΈΡ‚Π°Ρ‚ΡŒ ΠΈ ΠΏΠΎΠ΄ΠΎΠ±Ρ€Π°Ρ‚ΡŒ Π½Π°ΠΈΠ±ΠΎΠ»Π΅Π΅ подходящий Π²Π°Ρ€ΠΈΠ°Π½Ρ‚. Основная функция Π΄Π°Π½Π½ΠΎΠ³ΠΎ устройства Π·Π°ΠΊΠ»ΡŽΡ‡Π°Π΅Ρ‚ΡΡ Π² Π½Π°ΠΊΠΎΠΏΠ»Π΅Π½ΠΈΠΈ ΠΎΠΏΡ€Π΅Π΄Π΅Π»Π΅Π½Π½ΠΎΠ³ΠΎ количСства элСктроэнСргии. ΠŸΡ€ΠΎΡΡ‚Π΅ΠΉΡˆΠ°Ρ систСма Π²ΠΊΠ»ΡŽΡ‡Π°Π΅Ρ‚ Π² сСбя Π΄Π²Π° элСктрода ΠΈΠ»ΠΈ ΠΎΠ±ΠΊΠ»Π°Π΄ΠΊΠΈ, Ρ€Π°Π·Π΄Π΅Π»Π΅Π½Π½Ρ‹Π΅ ΠΌΠ΅ΠΆΠ΄Ρƒ собой диэлСктриком.

Π’ Ρ‡Π΅ΠΌ измСряСтся Π΅ΠΌΠΊΠΎΡΡ‚ΡŒ кондСнсатора

Одной ΠΈΠ· Π²Π°ΠΆΠ½Π΅ΠΉΡˆΠΈΡ… характСристик кондСнсатора являСтся Π΅Π³ΠΎ Π΅ΠΌΠΊΠΎΡΡ‚ΡŒ. Π”Π°Π½Π½Ρ‹ΠΉ ΠΏΠ°Ρ€Π°ΠΌΠ΅Ρ‚Ρ€ опрСдСляСтся количСством элСктроэнСргии, Π½Π°ΠΊΠ°ΠΏΠ»ΠΈΠ²Π°Π΅ΠΌΠΎΠΉ этим ΠΏΡ€ΠΈΠ±ΠΎΡ€ΠΎΠΌ. НакоплСниС происходит Π² Π²ΠΈΠ΄Π΅ элСктронов. Π˜Ρ… количСство, ΠΏΠΎΠΌΠ΅Ρ‰Π°ΡŽΡ‰Π΅Π΅ΡΡ Π² кондСнсаторС, опрСдСляСт Π²Π΅Π»ΠΈΡ‡ΠΈΠ½Ρƒ Смкости ΠΊΠΎΠ½ΠΊΡ€Π΅Ρ‚Π½ΠΎΠ³ΠΎ устройства.

Для измСрСния Смкости примСняСтся Π΅Π΄ΠΈΠ½ΠΈΡ†Π° — Ρ„Π°Ρ€Π°Π΄Π°. Π•ΠΌΠΊΠΎΡΡ‚ΡŒ кондСнсатора Π² 1 Ρ„Π°Ρ€Π°Π΄Ρƒ соотвСтствуСт элСктричСскому заряду Π² 1 ΠΊΡƒΠ»ΠΎΠ½, Π° Π½Π° ΠΎΠ±ΠΊΠ»Π°Π΄ΠΊΠ°Ρ… Ρ€Π°Π·Π½ΠΎΡΡ‚ΡŒ ΠΏΠΎΡ‚Π΅Π½Ρ†ΠΈΠ°Π»ΠΎΠ² Ρ€Π°Π²Π½Π° 1 Π²ΠΎΠ»ΡŒΡ‚Ρƒ. Π­Ρ‚Π° классичСская Ρ„ΠΎΡ€ΠΌΡƒΠ»ΠΈΡ€ΠΎΠ²ΠΊΠ° Π½Π΅ ΠΏΠΎΠ΄Ρ…ΠΎΠ΄ΠΈΡ‚ для практичСских расчСтов, ΠΏΠΎΡΠΊΠΎΠ»ΡŒΠΊΡƒ Π² кондСнсаторС ΡΠΎΠ±ΠΈΡ€Π°ΡŽΡ‚ΡΡ Π½Π΅ заряды, Π° элСктроны. Π•ΠΌΠΊΠΎΡΡ‚ΡŒ любого кондСнсатора находится Π² прямой зависимости ΠΎΡ‚ объСма элСктронов, способных Π½Π°ΠΊΠ°ΠΏΠ»ΠΈΠ²Π°Ρ‚ΡŒΡΡ ΠΏΡ€ΠΈ Π½ΠΎΡ€ΠΌΠ°Π»ΡŒΠ½ΠΎΠΌ Ρ€Π°Π±ΠΎΡ‡Π΅ΠΌ Ρ€Π΅ΠΆΠΈΠΌΠ΅. Для обозначСния Смкости всС Ρ€Π°Π²Π½ΠΎ ΠΈΡΠΏΠΎΠ»ΡŒΠ·ΡƒΠ΅Ρ‚ΡΡ Ρ„Π°Ρ€Π°Π΄Π°, Π° количСствСнныС ΠΏΠ°Ρ€Π°ΠΌΠ΅Ρ‚Ρ€Ρ‹ ΠΎΠΏΡ€Π΅Π΄Π΅Π»ΡΡŽΡ‚ΡΡ ΠΏΠΎ Ρ„ΠΎΡ€ΠΌΡƒΠ»Π΅: Π‘ = Q / U, Π³Π΄Π΅ Π‘ ΠΎΠ·Π½Π°Ρ‡Π°Π΅Ρ‚ Π΅ΠΌΠΊΠΎΡΡ‚ΡŒ, Q — заряд Π² ΠΊΡƒΠ»ΠΎΠ½Π°Ρ…, Π° U являСтся напряТСниСм. Π’Π°ΠΊΠΈΠΌ ΠΎΠ±Ρ€Π°Π·ΠΎΠΌ, просматриваСтся взаимная связь заряда ΠΈ напряТСния, ΠΎΠΊΠ°Π·Ρ‹Π²Π°ΡŽΡ‰ΠΈΡ… влияниС Π½Π° ΡΠΏΠΎΡΠΎΠ±Π½ΠΎΡΡ‚ΡŒ кондСнсатора ΠΊ накоплСнию ΠΈ ΡƒΠ΄Π΅Ρ€ΠΆΠ°Π½ΠΈΡŽ ΠΎΠΏΡ€Π΅Π΄Π΅Π»Π΅Π½Π½ΠΎΠ³ΠΎ количСства элСктричСства.

Для расчСтов ΠΈΡΠΏΠΎΠ»ΡŒΠ·ΡƒΠ΅Ρ‚ΡΡ Ρ„ΠΎΡ€ΠΌΡƒΠ»Π°:
Π² ΠΊΠΎΡ‚ΠΎΡ€ΠΎΠΉ Ξ΅ 0
= 8,854187817 Ρ… 10 -12 Ρ„/ΠΌ прСдставляСт собой ΠΏΠΎΡΡ‚ΠΎΡΠ½Π½ΡƒΡŽ Π²Π΅Π»ΠΈΡ‡ΠΈΠ½Ρƒ. ΠŸΡ€ΠΎΡ‡ΠΈΠ΅ Π²Π΅Π»ΠΈΡ‡ΠΈΠ½Ρ‹: Ξ΅ — являСтся диэлСктричСской ΠΏΡ€ΠΎΠ½ΠΈΡ†Π°Π΅ΠΌΠΎΡΡ‚ΡŒΡŽ диэлСктрика, находящСгося ΠΌΠ΅ΠΆΠ΄Ρƒ ΠΎΠ±ΠΊΠ»Π°Π΄ΠΊΠ°ΠΌΠΈ, S — ΠΎΠ·Π½Π°Ρ‡Π°Π΅Ρ‚ ΠΏΠ»ΠΎΡ‰Π°Π΄ΡŒ ΠΎΠ±ΠΊΠ»Π°Π΄ΠΊΠΈ, Π° d — Π·Π°Π·ΠΎΡ€ ΠΌΠ΅ΠΆΠ΄Ρƒ ΠΎΠ±ΠΊΠ»Π°Π΄ΠΊΠ°ΠΌΠΈ.

Π€ΠΎΡ€ΠΌΡƒΠ»Π° энСргии кондСнсатора

Π‘ Π΅ΠΌΠΊΠΎΡΡ‚ΡŒΡŽ самым тСсным ΠΎΠ±Ρ€Π°Π·ΠΎΠΌ связана другая Π²Π΅Π»ΠΈΡ‡ΠΈΠ½Π°, извСстная ΠΊΠ°ΠΊ . ПослС зарядки любого кондСнсатора, Π² Π½Π΅ΠΌ образуСтся ΠΎΠΏΡ€Π΅Π΄Π΅Π»Π΅Π½Π½ΠΎΠ΅ количСство энСргии, ΠΊΠΎΡ‚ΠΎΡ€ΠΎΠ΅ Π² дальнСйшСм выдСляСтся Π² процСссС разрядки. Π‘ этой ΠΏΠΎΡ‚Π΅Π½Ρ†ΠΈΠ°Π»ΡŒΠ½ΠΎΠΉ энСргиСй Π²ΡΡ‚ΡƒΠΏΠ°ΡŽΡ‚ Π²ΠΎ взаимодСйствиС ΠΎΠ±ΠΊΠ»Π°Π΄ΠΊΠΈ кондСнсатора. Π’ Π½ΠΈΡ… ΠΎΠ±Ρ€Π°Π·ΡƒΡŽΡ‚ΡΡ Ρ€Π°Π·Π½ΠΎΠΈΠΌΠ΅Π½Π½Ρ‹Π΅ заряды, ΠΏΡ€ΠΈΡ‚ΡΠ³ΠΈΠ²Π°ΡŽΡ‰ΠΈΠ΅ΡΡ Π΄Ρ€ΡƒΠ³ ΠΊ Π΄Ρ€ΡƒΠ³Ρƒ.

Π’ процСссС зарядки происходит расходованиС энСргии внСшнСго источника для раздСлСния зарядов с ΠΏΠΎΠ»ΠΎΠΆΠΈΡ‚Π΅Π»ΡŒΠ½Ρ‹ΠΌ ΠΈ ΠΎΡ‚Ρ€ΠΈΡ†Π°Ρ‚Π΅Π»ΡŒΠ½Ρ‹ΠΌ Π·Π½Π°Ρ‡Π΅Π½ΠΈΠ΅ΠΌ, ΠΊΠΎΡ‚ΠΎΡ€Ρ‹Π΅, Π·Π°Ρ‚Π΅ΠΌ Ρ€Π°ΡΠΏΠΎΠ»Π°Π³Π°ΡŽΡ‚ΡΡ Π½Π° ΠΎΠ±ΠΊΠ»Π°Π΄ΠΊΠ°Ρ… кондСнсатора. ΠŸΠΎΡΡ‚ΠΎΠΌΡƒ Π² соотвСтствии с Π·Π°ΠΊΠΎΠ½ΠΎΠΌ сохранСния энСргии, ΠΎΠ½Π° Π½Π΅ исчСзаСт бСсслСдно, Π° остаСтся Π²Π½ΡƒΡ‚Ρ€ΠΈ кондСнсатора Π² Π²ΠΈΠ΄Π΅ элСктричСского поля, сосрСдоточСнного ΠΌΠ΅ΠΆΠ΄Ρƒ пластинами. Π Π°Π·Π½ΠΎΠΈΠΌΠ΅Π½Π½Ρ‹Π΅ заряды ΠΎΠ±Ρ€Π°Π·ΡƒΡŽΡ‚ взаимодСйствиС ΠΈ ΠΏΠΎΡΠ»Π΅Π΄ΡƒΡŽΡ‰Π΅Π΅ притяТСниС ΠΎΠ±ΠΊΠ»Π°Π΄ΠΎΠΊ ΠΌΠ΅ΠΆΠ΄Ρƒ собой.

КаТдая пластина кондСнсатора ΠΏΠΎΠ΄ дСйствиСм заряда создаСт Π½Π°ΠΏΡ€ΡΠΆΠ΅Π½Π½ΠΎΡΡ‚ΡŒ элСктричСского поля, Ρ€Π°Π²Π½ΡƒΡŽ Π•/2. ΠžΠ±Ρ‰Π΅Π΅ ΠΏΠΎΠ»Π΅ Π±ΡƒΠ΄Π΅Ρ‚ ΡΠΊΠ»Π°Π΄Ρ‹Π²Π°Ρ‚ΡŒΡΡ ΠΈΠ· ΠΎΠ±ΠΎΠΈΡ… ΠΏΠΎΠ»Π΅ΠΉ, Π²ΠΎΠ·Π½ΠΈΠΊΠ°ΡŽΡ‰ΠΈΡ… Π² ΠΊΠ°ΠΆΠ΄ΠΎΠΉ ΠΎΠ±ΠΊΠ»Π°Π΄ΠΊΠ΅ с ΠΎΠ΄ΠΈΠ½Π°ΠΊΠΎΠ²Ρ‹ΠΌΠΈ зарядами, ΠΈΠΌΠ΅ΡŽΡ‰ΠΈΠΌΠΈ ΠΏΡ€ΠΎΡ‚ΠΈΠ²ΠΎΠΏΠΎΠ»ΠΎΠΆΠ½Ρ‹Π΅ значСния.

Π’Π°ΠΊΠΈΠΌ ΠΎΠ±Ρ€Π°Π·ΠΎΠΌ, энСргия кондСнсатора выраТаСтся Ρ„ΠΎΡ€ΠΌΡƒΠ»ΠΎΠΉ: W=q(E/2)d. Π’ свою ΠΎΡ‡Π΅Ρ€Π΅Π΄ΡŒ, напряТСниС выраТаСтся с ΠΏΠΎΠΌΠΎΡ‰ΡŒΡŽ понятий напряТСнности ΠΈ расстояния ΠΈ прСдставляСтся Π² Π²ΠΈΠ΄Π΅ Ρ„ΠΎΡ€ΠΌΡƒΠ»Ρ‹ U=Ed. Π­Ρ‚ΠΎ Π·Π½Π°Ρ‡Π΅Π½ΠΈΠ΅, подставлСнноС Π² ΠΏΠ΅Ρ€Π²ΡƒΡŽ Ρ„ΠΎΡ€ΠΌΡƒΠ»Ρƒ, ΠΎΡ‚ΠΎΠ±Ρ€Π°ΠΆΠ°Π΅Ρ‚ ΡΠ½Π΅Ρ€Π³ΠΈΡŽ кондСнсатора Π² Ρ‚Π°ΠΊΠΎΠΌ Π²ΠΈΠ΄Π΅:W=qU/2. Для получСния ΠΎΠΊΠΎΠ½Ρ‡Π°Ρ‚Π΅Π»ΡŒΠ½ΠΎΠ³ΠΎ Ρ€Π΅Π·ΡƒΠ»ΡŒΡ‚Π°Ρ‚Π° Π½Π΅ΠΎΠ±Ρ…ΠΎΠ΄ΠΈΠΌΠΎ ΠΈΡΠΏΠΎΠ»ΡŒΠ·ΠΎΠ²Π°Ρ‚ΡŒ ΠΎΠΏΡ€Π΅Π΄Π΅Π»Π΅Π½ΠΈΠ΅ Смкости: C=q/U, ΠΈ Π² ΠΊΠΎΠ½Ρ†Π΅ ΠΊΠΎΠ½Ρ†ΠΎΠ² энСргия заряТСнного кондСнсатора Π±ΡƒΠ΄Π΅Ρ‚ Π²Ρ‹Π³Π»ΡΠ΄Π΅Ρ‚ΡŒ ΡΠ»Π΅Π΄ΡƒΡŽΡ‰ΠΈΠΌ ΠΎΠ±Ρ€Π°Π·ΠΎΠΌ: W эл = CU 2 /2.

Π€ΠΎΡ€ΠΌΡƒΠ»Π° заряда кондСнсатора

Для выполнСния зарядки, кондСнсатор Π΄ΠΎΠ»ΠΆΠ΅Π½ Π±Ρ‹Ρ‚ΡŒ ΠΏΠΎΠ΄ΠΊΠ»ΡŽΡ‡Π΅Π½ ΠΊ Ρ†Π΅ΠΏΠΈ постоянного Ρ‚ΠΎΠΊΠ°. Π‘ этой Ρ†Π΅Π»ΡŒΡŽ ΠΌΠΎΠΆΠ΅Ρ‚ ΠΈΡΠΏΠΎΠ»ΡŒΠ·ΠΎΠ²Π°Ρ‚ΡŒΡΡ Π³Π΅Π½Π΅Ρ€Π°Ρ‚ΠΎΡ€. Π£ ΠΊΠ°ΠΆΠ΄ΠΎΠ³ΠΎ Π³Π΅Π½Π΅Ρ€Π°Ρ‚ΠΎΡ€Π° имССтся Π²Π½ΡƒΡ‚Ρ€Π΅Π½Π½Π΅Π΅ сопротивлСниС. ΠŸΡ€ΠΈ Π·Π°ΠΌΡ‹ΠΊΠ°Π½ΠΈΠΈ Ρ†Π΅ΠΏΠΈ происходит зарядка кондСнсатора. ΠœΠ΅ΠΆΠ΄Ρƒ Π΅Π³ΠΎ ΠΎΠ±ΠΊΠ»Π°Π΄ΠΊΠ°ΠΌΠΈ появляСтся напряТСниС, Ρ€Π°Π²Π½ΠΎΠ΅ элСктродвиТущСй силС Π³Π΅Π½Π΅Ρ€Π°Ρ‚ΠΎΡ€Π°: U c = E.

Обкладка, ΠΏΠΎΠ΄ΠΊΠ»ΡŽΡ‡Π΅Π½Π½Π°Ρ ΠΊ ΠΏΠΎΠ»ΠΎΠΆΠΈΡ‚Π΅Π»ΡŒΠ½ΠΎΠΌΡƒ ΠΏΠΎΠ»ΡŽΡΡƒ Π³Π΅Π½Π΅Ρ€Π°Ρ‚ΠΎΡ€Π°, заряТаСтся ΠΏΠΎΠ»ΠΎΠΆΠΈΡ‚Π΅Π»ΡŒΠ½ΠΎ (+q), Π° другая ΠΎΠ±ΠΊΠ»Π°Π΄ΠΊΠ° ΠΏΠΎΠ»ΡƒΡ‡Π°Π΅Ρ‚ Ρ€Π°Π²Π½ΠΎΠ·Π½Π°Ρ‡Π½Ρ‹ΠΉ заряд с ΠΎΡ‚Ρ€ΠΈΡ†Π°Ρ‚Π΅Π»ΡŒΠ½ΠΎΠΉ Π²Π΅Π»ΠΈΡ‡ΠΈΠ½ΠΎΠΉ (- q). Π’Π΅Π»ΠΈΡ‡ΠΈΠ½Π° заряда q находится Π² прямой ΠΏΡ€ΠΎΠΏΠΎΡ€Ρ†ΠΈΠΎΠ½Π°Π»ΡŒΠ½ΠΎΠΉ зависимости с Π΅ΠΌΠΊΠΎΡΡ‚ΡŒΡŽ кондСнсатора Π‘ ΠΈ напряТСниСм Π½Π° ΠΎΠ±ΠΊΠ»Π°Π΄ΠΊΠ°Ρ… Uc. Π­Ρ‚Π° Π·Π°Π²ΠΈΡΠΈΠΌΠΎΡΡ‚ΡŒ выраТаСтся Ρ„ΠΎΡ€ΠΌΡƒΠ»ΠΎΠΉ: q = C x Uc.

Π’ процСссС зарядки ΠΎΠ΄Π½Π° ΠΈΠ· ΠΎΠ±ΠΊΠ»Π°Π΄ΠΎΠΊ кондСнсатора ΠΏΡ€ΠΈΠΎΠ±Ρ€Π΅Ρ‚Π°Π΅Ρ‚, Π° другая тСряСт ΠΎΠΏΡ€Π΅Π΄Π΅Π»Π΅Π½Π½ΠΎΠ΅ количСство элСктронов. Они пСрСносятся ΠΏΠΎ внСшнСй Ρ†Π΅ΠΏΠΈ ΠΏΠΎΠ΄ влияниСм элСктродвиТущСй силы Π³Π΅Π½Π΅Ρ€Π°Ρ‚ΠΎΡ€Π°. Π’Π°ΠΊΠΎΠ΅ ΠΏΠ΅Ρ€Π΅ΠΌΠ΅Ρ‰Π΅Π½ΠΈΠ΅ являСтся элСктричСским Ρ‚ΠΎΠΊΠΎΠΌ, извСстным Π΅Ρ‰Π΅ ΠΊΠ°ΠΊ зарядный Смкостной Ρ‚ΠΎΠΊ (IΠ·Π°Ρ€).

Π’Π΅Ρ‡Π΅Π½ΠΈΠ΅ зарядного Ρ‚ΠΎΠΊΠ° Π² Ρ†Π΅ΠΏΠΈ происходит практичСски Π·Π° тысячныС Π΄ΠΎΠ»ΠΈ сСкунды, Π΄ΠΎ Ρ‚ΠΎΠ³ΠΎ ΠΌΠΎΠΌΠ΅Π½Ρ‚Π°, ΠΏΠΎΠΊΠ° напряТСниС кондСнсатора Π½Π΅ станСт Ρ€Π°Π²Π½Ρ‹ΠΌ элСктродвиТущСй силС Π³Π΅Π½Π΅Ρ€Π°Ρ‚ΠΎΡ€Π°. НапряТСниС увСличиваСтся ΠΏΠ»Π°Π²Π½ΠΎ, Π° ΠΏΠΎΡ‚ΠΎΠΌ постСпСнно замСдляСтся. Π”Π°Π»Π΅Π΅ Π·Π½Π°Ρ‡Π΅Π½ΠΈΠ΅ напряТСния кондСнсатора Π±ΡƒΠ΄Π΅Ρ‚ постоянным. Π’ΠΎ врСмя зарядки ΠΏΠΎ Ρ†Π΅ΠΏΠΈ Ρ‚Π΅Ρ‡Π΅Ρ‚ зарядный Ρ‚ΠΎΠΊ. Π’ самом Π½Π°Ρ‡Π°Π»Π΅ ΠΎΠ½ достигаСт максимальной Π²Π΅Π»ΠΈΡ‡ΠΈΠ½Ρ‹, Ρ‚Π°ΠΊ ΠΊΠ°ΠΊ напряТСниС кондСнсатора ΠΈΠΌΠ΅Π΅Ρ‚ Π½ΡƒΠ»Π΅Π²ΠΎΠ΅ Π·Π½Π°Ρ‡Π΅Π½ΠΈΠ΅. Богласно Π·Π°ΠΊΠΎΠ½Π° Ома I Π·Π°Ρ€ = Π•/R i , ΠΏΠΎΡΠΊΠΎΠ»ΡŒΠΊΡƒ ΠΊ ΡΠΎΠΏΡ€ΠΎΡ‚ΠΈΠ²Π»Π΅Π½ΠΈΡŽ Ri ΠΏΡ€ΠΈΠ»ΠΎΠΆΠ΅Π½Π° вся Π­Π”Π‘ Π³Π΅Π½Π΅Ρ€Π°Ρ‚ΠΎΡ€Π°.

Π€ΠΎΡ€ΠΌΡƒΠ»Π° Ρ‚ΠΎΠΊΠ° ΡƒΡ‚Π΅Ρ‡ΠΊΠΈ кондСнсатора

Π’ΠΎΠΊ ΡƒΡ‚Π΅Ρ‡ΠΊΠΈ кондСнсатора Π²ΠΏΠΎΠ»Π½Π΅ ΠΌΠΎΠΆΠ½ΠΎ ΡΡ€Π°Π²Π½ΠΈΡ‚ΡŒ с воздСйствиСм ΠΏΠΎΠ΄ΠΊΠ»ΡŽΡ‡Π΅Π½Π½ΠΎΠ³ΠΎ ΠΊ Π½Π΅ΠΌΡƒ рСзистора с ΠΊΠ°ΠΊΠΈΠΌ-Π»ΠΈΠ±ΠΎ сопротивлСниСм R. Π’ΠΎΠΊ ΡƒΡ‚Π΅Ρ‡ΠΊΠΈ тСсно связан с Ρ‚ΠΈΠΏΠΎΠΌ кондСнсатора ΠΈ качСством ΠΈΡΠΏΠΎΠ»ΡŒΠ·ΡƒΠ΅ΠΌΠΎΠ³ΠΎ диэлСктрика. ΠšΡ€ΠΎΠΌΠ΅ Ρ‚ΠΎΠ³ΠΎ, Π²Π°ΠΆΠ½Ρ‹ΠΌ Ρ„Π°ΠΊΡ‚ΠΎΡ€ΠΎΠΌ становится конструкция корпуса ΠΈ ΡΡ‚Π΅ΠΏΠ΅Π½ΡŒ Π΅Π³ΠΎ загрязнСнности.

НСкоторыС кондСнсаторы ΠΈΠΌΠ΅ΡŽΡ‚ Π½Π΅Π³Π΅Ρ€ΠΌΠ΅Ρ‚ΠΈΡ‡Π½Ρ‹ΠΉ корпус, Ρ‡Ρ‚ΠΎ ΠΏΡ€ΠΈΠ²ΠΎΠ΄ΠΈΡ‚ ΠΊ ΠΏΡ€ΠΎΠ½ΠΈΠΊΠ½ΠΎΠ²Π΅Π½ΠΈΡŽ Π²Π»Π°Π³ΠΈ ΠΈΠ· Π²ΠΎΠ·Π΄ΡƒΡ…Π° ΠΈ Π²ΠΎΠ·Ρ€Π°ΡΡ‚Π°Π½ΠΈΡŽ Ρ‚ΠΎΠΊΠ° ΡƒΡ‚Π΅Ρ‡ΠΊΠΈ. Π’ ΠΏΠ΅Ρ€Π²ΡƒΡŽ ΠΎΡ‡Π΅Ρ€Π΅Π΄ΡŒ это касаСтся устройств, Π³Π΄Π΅ Π² качСствС диэлСктрика использована промаслСнная Π±ΡƒΠΌΠ°Π³Π°. Π—Π½Π°Ρ‡ΠΈΡ‚Π΅Π»ΡŒΠ½Ρ‹Π΅ Ρ‚ΠΎΠΊΠΈ ΡƒΡ‚Π΅Ρ‡ΠΊΠΈ Π²ΠΎΠ·Π½ΠΈΠΊΠ°ΡŽΡ‚ ΠΈΠ·-Π·Π° сниТСния элСктричСского сопротивлСния изоляции. Π’ Ρ€Π΅Π·ΡƒΠ»ΡŒΡ‚Π°Ρ‚Π΅ Π½Π°Ρ€ΡƒΡˆΠ°Π΅Ρ‚ΡΡ основная функция кондСнсатора — ΡΠΏΠΎΡΠΎΠ±Π½ΠΎΡΡ‚ΡŒ ΠΏΠΎΠ»ΡƒΡ‡Π°Ρ‚ΡŒ ΠΈ ΡΠΎΡ…Ρ€Π°Π½ΡΡ‚ΡŒ заряд элСктричСского Ρ‚ΠΎΠΊΠ°.

Основная Ρ„ΠΎΡ€ΠΌΡƒΠ»Π° для расчСта выглядит ΡΠ»Π΅Π΄ΡƒΡŽΡ‰ΠΈΠΌ ΠΎΠ±Ρ€Π°Π·ΠΎΠΌ: I ΡƒΡ‚ = U/R d , Π³Π΄Π΅ I ΡƒΡ‚, — это Ρ‚ΠΎΠΊ ΡƒΡ‚Π΅Ρ‡ΠΊΠΈ, U — напряТСниС, ΠΏΡ€ΠΈΠ»Π°Π³Π°Π΅ΠΌΠΎΠ΅ ΠΊ кондСнсатору, Π° R d — сопротивлСниС изоляции.

Β§

6. Заряд ΠΈ
разряд кондСнсатора

Π§Ρ‚ΠΎΠ±Ρ‹ Π·Π°Ρ€ΡΠ΄ΠΈΡ‚ΡŒ
кондСнсатор, Π½Π°Π΄ΠΎ, Ρ‡Ρ‚ΠΎΠ±Ρ‹ свободныС элСктроны ΠΏΠ΅Ρ€Π΅ΡˆΠ»ΠΈ ΠΈΠ· ΠΎΠ΄Π½ΠΎΠΉ ΠΎΠ±ΠΊΠ»Π°Π΄ΠΊΠΈ Π½Π°
Π΄Ρ€ΡƒΠ³ΡƒΡŽ. ΠŸΠ΅Ρ€Π΅Ρ…ΠΎΠ΄ элСктронов с ΠΎΠ΄Π½ΠΎΠΉ ΠΎΠ±ΠΊΠ»Π°Π΄ΠΊΠΈ кондСнсатора Π½Π° Π΄Ρ€ΡƒΠ³ΡƒΡŽ происходит
ΠΏΠΎΠ΄ дСйствиСм напряТСния источника ΠΏΠΎ ΠΏΡ€ΠΎΠ²ΠΎΠ΄Π°ΠΌ, ΡΠΎΠ΅Π΄ΠΈΠ½ΡΡŽΡ‰ΠΈΠΌ этот источник с
ΠΎΠ±ΠΊΠ»Π°Π΄ΠΊΠ°ΠΌΠΈ кондСнсатора.

Π’ ΠΌΠΎΠΌΠ΅Π½Ρ‚
Π²ΠΊΠ»ΡŽΡ‡Π΅Π½ΠΈΡ кондСнсатора зарядов Π½Π° Π΅Π³ΠΎ ΠΎΠ±ΠΊΠ»Π°Π΄ΠΊΠ°Ρ… Π½Π΅Ρ‚ ΠΈ напряТСниС Π½Π° Π½Π΅ΠΌ Ρ€Π°Π²Π½ΠΎ
Π½ΡƒΠ»ΡŽ ΞΌ
с =0.
ΠŸΠΎΡΡ‚ΠΎΠΌΡƒ зарядный Ρ‚ΠΎΠΊ опрСдСляСтся Π²Π½ΡƒΡ‚Ρ€Π΅Π½Π½ΠΈΠΌ сопротивлСниСм источника r
Π² ΠΈ ΠΈΠΌΠ΅Π΅Ρ‚
Π½Π°ΠΈΠ±ΠΎΠ»ΡŒΡˆΡƒΡŽ Π²Π΅Π»ΠΈΡ‡ΠΈΠ½Ρƒ:

I Π— max =E/ r Π².

По ΠΌΠ΅Ρ€Π΅
накоплСния зарядов Π½Π° ΠΎΠ±ΠΊΠ»Π°Π΄ΠΊΠ°Ρ… кондСнсатора напряТСниС Π½Π° Π½Π΅ΠΌ увСличиваСтся ΠΈ
ΠΏΠ°Π΄Π΅Π½ΠΈΠ΅ напряТСния Π½Π° Π²Π½ΡƒΡ‚Ρ€Π΅Π½Π½Π΅ΠΌ сопротивлСнии источника Π±ΡƒΠ΄Π΅Ρ‚ Ρ€Π°Π²Π½ΠΎ разности
Π­Π”Π‘ источника ΠΈ напряТСния Π½Π° кондСнсаторС (Π•- ΞΌ
с). ΡΠ»Π΅Π΄ΠΎΠ²Π°Ρ‚Π΅Π»ΡŒΠ½ΠΎ, зарядный Ρ‚ΠΎΠΊ

i Π· =(Π•- ΞΌ с)/ r Π².

Π’Π°ΠΊΠΈΠΌ ΠΎΠ±Ρ€Π°Π·ΠΎΠΌ, с
ΡƒΠ²Π΅Π»ΠΈΡ‡Π΅Π½ΠΈΠ΅ΠΌ напряТСния Π½Π° кондСнсаторС Ρ‚ΠΎΠΊ заряда снизится ΠΈ ΠΏΡ€ΠΈ ΞΌ
с =Π•
становится Ρ€Π°Π²Π½Ρ‹ΠΌ Π½ΡƒΠ»ΡŽ. ΠŸΡ€ΠΎΡ†Π΅ΡΡ измСнСния напряТСния Π½Π° кондСнсаторС ΠΈ Ρ‚ΠΎΠΊΠ°
заряда Π²ΠΎ Π²Ρ€Π΅ΠΌΠ΅Π½ΠΈ ΠΈΠ·ΠΎΠ±Ρ€Π°ΠΆΠ΅Π½ Π½Π° рис. 1. Π’ самом Π½Π°Ρ‡Π°Π»Π΅ заряда напряТСниС Π½Π° кондСнсаторС
Ρ€Π΅Π·ΠΊΠΎ возрастаСт, Ρ‚Π°ΠΊ ΠΊΠ°ΠΊ зарядный Ρ‚ΠΎΠΊ ΠΈΠΌΠ΅Π΅Ρ‚ наибольшСС Π·Π½Π°Ρ‡Π΅Π½ΠΈΠ΅ ΠΈ Π½Π°ΠΊΠΎΠΏΠ»Π΅Π½ΠΈΠ΅
зарядов Π½Π° ΠΎΠ±ΠΊΠ»Π°Π΄ΠΊΠ°Ρ… кондСнсатора происходит интСнсивно. По ΠΌΠ΅Ρ€Π΅ ΠΏΠΎΠ²Ρ‹ΡˆΠ΅Π½ΠΈΡ
напряТСния Π½Π° кондСнсаторС зарядный Ρ‚ΠΎΠΊ ΡƒΠΌΠ΅Π½ΡŒΡˆΠ°Π΅Ρ‚ΡΡ ΠΈ Π½Π°ΠΊΠΎΠΏΠ»Π΅Π½ΠΈΠ΅ зарядов Π½Π°
ΠΎΠ±ΠΊΠ»Π°Π΄ΠΊΠ°Ρ… замСдляСтся. ΠŸΡ€ΠΎΠ΄ΠΎΠ»ΠΆΠΈΡ‚Π΅Π»ΡŒΠ½ΠΎΡΡ‚ΡŒ заряда кондСнсатора зависит ΠΎΡ‚ Π΅Π³ΠΎ
Смкости ΠΈ сопротивлСния Ρ†Π΅ΠΏΠΈ, ΡƒΠ²Π΅Π»ΠΈΡ‡Π΅Π½ΠΈΠ΅ ΠΊΠΎΡ‚ΠΎΡ€Ρ‹Ρ… ΠΏΡ€ΠΈΠ²ΠΎΠ΄ΠΈΡ‚ ΠΊ Π²ΠΎΠ·Ρ€Π°ΡΡ‚Π°Π½ΠΈΡŽ
ΠΏΡ€ΠΎΠ΄ΠΎΠ»ΠΆΠΈΡ‚Π΅Π»ΡŒΠ½ΠΎΡΡ‚ΠΈ заряда. Π‘ ΡƒΠ²Π΅Π»ΠΈΡ‡Π΅Π½ΠΈΠ΅ΠΌ Смкости кондСнсатора, возрастаСт
количСство зарядов, Π½Π°ΠΊΠ°ΠΏΠ»ΠΈΠ²Π°Π΅ΠΌΡ‹Ρ… Π½Π° Π΅Π³ΠΎ пластинах, Π° Ссли ΡƒΠ²Π΅Π»ΠΈΡ‡ΠΈΡ‚ΡŒ
сопротивлСниС Ρ†Π΅ΠΏΠΈ ΡƒΠΌΠ΅Π½ΡŒΡˆΠΈΡ‚ΡΡ ΠΈ зарядный Ρ‚ΠΎΠΊ, Π° это замСдляСт процСсс
накоплСния зарядов Π½Π° этих ΠΎΠ±ΠΊΠ»Π°Π΄ΠΊΠ°Ρ….

Если ΠΎΠ±ΠΊΠ»Π°Π΄ΠΊΠΈ
заряТСнного кондСнсатора ΠΏΠΎΠ΄ΠΊΠ»ΡŽΡ‡ΠΈΡ‚ΡŒ ΠΊ ΠΊΠ°ΠΊΠΎΠΌΡƒ-Π»ΠΈΠ±ΠΎ ΡΠΎΠΏΡ€ΠΎΡ‚ΠΈΠ²Π»Π΅Π½ΠΈΡŽ R
, Ρ‚ΠΎ Π·Π° счСт напряТСния Π½Π°
кондСнсаторС Π±ΡƒΠ΄Π΅Ρ‚ ΠΏΡ€ΠΎΡ‚Π΅ΠΊΠ°Ρ‚ΡŒ разрядный Ρ‚ΠΎΠΊ кондСнсатора. ΠŸΡ€ΠΈ разрядС
кондСнсатора элСктронысодной пластины (ΠΏΡ€ΠΈ ΠΈΡ… ΠΈΠ·Π±Ρ‹Ρ‚ΠΊΠ΅) Π±ΡƒΠ΄ΡƒΡ‚
ΠΏΠ΅Ρ€Π΅Ρ…ΠΎΠ΄ΠΈΡ‚ΡŒ Π½Π° Π΄Ρ€ΡƒΠ³ΡƒΡŽ (ΠΏΡ€ΠΈ ΠΈΡ… нСдостаткС) ΠΈ Π±ΡƒΠ΄Π΅Ρ‚ продолТаСтся Π΄ΠΎ Ρ‚Π΅Ρ… ΠΏΠΎΡ€, ΠΏΠΎΠΊΠ°
ΠΏΠΎΡ‚Π΅Π½Ρ†ΠΈΠ°Π»Ρ‹ ΠΎΠ±ΠΊΠ»Π°Π΄ΠΎΠΊ Π½Π΅ Π²Ρ‹Ρ€Π°Π²Π½ΡΡŽΡ‚ΡΡ, Ρ‚. Π΅. напряТСниС Π½Π° кондСнсаторС станСт
Ρ€Π°Π²Π½Ρ‹ΠΌ Π½ΡƒΠ»ΡŽ. ИзмСнСниС напряТСния Π² процСссС разряда кондСнсатора ΠΈΠ·ΠΎΠ±Ρ€Π°ΠΆΠ΅Π½ΠΎ Π½Π°
рис. 2. Π’ΠΎΠΊ разряда кондСнсатора ΠΏΡ€ΠΎΠΏΠΎΡ€Ρ†ΠΈΠΎΠ½Π°Π»Π΅Π½ Π½Π°ΠΏΡ€ΡΠΆΠ΅Π½ΠΈΡŽ Π½Π° кондСнсаторС (i
Ρ€ =ΞΌ
с /R
), ΠΈ Π΅Π³ΠΎ ΠΈΠ·ΠΌΠ΅Π½Π΅Π½ΠΈΠ΅ Π²ΠΎ
Π²Ρ€Π΅ΠΌΠ΅Π½ΠΈ ΠΏΠΎΠ΄ΠΎΠ±Π½ΠΎ измСнСнию напряТСния.

Π’ Π½Π°Ρ‡Π°Π»ΡŒΠ½Ρ‹ΠΉ
ΠΌΠΎΠΌΠ΅Π½Ρ‚ разряда напряТСниС Π½Π° кондСнсаторС наибольшСС (ΞΌ
с =Π•) ΠΈ разрядный Ρ‚ΠΎΠΊ ΠΌΠ°ΠΊΡΠΈΠΌΠ°Π»ΡŒΠ½Ρ‹ΠΉ (I
Ρ€ max
=E
/R
), Ρ‚Π°ΠΊ Ρ‡Ρ‚ΠΎ разряд происходит быстро. ΠŸΡ€ΠΈ ΠΏΠΎΠ½ΠΈΠΆΠ΅Π½ΠΈΠΈ напряТСния, Ρ‚ΠΎΠΊ
разряда сниТаСтся ΠΈ процСсс ΠΏΠ΅Ρ€Π΅Ρ…ΠΎΠ΄Π° зарядов с ΠΎΠ΄Π½ΠΎΠΉ ΠΎΠ±ΠΊΠ»Π°Π΄ΠΊΠΈ Π½Π° Π΄Ρ€ΡƒΠ³ΡƒΡŽ затормаТиваСтся.

ВрСмя процСсса
разряда кондСнсатора зависит ΠΎΡ‚ сопротивлСния Ρ†Π΅ΠΏΠΈ ΠΈ Смкости кондСнсатора,
ΠΏΡ€ΠΈΡ‡Π΅ΠΌ возрастаниС ΠΊΠ°ΠΊ сопротивлСния, Ρ‚Π°ΠΊ ΠΈ Смкости ΡƒΠ²Π΅Π»ΠΈΡ‡ΠΈΠ²Π°Π΅Ρ‚
ΠΏΡ€ΠΎΠ΄ΠΎΠ»ΠΆΠΈΡ‚Π΅Π»ΡŒΠ½ΠΎΡΡ‚ΡŒ разряда. Π‘ ΡƒΠ²Π΅Π»ΠΈΡ‡Π΅Π½ΠΈΠ΅ΠΌ сопротивлСния разрядный Ρ‚ΠΎΠΊ сниТаСтся,
замСдляСтся процСсс пСрСноски зарядов с ΠΎΠ΄Π½ΠΎΠΉ Π½Π° Π΄Ρ€ΡƒΠ³ΡƒΡŽ ΠΎΠ±ΠΊΠ»Π°Π΄ΠΎΠΊ; с ΡƒΠ²Π΅Π»ΠΈΡ‡Π΅Π½ΠΈΠ΅ΠΌ
Смкости кондСнсатора ΠΏΠΎΠ²Ρ‹ΡˆΠ°Π΅Ρ‚ΡΡ заряд Π½Π° ΠΎΠ±ΠΊΠ»Π°Π΄ΠΊΠ°Ρ….

Π’Π°ΠΊΠΈΠΌ ΠΎΠ±Ρ€Π°Π·ΠΎΠΌ, Π² Ρ†Π΅ΠΏΠΈ, содСрТащСй кондСнсатор,
Ρ‚ΠΎΠΊ ΠΏΡ€ΠΎΡ…ΠΎΠ΄ΠΈΡ‚ Ρ‚ΠΎΠ»ΡŒΠΊΠΎ Π² процСссС Π΅Π³ΠΎ заряда ΠΈ разряда, Ρ‚. Π΅. ΠΊΠΎΠ³Π΄Π° напряТСниС Π½Π°
ΠΎΠ±ΠΊΠ»Π°Π΄ΠΊΠ°Ρ… ΠΏΡ€Π΅Ρ‚Π΅Ρ€ΠΏΠ΅Π²Π°Π΅Ρ‚ ΠΈΠ·ΠΌΠ΅Π½Π΅Π½ΠΈΠ΅ Π²ΠΎ Π²Ρ€Π΅ΠΌΠ΅Π½ΠΈ. ΠŸΡ€ΠΈ постоянствС напряТСния Ρ‚ΠΎΠΊ
Ρ‡Π΅Ρ€Π΅Π· кондСнсатор Π½Π΅ ΠΏΡ€ΠΎΡ…ΠΎΠ΄ΠΈΡ‚, Ρ‚. Π΅. кондСнсатор Π½Π΅ пропускаСт постоянный Ρ‚ΠΎΠΊ,
Ρ‚Π°ΠΊ ΠΊΠ°ΠΊ ΠΌΠ΅ΠΆΠ΄Ρƒ Π΅Π³ΠΎ ΠΎΠ±ΠΊΠ»Π°Π΄ΠΊΠ°ΠΌΠΈ ΠΏΠΎΠΌΠ΅Ρ‰Π΅Π½ диэлСктрик ΠΈ Π² Ρ€Π΅Π·ΡƒΠ»ΡŒΡ‚Π°Ρ‚Π΅ этого Ρ†Π΅ΠΏΡŒ
Ρ€Π°Π·ΠΎΠΌΠΊΠ½ΡƒΡ‚Π°.

ΠŸΡ€ΠΈ зарядкС
кондСнсатора, послСдний способСн Π½Π°ΠΊΠ°ΠΏΠ»ΠΈΠ²Π°Ρ‚ΡŒ ΡΠ»Π΅ΠΊΡ‚Ρ€ΠΈΡ‡Π΅ΡΠΊΡƒΡŽ ΡΠ½Π΅Ρ€Π³ΠΈΡŽ, потрСбляя
Π΅Π΅ ΠΎΡ‚ энСргоисточника. НакоплСнная энСргия сохраняСтся ΠΎΠΏΡ€Π΅Π΄Π΅Π»Π΅Π½Π½ΠΎΠ΅ врСмя. ΠŸΡ€ΠΈ
разрядС кондСнсатора эта энСргия ΠΏΠ΅Ρ€Π΅Ρ…ΠΎΠ΄ΠΈΡ‚ ΠΊ разрядному рСзистору, нагрСвая
Π΅Π³ΠΎ, Ρ‚. Π΅. ΡΠ½Π΅Ρ€Π³ΠΈΡŽ элСктричСского поля прСвращаСтся Π² Ρ‚Π΅ΠΏΠ»ΠΎΠ²ΡƒΡŽ. Π§Π΅ΠΌ Π²Ρ‹ΡˆΠ΅
Π΅ΠΌΠΊΠΎΡΡ‚ΡŒ кондСнсатора ΠΈ напряТСниС Π½Π° Π΅Π³ΠΎ ΠΎΠ±ΠΊΠ»Π°Π΄ΠΊΠ°Ρ…, Ρ‚Π΅ΠΌ Π±ΡƒΠ΄Π΅Ρ‚ большС энСргии,
запасСнной Π½Π° Π½Π΅ΠΌ. ЭнСргия элСктричСского поля кондСнсатора опрСдСляСтся
ΡΠ»Π΅Π΄ΡƒΡŽΡ‰ΠΈΠΌ Π²Ρ‹Ρ€Π°ΠΆΠ΅Π½ΠΈΠ΅ΠΌ

W=CU 2 /2.

Если кондСнсатор
Π΅ΠΌΠΊΠΎΡΡ‚ΡŒΡŽ 100 ΠΌΠΊΠ€ заряТСн Π΄ΠΎ напряТСния 200 Π’, Ρ‚ΠΎ энСргия, запасСнная Π²
элСктричСском ΠΏΠΎΠ»Π΅ кондСнсатора, W
=100Β·
10 -6 Β·
200 2 /2=2 Π”ΠΆ.

Π’Π°ΠΌ понадобится

  • — Π·Π½Π°Π½ΠΈΠ΅ Смкости ΠΈΠ»ΠΈ гСомСтричСских ΠΈ физичСских ΠΏΠ°Ρ€Π°ΠΌΠ΅Ρ‚Ρ€ΠΎΠ² кондСнсатора;
  • — Π·Π½Π°Π½ΠΈΠ΅ энСргии ΠΈΠ»ΠΈ заряда Π½Π° кондСнсаторС.

Π˜Π½ΡΡ‚Ρ€ΡƒΠΊΡ†ΠΈΡ

НайдитС напряТСниС ΠΌΠ΅ΠΆΠ΄Ρƒ пластинами кондСнсатора, Ссли извСстна тСкущая Π²Π΅Π»ΠΈΡ‡ΠΈΠ½Π° Π½Π°ΠΊΠΎΠΏΠ»Π΅Π½Π½ΠΎΠΉ ΠΈΠΌ энСргии, Π° Ρ‚Π°ΠΊΠΆΠ΅ Π΅Π³ΠΎ Π΅ΠΌΠΊΠΎΡΡ‚ΡŒ. ЭнСргия, запасСнная кондСнсатором, ΠΌΠΎΠΆΠ΅Ρ‚ Π±Ρ‹Ρ‚ΡŒ вычислСна ΠΏΠΎ Ρ„ΠΎΡ€ΠΌΡƒΠ»Π΅ W=(Cβˆ™UΒ²)/2, Π³Π΄Π΅ C — Π΅ΠΌΠΊΠΎΡΡ‚ΡŒ, Π° U — напряТСниС ΠΌΠ΅ΠΆΠ΄Ρƒ пластинами. Π’Π°ΠΊΠΈΠΌ ΠΎΠ±Ρ€Π°Π·ΠΎΠΌ, Π·Π½Π°Ρ‡Π΅Π½ΠΈΠ΅ напряТСния ΠΌΠΎΠΆΠ΅Ρ‚ Π±Ρ‹Ρ‚ΡŒ ΠΏΠΎΠ»ΡƒΡ‡Π΅Π½ΠΎ ΠΊΠ°ΠΊ ΠΊΠΎΡ€Π΅Π½ΡŒ ΠΈΠ· ΡƒΠ΄Π²ΠΎΠ΅Π½Π½ΠΎΠ³ΠΎ значСния энСргии, Π΄Π΅Π»Π΅Π½Π½ΠΎΠ³ΠΎ Π½Π° Π΅ΠΌΠΊΠΎΡΡ‚ΡŒ. Π’ΠΎ Π΅ΡΡ‚ΡŒ, ΠΎΠ½ΠΎ Π±ΡƒΠ΄Π΅Ρ‚ Ρ€Π°Π²Π½ΠΎ: U=√(2βˆ™W/C).

ЭнСргия, запасСнная кондСнсатором, Ρ‚Π°ΠΊΠΆΠ΅ ΠΌΠΎΠΆΠ΅Ρ‚ Π±Ρ‹Ρ‚ΡŒ вычислСна Π½Π° основании значСния содСрТащСгося Π² Π½Π΅ΠΌ заряда (количСства ) ΠΈ напряТСния ΠΌΠ΅ΠΆΠ΄Ρƒ ΠΎΠ±ΠΊΠ»Π°Π΄ΠΊΠ°ΠΌΠΈ. Π€ΠΎΡ€ΠΌΡƒΠ»Π°, Π·Π°Π΄Π°ΡŽΡ‰Π°Ρ соотвСтствиС ΠΌΠ΅ΠΆΠ΄Ρƒ этими ΠΏΠ°Ρ€Π°ΠΌΠ΅Ρ‚Ρ€Π°ΠΌΠΈ, ΠΈΠΌΠ΅Π΅Ρ‚ Π²ΠΈΠ΄: W=qβˆ™U/2 (Π³Π΄Π΅ q — заряд). Π‘Π»Π΅Π΄ΠΎΠ²Π°Ρ‚Π΅Π»ΡŒΠ½ΠΎ, зная ΡΠ½Π΅Ρ€Π³ΠΈΡŽ ΠΈ , ΠΌΠΎΠΆΠ½ΠΎ Π²Ρ‹Ρ‡ΠΈΡΠ»ΠΈΡ‚ΡŒ напряТСниС ΠΌΠ΅ΠΆΠ΄Ρƒ Π΅Π³ΠΎ пластинами ΠΏΠΎ Ρ„ΠΎΡ€ΠΌΡƒΠ»Π΅: U=2βˆ™W/q.

ΠŸΠΎΡΠΊΠΎΠ»ΡŒΠΊΡƒ заряд Π½Π° кондСнсаторС ΠΏΡ€ΠΎΠΏΠΎΡ€Ρ†ΠΈΠΎΠ½Π°Π»Π΅Π½ ΠΊΠ°ΠΊ ΠΏΡ€ΠΈΠ»ΠΎΠΆΠ΅Π½Π½ΠΎΠΌΡƒ ΠΊ Π΅Π³ΠΎ пластинам Π½Π°ΠΏΡ€ΡΠΆΠ΅Π½ΠΈΡŽ, Ρ‚Π°ΠΊ ΠΈ Смкости устройства (ΠΎΠ½ опрСдСляСтся Ρ„ΠΎΡ€ΠΌΡƒΠ»ΠΎΠΉ q=Cβˆ™U), Ρ‚ΠΎ, зная заряд ΠΈ Π΅ΠΌΠΊΠΎΡΡ‚ΡŒ, ΠΌΠΎΠΆΠ½ΠΎ Π½Π°ΠΉΡ‚ΠΈ ΠΈ напряТСниС. БоотвСтствСнно, для провСдСния расчСта ΠΈΡΠΏΠΎΠ»ΡŒΠ·ΡƒΠΉΡ‚Π΅ Ρ„ΠΎΡ€ΠΌΡƒΠ»Ρƒ: U=q/C.

Для получСния значСния напряТСния Π½Π° кондСнсаторС с извСстными гСомСтричСскими ΠΈ ΠΏΠ°Ρ€Π°ΠΌΠ΅Ρ‚Ρ€Π°ΠΌΠΈ, сначала рассчитайтС Π΅Π³ΠΎ Π΅ΠΌΠΊΠΎΡΡ‚ΡŒ. Для простого плоского кондСнсатора, состоящСго ΠΈΠ· Π΄Π²ΡƒΡ… проводящих пластин, Ρ€Π°Π·Π΄Π΅Π»Π΅Π½Π½Ρ‹Ρ… , расстояниС ΠΌΠ΅ΠΆΠ΄Ρƒ ΠΊΠΎΡ‚ΠΎΡ€Ρ‹ΠΌΠΈ ΠΏΡ€Π΅Π½Π΅Π±Ρ€Π΅ΠΆΠΈΠΌΠΎ ΠΌΠ°Π»ΠΎ ΠΏΠΎ ΡΡ€Π°Π²Π½Π΅Π½ΠΈΡŽ с ΠΈΡ… Ρ€Π°Π·ΠΌΠ΅Ρ€Π°ΠΌΠΈ, Π΅ΠΌΠΊΠΎΡΡ‚ΡŒ ΠΌΠΎΠΆΠ΅Ρ‚ Π±Ρ‹Ρ‚ΡŒ вычислСна ΠΏΠΎ Ρ„ΠΎΡ€ΠΌΡƒΠ»Π΅: C=(Ξ΅βˆ™Ξ΅0βˆ™S)/d. -12 Π€/ΠΌ), Ξ΅ — ΠΎΡ‚Π½ΠΎΡΠΈΡ‚Π΅Π»ΡŒΠ½Π°Ρ диэлСктричСская ΠΏΡ€ΠΎΠ½ΠΈΡ†Π°Π΅ΠΌΠΎΡΡ‚ΡŒ пространства ΠΌΠ΅ΠΆΠ΄Ρƒ пластинами (Π΅Π΅ ΠΌΠΎΠΆΠ½ΠΎ ΡƒΠ·Π½Π°Ρ‚ΡŒ ΠΈΠ· физичСских справочников). Вычислив Π΅ΠΌΠΊΠΎΡΡ‚ΡŒ, рассчитайтС напряТСниС ΠΎΠ΄Π½ΠΈΠΌ ΠΈΠ· ΠΌΠ΅Ρ‚ΠΎΠ΄ΠΎΠ², ΠΏΡ€ΠΈΠ²Π΅Π΄Π΅Π½Π½Ρ‹Ρ… Π² ΡˆΠ°Π³Π°Ρ… 1-3.

ΠžΠ±Ρ€Π°Ρ‚ΠΈΡ‚Π΅ Π²Π½ΠΈΠΌΠ°Π½ΠΈΠ΅

Для получСния ΠΊΠΎΡ€Ρ€Π΅ΠΊΡ‚Π½Ρ‹Ρ… Ρ€Π΅Π·ΡƒΠ»ΡŒΡ‚Π°Ρ‚ΠΎΠ² ΠΏΡ€ΠΈ вычислСнии напряТСний ΠΌΠ΅ΠΆΠ΄Ρƒ ΠΎΠ±ΠΊΠ»Π°Π΄ΠΊΠ°ΠΌΠΈ кондСнсаторов, ΠΏΠ΅Ρ€Π΅Π΄ ΠΏΡ€ΠΎΠ²Π΅Π΄Π΅Π½ΠΈΠ΅ΠΌ расчСтов ΠΏΡ€ΠΈΠ²ΠΎΠ΄ΠΈΡ‚Π΅ значСния всСх ΠΏΠ°Ρ€Π°ΠΌΠ΅Ρ‚Ρ€ΠΎΠ² Π² систСму БИ.

Для Ρ‚ΠΎΠ³ΠΎ Ρ‡Ρ‚ΠΎΠ±Ρ‹ Π·Π½Π°Ρ‚ΡŒ, ΠΌΠΎΠΆΠ½ΠΎ Π»ΠΈ ΠΈΡΠΏΠΎΠ»ΡŒΠ·ΠΎΠ²Π°Ρ‚ΡŒ Π² Ρ‚ΠΎΠΌ ΠΈΠ»ΠΈ ΠΈΠ½ΠΎΠΌ мСстС схСмы кондСнсатор, слСдуСт ΠΎΠΏΡ€Π΅Π΄Π΅Π»ΠΈΡ‚ΡŒ Π΅Π³ΠΎ . Бпособ нахоТдСния этого ΠΏΠ°Ρ€Π°ΠΌΠ΅Ρ‚Ρ€Π° зависит ΠΎΡ‚ Ρ‚ΠΎΠ³ΠΎ, ΠΊΠ°ΠΊΠΈΠΌ ΠΎΠ±Ρ€Π°Π·ΠΎΠΌ ΠΎΠ½ ΠΎΠ±ΠΎΠ·Π½Π°Ρ‡Π΅Π½ Π½Π° кондСнсаторС ΠΈ ΠΎΠ±ΠΎΠ·Π½Π°Ρ‡Π΅Π½ Π»ΠΈ Π²ΠΎΠΎΠ±Ρ‰Π΅.

Π’Π°ΠΌ понадобится

  • Π˜Π·ΠΌΠ΅Ρ€ΠΈΡ‚Π΅Π»ΡŒ Смкости

Π˜Π½ΡΡ‚Ρ€ΡƒΠΊΡ†ΠΈΡ

На ΠΊΡ€ΡƒΠΏΠ½Ρ‹Ρ… кондСнсаторах
Π΅ΠΌΠΊΠΎΡΡ‚ΡŒ
ΠΎΠ±Ρ‹Ρ‡Π½ΠΎ ΠΎΠ±ΠΎΠ·Π½Π°Ρ‡Π΅Π½Π° ΠΎΡ‚ΠΊΡ€Ρ‹Ρ‚Ρ‹ΠΌ тСкстом: 0,25 ΠΌΠΊΠ€ ΠΈΠ»ΠΈ 15 uF. Π’ этом случаС, способ Π΅Π΅ опрСдСлСния Ρ‚Ρ€ΠΈΠ²ΠΈΠ°Π»Π΅Π½.

На ΠΌΠ΅Π½Π΅Π΅ ΠΊΡ€ΡƒΠΏΠ½Ρ‹Ρ… кондСнсаторах
(Π² Ρ‚ΠΎΠΌ , SMD) Π΅ΠΌΠΊΠΎΡΡ‚ΡŒ
двумя ΠΈΠ»ΠΈ трСмя Ρ†ΠΈΡ„Ρ€Π°ΠΌΠΈ. Π’ ΠΏΠ΅Ρ€Π²ΠΎΠΌ случаС, ΠΎΠ½Π° ΠΎΠ±ΠΎΠ·Π½Π°Ρ‡Π΅Π½Π° Π² ΠΏΠΈΠΊΠΎΡ„Π°Ρ€Π°Π΄Π°Ρ…. Π’ΠΎ Π²Ρ‚ΠΎΡ€ΠΎΠΌ случаС, ΠΏΠ΅Ρ€Π²Ρ‹Π΅ Π΄Π²Π΅ Ρ†ΠΈΡ„Ρ€Ρ‹ Π΅ΠΌΠΊΠΎΡΡ‚ΡŒ
, Π° Ρ‚Ρ€Π΅Ρ‚ΡŒΡ — Π² ΠΊΠ°ΠΊΠΈΡ… Π΅Π΄ΠΈΠ½ΠΈΡ†Π°Ρ… ΠΎΠ½Π° Π²Ρ‹Ρ€Π°ΠΆΠ΅Π½Π°:1 — дСсятки ΠΏΠΈΠΊΠΎΡ„Π°Ρ€Π°Π΄;
2 — сотни ΠΏΠΈΠΊΠΎΡ„Π°Ρ€Π°Π΄;
3 — Π½Π°Π½ΠΎΡ„Π°Ρ€Π°Π΄Ρ‹;
4 — дСсятки Π½Π°Π½ΠΎΡ„Π°Ρ€Π°Π΄;
5 — Π΄ΠΎΠ»ΠΈ ΠΌΠΈΠΊΡ€ΠΎΡ„Π°Ρ€Π°Π΄Ρ‹.

БущСствуСт Ρ‚Π°ΠΊΠΆΠ΅ систСма обозначСния Смкости, ΠΈΡΠΏΠΎΠ»ΡŒΠ·ΡƒΡŽΡ‰Π°Ρ сочСтания латинских Π±ΡƒΠΊΠ² ΠΈ Ρ†ΠΈΡ„Ρ€. Π‘ΡƒΠΊΠ²Ρ‹ ΠΎΠ±ΠΎΠ·Π½Π°Ρ‡Π°ΡŽΡ‚ ΡΠ»Π΅Π΄ΡƒΡŽΡ‰ΠΈΠ΅ Ρ†ΠΈΡ„Ρ€Ρ‹:A — 10;
B — 11;
C — 12;
D — 13;
E — 15;
F — 16;
G — 18;
H — 20;
J — 22;
K — 24;
L — 27;
M — 30;
N — 33;
P — 36;
Q — 39;
R — 43;
S — 47;
T — 51;
U — 56;
V — 62;
W — 68;
X — 75;
Y — 82;
Z — 91.ΠŸΠΎΠ»ΡƒΡ‡Π΅Π½Π½ΠΎΠ΅ число слСдуСт ΡƒΠΌΠ½ΠΎΠΆΠΈΡ‚ΡŒ Π½Π° число 10, ΠΏΡ€Π΅Π΄Π²Π°Ρ€ΠΈΡ‚Π΅Π»ΡŒΠ½ΠΎ Π²ΠΎΠ·Π²Π΅Π΄Π΅Π½Π½ΠΎΠ΅ Π² ΡΡ‚Π΅ΠΏΠ΅Π½ΡŒ, Ρ€Π°Π²Π½ΡƒΡŽ Ρ†ΠΈΡ„Ρ€Π΅, ΡΠ»Π΅Π΄ΡƒΡŽΡ‰Π΅ΠΉ послС . Π Π΅Π·ΡƒΠ»ΡŒΡ‚Π°Ρ‚ Π±ΡƒΠ΄Π΅Ρ‚ Π²Ρ‹Ρ€Π°ΠΆΠ΅Π½ Π² ΠΏΠΈΠΊΠΎΡ„Π°Ρ€Π°Π΄Π°Ρ….

Π’ΡΡ‚Ρ€Π΅Ρ‡Π°ΡŽΡ‚ΡΡ кондСнсаторы, Π΅ΠΌΠΊΠΎΡΡ‚ΡŒ
Π½Π° ΠΊΠΎΡ‚ΠΎΡ€Ρ‹Ρ… Π½Π΅ ΠΎΠ±ΠΎΠ·Π½Π°Ρ‡Π΅Π½Π° Π²ΠΎΠΎΠ±Ρ‰Π΅. Π’Ρ‹ навСрняка встрСчали ΠΈΡ…, Π² , Π² стартСрах Π»Π°ΠΌΠΏ Π΄Π½Π΅Π²Π½ΠΎΠ³ΠΎ . Π’ этом случаС, ΠΈΠ·ΠΌΠ΅Ρ€ΠΈΡ‚ΡŒ Π΅ΠΌΠΊΠΎΡΡ‚ΡŒ
ΠΌΠΎΠΆΠ½ΠΎ Ρ‚ΠΎΠ»ΡŒΠΊΠΎ ΡΠΏΠ΅Ρ†ΠΈΠ°Π»ΡŒΠ½Ρ‹ΠΌ ΠΏΡ€ΠΈΠ±ΠΎΡ€ΠΎΠΌ. Они Ρ†ΠΈΡ„Ρ€ΠΎΠ²Ρ‹ΠΌΠΈ ΠΈ мостовыми.Π’ любом случаС, Ссли кондСнсатор впаян Π² Ρ‚ΠΎ ΠΈΠ»ΠΈ ΠΈΠ½ΠΎΠ΅ устройство, Π΅Π³ΠΎ слСдуСт ΠΎΠ±Π΅ΡΡ‚ΠΎΡ‡ΠΈΡ‚ΡŒ, Ρ€Π°Π·Ρ€ΡΠ΄ΠΈΡ‚ΡŒ Π² Π½Π΅ΠΌ кондСнсаторы Ρ„ΠΈΠ»ΡŒΡ‚Ρ€Π° ΠΈ сам кондСнсатор, Π΅ΠΌΠΊΠΎΡΡ‚ΡŒ
ΠΊΠΎΡ‚ΠΎΡ€ΠΎΠ³ΠΎ слСдуСт ΠΈΠ·ΠΌΠ΅Ρ€ΠΈΡ‚ΡŒ, ΠΈ лишь послС этого Π²Ρ‹ΠΏΠ°ΡΡ‚ΡŒ Π΅Π³ΠΎ. Π—Π°Ρ‚Π΅ΠΌ Π΅Π³ΠΎ Π½Π΅ΠΎΠ±Ρ…ΠΎΠ΄ΠΈΠΌΠΎ ΠΏΠΎΠ΄ΠΊΠ»ΡŽΡ‡ΠΈΡ‚ΡŒ ΠΊ ΠΏΡ€ΠΈΠ±ΠΎΡ€Ρƒ.На Ρ†ΠΈΡ„Ρ€ΠΎΠ²ΠΎΠΌ ΠΈΠ·ΠΌΠ΅Ρ€ΠΈΡ‚Π΅Π»Π΅ сначала Π²Ρ‹Π±ΠΈΡ€Π°ΡŽΡ‚ самый Π³Ρ€ΡƒΠ±Ρ‹ΠΉ ΠΏΡ€Π΅Π΄Π΅Π», Π·Π°Ρ‚Π΅ΠΌ ΠΏΠ΅Ρ€Π΅ΠΊΠ»ΡŽΡ‡Π°ΡŽΡ‚ Π΅Π³ΠΎ Π΄ΠΎ Ρ‚Π΅Ρ… ΠΏΠΎΡ€, ΠΏΠΎΠΊΠ° ΠΎΠ½ Π½Π΅ ΠΏΠΎΠΊΠ°ΠΆΠ΅Ρ‚ ΠΏΠ΅Ρ€Π΅Π³Ρ€ΡƒΠ·ΠΊΡƒ. ПослС этого ΠΏΠ΅Ρ€Π΅ΠΊΠ»ΡŽΡ‡Π°Ρ‚Π΅Π»ΡŒ пСрСводят Π½Π° ΠΎΠ΄ΠΈΠ½ ΠΏΡ€Π΅Π΄Π΅Π» Π½Π°Π·Π°Π΄ ΠΈ Ρ‡ΠΈΡ‚Π°ΡŽΡ‚ показания, Π° ΠΏΠΎ полоТСнию ΠΏΠ΅Ρ€Π΅ΠΊΠ»ΡŽΡ‡Π°Ρ‚Π΅Π»Ρ ΠΎΠΏΡ€Π΅Π΄Π΅Π»ΡΡŽΡ‚, Π² ΠΊΠ°ΠΊΠΈΡ… Π΅Π΄ΠΈΠ½ΠΈΡ†Π°Ρ… ΠΎΠ½ΠΈ Π²Ρ‹Ρ€Π°ΠΆΠ΅Π½Ρ‹.На мостовом ΠΈΠ·ΠΌΠ΅Ρ€ΠΈΡ‚Π΅Π»Π΅, ΠΏΠΎΡΠ»Π΅Π΄ΠΎΠ²Π°Ρ‚Π΅Π»ΡŒΠ½ΠΎ ΠΏΠ΅Ρ€Π΅ΠΊΠ»ΡŽΡ‡Π°Ρ , Π½Π° ΠΊΠ°ΠΆΠ΄ΠΎΠΌ ΠΈΠ· Π½ΠΈΡ… ΠΏΡ€ΠΎΠΊΡ€ΡƒΡ‡ΠΈΠ²Π°ΡŽΡ‚ рСгулятор ΠΈΠ· ΠΎΠ΄Π½ΠΎΠ³ΠΎ ΠΊΠΎΠ½Ρ†Π° ΡˆΠΊΠ°Π»Ρ‹ Π² Π΄Ρ€ΡƒΠ³ΠΎΠΉ, ΠΏΠΎΠΊΠ° Π·Π²ΡƒΠΊ ΠΈΠ· Π΄ΠΈΠ½Π°ΠΌΠΈΠΊΠ° Π½Π΅ исчСзнСт. Π”ΠΎΠ±ΠΈΠ²ΡˆΠΈΡΡŒ исчСзновСния , ΠΏΠΎ шкалС рСгулятора ΡΡ‡ΠΈΡ‚Ρ‹Π²Π°ΡŽΡ‚ Ρ€Π΅Π·ΡƒΠ»ΡŒΡ‚Π°Ρ‚, Π° Π΅Π΄ΠΈΠ½ΠΈΡ†Ρ‹, Π² ΠΊΠΎΡ‚ΠΎΡ€Ρ‹Ρ… ΠΎΠ½ Π²Ρ‹Ρ€Π°ΠΆΠ΅Π½, Ρ‚Π°ΠΊΠΆΠ΅ ΠΎΠΏΡ€Π΅Π΄Π΅Π»ΡΡŽΡ‚ ΠΏΠΎ полоТСнию ΠΏΠ΅Ρ€Π΅ΠΊΠ»ΡŽΡ‡Π°Ρ‚Π΅Π»Ρ.Π—Π°Ρ‚Π΅ΠΌ кондСнсатор ΡƒΡΡ‚Π°Π½Π°Π²Π»ΠΈΠ²Π°ΡŽΡ‚ ΠΎΠ±Ρ€Π°Ρ‚Π½ΠΎ Π² устройство.

ΠžΠ±Ρ€Π°Ρ‚ΠΈΡ‚Π΅ Π²Π½ΠΈΠΌΠ°Π½ΠΈΠ΅

Никогда Π½Π΅ ΠΏΠΎΠ΄ΠΊΠ»ΡŽΡ‡Π°ΠΉΡ‚Π΅ ΠΊ ΠΈΠ·ΠΌΠ΅Ρ€ΠΈΡ‚Π΅Π»ΡŽ заряТСнныС кондСнсаторы.

Π˜ΡΡ‚ΠΎΡ‡Π½ΠΈΠΊΠΈ:

  • Π‘ΠΏΡ€Π°Π²ΠΎΡ‡Π½ΠΈΠΊ ΠΏΠΎ систСмам обозначСния Смкости

Найти Π·Π½Π°Ρ‡Π΅Π½ΠΈΠ΅ элСктричСского заряда
ΠΌΠΎΠΆΠ½ΠΎ двумя способами. ΠŸΠ΅Ρ€Π²Ρ‹ΠΉ – ΠΈΠ·ΠΌΠ΅Ρ€ΠΈΡ‚ΡŒ силу взаимодСйствия нСизвСстного заряда
с извСстным ΠΈ с ΠΏΠΎΠΌΠΎΡ‰ΡŒΡŽ Π·Π°ΠΊΠΎΠ½Π° ΠšΡƒΠ»ΠΎΠ½Π° Ρ€Π°ΡΡΡ‡ΠΈΡ‚Π°Ρ‚ΡŒ Π΅Π³ΠΎ Π·Π½Π°Ρ‡Π΅Π½ΠΈΠ΅. Π’Ρ‚ΠΎΡ€ΠΎΠΉ – внСсти заряд Π² извСстноС элСктричСскоС ΠΏΠΎΠ»Π΅ ΠΈ ΠΈΠ·ΠΌΠ΅Ρ€ΠΈΡ‚ΡŒ силу, с ΠΊΠΎΡ‚ΠΎΡ€ΠΎΠΉ ΠΎΠ½ΠΎ дСйствуСт Π½Π° Π½Π΅Π³ΠΎ. Для измСрСния заряда
ΠΏΡ€ΠΎΡ‚Π΅ΠΊΠ°ΡŽΡ‰Π΅Π³ΠΎ Ρ‡Π΅Ρ€Π΅Π· ΠΏΠΎΠΏΠ΅Ρ€Π΅Ρ‡Π½ΠΎΠ΅ сСчСниС ΠΏΡ€ΠΎΠ²ΠΎΠ΄Π½ΠΈΠΊΠ° Π·Π° ΠΎΠΏΡ€Π΅Π΄Π΅Π»Π΅Π½Π½ΠΎΠ΅ врСмя ΠΈΠ·ΠΌΠ΅Ρ€ΡŒΡ‚Π΅ силу Ρ‚ΠΎΠΊΠ° ΠΈ ΡƒΠΌΠ½ΠΎΠΆΡŒΡ‚Π΅ Π΅Π΅ Π½Π° Π·Π½Π°Ρ‡Π΅Π½ΠΈΠ΅ Π²Ρ€Π΅ΠΌΠ΅Π½ΠΈ.

Π’Π°ΠΌ понадобится

  • Ρ‡ΡƒΠ²ΡΡ‚Π²ΠΈΡ‚Π΅Π»ΡŒΠ½Ρ‹ΠΉ Π΄ΠΈΠ½Π°ΠΌΠΎΠΌΠ΅Ρ‚Ρ€, сСкундомСр, Π°ΠΌΠΏΠ΅Ρ€ΠΌΠ΅Ρ‚Ρ€, ΠΈΠ·ΠΌΠ΅Ρ€ΠΈΡ‚Π΅Π»ΡŒ элСктростатичСского поля, Π²ΠΎΠ·Π΄ΡƒΡˆΠ½Ρ‹ΠΉ кондСнсатор.

Π˜Π½ΡΡ‚Ρ€ΡƒΠΊΡ†ΠΈΡ

Π˜Π·ΠΌΠ΅Ρ€Π΅Π½ΠΈΠ΅ заряда
ΠΏΡ€ΠΈ Π΅Π³ΠΎ с извСстным зарядомЕсли извСстСн ΠΎΠ΄Π½ΠΎΠ³ΠΎ Ρ‚Π΅Π»Π°, поднСситС ΠΊ Π½Π΅ΠΌΡƒ нСизвСстный заряд ΠΈ ΠΈΠ·ΠΌΠ΅Ρ€ΡŒΡ‚Π΅ ΠΌΠ΅ΠΆΠ΄Ρƒ Π½ΠΈΠΌΠΈ Π² ΠΌΠ΅Ρ‚Ρ€Π°Ρ…. Заряды Π½Π°Ρ‡Π½ΡƒΡ‚ Π²Π·Π°ΠΈΠΌΠΎΠ΄Π΅ΠΉΡΡ‚Π²ΠΎΠ²Π°Ρ‚ΡŒ. Π‘ ΠΏΠΎΠΌΠΎΡ‰ΡŒΡŽ Π΄ΠΈΠ½Π°ΠΌΠΎΠΌΠ΅Ρ‚Ρ€Π° ΠΈΠ·ΠΌΠ΅Ρ€ΡŒΡ‚Π΅ силу ΠΈΡ… взаимодСйствия. РассчитайтС Π·Π½Π°Ρ‡Π΅Π½ΠΈΠ΅ нСизвСстного заряда
— для этого ΠΊΠ²Π°Π΄Ρ€Π°Ρ‚ ΠΈΠ·ΠΌΠ΅Ρ€Π΅Π½Π½ΠΎΠ³ΠΎ расстояния ΡƒΠΌΠ½ΠΎΠΆΡŒΡ‚Π΅ Π½Π° Π·Π½Π°Ρ‡Π΅Π½ΠΈΠ΅ силы ΠΈ ΠΏΠΎΠ΄Π΅Π»ΠΈΡ‚Π΅ Π½Π° извСстный заряд. 9)). Если заряды ΠΎΡ‚Ρ‚Π°Π»ΠΊΠΈΠ²Π°ΡŽΡ‚ΡΡ, Ρ‚ΠΎ ΠΎΠ½ΠΈ ΠΎΠ΄Π½ΠΎΠΈΠΌΠ΅Π½Π½Ρ‹Π΅, Ссли ΠΆΠ΅ ΠΏΡ€ΠΈΡ‚ΡΠ³ΠΈΠ²Π°ΡŽΡ‚ΡΡ – Ρ€Π°Π·Π½ΠΎΠΈΠΌΠ΅Π½Π½Ρ‹Π΅.

Π˜Π·ΠΌΠ΅Ρ€Π΅Π½ΠΈΠ΅ значСния заряда
, внСсСнного Π² элСктричСскоС ΠΏΠΎΠ»Π΅Π˜Π·ΠΌΠ΅Ρ€ΡŒΡ‚Π΅ Π·Π½Π°Ρ‡Π΅Π½ΠΈΠ΅ постоянного элСктричСского поля ΡΠΏΠ΅Ρ†ΠΈΠ°Π»ΡŒΠ½Ρ‹ΠΌ ΠΏΡ€ΠΈΠ±ΠΎΡ€ΠΎΠΌ (ΠΈΠ·ΠΌΠ΅Ρ€ΠΈΡ‚Π΅Π»ΡŒ элСктричСского поля). Если Ρ‚Π°ΠΊΠΎΠ³ΠΎ ΠΏΡ€ΠΈΠ±ΠΎΡ€Π° Π½Π΅Ρ‚, Π²ΠΎΠ·ΡŒΠΌΠΈΡ‚Π΅ Π²ΠΎΠ·Π΄ΡƒΡˆΠ½Ρ‹ΠΉ кондСнсатор, зарядитС Π΅Π³ΠΎ, ΠΈΠ·ΠΌΠ΅Ρ€ΡŒΡ‚Π΅ напряТСниС Π½Π° Π΅Π³ΠΎ ΠΎΠ±ΠΊΠ»Π°Π΄ΠΊΠ°Ρ… ΠΈ ΠΏΠΎΠ΄Π΅Π»ΠΈΡ‚Π΅ Π½Π΅ расстояниС ΠΌΠ΅ΠΆΠ΄Ρƒ пластинами – это ΠΈ Π±ΡƒΠ΄Π΅Ρ‚ Π·Π½Π°Ρ‡Π΅Π½ΠΈΠ΅ элСктричСского поля Π²Π½ΡƒΡ‚Ρ€ΠΈ кондСнсатора Π² Π²ΠΎΠ»ΡŒΡ‚Π°Ρ… Π½Π° ΠΌΠ΅Ρ‚Ρ€. ВнСситС Π² ΠΏΠΎΠ»Π΅ нСизвСстный заряд. Π‘ ΠΏΠΎΠΌΠΎΡ‰ΡŒΡŽ Ρ‡ΡƒΠ²ΡΡ‚Π²ΠΈΡ‚Π΅Π»ΡŒΠ½ΠΎΠ³ΠΎ Π΄ΠΈΠ½Π°ΠΌΠΎΠΌΠ΅Ρ‚Ρ€Π° ΠΈΠ·ΠΌΠ΅Ρ€ΡŒΡ‚Π΅ силу, которая Π½Π° Π½Π΅Π³ΠΎ дСйствуСт. Π˜Π·ΠΌΠ΅Ρ€Π΅Π½ΠΈΠ΅ ΠΏΡ€ΠΎΠ²ΠΎΠ΄ΠΈΡ‚Π΅ Π² . ΠŸΠΎΠ΄Π΅Π»ΠΈΡ‚Π΅ Π·Π½Π°Ρ‡Π΅Π½ΠΈΠ΅ силы Π½Π° Π½Π°ΠΏΡ€ΡΠΆΠ΅Π½Π½ΠΎΡΡ‚ΡŒ элСктричСского поля. Π Π΅Π·ΡƒΠ»ΡŒΡ‚Π°Ρ‚ΠΎΠΌ Π±ΡƒΠ΄Π΅Ρ‚ Π·Π½Π°Ρ‡Π΅Π½ΠΈΠ΅ заряда
Π² ΠšΡƒΠ»ΠΎΠ½Π°Ρ… (q=F/Π•).

Π˜Π·ΠΌΠ΅Ρ€Π΅Π½ΠΈΠ΅ заряда
, ΠΏΡ€ΠΎΡ‚Π΅ΠΊΠ°ΡŽΡ‰Π΅Π³ΠΎ Ρ‡Π΅Ρ€Π΅Π· ΠΏΠΎΠΏΠ΅Ρ€Π΅Ρ‡Π½ΠΎΠ΅ ΠΏΡ€ΠΎΠ²ΠΎΠ΄Π½ΠΈΠΊΠ°Π‘ΠΎΠ±Π΅Ρ€ΠΈΡ‚Π΅ ΡΠ»Π΅ΠΊΡ‚Ρ€ΠΈΡ‡Π΅ΡΠΊΡƒΡŽ Ρ†Π΅ΠΏΡŒ с ΠΏΡ€ΠΎΠ²ΠΎΠ΄Π½ΠΈΠΊΠ°ΠΌΠΈ ΠΈ ΠΏΠΎΡΠ»Π΅Π΄ΠΎΠ²Π°Ρ‚Π΅Π»ΡŒΠ½ΠΎ ΠΏΠΎΠ΄ΠΊΠ»ΡŽΡ‡ΠΈΡ‚Π΅ ΠΊ Π½Π΅ΠΉ Π°ΠΌΠΏΠ΅Ρ€ΠΌΠ΅Ρ‚Ρ€. Π—Π°ΠΌΠΊΠ½ΠΈΡ‚Π΅ Π΅Π΅ Π½Π° источник Ρ‚ΠΎΠΊΠ° ΠΈ ΠΈΠ·ΠΌΠ΅Ρ€ΡŒΡ‚Π΅ силу Ρ‚ΠΎΠΊΠ° с ΠΏΠΎΠΌΠΎΡ‰ΡŒΡŽ Π°ΠΌΠΏΠ΅Ρ€ΠΌΠ΅Ρ‚Ρ€Π° Π² Π°ΠΌΠΏΠ΅Ρ€Π°Ρ…. ΠžΠ΄Π½ΠΎΠ²Ρ€Π΅ΠΌΠ΅Π½Π½ΠΎ сСкундомСром засСкитС , Π² ΠΊΠΎΡ‚ΠΎΡ€ΠΎΠ³ΠΎ Π² Ρ†Π΅ΠΏΠΈ Π±Ρ‹Π» элСктричСский Ρ‚ΠΎΠΊ. Π£ΠΌΠ½ΠΎΠΆΠΈΠ² Π·Π½Π°Ρ‡Π΅Π½ΠΈΠ΅ силы Ρ‚ΠΎΠΊΠ° Π½Π° ΠΏΠΎΠ»ΡƒΡ‡Π΅Π½Π½ΠΎΠ΅ врСмя, ΡƒΠ·Π½Π°ΠΉΡ‚Π΅ заряд, Ρ‡Π΅Ρ€Π΅Π· ΠΏΠΎΠΏΠ΅Ρ€Π΅Ρ‡Π½ΠΎΠ΅ сСчСниС ΠΊΠ°ΠΆΠ΄ΠΎΠ³ΠΎ Π·Π° это врСмя (q=I t). ΠŸΡ€ΠΈ измСрСниях слСдитС, Ρ‡Ρ‚ΠΎΠ±Ρ‹ ΠΏΡ€ΠΎΠ²ΠΎΠ΄Π½ΠΈΠΊΠΈ Π½Π΅ ΠΏΠ΅Ρ€Π΅Π³Ρ€Π΅Π²Π°Π»ΠΈΡΡŒ ΠΈ Π½Π΅ ΠΏΡ€ΠΎΠΈΠ·ΠΎΡˆΠ»ΠΎ ΠΊΠΎΡ€ΠΎΡ‚ΠΊΠΎΠ΅ Π·Π°ΠΌΡ‹ΠΊΠ°Π½ΠΈΠ΅.

ΠšΠΎΠ½Π΄Π΅Π½ΡΠ°Ρ‚ΠΎΡ€ΠΎΠΌ называСтся устройство, способноС Π½Π°ΠΊΠ°ΠΏΠ»ΠΈΠ²Π°Ρ‚ΡŒ элСктричСскиС заряды. ΠšΠΎΠ»ΠΈΡ‡Π΅ΡΡ‚Π²ΠΎ Π½Π°ΠΊΠ°ΠΏΠ»ΠΈΠ²Π°Π΅ΠΌΠΎΠΉ элСктричСской энСргии Π² кондСнсаторС характСризуСтся Π΅Π³ΠΎ Π΅ΠΌΠΊΠΎΡΡ‚ΡŒΡŽ
. Она измСряСтся Π² Ρ„Π°Ρ€Π°Π΄Π°Ρ…. БчитаСтся, Ρ‡Ρ‚ΠΎ Π΅ΠΌΠΊΠΎΡΡ‚ΡŒ Π² ΠΎΠ΄ΠΈΠ½ Ρ„Π°Ρ€Π°Π΄ соотвСтствуСт кондСнсатору, заряТСнному элСктричСским зарядом Π² ΠΎΠ΄ΠΈΠ½ ΠΊΡƒΠ»ΠΎΠ½ ΠΏΡ€ΠΈ разности ΠΏΠΎΡ‚Π΅Π½Ρ†ΠΈΠ°Π»ΠΎΠ² Π½Π° Π΅Π³ΠΎ ΠΎΠ±ΠΊΠ»Π°Π΄ΠΊΠ°Ρ… Π² ΠΎΠ΄ΠΈΠ½ Π²ΠΎΠ»ΡŒΡ‚.

Π˜Π½ΡΡ‚Ρ€ΡƒΠΊΡ†ΠΈΡ

ΠžΠΏΡ€Π΅Π΄Π΅Π»ΠΈΡ‚Π΅ Π΅ΠΌΠΊΠΎΡΡ‚ΡŒ плоского кондСнсатора
ΠΏΠΎ Ρ„ΠΎΡ€ΠΌΡƒΠ»Π΅ Π‘ = S e e0/d, Π³Π΄Π΅ S — ΠΏΠ»ΠΎΡ‰Π°Π΄ΡŒ повСрхности ΠΎΠ΄Π½ΠΎΠΉ пластины, d — ΠΌΠ΅ΠΆΠ΄Ρƒ пластинами, e — ΠΎΡ‚Π½ΠΎΡΠΈΡ‚Π΅Π»ΡŒΠ½Π°Ρ диэлСктричСская ΠΏΡ€ΠΎΠ½ΠΈΡ†Π°Π΅ΠΌΠΎΡΡ‚ΡŒ , Π·Π°ΠΏΠΎΠ»Π½ΡΡŽΡ‰Π΅ΠΉ пространство ΠΌΠ΅ΠΆΠ΄Ρƒ пластинами (Π² Π²Π°ΠΊΡƒΡƒΠΌΠ΅ ΠΎΠ½Π° Ρ€Π°Π²Π½Π° ), e0 — элСктричСская постоянная, равная 8,854187817 10(-12) Π€/ΠΌ. Π˜ΡΡ…ΠΎΠ΄Ρ ΠΈΠ· ΠΏΡ€ΠΈΠ²Π΅Π΄Π΅Π½Π½ΠΎΠΉ Ρ„ΠΎΡ€ΠΌΡƒΠ»Ρ‹, Π²Π΅Π»ΠΈΡ‡ΠΈΠ½Π° Смкости Π±ΡƒΠ΄Π΅Ρ‚ Π·Π°Π²ΠΈΡΠ΅Ρ‚ΡŒ ΠΎΡ‚ ΠΏΠ»ΠΎΡ‰Π°Π΄ΠΈ ΠΏΡ€ΠΎΠ²ΠΎΠ΄Π½ΠΈΠΊΠΎΠ², ΠΌΠ΅ΠΆΠ΄Ρƒ Π½ΠΈΠΌΠΈ ΠΈ ΠΎΡ‚ ΠΌΠ°Ρ‚Π΅Ρ€ΠΈΠ°Π»Π° диэлСктрика. Π’ качСствС диэлСктрика ΠΌΠΎΠΆΠ΅Ρ‚ ΠΏΡ€ΠΈΠΌΠ΅Π½ΡΡ‚ΡŒΡΡ ΠΈΠ»ΠΈ слюда.

ВычислитС Π΅ΠΌΠΊΠΎΡΡ‚ΡŒ сфСричСского кондСнсатора
ΠΏΠΎ Ρ„ΠΎΡ€ΠΌΡƒΠ»Π΅ Π‘ = (4П e0 RΒ²)/d, Π³Π΄Π΅ П — число Β«ΠΏΠΈΒ», R — радиус сфСры, d — Π²Π΅Π»ΠΈΡ‡ΠΈΠ½Π° Π·Π°Π·ΠΎΡ€Π° ΠΌΠ΅ΠΆΠ΄Ρƒ Π΅Π³ΠΎ сфСрами.Π’Π΅Π»ΠΈΡ‡ΠΈΠ½Π° Смкости сфСричСского кондСнсатора
прямо ΠΏΡ€ΠΎΠΏΠΎΡ€Ρ†ΠΈΠΎΠ½Π°Π»ΡŒΠ½Π° концСнтричСской сфСры ΠΈ ΠΎΠ±Ρ€Π°Ρ‚Π½ΠΎ ΠΏΡ€ΠΎΠΏΠΎΡ€Ρ†ΠΈΠΎΠ½Π°Π»ΡŒΠ½Π° Ρ€Π°ΡΡΡ‚ΠΎΡΠ½ΠΈΡŽ ΠΌΠ΅ΠΆΠ΄Ρƒ сфСрами.

РассчитайтС Π΅ΠΌΠΊΠΎΡΡ‚ΡŒ цилиндричСского кондСнсатора
ΠΏΠΎ Ρ„ΠΎΡ€ΠΌΡƒΠ»Π΅ Π‘ = (2П e e0 L R1)/(R2-R1), Π³Π΄Π΅ L — Π΄Π»ΠΈΠ½Π° кондСнсатора
, П — число Β«ΠΏΠΈΒ», R1 ΠΈ R2 — радиусы Π΅Π³ΠΎ цилиндричСских ΠΎΠ±ΠΊΠ»Π°Π΄ΠΎΠΊ.

Если кондСнсаторы Π² Ρ†Π΅ΠΏΠΈ соСдинСны ΠΏΠ°Ρ€Π°Π»Π»Π΅Π»ΡŒΠ½ΠΎ, рассчитайтС ΠΈΡ… ΠΎΠ±Ρ‰ΡƒΡŽ Π΅ΠΌΠΊΠΎΡΡ‚ΡŒ ΠΏΠΎ Ρ„ΠΎΡ€ΠΌΡƒΠ»Π΅ Π‘ = Π‘1+Π‘2+…+Π‘n, Π³Π΄Π΅ Π‘1, Π‘2,…Бn – Смкости ΠΏΠ°Ρ€Π°Π»Π»Π΅Π»ΡŒΠ½ΠΎ соСдинСнных кондСнсаторов.

ВычислитС ΠΎΠ±Ρ‰ΡƒΡŽ Π΅ΠΌΠΊΠΎΡΡ‚ΡŒ ΠΏΠΎΡΠ»Π΅Π΄ΠΎΠ²Π°Ρ‚Π΅Π»ΡŒΠ½ΠΎ соСдинСнных кондСнсаторов ΠΏΠΎ Ρ„ΠΎΡ€ΠΌΡƒΠ»Π΅ 1/Π‘ = 1/Π‘1+1/Π‘2+…+1/Π‘n, Π³Π΄Π΅ Π‘1, Π‘2,…Бn — Смкости ΠΏΠΎΡΠ»Π΅Π΄ΠΎΠ²Π°Ρ‚Π΅Π»ΡŒΠ½ΠΎ соСдинСнных кондСнсаторов.

ΠžΠ±Ρ€Π°Ρ‚ΠΈΡ‚Π΅ Π²Π½ΠΈΠΌΠ°Π½ΠΈΠ΅

На любом кондСнсаторС ΠΎΠ±ΡΠ·Π°Ρ‚Π΅Π»ΡŒΠ½ΠΎ Π΄ΠΎΠ»ΠΆΠ½Π° Π±Ρ‹Ρ‚ΡŒ нанСсСна ΠΌΠ°Ρ€ΠΊΠΈΡ€ΠΎΠ²ΠΊΠ°, которая ΠΌΠΎΠΆΠ΅Ρ‚ Π±Ρ‹Ρ‚ΡŒ Π±ΡƒΠΊΠ²Π΅Π½Π½ΠΎ-цифровая ΠΈΠ»ΠΈ цвСтовая. ΠœΠ°Ρ€ΠΊΠΈΡ€ΠΎΠ²ΠΊΠ° ΠΎΡ‚Ρ€Π°ΠΆΠ°Π΅Ρ‚ Π΅Π³ΠΎ ΠΏΠ°Ρ€Π°ΠΌΠ΅Ρ‚Ρ€Ρ‹.

Π˜ΡΡ‚ΠΎΡ‡Π½ΠΈΠΊΠΈ:

  • ЦвСтовая ΠΌΠ°Ρ€ΠΊΠΈΡ€ΠΎΠ²ΠΊΠ° рСзисторов, кондСнсаторов ΠΈ индуктивностСй

Π•ΠΌΠΊΠΎΡΡ‚ΡŒ – Π²Π΅Π»ΠΈΡ‡ΠΈΠ½Π°, Π² систСмС БИ выраТаСмая Π² Ρ„Π°Ρ€Π°Π΄Π°Ρ…. Π₯отя ΠΈΡΠΏΠΎΠ»ΡŒΠ·ΡƒΡŽΡ‚ΡΡ, фактичСски, лишь ΠΏΡ€ΠΎΠΈΠ·Π²ΠΎΠ΄Π½Ρ‹Π΅ ΠΎΡ‚ Π½Π΅Π΅ – ΠΌΠΈΠΊΡ€ΠΎΡ„Π°Ρ€Π°Π΄Ρ‹, ΠΏΠΈΠΊΠΎΡ„Π°Ρ€Π°Π΄Ρ‹ ΠΈ Ρ‚Π°ΠΊ Π΄Π°Π»Π΅Π΅. Π§Ρ‚ΠΎ касаСтся элСктроСмкости плоского кондСнсатора, ΠΎΠ½Π° зависит ΠΎΡ‚ Π·Π°Π·ΠΎΡ€Π° ΠΌΠ΅ΠΆ ΠΎΠ±ΠΊΠ»Π°Π΄ΠΎΠΊ ΠΈ ΠΈΡ… ΠΏΠ»ΠΎΡ‰Π°Π΄ΠΈ, ΠΎΡ‚ Π²ΠΈΠ΄Π° диэлСктрика, Π² Π΄Π°Π½Π½ΠΎΠΌ Π·Π°Π·ΠΎΡ€Π΅ располоТСнного.

Π˜Π½ΡΡ‚Ρ€ΡƒΠΊΡ†ΠΈΡ

Π’ Ρ‚ΠΎΠΌ случаС, Ссли ΠΎΠ±ΠΊΠ»Π°Π΄ΠΊΠΈ кондСнсатора ΠΈΠΌΠ΅ΡŽΡ‚ ΠΎΠ΄ΠΈΠ½Π°ΠΊΠΎΠ²ΡƒΡŽ ΠΏΠ»ΠΎΡ‰Π°Π΄ΡŒ ΠΈ ΠΈΠΌΠ΅ΡŽΡ‚ располоТСниС строго ΠΎΠ΄Π½Π° Π½Π°Π΄ Π΄Ρ€ΡƒΠ³ΠΎΠΉ, рассчитайтС ΠΏΠ»ΠΎΡ‰Π°Π΄ΡŒ ΠΎΠ΄Π½ΠΎΠΉ ΠΈΠ· ΠΎΠ±ΠΊΠ»Π°Π΄ΠΎΠΊ – любой. Если ΠΆΠ΅ ΠΎΠ΄Π½Π° ΠΈΠ· Π½ΠΈΡ… ΠΎΡ‚Π½ΠΎΡΠΈΡ‚Π΅Π»ΡŒΠ½ΠΎ Π΄Ρ€ΡƒΠ³ΠΎΠΉ смСщСна Π»ΠΈΠ±ΠΎ ΠΎΠ½ΠΈ Ρ€Π°Π·Π½Ρ‹Π΅ , Π½ΡƒΠΆΠ½ΠΎ Ρ€Π°ΡΡΡ‡ΠΈΡ‚Ρ‹Π²Π°Ρ‚ΡŒ ΠΏΠ»ΠΎΡ‰Π°Π΄ΡŒ области, Π² ΠΊΠΎΡ‚ΠΎΡ€ΠΎΠΉ ΠΎΠ±ΠΊΠ»Π°Π΄ΠΊΠΈ Π΄Ρ€ΡƒΠ³ Π΄Ρ€ΡƒΠΆΠΊΡƒ ΠΏΠ΅Ρ€Π΅ΠΊΡ€Ρ‹Π²Π°ΡŽΡ‚.

ΠŸΡ€ΠΈ этом ΠΈΡΠΏΠΎΠ»ΡŒΠ·ΡƒΡŽΡ‚ΡΡ общСпринятыС Ρ„ΠΎΡ€ΠΌΡƒΠ»Ρ‹, Ρ€Π°ΡΡΡ‡ΠΈΡ‚Ρ‹Π²Π°Ρ‚ΡŒ ΠΏΠ»ΠΎΡ‰Π°Π΄ΠΈ Ρ‚Π°ΠΊΠΈΡ… гСомСтричСских Ρ„ΠΈΠ³ΡƒΡ€, ΠΊΠ°ΠΊ ΠΊΡ€ΡƒΠ³ (S=Ο€(R^2)), ΠΏΡ€ΡΠΌΠΎΡƒΠ³ΠΎΠ»ΡŒΠ½ΠΈΠΊΠ° (S=ab), Π΅Π³ΠΎ частного случая – ΠΊΠ²Π°Π΄Ρ€Π°Ρ‚Π° (S=a^2) – ΠΈ Π΄Ρ€ΡƒΠ³ΠΈΡ…. (-12) Π€/ΠΌ ΠΈ являСтся, ΠΏΠΎ сути, диэлСктричСской ΠΏΡ€ΠΎΠ½ΠΈΡ†Π°Π΅ΠΌΠΎΡΡ‚ΡŒΡŽ Π²Π°ΠΊΡƒΡƒΠΌΠ°.

8.1 ΠšΠΎΠ½Π΄Π΅Π½ΡΠ°Ρ‚ΠΎΡ€Ρ‹ ΠΈ Π΅ΠΌΠΊΠΎΡΡ‚ΡŒ — University Physics Volume 2

Π¦Π΅Π»ΠΈ обучСния

К ΠΊΠΎΠ½Ρ†Ρƒ этого Ρ€Π°Π·Π΄Π΅Π»Π° Π²Ρ‹ смоТСтС:

  • ΠžΠ±ΡŠΡΡΠ½ΠΈΡ‚Π΅ понятиС кондСнсатора ΠΈ Π΅Π³ΠΎ Смкости
  • ΠžΠΏΠΈΡˆΠΈΡ‚Π΅, ΠΊΠ°ΠΊ ΠΎΡ†Π΅Π½ΠΈΡ‚ΡŒ Π΅ΠΌΠΊΠΎΡΡ‚ΡŒ систСмы ΠΏΡ€ΠΎΠ²ΠΎΠ΄ΠΎΠ²

ΠšΠΎΠ½Π΄Π΅Π½ΡΠ°Ρ‚ΠΎΡ€ — это устройство, ΠΈΡΠΏΠΎΠ»ΡŒΠ·ΡƒΠ΅ΠΌΠΎΠ΅ для хранСния элСктричСского заряда ΠΈ элСктричСской энСргии. ΠšΠΎΠ½Π΄Π΅Π½ΡΠ°Ρ‚ΠΎΡ€Ρ‹ ΠΎΠ±Ρ‹Ρ‡Π½ΠΎ состоят ΠΈΠ· Π΄Π²ΡƒΡ… элСктричСских ΠΏΡ€ΠΎΠ²ΠΎΠ΄Π½ΠΈΠΊΠΎΠ², Ρ€Π°Π·Π΄Π΅Π»Π΅Π½Π½Ρ‹Ρ… расстояниСм.(ΠžΠ±Ρ€Π°Ρ‚ΠΈΡ‚Π΅ Π²Π½ΠΈΠΌΠ°Π½ΠΈΠ΅, Ρ‡Ρ‚ΠΎ Ρ‚Π°ΠΊΠΈΠ΅ элСктричСскиС ΠΏΡ€ΠΎΠ²ΠΎΠ΄Π½ΠΈΠΊΠΈ ΠΈΠ½ΠΎΠ³Π΄Π° Π½Π°Π·Ρ‹Π²Π°ΡŽΡ‚ «элСктродами», Π½ΠΎ, Ρ‚ΠΎΡ‡Π½Π΅Π΅, ΠΎΠ½ΠΈ Β«ΠΎΠ±ΠΊΠ»Π°Π΄ΠΊΠΈ кондСнсатора».) ΠŸΡ€ΠΎΡΡ‚Ρ€Π°Π½ΡΡ‚Π²ΠΎ ΠΌΠ΅ΠΆΠ΄Ρƒ кондСнсаторами ΠΌΠΎΠΆΠ΅Ρ‚ Π±Ρ‹Ρ‚ΡŒ просто Π²Π°ΠΊΡƒΡƒΠΌΠΎΠΌ, ΠΈ Π² этом случаС кондСнсатор Π±ΡƒΠ΄Π΅Ρ‚ извСстСн ΠΊΠ°ΠΊ Β«Π’Π°ΠΊΡƒΡƒΠΌΠ½Ρ‹ΠΉ кондСнсатор». Однако пространство ΠΎΠ±Ρ‹Ρ‡Π½ΠΎ заполняСтся ΠΈΠ·ΠΎΠ»ΠΈΡ€ΡƒΡŽΡ‰ΠΈΠΌ ΠΌΠ°Ρ‚Π΅Ρ€ΠΈΠ°Π»ΠΎΠΌ, извСстным ΠΊΠ°ΠΊ диэлСктрик. (Π’Ρ‹ ΡƒΠ·Π½Π°Π΅Ρ‚Π΅ большС ΠΎ диэлСктриках Π² Ρ€Π°Π·Π΄Π΅Π»Π°Ρ…, посвящСнных диэлСктрикам, Π΄Π°Π»Π΅Π΅ Π² этой Π³Π»Π°Π²Π΅.) ОбъСм накопитСля Π² кондСнсаторС опрСдСляСтся свойством, Π½Π°Π·Ρ‹Π²Π°Π΅ΠΌΡ‹ΠΌ Π΅ΠΌΠΊΠΎΡΡ‚ΡŒΡŽ , , ΠΎ ΠΊΠΎΡ‚ΠΎΡ€ΠΎΠΌ Π²Ρ‹ ΡƒΠ·Π½Π°Π΅Ρ‚Π΅ большС Ρ‡ΡƒΡ‚ΡŒ ΠΏΠΎΠ·ΠΆΠ΅ Π² этом Ρ€Π°Π·Π΄Π΅Π»Π΅.

ΠšΠΎΠ½Π΄Π΅Π½ΡΠ°Ρ‚ΠΎΡ€Ρ‹

ΠΈΠΌΠ΅ΡŽΡ‚ Ρ€Π°Π·Π»ΠΈΡ‡Π½Ρ‹Π΅ примСнСния: ΠΎΡ‚ Ρ„ΠΈΠ»ΡŒΡ‚Ρ€Π°Ρ†ΠΈΠΈ статичСского элСктричСства, Ρ€Π°Π΄ΠΈΠΎΠΏΡ€ΠΈΠ΅ΠΌΠ° Π΄ΠΎ накоплСния энСргии Π² дСфибрилляторах сСрдца. ΠžΠ±Ρ‹Ρ‡Π½ΠΎ Π² ΠΏΡ€ΠΎΠΌΡ‹ΡˆΠ»Π΅Π½Π½Ρ‹Ρ… кондСнсаторах Π΄Π²Π΅ токопроводящиС части располоТСны Π±Π»ΠΈΠ·ΠΊΠΎ Π΄Ρ€ΡƒΠ³ ΠΊ Π΄Ρ€ΡƒΠ³Ρƒ, Π½ΠΎ Π½Π΅ ΡΠΎΠΏΡ€ΠΈΠΊΠ°ΡΠ°ΡŽΡ‚ΡΡ, ΠΊΠ°ΠΊ Π½Π° рис. 8.2. Π’ Π±ΠΎΠ»ΡŒΡˆΠΈΠ½ΡΡ‚Π²Π΅ случаСв ΠΌΠ΅ΠΆΠ΄Ρƒ двумя пластинами ΠΈΡΠΏΠΎΠ»ΡŒΠ·ΡƒΠ΅Ρ‚ΡΡ диэлСктрик. Когда ΠΊΠ»Π΅ΠΌΠΌΡ‹ Π±Π°Ρ‚Π°Ρ€Π΅ΠΈ ΠΏΠΎΠ΄ΠΊΠ»ΡŽΡ‡Π΅Π½Ρ‹ ΠΊ ΠΏΠ΅Ρ€Π²ΠΎΠ½Π°Ρ‡Π°Π»ΡŒΠ½ΠΎ нСзаряТСнному кондСнсатору, ΠΏΠΎΡ‚Π΅Π½Ρ†ΠΈΠ°Π» Π±Π°Ρ‚Π°Ρ€Π΅ΠΈ ΠΏΠ΅Ρ€Π΅ΠΌΠ΅Ρ‰Π°Π΅Ρ‚ нСбольшой заряд Π²Π΅Π»ΠΈΡ‡ΠΈΠ½ΠΎΠΉ Q с ΠΏΠΎΠ»ΠΎΠΆΠΈΡ‚Π΅Π»ΡŒΠ½ΠΎΠΉ пластины Π½Π° ΠΎΡ‚Ρ€ΠΈΡ†Π°Ρ‚Π΅Π»ΡŒΠ½ΡƒΡŽ.ΠšΠΎΠ½Π΄Π΅Π½ΡΠ°Ρ‚ΠΎΡ€ Π² Ρ†Π΅Π»ΠΎΠΌ остаСтся Π½Π΅ΠΉΡ‚Ρ€Π°Π»ΡŒΠ½Ρ‹ΠΌ, Π½ΠΎ с зарядами + Q + Q ΠΈ βˆ’Q βˆ’ Q, располоТСнными Π½Π° ΠΏΡ€ΠΎΡ‚ΠΈΠ²ΠΎΠΏΠΎΠ»ΠΎΠΆΠ½Ρ‹Ρ… пластинах.

Π€ΠΈΠ³ΡƒΡ€Π°
8,2

Оба кондСнсатора, ΠΏΠΎΠΊΠ°Π·Π°Π½Π½Ρ‹Π΅ здСсь, Π±Ρ‹Π»ΠΈ ΠΈΠ·Π½Π°Ρ‡Π°Π»ΡŒΠ½ΠΎ разряТСны ΠΏΠ΅Ρ€Π΅Π΄ ΠΏΠΎΠ΄ΠΊΠ»ΡŽΡ‡Π΅Π½ΠΈΠ΅ΠΌ ΠΊ Π±Π°Ρ‚Π°Ρ€Π΅Π΅. Π’Π΅ΠΏΠ΅Ρ€ΡŒ Ρƒ Π½ΠΈΡ… Π½Π° пластинах Π΅ΡΡ‚ΡŒ заряды + Q + Q ΠΈ βˆ’Q βˆ’ Q (соотвСтствСнно). (a) ΠšΠΎΠ½Π΄Π΅Π½ΡΠ°Ρ‚ΠΎΡ€ с ΠΏΠ°Ρ€Π°Π»Π»Π΅Π»ΡŒΠ½Ρ‹ΠΌΠΈ пластинами состоит ΠΈΠ· Π΄Π²ΡƒΡ… пластин ΠΏΡ€ΠΎΡ‚ΠΈΠ²ΠΎΠΏΠΎΠ»ΠΎΠΆΠ½ΠΎΠ³ΠΎ заряда с ΠΏΠ»ΠΎΡ‰Π°Π΄ΡŒΡŽ A, , Ρ€Π°Π·Π΄Π΅Π»Π΅Π½Π½Ρ‹Ρ… расстояниСм d . (b) ΠšΠ°Ρ‚Π°Π½Ρ‹ΠΉ кондСнсатор ΠΈΠΌΠ΅Π΅Ρ‚ диэлСктричСский ΠΌΠ°Ρ‚Π΅Ρ€ΠΈΠ°Π» ΠΌΠ΅ΠΆΠ΄Ρƒ двумя проводящими листами (пластинами).

БистСма, состоящая ΠΈΠ· Π΄Π²ΡƒΡ… ΠΈΠ΄Π΅Π½Ρ‚ΠΈΡ‡Π½Ρ‹Ρ… ΠΏΠ°Ρ€Π°Π»Π»Π΅Π»ΡŒΠ½ΠΎ проводящих пластин, Ρ€Π°Π·Π΄Π΅Π»Π΅Π½Π½Ρ‹Ρ… расстояниСм, называСтся кондСнсатором с ΠΏΠ°Ρ€Π°Π»Π»Π΅Π»ΡŒΠ½Ρ‹ΠΌΠΈ пластинами (рис. 8.3). Π’Π΅Π»ΠΈΡ‡ΠΈΠ½Π° элСктричСского поля Π² пространствС ΠΌΠ΅ΠΆΠ΄Ρƒ ΠΏΠ°Ρ€Π°Π»Π»Π΅Π»ΡŒΠ½Ρ‹ΠΌΠΈ пластинами составляСт E = Οƒ / Ξ΅0E = Οƒ / Ξ΅0, Π³Π΄Π΅ σσ ΠΎΠ±ΠΎΠ·Π½Π°Ρ‡Π°Π΅Ρ‚ ΠΏΠΎΠ²Π΅Ρ€Ρ…Π½ΠΎΡΡ‚Π½ΡƒΡŽ ΠΏΠ»ΠΎΡ‚Π½ΠΎΡΡ‚ΡŒ заряда Π½Π° ΠΎΠ΄Π½ΠΎΠΉ пластинС (Π½Π°ΠΏΠΎΠΌΠ½ΠΈΠΌ, Ρ‡Ρ‚ΠΎ σσ — это заряд Q Π½Π° ΠΏΠ»ΠΎΡ‰Π°Π΄ΡŒ повСрхности A ). Π’Π°ΠΊΠΈΠΌ ΠΎΠ±Ρ€Π°Π·ΠΎΠΌ, Π²Π΅Π»ΠΈΡ‡ΠΈΠ½Π° поля прямо ΠΏΡ€ΠΎΠΏΠΎΡ€Ρ†ΠΈΠΎΠ½Π°Π»ΡŒΠ½Π° Q .

Π€ΠΈΠ³ΡƒΡ€Π°
8,3

Π Π°Π·Π΄Π΅Π»Π΅Π½ΠΈΠ΅ зарядов Π² кондСнсаторС ΠΏΠΎΠΊΠ°Π·Ρ‹Π²Π°Π΅Ρ‚, Ρ‡Ρ‚ΠΎ заряды ΠΎΡΡ‚Π°ΡŽΡ‚ΡΡ Π½Π° повСрхности пластин кондСнсатора.Π›ΠΈΠ½ΠΈΠΈ элСктричСского поля Π² кондСнсаторС с ΠΏΠ°Ρ€Π°Π»Π»Π΅Π»ΡŒΠ½Ρ‹ΠΌΠΈ пластинами Π½Π°Ρ‡ΠΈΠ½Π°ΡŽΡ‚ΡΡ с ΠΏΠΎΠ»ΠΎΠΆΠΈΡ‚Π΅Π»ΡŒΠ½Ρ‹Ρ… зарядов ΠΈ Π·Π°ΠΊΠ°Π½Ρ‡ΠΈΠ²Π°ΡŽΡ‚ΡΡ ΠΎΡ‚Ρ€ΠΈΡ†Π°Ρ‚Π΅Π»ΡŒΠ½Ρ‹ΠΌΠΈ зарядами. Π’Π΅Π»ΠΈΡ‡ΠΈΠ½Π° элСктричСского поля Π² пространствС ΠΌΠ΅ΠΆΠ΄Ρƒ пластинами прямо ΠΏΡ€ΠΎΠΏΠΎΡ€Ρ†ΠΈΠΎΠ½Π°Π»ΡŒΠ½Π° количСству заряда Π½Π° кондСнсаторС.

ΠšΠΎΠ½Π΄Π΅Π½ΡΠ°Ρ‚ΠΎΡ€Ρ‹ с Ρ€Π°Π·Π½Ρ‹ΠΌΠΈ физичСскими характСристиками (Ρ‚Π°ΠΊΠΈΠΌΠΈ ΠΊΠ°ΠΊ Ρ„ΠΎΡ€ΠΌΠ° ΠΈ Ρ€Π°Π·ΠΌΠ΅Ρ€ пластин) Π½Π°ΠΊΠ°ΠΏΠ»ΠΈΠ²Π°ΡŽΡ‚ Ρ€Π°Π·Π½ΠΎΠ΅ количСство заряда для ΠΎΠ΄Π½ΠΎΠ³ΠΎ ΠΈ Ρ‚ΠΎΠ³ΠΎ ΠΆΠ΅ ΠΏΡ€ΠΈΠ»ΠΎΠΆΠ΅Π½Π½ΠΎΠ³ΠΎ напряТСния Π’ Π½Π° своих пластинах. Π•ΠΌΠΊΠΎΡΡ‚ΡŒ C кондСнсатора опрСдСляСтся ΠΊΠ°ΠΊ ΠΎΡ‚Π½ΠΎΡˆΠ΅Π½ΠΈΠ΅ максимального заряда Q , ΠΊΠΎΡ‚ΠΎΡ€Ρ‹ΠΉ ΠΌΠΎΠΆΠ΅Ρ‚ Ρ…Ρ€Π°Π½ΠΈΡ‚ΡŒΡΡ Π² кондСнсаторС, ΠΊ ΠΏΡ€ΠΈΠ»ΠΎΠΆΠ΅Π½Π½ΠΎΠΌΡƒ Π½Π°ΠΏΡ€ΡΠΆΠ΅Π½ΠΈΡŽ Π’, Π½Π° Π΅Π³ΠΎ пластинах.Π”Ρ€ΡƒΠ³ΠΈΠΌΠΈ словами, Π΅ΠΌΠΊΠΎΡΡ‚ΡŒ — это наибольшая Π²Π΅Π»ΠΈΡ‡ΠΈΠ½Π° заряда Π½Π° Π²ΠΎΠ»ΡŒΡ‚, которая ΠΌΠΎΠΆΠ΅Ρ‚ Ρ…Ρ€Π°Π½ΠΈΡ‚ΡŒΡΡ Π½Π° устройствС:

Π•Π΄ΠΈΠ½ΠΈΡ†Π° измСрСния Смкости Π² систСмС БИ — Ρ„Π°Ρ€Π°Π΄ (Π€), названная Π² Ρ‡Π΅ΡΡ‚ΡŒ Майкла ЀарадСя (1791–1867). ΠŸΠΎΡΠΊΠΎΠ»ΡŒΠΊΡƒ Π΅ΠΌΠΊΠΎΡΡ‚ΡŒ — это заряд Π½Π° Π΅Π΄ΠΈΠ½ΠΈΡ†Ρƒ напряТСния, ΠΎΠ΄ΠΈΠ½ Ρ„Π°Ρ€Π°Π΄ Ρ€Π°Π²Π΅Π½ ΠΎΠ΄Π½ΠΎΠΌΡƒ ΠΊΡƒΠ»ΠΎΠ½Ρƒ Π½Π° ΠΎΠ΄ΠΈΠ½ Π²ΠΎΠ»ΡŒΡ‚, ΠΈΠ»ΠΈ

.

По ΠΎΠΏΡ€Π΅Π΄Π΅Π»Π΅Π½ΠΈΡŽ, кондСнсатор Π΅ΠΌΠΊΠΎΡΡ‚ΡŒΡŽ 1,0 ΠΌΠΊΠ€ ΠΌΠΎΠΆΠ΅Ρ‚ ΡΠΎΡ…Ρ€Π°Π½ΡΡ‚ΡŒ заряд 1,0 К (ΠΎΡ‡Π΅Π½ΡŒ большой заряд), ΠΊΠΎΠ³Π΄Π° Ρ€Π°Π·Π½ΠΎΡΡ‚ΡŒ ΠΏΠΎΡ‚Π΅Π½Ρ†ΠΈΠ°Π»ΠΎΠ² ΠΌΠ΅ΠΆΠ΄Ρƒ Π΅Π³ΠΎ пластинами составляСт всСго 1,0 Π’. Π‘Π»Π΅Π΄ΠΎΠ²Π°Ρ‚Π΅Π»ΡŒΠ½ΠΎ, ΠΎΠ΄ΠΈΠ½ Ρ„Π°Ρ€Π°Π΄ являСтся ΠΎΡ‡Π΅Π½ΡŒ большой Π΅ΠΌΠΊΠΎΡΡ‚ΡŒΡŽ. Π’ΠΈΠΏΠΈΡ‡Π½Ρ‹Π΅ значСния Смкости Π²Π°Ρ€ΡŒΠΈΡ€ΡƒΡŽΡ‚ΡΡ ΠΎΡ‚ ΠΏΠΈΠΊΠΎΡ„Π°Ρ€Π°Π΄ (1 ΠΏΠ€ = 10-12 Π€) (1 ΠΏΠ€ = 10-12 Π€) Π΄ΠΎ ΠΌΠΈΠ»Π»ΠΈΡ„Π°Ρ€Π°Π΄ΠΎΠ² (1 ΠΌΠ€ = 10-3 Π€) (1 ΠΌΠ€ = 10-3 Π€), Π²ΠΊΠ»ΡŽΡ‡Π°Ρ ΠΌΠΈΠΊΡ€ΠΎΡ„Π°Ρ€Π°Π΄Ρ‹ (1 ΠΌΠΊΠ€ = 10-6 Π€ 1 ΠΌΠΊΠ€ = 10- 6F). ΠšΠΎΠ½Π΄Π΅Π½ΡΠ°Ρ‚ΠΎΡ€Ρ‹ ΠΌΠΎΠ³ΡƒΡ‚ Π±Ρ‹Ρ‚ΡŒ Ρ€Π°Π·Π½Ρ‹Ρ… Ρ„ΠΎΡ€ΠΌ ΠΈ Ρ€Π°Π·ΠΌΠ΅Ρ€ΠΎΠ² (рис. 8.4).

Π€ΠΈΠ³ΡƒΡ€Π°
8,4

Π­Ρ‚ΠΎ Π½Π΅ΠΊΠΎΡ‚ΠΎΡ€Ρ‹Π΅ Ρ‚ΠΈΠΏΠΈΡ‡Π½Ρ‹Π΅ кондСнсаторы, ΠΈΡΠΏΠΎΠ»ΡŒΠ·ΡƒΠ΅ΠΌΡ‹Π΅ Π² элСктронных устройствах. Π Π°Π·ΠΌΠ΅Ρ€ кондСнсатора Π½Π΅ ΠΎΠ±ΡΠ·Π°Ρ‚Π΅Π»ΡŒΠ½ΠΎ зависит ΠΎΡ‚ Π΅Π³ΠΎ Смкости. (Π˜ΡΡ‚ΠΎΡ‡Π½ΠΈΠΊ: Windell Oskay)

РасчСт Смкости

ΠœΡ‹ ΠΌΠΎΠΆΠ΅ΠΌ Ρ€Π°ΡΡΡ‡ΠΈΡ‚Π°Ρ‚ΡŒ Π΅ΠΌΠΊΠΎΡΡ‚ΡŒ ΠΏΠ°Ρ€Ρ‹ ΠΏΡ€ΠΎΠ²ΠΎΠ΄ΠΎΠ² с ΠΏΠΎΠΌΠΎΡ‰ΡŒΡŽ ΡΠ»Π΅Π΄ΡƒΡŽΡ‰Π΅Π³ΠΎ стандартного ΠΏΠΎΠ΄Ρ…ΠΎΠ΄Π°.

БтратСгия Ρ€Π΅ΡˆΠ΅Π½ΠΈΡ ΠΏΡ€ΠΎΠ±Π»Π΅ΠΌ

РасчСт Смкости
  1. ΠŸΡ€Π΅Π΄ΠΏΠΎΠ»ΠΎΠΆΠΈΠΌ, Ρ‡Ρ‚ΠΎ кондСнсатор ΠΈΠΌΠ΅Π΅Ρ‚ заряд Q .
  2. ΠžΠΏΡ€Π΅Π΄Π΅Π»ΠΈΡ‚ΡŒ элСктричСскоС ΠΏΠΎΠ»Π΅ E β†’ E β†’ ΠΌΠ΅ΠΆΠ΄Ρƒ ΠΏΡ€ΠΎΠ²ΠΎΠ΄Π½ΠΈΠΊΠ°ΠΌΠΈ. Если Π² располоТСнии ΠΏΡ€ΠΎΠ²ΠΎΠ΄Π½ΠΈΠΊΠΎΠ² присутствуСт симмСтрия, Π²Ρ‹ ΠΌΠΎΠΆΠ΅Ρ‚Π΅ ΠΈΡΠΏΠΎΠ»ΡŒΠ·ΠΎΠ²Π°Ρ‚ΡŒ Π·Π°ΠΊΠΎΠ½ Гаусса для этого расчСта.
  3. НайдитС Ρ€Π°Π·Π½ΠΎΡΡ‚ΡŒ ΠΏΠΎΡ‚Π΅Π½Ρ†ΠΈΠ°Π»ΠΎΠ² ΠΌΠ΅ΠΆΠ΄Ρƒ ΠΏΡ€ΠΎΠ²ΠΎΠ΄Π½ΠΈΠΊΠ°ΠΌΠΈ ΠΈΠ·
    VB βˆ’ VA = βˆ’ABE β†’ Β· dl β†’, VB βˆ’ VA = βˆ’ABE β†’ Β· dl β†’,

    8,2

    Π³Π΄Π΅ ΠΏΡƒΡ‚ΡŒ интСгрирования Π²Π΅Π΄Π΅Ρ‚ ΠΎΡ‚ ΠΎΠ΄Π½ΠΎΠ³ΠΎ ΠΏΡ€ΠΎΠ²ΠΎΠ΄Π½ΠΈΠΊΠ° ΠΊ Π΄Ρ€ΡƒΠ³ΠΎΠΌΡƒ. Π’ΠΎΠ³Π΄Π° Π²Π΅Π»ΠΈΡ‡ΠΈΠ½Π° разности ΠΏΠΎΡ‚Π΅Π½Ρ†ΠΈΠ°Π»ΠΎΠ² Ρ€Π°Π²Π½Π° V = | VB-VA | V = | VB-VA |.

  4. Зная Π’, , ΠΎΠΏΡ€Π΅Π΄Π΅Π»ΠΈΡ‚Π΅ Π΅ΠΌΠΊΠΎΡΡ‚ΡŒ нСпосрСдствСнно ΠΈΠ· уравнСния 8.1.

Π§Ρ‚ΠΎΠ±Ρ‹ ΠΏΠΎΠΊΠ°Π·Π°Ρ‚ΡŒ, ΠΊΠ°ΠΊ Ρ€Π°Π±ΠΎΡ‚Π°Π΅Ρ‚ эта ΠΏΡ€ΠΎΡ†Π΅Π΄ΡƒΡ€Π°, ΠΌΡ‹ Ρ‚Π΅ΠΏΠ΅Ρ€ΡŒ вычисляСм Смкости ΠΏΠ°Ρ€Π°Π»Π»Π΅Π»ΡŒΠ½Ρ‹Ρ… пластин, сфСричСских ΠΈ цилиндричСских кондСнсаторов. Π’ΠΎ всСх случаях ΠΌΡ‹ ΠΏΡ€Π΅Π΄ΠΏΠΎΠ»Π°Π³Π°Π΅ΠΌ Π²Π°ΠΊΡƒΡƒΠΌΠ½Ρ‹Π΅ кондСнсаторы (пустыС кондСнсаторы) Π±Π΅Π· диэлСктричСского вСщСства Π² пространствС ΠΌΠ΅ΠΆΠ΄Ρƒ ΠΏΡ€ΠΎΠ²ΠΎΠ΄Π½ΠΈΠΊΠ°ΠΌΠΈ.

ΠšΠΎΠ½Π΄Π΅Π½ΡΠ°Ρ‚ΠΎΡ€ с ΠΏΠ°Ρ€Π°Π»Π»Π΅Π»ΡŒΠ½Ρ‹ΠΌΠΈ пластинами

ΠšΠΎΠ½Π΄Π΅Π½ΡΠ°Ρ‚ΠΎΡ€ с ΠΏΠ°Ρ€Π°Π»Π»Π΅Π»ΡŒΠ½Ρ‹ΠΌΠΈ пластинами (рисунок 8.5) ΠΈΠΌΠ΅Π΅Ρ‚ Π΄Π²Π΅ ΠΈΠ΄Π΅Π½Ρ‚ΠΈΡ‡Π½Ρ‹Π΅ токопроводящиС пластины, каТдая с ΠΏΠ»ΠΎΡ‰Π°Π΄ΡŒΡŽ повСрхности A, , Ρ€Π°Π·Π΄Π΅Π»Π΅Π½Π½Ρ‹ΠΌΠΈ расстояниСм d . Когда Π½Π° кондСнсатор подаСтся напряТСниС Π’, , ΠΎΠ½ сохраняСт заряд Q , ΠΊΠ°ΠΊ ΠΏΠΎΠΊΠ°Π·Π°Π½ΠΎ. ΠœΡ‹ ΠΌΠΎΠΆΠ΅ΠΌ ΡƒΠ²ΠΈΠ΄Π΅Ρ‚ΡŒ, ΠΊΠ°ΠΊ Π΅Π³ΠΎ Π΅ΠΌΠΊΠΎΡΡ‚ΡŒ ΠΌΠΎΠΆΠ΅Ρ‚ Π·Π°Π²ΠΈΡΠ΅Ρ‚ΡŒ ΠΎΡ‚ A ΠΈ d , рассмотрСв характСристики кулоновской силы. ΠœΡ‹ Π·Π½Π°Π΅ΠΌ, Ρ‡Ρ‚ΠΎ сила ΠΌΠ΅ΠΆΠ΄Ρƒ зарядами увСличиваСтся с ΡƒΠ²Π΅Π»ΠΈΡ‡Π΅Π½ΠΈΠ΅ΠΌ заряда ΠΈ ΡƒΠΌΠ΅Π½ΡŒΡˆΠ°Π΅Ρ‚ΡΡ с расстояниСм ΠΌΠ΅ΠΆΠ΄Ρƒ Π½ΠΈΠΌΠΈ. Π‘Π»Π΅Π΄ΡƒΠ΅Ρ‚ ΠΎΠΆΠΈΠ΄Π°Ρ‚ΡŒ, Ρ‡Ρ‚ΠΎ Ρ‡Π΅ΠΌ большС пластины, Ρ‚Π΅ΠΌ большС заряда ΠΎΠ½ΠΈ ΠΌΠΎΠ³ΡƒΡ‚ Ρ…Ρ€Π°Π½ΠΈΡ‚ΡŒ.Π’Π°ΠΊΠΈΠΌ ΠΎΠ±Ρ€Π°Π·ΠΎΠΌ, C Π΄ΠΎΠ»ΠΆΠ½ΠΎ Π±Ρ‹Ρ‚ΡŒ большС для большСго значСния A . Π’ΠΎΡ‡Π½ΠΎ Ρ‚Π°ΠΊ ΠΆΠ΅, Ρ‡Π΅ΠΌ Π±Π»ΠΈΠΆΠ΅ пластины Π΄Ρ€ΡƒΠ³ ΠΊ Π΄Ρ€ΡƒΠ³Ρƒ, Ρ‚Π΅ΠΌ сильнСС Π½Π° Π½ΠΈΡ… притяТСниС ΠΏΡ€ΠΎΡ‚ΠΈΠ²ΠΎΠΏΠΎΠ»ΠΎΠΆΠ½Ρ‹Ρ… зарядов. Π‘Π»Π΅Π΄ΠΎΠ²Π°Ρ‚Π΅Π»ΡŒΠ½ΠΎ, C Π΄ΠΎΠ»ΠΆΠ½ΠΎ Π±Ρ‹Ρ‚ΡŒ большС для мСньшСго d .

Π€ΠΈΠ³ΡƒΡ€Π°
8,5

Π’ кондСнсаторС с ΠΏΠ°Ρ€Π°Π»Π»Π΅Π»ΡŒΠ½Ρ‹ΠΌΠΈ пластинами, Ρ€Π°Π·Π΄Π΅Π»Π΅Π½Π½Ρ‹Π΅ пластинами Π½Π° расстояниС d , каТдая пластина ΠΈΠΌΠ΅Π΅Ρ‚ ΠΎΠ΄ΠΈΠ½Π°ΠΊΠΎΠ²ΡƒΡŽ ΠΏΠ»ΠΎΡ‰Π°Π΄ΡŒ повСрхности A, .

ΠžΠΏΡ€Π΅Π΄Π΅Π»ΠΈΠΌ ΠΏΠ»ΠΎΡ‚Π½ΠΎΡΡ‚ΡŒ повСрхностного заряда σσ Π½Π° пластинах ΠΊΠ°ΠΊ

Из ΠΏΡ€Π΅Π΄Ρ‹Π΄ΡƒΡ‰ΠΈΡ… Π³Π»Π°Π² ΠΌΡ‹ Π·Π½Π°Π΅ΠΌ, Ρ‡Ρ‚ΠΎ ΠΊΠΎΠ³Π΄Π° d ΠΌΠ°Π»ΠΎ, элСктричСскоС ΠΏΠΎΠ»Π΅ ΠΌΠ΅ΠΆΠ΄Ρƒ пластинами довольно ΠΎΠ΄Π½ΠΎΡ€ΠΎΠ΄Π½ΠΎ (Π±Π΅Π· ΡƒΡ‡Π΅Ρ‚Π° ΠΊΡ€Π°Π΅Π²Ρ‹Ρ… эффСктов) ΠΈ Ρ‡Ρ‚ΠΎ Π΅Π³ΠΎ Π²Π΅Π»ΠΈΡ‡ΠΈΠ½Π° опрСдСляСтся ΠΊΠ°ΠΊ

.

Π³Π΄Π΅ постоянная Ξ΅0Ξ΅0 — диэлСктричСская ΠΏΡ€ΠΎΠ½ΠΈΡ†Π°Π΅ΠΌΠΎΡΡ‚ΡŒ свободного пространства, Ξ΅0 = 8.85 Γ— 10–12Π€ / ΠΌ. Ξ•0 = 8,85 Γ— 10–12Π€ / ΠΌ. Π•Π΄ΠΈΠ½ΠΈΡ†Π° БИ Π² Π€ / ΠΌ эквивалСнтна C2 / N Β· m2.C2 / N Β· m2. ΠŸΠΎΡΠΊΠΎΠ»ΡŒΠΊΡƒ элСктричСскоС ΠΏΠΎΠ»Π΅ E β†’ E β†’ ΠΌΠ΅ΠΆΠ΄Ρƒ пластинами ΠΎΠ΄Π½ΠΎΡ€ΠΎΠ΄Π½ΠΎ, Ρ€Π°Π·Π½ΠΎΡΡ‚ΡŒ ΠΏΠΎΡ‚Π΅Π½Ρ†ΠΈΠ°Π»ΠΎΠ² ΠΌΠ΅ΠΆΠ΄Ρƒ пластинами составляСт

.
V = Ed = ΟƒdΞ΅0 = QdΞ΅0A.V = Ed = ΟƒdΞ΅0 = QdΞ΅0A.

Π‘Π»Π΅Π΄ΠΎΠ²Π°Ρ‚Π΅Π»ΡŒΠ½ΠΎ, ΡƒΡ€Π°Π²Π½Π΅Π½ΠΈΠ΅ 8.1 Π΄Π°Π΅Ρ‚ Π΅ΠΌΠΊΠΎΡΡ‚ΡŒ кондСнсатора с ΠΏΠ°Ρ€Π°Π»Π»Π΅Π»ΡŒΠ½Ρ‹ΠΌΠΈ пластинами ΠΊΠ°ΠΊ

C = QV = QQd / Ξ΅0A = Ξ΅0Ad.C = QV = QQd / Ξ΅0A = Ξ΅0Ad.

8,3

ΠžΠ±Ρ€Π°Ρ‚ΠΈΡ‚Π΅ Π²Π½ΠΈΠΌΠ°Π½ΠΈΠ΅ Π½Π° это ΡƒΡ€Π°Π²Π½Π΅Π½ΠΈΠ΅, Ρ‡Ρ‚ΠΎ Π΅ΠΌΠΊΠΎΡΡ‚ΡŒ являСтся Ρ„ΡƒΠ½ΠΊΡ†ΠΈΠ΅ΠΉ Ρ‚ΠΎΠ»ΡŒΠΊΠΎ Π³Π΅ΠΎΠΌΠ΅Ρ‚Ρ€ΠΈΠΈ ΠΈ Ρ‚ΠΎΠ³ΠΎ, ΠΊΠ°ΠΊΠΎΠΉ ΠΌΠ°Ρ‚Π΅Ρ€ΠΈΠ°Π» заполняСт пространство ΠΌΠ΅ΠΆΠ΄Ρƒ пластинами (Π² Π΄Π°Π½Π½ΠΎΠΌ случаС Π²Π°ΠΊΡƒΡƒΠΌ) этого кондСнсатора.ЀактичСски, это Π²Π΅Ρ€Π½ΠΎ Π½Π΅ Ρ‚ΠΎΠ»ΡŒΠΊΠΎ для кондСнсатора с ΠΏΠ°Ρ€Π°Π»Π»Π΅Π»ΡŒΠ½Ρ‹ΠΌΠΈ пластинами, Π½ΠΎ ΠΈ для всСх кондСнсаторов: Π΅ΠΌΠΊΠΎΡΡ‚ΡŒ Π½Π΅ зависит ΠΎΡ‚ Q ΠΈΠ»ΠΈ Π’ . Если заряд измСняСтся, соотвСтствСнно измСняСтся ΠΈ ΠΏΠΎΡ‚Π΅Π½Ρ†ΠΈΠ°Π», Ρ‚Π°ΠΊ Ρ‡Ρ‚ΠΎ Q / V остаСтся постоянным.

ΠŸΡ€ΠΈΠΌΠ΅Ρ€
8.1

Π•ΠΌΠΊΠΎΡΡ‚ΡŒ ΠΈ заряд Π² кондСнсаторС с ΠΏΠ°Ρ€Π°Π»Π»Π΅Π»ΡŒΠ½Ρ‹ΠΌΠΈ пластинами

(Π°) Какова Π΅ΠΌΠΊΠΎΡΡ‚ΡŒ пустого кондСнсатора с ΠΏΠ°Ρ€Π°Π»Π»Π΅Π»ΡŒΠ½Ρ‹ΠΌΠΈ пластинами с мСталличСскими пластинами, каТдая ΠΈΠ· ΠΊΠΎΡ‚ΠΎΡ€Ρ‹Ρ… ΠΈΠΌΠ΅Π΅Ρ‚ ΠΏΠ»ΠΎΡ‰Π°Π΄ΡŒ 1?00ΠΌ21.00ΠΌ2, Ρ‡Π΅Ρ€Π΅Π· 1,00 ΠΌΠΌ? (b) Бколько заряда хранится Π² этом кондСнсаторС, Ссли ΠΊ Π½Π΅ΠΌΡƒ ΠΏΡ€ΠΈΠ»ΠΎΠΆΠ΅Π½ΠΎ напряТСниС 3,00 Γ— 103 Π’3,00 Γ— 103 Π’?

БтратСгия

ΠžΠΏΡ€Π΅Π΄Π΅Π»Π΅Π½ΠΈΠ΅ Смкости C прСдставляСт собой прямоС ΠΏΡ€ΠΈΠΌΠ΅Π½Π΅Π½ΠΈΠ΅ уравнСния 8.3. Найдя C , ΠΌΡ‹ смоТСм Π½Π°ΠΉΡ‚ΠΈ Π½Π°ΠΊΠΎΠΏΠ»Π΅Π½Π½Ρ‹ΠΉ заряд, ΠΈΡΠΏΠΎΠ»ΡŒΠ·ΡƒΡ ΡƒΡ€Π°Π²Π½Π΅Π½ΠΈΠ΅ 8.1.

РСшСниС
  1. Π’Π²ΠΎΠ΄ Π·Π°Π΄Π°Π½Π½Ρ‹Ρ… Π·Π½Π°Ρ‡Π΅Π½ΠΈΠΉ Π² ΡƒΡ€Π°Π²Π½Π΅Π½ΠΈΠ΅ 8.3 Π΄Π°Π΅Ρ‚
    C = Ξ΅0Ad = (8,85 Γ— 10βˆ’12Fm) 1,00ΠΌ21,00 Γ— 10βˆ’3ΠΌ = 8,85 Γ— 10βˆ’9F = 8,85nF.C = Ξ΅0Ad = (8,85 Γ— 10βˆ’12Fm) 1,00ΠΌ21.00 Γ— 10βˆ’3m = 8,85 Γ— 10βˆ’9F = 8,85 Π½Π€.
    Π­Ρ‚ΠΎ нСбольшоС Π·Π½Π°Ρ‡Π΅Π½ΠΈΠ΅ Смкости ΡƒΠΊΠ°Π·Ρ‹Π²Π°Π΅Ρ‚ Π½Π° Ρ‚ΠΎ, насколько слоТно ΠΈΠ·Π³ΠΎΡ‚ΠΎΠ²ΠΈΡ‚ΡŒ устройство с большой Π΅ΠΌΠΊΠΎΡΡ‚ΡŒΡŽ.
  2. ΠžΠ±Ρ€Π°Ρ‰Π΅Π½ΠΈΠ΅ уравнСния 8. 1 ΠΈ Π²Π²ΠΎΠ΄ извСстных Π·Π½Π°Ρ‡Π΅Π½ΠΈΠΉ Π² это ΡƒΡ€Π°Π²Π½Π΅Π½ΠΈΠ΅ Π΄Π°Π΅Ρ‚
    Q = CV = (8,85 Γ— 10–9F) (3,00 Γ— 103 Π’) = 26,6 мкКл. Q = CV = (8,85 Γ— 10–9F) (3,00 Γ— 103 Π’) = 26,6 мкКл.
Π—Π½Π°Ρ‡Π΅Π½ΠΈΠ΅

Π­Ρ‚ΠΎΡ‚ заряд лишь Π½Π΅ΠΌΠ½ΠΎΠ³ΠΎ большС, Ρ‡Π΅ΠΌ Π² Ρ‚ΠΈΠΏΠΈΡ‡Π½Ρ‹Ρ… прилоТСниях для статичСского элСктричСства. ΠŸΠΎΡΠΊΠΎΠ»ΡŒΠΊΡƒ Π²ΠΎΠ·Π΄ΡƒΡ… Ρ€Π°Π·Ρ€ΡƒΡˆΠ°Π΅Ρ‚ΡΡ (становится проводящим) ΠΏΡ€ΠΈ напряТСнности элСктричСского поля ΠΎΠΊΠΎΠ»ΠΎ 3.0 ΠœΠ’ / ΠΌ, Π½Π° этом кондСнсаторС большС нСльзя Π½Π°ΠΊΠ°ΠΏΠ»ΠΈΠ²Π°Ρ‚ΡŒ заряд ΠΏΡ€ΠΈ ΡƒΠ²Π΅Π»ΠΈΡ‡Π΅Π½ΠΈΠΈ напряТСния.

ΠŸΡ€ΠΈΠΌΠ΅Ρ€
8,2

1-Π€ кондСнсатор с ΠΏΠ°Ρ€Π°Π»Π»Π΅Π»ΡŒΠ½Ρ‹ΠΌΠΈ пластинами

ΠŸΡ€Π΅Π΄ΠΏΠΎΠ»ΠΎΠΆΠΈΠΌ, Π²Ρ‹ Ρ…ΠΎΡ‚ΠΈΡ‚Π΅ ΡΠΊΠΎΠ½ΡΡ‚Ρ€ΡƒΠΈΡ€ΠΎΠ²Π°Ρ‚ΡŒ кондСнсатор с ΠΏΠ°Ρ€Π°Π»Π»Π΅Π»ΡŒΠ½Ρ‹ΠΌΠΈ пластинами Π΅ΠΌΠΊΠΎΡΡ‚ΡŒΡŽ 1,0 F. ΠšΠ°ΠΊΡƒΡŽ ΠΏΠ»ΠΎΡ‰Π°Π΄ΡŒ Π²Ρ‹ Π΄ΠΎΠ»ΠΆΠ½Ρ‹ ΠΈΡΠΏΠΎΠ»ΡŒΠ·ΠΎΠ²Π°Ρ‚ΡŒ для ΠΊΠ°ΠΆΠ΄ΠΎΠΉ пластины, Ссли пластины Ρ€Π°Π·Π΄Π΅Π»Π΅Π½Ρ‹ Π½Π° 1,0 ΠΌΠΌ?

РСшСниС

ΠŸΡ€Π΅ΠΎΠ±Ρ€Π°Π·ΡƒΡ ΡƒΡ€Π°Π²Π½Π΅Π½ΠΈΠ΅ 8.3, ΠΏΠΎΠ»ΡƒΡ‡Π°Π΅ΠΌ
A = CdΞ΅0 = (1.0F) (1.0 Γ— 10βˆ’3m) 8.85 Γ— 10βˆ’12F / m = 1.1 Γ— 108m2 A = CdΞ΅0 = (1.0F) (1.0 Γ— 10βˆ’3m) 8,85 Γ— 10βˆ’12F / m = 1,1 Γ— 108 ΠΌ2.

КаТдая квадратная пластина Π΄ΠΎΠ»ΠΆΠ½Π° Π±Ρ‹Ρ‚ΡŒ 10 ΠΊΠΌ Π² ΠΏΠΎΠΏΠ΅Ρ€Π΅Ρ‡Π½ΠΈΠΊΠ΅. РаньшС Π±Ρ‹Π»ΠΎ ΠΎΠ±Ρ‹Ρ‡Π½Ρ‹ΠΌ Ρ€ΠΎΠ·Ρ‹Π³Ρ€Ρ‹ΡˆΠ΅ΠΌ — ΠΏΠΎΠΏΡ€ΠΎΡΠΈΡ‚ΡŒ студСнта ΠΏΠΎΠΉΡ‚ΠΈ Π² склад Π»Π°Π±ΠΎΡ€Π°Ρ‚ΠΎΡ€ΠΈΠΈ ΠΈ ΠΏΠΎΠΏΡ€ΠΎΡΠΈΡ‚ΡŒ кондСнсатор с ΠΏΠ°Ρ€Π°Π»Π»Π΅Π»ΡŒΠ½Ρ‹ΠΌΠΈ пластинами 1F, ΠΏΠΎΠΊΠ° ΠΎΠ±ΡΠ»ΡƒΠΆΠΈΠ²Π°ΡŽΡ‰ΠΈΠΉ пСрсонал Π½Π΅ устанСт ΠΎΡ‚ ΡˆΡƒΡ‚ΠΎΠΊ.

ΠŸΡ€ΠΎΠ²Π΅Ρ€ΡŒΡ‚Π΅ своС ΠΏΠΎΠ½ΠΈΠΌΠ°Π½ΠΈΠ΅
8.1

ΠŸΡ€ΠΎΠ²Π΅Ρ€ΡŒΡ‚Π΅ своС ΠΏΠΎΠ½ΠΈΠΌΠ°Π½ΠΈΠ΅ Π•ΠΌΠΊΠΎΡΡ‚ΡŒ кондСнсатора с ΠΏΠ°Ρ€Π°Π»Π»Π΅Π»ΡŒΠ½Ρ‹ΠΌΠΈ пластинами составляСт 2,0 ΠΏΠ€. Если ΠΏΠ»ΠΎΡ‰Π°Π΄ΡŒ ΠΊΠ°ΠΆΠ΄ΠΎΠΉ пластины составляСт 2,4 см 22,4 см2, ΠΊΠ°ΠΊΠΎΠ²ΠΎ расстояниС ΠΌΠ΅ΠΆΠ΄Ρƒ пластинами?

ΠŸΡ€ΠΎΠ²Π΅Ρ€ΡŒΡ‚Π΅ своС ΠΏΠΎΠ½ΠΈΠΌΠ°Π½ΠΈΠ΅
8,2

ΠŸΡ€ΠΎΠ²Π΅Ρ€ΡŒΡ‚Π΅ своС ΠΏΠΎΠ½ΠΈΠΌΠ°Π½ΠΈΠ΅ Π£Π±Π΅Π΄ΠΈΡ‚Π΅ΡΡŒ, Ρ‡Ρ‚ΠΎ Οƒ / VΟƒ / V ΠΈ Ξ΅0 / dΞ΅0 / d ΠΈΠΌΠ΅ΡŽΡ‚ ΠΎΠ΄ΠΈΠ½Π°ΠΊΠΎΠ²Ρ‹Π΅ физичСскиС Π΅Π΄ΠΈΠ½ΠΈΡ†Ρ‹.

БфСричСский кондСнсатор

БфСричСский кондСнсатор — это Π΅Ρ‰Π΅ ΠΎΠ΄ΠΈΠ½ Π½Π°Π±ΠΎΡ€ ΠΏΡ€ΠΎΠ²ΠΎΠ΄Π½ΠΈΠΊΠΎΠ², Π΅ΠΌΠΊΠΎΡΡ‚ΡŒ ΠΊΠΎΡ‚ΠΎΡ€Ρ‹Ρ… ΠΌΠΎΠΆΠ½ΠΎ Π»Π΅Π³ΠΊΠΎ ΠΎΠΏΡ€Π΅Π΄Π΅Π»ΠΈΡ‚ΡŒ (рис. 8.6). Он состоит ΠΈΠ· Π΄Π²ΡƒΡ… концСнтричСских проводящих сфСричСских ΠΎΠ±ΠΎΠ»ΠΎΡ‡Π΅ΠΊ радиусов R1R1 (внутрСнняя ΠΎΠ±ΠΎΠ»ΠΎΡ‡ΠΊΠ°) ΠΈ R2R2 (внСшняя ΠΎΠ±ΠΎΠ»ΠΎΡ‡ΠΊΠ°). ΠžΠ±ΠΎΠ»ΠΎΡ‡ΠΊΠ°ΠΌ ΠΏΡ€ΠΈΠΏΠΈΡΡ‹Π²Π°ΡŽΡ‚ΡΡ Ρ€Π°Π²Π½Ρ‹Π΅ ΠΈ ΠΏΡ€ΠΎΡ‚ΠΈΠ²ΠΎΠΏΠΎΠ»ΠΎΠΆΠ½Ρ‹Π΅ заряды + Q + Q ΠΈ βˆ’Q βˆ’ Q соотвСтствСнно. Из-Π·Π° симмСтрии элСктричСскоС ΠΏΠΎΠ»Π΅ ΠΌΠ΅ΠΆΠ΄Ρƒ ΠΎΠ±ΠΎΠ»ΠΎΡ‡ΠΊΠ°ΠΌΠΈ Π½Π°ΠΏΡ€Π°Π²Π»Π΅Π½ΠΎ Ρ€Π°Π΄ΠΈΠ°Π»ΡŒΠ½ΠΎ Π½Π°Ρ€ΡƒΠΆΡƒ. ΠœΡ‹ ΠΌΠΎΠΆΠ΅ΠΌ ΠΏΠΎΠ»ΡƒΡ‡ΠΈΡ‚ΡŒ Π²Π΅Π»ΠΈΡ‡ΠΈΠ½Ρƒ поля, ΠΏΡ€ΠΈΠΌΠ΅Π½ΠΈΠ² Π·Π°ΠΊΠΎΠ½ Гаусса ΠΊ сфСричСской гауссовой повСрхности радиусом r , ΠΊΠΎΠ½Ρ†Π΅Π½Ρ‚Ρ€ΠΈΡ‡Π½ΠΎΠΉ ΠΎΠ±ΠΎΠ»ΠΎΡ‡ΠΊΠ°ΠΌ.dr) = Q4πΡ0∫R1R2drr2 = Q4πΡ0 (1R1βˆ’1R2).

Π’ этом ΡƒΡ€Π°Π²Π½Π΅Π½ΠΈΠΈ Ρ€Π°Π·Π½ΠΎΡΡ‚ΡŒ ΠΏΠΎΡ‚Π΅Π½Ρ†ΠΈΠ°Π»ΠΎΠ² ΠΌΠ΅ΠΆΠ΄Ρƒ пластинами Ρ€Π°Π²Π½Π° V = — (V2 βˆ’ V1) = V1 βˆ’ V2V = — (V2 βˆ’ V1) = V1 βˆ’ V2. ΠŸΠΎΠ΄ΡΡ‚Π°Π²Π»ΡΠ΅ΠΌ этот Ρ€Π΅Π·ΡƒΠ»ΡŒΡ‚Π°Ρ‚ Π² ΡƒΡ€Π°Π²Π½Π΅Π½ΠΈΠ΅ 8.1, Ρ‡Ρ‚ΠΎΠ±Ρ‹ Π½Π°ΠΉΡ‚ΠΈ Π΅ΠΌΠΊΠΎΡΡ‚ΡŒ сфСричСского кондСнсатора:

C = QV = 4πΡ0R1R2R2 βˆ’ R1.C = QV = 4πΡ0R1R2R2 βˆ’ R1.

8,4

Π€ΠΈΠ³ΡƒΡ€Π°
8,6

БфСричСский кондСнсатор состоит ΠΈΠ· Π΄Π²ΡƒΡ… концСнтричСских проводящих сфСр. ΠžΠ±Ρ€Π°Ρ‚ΠΈΡ‚Π΅ Π²Π½ΠΈΠΌΠ°Π½ΠΈΠ΅, Ρ‡Ρ‚ΠΎ заряды Π½Π° ΠΏΡ€ΠΎΠ²ΠΎΠ΄Π½ΠΈΠΊΠ΅ находятся Π½Π° Π΅Π³ΠΎ повСрхности.

ΠŸΡ€ΠΈΠΌΠ΅Ρ€
8.3

Π•ΠΌΠΊΠΎΡΡ‚ΡŒ ΠΈΠ·ΠΎΠ»ΠΈΡ€ΠΎΠ²Π°Π½Π½ΠΎΠΉ сфСры

ВычислитС Π΅ΠΌΠΊΠΎΡΡ‚ΡŒ ΠΎΠ΄ΠΈΠ½ΠΎΡ‡Π½ΠΎΠΉ ΠΈΠ·ΠΎΠ»ΠΈΡ€ΠΎΠ²Π°Π½Π½ΠΎΠΉ проводящСй сфСры радиуса R1R1 ΠΈ сравнитС Π΅Π΅ с ΡƒΡ€Π°Π²Π½Π΅Π½ΠΈΠ΅ΠΌ 8.4 Π² ΠΏΡ€Π΅Π΄Π΅Π»Π΅ R2 β†’ ∞R2 β†’ ∞.

БтратСгия

ΠœΡ‹ ΠΏΡ€Π΅Π΄ΠΏΠΎΠ»Π°Π³Π°Π΅ΠΌ, Ρ‡Ρ‚ΠΎ заряд Π½Π° сфСрС Ρ€Π°Π²Π΅Π½ Q , ΠΈ поэтому слСдуСм Ρ‡Π΅Ρ‚Ρ‹Ρ€Π΅ΠΌ шагам, описанным Ρ€Π°Π½Π΅Π΅. ΠœΡ‹ Ρ‚Π°ΠΊΠΆΠ΅ ΠΏΡ€Π΅Π΄ΠΏΠΎΠ»Π°Π³Π°Π΅ΠΌ, Ρ‡Ρ‚ΠΎ Π΄Ρ€ΡƒΠ³ΠΎΠΉ ΠΏΡ€ΠΎΠ²ΠΎΠ΄Π½ΠΈΠΊ прСдставляСт собой ΠΊΠΎΠ½Ρ†Π΅Π½Ρ‚Ρ€ΠΈΡ‡Π΅ΡΠΊΡƒΡŽ ΠΏΠΎΠ»ΡƒΡŽ сфСру бСсконСчного радиуса.

РСшСниС

На внСшнСй сторонС ΠΈΠ·ΠΎΠ»ΠΈΡ€ΠΎΠ²Π°Π½Π½ΠΎΠΉ проводящСй сфСры элСктричСскоС ΠΏΠΎΠ»Π΅ задаСтся ΡƒΡ€Π°Π²Π½Π΅Π½ΠΈΠ΅ΠΌ 8.dr) = Q4πΡ0∫R1 + ∞drr2 = 14πΡ0QR1.

Π’Π°ΠΊΠΈΠΌ ΠΎΠ±Ρ€Π°Π·ΠΎΠΌ, Π΅ΠΌΠΊΠΎΡΡ‚ΡŒ ΠΈΠ·ΠΎΠ»ΠΈΡ€ΠΎΠ²Π°Π½Π½ΠΎΠΉ сфСры Ρ€Π°Π²Π½Π°

.
C = QV = Q4πΡ0R1Q = 4πΡ0R1.C = QV = Q4πΡ0R1Q = 4πΡ0R1.

Π—Π½Π°Ρ‡Π΅Π½ΠΈΠ΅

Π’ΠΎΡ‚ ΠΆΠ΅ Ρ€Π΅Π·ΡƒΠ»ΡŒΡ‚Π°Ρ‚ ΠΌΠΎΠΆΠ½ΠΎ ΠΏΠΎΠ»ΡƒΡ‡ΠΈΡ‚ΡŒ, взяв ΠΏΡ€Π΅Π΄Π΅Π» уравнСния 8.4 ΠΏΡ€ΠΈ R2 β†’ ∞R2 β†’ ∞. Π’Π°ΠΊΠΈΠΌ ΠΎΠ±Ρ€Π°Π·ΠΎΠΌ, одиночная изолированная сфСра эквивалСнтна сфСричСскому кондСнсатору, внСшняя ΠΎΠ±ΠΎΠ»ΠΎΡ‡ΠΊΠ° ΠΊΠΎΡ‚ΠΎΡ€ΠΎΠ³ΠΎ ΠΈΠΌΠ΅Π΅Ρ‚ бСсконСчно большой радиус.

ΠŸΡ€ΠΎΠ²Π΅Ρ€ΡŒΡ‚Π΅ своС ΠΏΠΎΠ½ΠΈΠΌΠ°Π½ΠΈΠ΅
8,3

ΠŸΡ€ΠΎΠ²Π΅Ρ€ΡŒΡ‚Π΅ своС ΠΏΠΎΠ½ΠΈΠΌΠ°Π½ΠΈΠ΅ Радиус внСшнСй сфСры сфСричСского кондСнсатора Π² ΠΏΡΡ‚ΡŒ Ρ€Π°Π· большС радиуса Π΅Π³ΠΎ Π²Π½ΡƒΡ‚Ρ€Π΅Π½Π½Π΅ΠΉ ΠΎΠ±ΠΎΠ»ΠΎΡ‡ΠΊΠΈ.ΠšΠ°ΠΊΠΎΠ²Ρ‹ Ρ€Π°Π·ΠΌΠ΅Ρ€Ρ‹ этого кондСнсатора, Ссли Π΅Π³ΠΎ Π΅ΠΌΠΊΠΎΡΡ‚ΡŒ 5,00 ΠΏΠ€?

ЦилиндричСский кондСнсатор

ЦилиндричСский кондСнсатор состоит ΠΈΠ· Π΄Π²ΡƒΡ… концСнтричСских проводящих Ρ†ΠΈΠ»ΠΈΠ½Π΄Ρ€ΠΎΠ² (рисунок 8. dA = E (2Ο€rl) = QΞ΅0.dr) = Q2πΡ0l∫R1R2drr = Q2πΡ0llnr | R1R2 = Q2πΡ0llnR2R1.

Π’Π°ΠΊΠΈΠΌ ΠΎΠ±Ρ€Π°Π·ΠΎΠΌ, Π΅ΠΌΠΊΠΎΡΡ‚ΡŒ цилиндричСского кондСнсатора

C = QV = 2πΡ0lln (R2 / R1). C = QV = 2πΡ0lln (R2 / R1).

8,6

Как ΠΈ Π² Π΄Ρ€ΡƒΠ³ΠΈΡ… случаях, эта Π΅ΠΌΠΊΠΎΡΡ‚ΡŒ зависит Ρ‚ΠΎΠ»ΡŒΠΊΠΎ ΠΎΡ‚ Π³Π΅ΠΎΠΌΠ΅Ρ‚Ρ€ΠΈΠΈ располоТСния ΠΏΡ€ΠΎΠ²ΠΎΠ΄Π½ΠΈΠΊΠΎΠ². Π’Π°ΠΆΠ½Ρ‹ΠΌ ΠΏΡ€ΠΈΠΌΠ΅Π½Π΅Π½ΠΈΠ΅ΠΌ уравнСния 8.6 являСтся ΠΎΠΏΡ€Π΅Π΄Π΅Π»Π΅Π½ΠΈΠ΅ Смкости Π½Π° Π΅Π΄ΠΈΠ½ΠΈΡ†Ρƒ Π΄Π»ΠΈΠ½Ρ‹ коаксиального кабСля , ΠΊΠΎΡ‚ΠΎΡ€Ρ‹ΠΉ ΠΎΠ±Ρ‹Ρ‡Π½ΠΎ ΠΈΡΠΏΠΎΠ»ΡŒΠ·ΡƒΠ΅Ρ‚ΡΡ для ΠΏΠ΅Ρ€Π΅Π΄Π°Ρ‡ΠΈ ΠΈΠ·ΠΌΠ΅Π½ΡΡŽΡ‰ΠΈΡ…ΡΡ Π²ΠΎ Π²Ρ€Π΅ΠΌΠ΅Π½ΠΈ элСктричСских сигналов. ΠšΠΎΠ°ΠΊΡΠΈΠ°Π»ΡŒΠ½Ρ‹ΠΉ кабСль состоит ΠΈΠ· Π΄Π²ΡƒΡ… концСнтричСских цилиндричСских ΠΏΡ€ΠΎΠ²ΠΎΠ΄Π½ΠΈΠΊΠΎΠ², Ρ€Π°Π·Π΄Π΅Π»Π΅Π½Π½Ρ‹Ρ… изоляционным ΠΌΠ°Ρ‚Π΅Ρ€ΠΈΠ°Π»ΠΎΠΌ.(Π—Π΄Π΅ΡΡŒ ΠΌΡ‹ ΠΏΡ€Π΅Π΄ΠΏΠΎΠ»Π°Π³Π°Π΅ΠΌ Π½Π°Π»ΠΈΡ‡ΠΈΠ΅ Π²Π°ΠΊΡƒΡƒΠΌΠ° ΠΌΠ΅ΠΆΠ΄Ρƒ ΠΏΡ€ΠΎΠ²ΠΎΠ΄Π½ΠΈΠΊΠ°ΠΌΠΈ, Π½ΠΎ Ρ„ΠΈΠ·ΠΈΠΊΠ° качСствСнно ΠΏΠΎΡ‡Ρ‚ΠΈ такая ΠΆΠ΅, ΠΊΠΎΠ³Π΄Π° пространство ΠΌΠ΅ΠΆΠ΄Ρƒ ΠΏΡ€ΠΎΠ²ΠΎΠ΄Π½ΠΈΠΊΠ°ΠΌΠΈ Π·Π°ΠΏΠΎΠ»Π½Π΅Π½ΠΎ диэлСктриком.) Π­Ρ‚Π° конфигурация экранируСт элСктричСский сигнал, Ρ€Π°ΡΠΏΡ€ΠΎΡΡ‚Ρ€Π°Π½ΡΡŽΡ‰ΠΈΠΉΡΡ ΠΏΠΎ Π²Π½ΡƒΡ‚Ρ€Π΅Π½Π½Π΅ΠΌΡƒ ΠΏΡ€ΠΎΠ²ΠΎΠ΄Π½ΠΈΠΊΡƒ, ΠΎΡ‚ ΠΏΠ°Ρ€Π°Π·ΠΈΡ‚Π½Ρ‹Ρ… элСктричСских ΠΏΠΎΠ»Π΅ΠΉ, Π²Π½Π΅ΡˆΠ½ΠΈΡ… ΠΏΠΎ ΠΎΡ‚Π½ΠΎΡˆΠ΅Π½ΠΈΡŽ ΠΊ ΠΏΡ€ΠΎΠ²ΠΎΠ΄Π½ΠΈΠΊΡƒ. кабСль. Π’ΠΎΠΊ Ρ‚Π΅Ρ‡Π΅Ρ‚ Π² ΠΏΡ€ΠΎΡ‚ΠΈΠ²ΠΎΠΏΠΎΠ»ΠΎΠΆΠ½Ρ‹Ρ… направлСниях Π²ΠΎ Π²Π½ΡƒΡ‚Ρ€Π΅Π½Π½Π΅ΠΌ ΠΈ внСшнСм ΠΏΡ€ΠΎΠ²ΠΎΠ΄Π½ΠΈΠΊΠ°Ρ…, ΠΏΡ€ΠΈ этом внСшний ΠΏΡ€ΠΎΠ²ΠΎΠ΄ ΠΎΠ±Ρ‹Ρ‡Π½ΠΎ Π·Π°Π·Π΅ΠΌΠ»Π΅Π½. Π’Π΅ΠΏΠ΅Ρ€ΡŒ ΠΈΠ· уравнСния 8.6 Π΅ΠΌΠΊΠΎΡΡ‚ΡŒ коаксиального кабСля Π½Π° Π΅Π΄ΠΈΠ½ΠΈΡ†Ρƒ Π΄Π»ΠΈΠ½Ρ‹ Ρ€Π°Π²Π½Π°

.
Cl = 2πΡ0ln (R2 / R1).Cl = 2πΡ0ln (R2 / R1).

Π’ практичСских прилоТСниях Π²Π°ΠΆΠ½ΠΎ Π²Ρ‹Π±ΠΈΡ€Π°Ρ‚ΡŒ ΠΊΠΎΠ½ΠΊΡ€Π΅Ρ‚Π½Ρ‹Π΅ значСния C / l . Π­Ρ‚ΠΎ ΠΌΠΎΠΆΠ΅Ρ‚ Π±Ρ‹Ρ‚ΡŒ достигнуто Π·Π° счСт ΡΠΎΠΎΡ‚Π²Π΅Ρ‚ΡΡ‚Π²ΡƒΡŽΡ‰Π΅Π³ΠΎ Π²Ρ‹Π±ΠΎΡ€Π° радиусов ΠΏΡ€ΠΎΠ²ΠΎΠ΄Π½ΠΈΠΊΠΎΠ² ΠΈ изоляционного ΠΌΠ°Ρ‚Π΅Ρ€ΠΈΠ°Π»Π° ΠΌΠ΅ΠΆΠ΄Ρƒ Π½ΠΈΠΌΠΈ.

ΠŸΡ€ΠΎΠ²Π΅Ρ€ΡŒΡ‚Π΅ своС ΠΏΠΎΠ½ΠΈΠΌΠ°Π½ΠΈΠ΅
8,4

ΠŸΡ€ΠΎΠ²Π΅Ρ€ΡŒΡ‚Π΅ своС ΠΏΠΎΠ½ΠΈΠΌΠ°Π½ΠΈΠ΅ Когда цилиндричСский кондСнсатор ΠΏΠΎΠ»ΡƒΡ‡Π°Π΅Ρ‚ заряд 0,500 нКл, ΠΌΠ΅ΠΆΠ΄Ρƒ Ρ†ΠΈΠ»ΠΈΠ½Π΄Ρ€Π°ΠΌΠΈ измСряСтся Ρ€Π°Π·Π½ΠΎΡΡ‚ΡŒ ΠΏΠΎΡ‚Π΅Π½Ρ†ΠΈΠ°Π»ΠΎΠ² 20,0 Π’. Π°) Какова Π΅ΠΌΠΊΠΎΡΡ‚ΡŒ этой систСмы? (b) Если Ρ†ΠΈΠ»ΠΈΠ½Π΄Ρ€Ρ‹ 1.Π”Π»ΠΈΠ½Π° 0 ΠΌ, ΠΊΠ°ΠΊΠΎΠ²ΠΎ ΡΠΎΠΎΡ‚Π½ΠΎΡˆΠ΅Π½ΠΈΠ΅ ΠΈΡ… радиусов?

НСсколько Ρ‚ΠΈΠΏΠΎΠ² кондСнсаторов, ΠΊΠΎΡ‚ΠΎΡ€Ρ‹Π΅ ΠΌΠΎΠΆΠ½ΠΎ ΠΈΡΠΏΠΎΠ»ΡŒΠ·ΠΎΠ²Π°Ρ‚ΡŒ Π½Π° ΠΏΡ€Π°ΠΊΡ‚ΠΈΠΊΠ΅, ΠΏΠΎΠΊΠ°Π·Π°Π½Ρ‹ Π½Π° рис. 8.4. ΠžΠ±Ρ‹Ρ‡Π½Ρ‹Π΅ кондСнсаторы часто состоят ΠΈΠ· Π΄Π²ΡƒΡ… Π½Π΅Π±ΠΎΠ»ΡŒΡˆΠΈΡ… кусочков мСталличСской Ρ„ΠΎΠ»ΡŒΠ³ΠΈ, Ρ€Π°Π·Π΄Π΅Π»Π΅Π½Π½Ρ‹Ρ… двумя нСбольшими кусочками изоляции (см. Рисунок 8.2 (b)). ΠœΠ΅Ρ‚Π°Π»Π»ΠΈΡ‡Π΅ΡΠΊΠ°Ρ Ρ„ΠΎΠ»ΡŒΠ³Π° ΠΈ изоляция ΠΏΠΎΠΊΡ€Ρ‹Ρ‚Ρ‹ Π·Π°Ρ‰ΠΈΡ‚Π½Ρ‹ΠΌ ΠΏΠΎΠΊΡ€Ρ‹Ρ‚ΠΈΠ΅ΠΌ, Π° Π΄Π²Π° мСталличСских Π²Ρ‹Π²ΠΎΠ΄Π° ΠΈΡΠΏΠΎΠ»ΡŒΠ·ΡƒΡŽΡ‚ΡΡ для ΠΏΠΎΠ΄ΠΊΠ»ΡŽΡ‡Π΅Π½ΠΈΡ Ρ„ΠΎΠ»ΡŒΠ³ΠΈ ΠΊ внСшнСй Ρ†Π΅ΠΏΠΈ. НСкоторыС распространСнныС изоляционныС ΠΌΠ°Ρ‚Π΅Ρ€ΠΈΠ°Π»Ρ‹ — это слюда, ΠΊΠ΅Ρ€Π°ΠΌΠΈΠΊΠ°, Π±ΡƒΠΌΠ°Π³Π° ΠΈ Π°Π½Ρ‚ΠΈΠΏΡ€ΠΈΠ³Π°Ρ€Π½ΠΎΠ΅ ΠΏΠΎΠΊΡ€Ρ‹Ρ‚ΠΈΠ΅ Teflon β„’.

Π”Ρ€ΡƒΠ³ΠΎΠΉ популярный Ρ‚ΠΈΠΏ кондСнсатора — элСктролитичСский кондСнсатор.Он состоит ΠΈΠ· окислСнного ΠΌΠ΅Ρ‚Π°Π»Π»Π° Π² проводящСй пастС. ΠžΡΠ½ΠΎΠ²Π½Ρ‹ΠΌ прСимущСством элСктролитичСского кондСнсатора являСтся Π΅Π³ΠΎ высокая Π΅ΠΌΠΊΠΎΡΡ‚ΡŒ ΠΏΠΎ ΡΡ€Π°Π²Π½Π΅Π½ΠΈΡŽ с Π΄Ρ€ΡƒΠ³ΠΈΠΌΠΈ распространСнными Ρ‚ΠΈΠΏΠ°ΠΌΠΈ кондСнсаторов. НапримСр, Π΅ΠΌΠΊΠΎΡΡ‚ΡŒ ΠΎΠ΄Π½ΠΎΠ³ΠΎ Ρ‚ΠΈΠΏΠ° алюминиСвого элСктролитичСского кондСнсатора ΠΌΠΎΠΆΠ΅Ρ‚ Π΄ΠΎΡΡ‚ΠΈΠ³Π°Ρ‚ΡŒ 1,0 F. Однако Π²Ρ‹ Π΄ΠΎΠ»ΠΆΠ½Ρ‹ Π±Ρ‹Ρ‚ΡŒ остороТны ΠΏΡ€ΠΈ использовании элСктролитичСского кондСнсатора Π² Ρ†Π΅ΠΏΠΈ, ΠΏΠΎΡ‚ΠΎΠΌΡƒ Ρ‡Ρ‚ΠΎ ΠΎΠ½ Ρ€Π°Π±ΠΎΡ‚Π°Π΅Ρ‚ ΠΏΡ€Π°Π²ΠΈΠ»ΡŒΠ½ΠΎ Ρ‚ΠΎΠ»ΡŒΠΊΠΎ Ρ‚ΠΎΠ³Π΄Π°, ΠΊΠΎΠ³Π΄Π° мСталличСская Ρ„ΠΎΠ»ΡŒΠ³Π° находится ΠΏΠΎΠ΄ Π±ΠΎΠ»Π΅Π΅ высоким ΠΏΠΎΡ‚Π΅Π½Ρ†ΠΈΠ°Π»ΠΎΠΌ, Ρ‡Π΅ΠΌ проводящая паста. Когда Π²ΠΎΠ·Π½ΠΈΠΊΠ°Π΅Ρ‚ обратная поляризация, элСктролитичСскоС дСйствиС Ρ€Π°Π·Ρ€ΡƒΡˆΠ°Π΅Ρ‚ ΠΎΠΊΡΠΈΠ΄Π½ΡƒΡŽ ΠΏΠ»Π΅Π½ΠΊΡƒ.Π­Ρ‚ΠΎΡ‚ Ρ‚ΠΈΠΏ кондСнсатора Π½Π΅ ΠΌΠΎΠΆΠ΅Ρ‚ Π±Ρ‹Ρ‚ΡŒ ΠΏΠΎΠ΄ΠΊΠ»ΡŽΡ‡Π΅Π½ ΠΊ источнику ΠΏΠ΅Ρ€Π΅ΠΌΠ΅Π½Π½ΠΎΠ³ΠΎ Ρ‚ΠΎΠΊΠ°, ΠΏΠΎΡ‚ΠΎΠΌΡƒ Ρ‡Ρ‚ΠΎ Π² ΠΏΠΎΠ»ΠΎΠ²ΠΈΠ½Π΅ случаСв ΠΏΠ΅Ρ€Π΅ΠΌΠ΅Π½Π½ΠΎΠ΅ напряТСниС Π±ΡƒΠ΄Π΅Ρ‚ ΠΈΠΌΠ΅Ρ‚ΡŒ Π½Π΅ΠΏΡ€Π°Π²ΠΈΠ»ΡŒΠ½ΡƒΡŽ ΠΏΠΎΠ»ΡΡ€Π½ΠΎΡΡ‚ΡŒ, ΠΏΠΎΡΠΊΠΎΠ»ΡŒΠΊΡƒ ΠΏΠ΅Ρ€Π΅ΠΌΠ΅Π½Π½Ρ‹ΠΉ Ρ‚ΠΎΠΊ мСняСт свою ΠΏΠΎΠ»ΡΡ€Π½ΠΎΡΡ‚ΡŒ (см. Π‘Ρ…Π΅ΠΌΡ‹ ΠΏΠ΅Ρ€Π΅ΠΌΠ΅Π½Π½ΠΎΠ³ΠΎ Ρ‚ΠΎΠΊΠ° Π² цСпях ΠΏΠ΅Ρ€Π΅ΠΌΠ΅Π½Π½ΠΎΠ³ΠΎ Ρ‚ΠΎΠΊΠ°).

ΠšΠΎΠ½Π΄Π΅Π½ΡΠ°Ρ‚ΠΎΡ€ ΠΏΠ΅Ρ€Π΅ΠΌΠ΅Π½Π½ΠΎΠ³ΠΎ Ρ‚ΠΎΠΊΠ° (рисунок 8.8) ΠΈΠΌΠ΅Π΅Ρ‚ Π΄Π²Π° Π½Π°Π±ΠΎΡ€Π° ΠΏΠ°Ρ€Π°Π»Π»Π΅Π»ΡŒΠ½Ρ‹Ρ… пластин. Один Π½Π°Π±ΠΎΡ€ пластин Π·Π°ΠΊΡ€Π΅ΠΏΠ»Π΅Π½ (ΠΎΠ±ΠΎΠ·Π½Π°Ρ‡Π΅Π½ ΠΊΠ°ΠΊ «статор»), Π° Π΄Ρ€ΡƒΠ³ΠΎΠΉ Π½Π°Π±ΠΎΡ€ пластин ΠΏΡ€ΠΈΠΊΡ€Π΅ΠΏΠ»Π΅Π½ ΠΊ Π²Π°Π»Ρƒ, ΠΊΠΎΡ‚ΠΎΡ€Ρ‹ΠΉ ΠΌΠΎΠΆΠ΅Ρ‚ Π²Ρ€Π°Ρ‰Π°Ρ‚ΡŒΡΡ (обозначаСтся ΠΊΠ°ΠΊ Β«Ρ€ΠΎΡ‚ΠΎΡ€Β»). ΠŸΠΎΠ²ΠΎΡ€Π°Ρ‡ΠΈΠ²Π°Ρ Π²Π°Π», ΠΌΠΎΠΆΠ½ΠΎ ΠΈΠ·ΠΌΠ΅Π½ΡΡ‚ΡŒ ΠΏΠ»ΠΎΡ‰Π°Π΄ΡŒ ΠΏΠΎΠΏΠ΅Ρ€Π΅Ρ‡Π½ΠΎΠ³ΠΎ сСчСния Π² ΠΏΠ΅Ρ€Π΅ΠΊΡ€Ρ‹Ρ‚ΠΈΠΈ пластин; ΡΠ»Π΅Π΄ΠΎΠ²Π°Ρ‚Π΅Π»ΡŒΠ½ΠΎ, Π΅ΠΌΠΊΠΎΡΡ‚ΡŒ этой систСмы ΠΌΠΎΠΆΠ΅Ρ‚ Π±Ρ‹Ρ‚ΡŒ настроСна Π½Π° ΠΆΠ΅Π»Π°Π΅ΠΌΠΎΠ΅ Π·Π½Π°Ρ‡Π΅Π½ΠΈΠ΅.Настройка кондСнсатора Π½Π°Ρ…ΠΎΠ΄ΠΈΡ‚ ΠΏΡ€ΠΈΠΌΠ΅Π½Π΅Π½ΠΈΠ΅ Π² любом Ρ‚ΠΈΠΏΠ΅ Ρ€Π°Π΄ΠΈΠΎΠΏΠ΅Ρ€Π΅Π΄Π°Ρ‡ΠΈ ΠΈ ΠΏΡ€ΠΈ ΠΏΡ€ΠΈΠ΅ΠΌΠ΅ радиосигналов ΠΎΡ‚ элСктронных устройств. ΠšΠ°ΠΆΠ΄Ρ‹ΠΉ Ρ€Π°Π·, ΠΊΠΎΠ³Π΄Π° Π²Ρ‹ настраиваСтС Π°Π²Ρ‚ΠΎΠΌΠΎΠ±ΠΈΠ»ΡŒΠ½ΠΎΠ΅ Ρ€Π°Π΄ΠΈΠΎ Π½Π° Π»ΡŽΠ±ΠΈΠΌΡƒΡŽ ΡΡ‚Π°Π½Ρ†ΠΈΡŽ, Π΄ΡƒΠΌΠ°ΠΉΡ‚Π΅ ΠΎ Смкости.

Π€ΠΈΠ³ΡƒΡ€Π°
8,8

Π’ ΠΏΠ΅Ρ€Π΅ΠΌΠ΅Π½Π½ΠΎΠΌ Π²ΠΎΠ·Π΄ΡƒΡˆΠ½ΠΎΠΌ кондСнсаторС Π΅ΠΌΠΊΠΎΡΡ‚ΡŒ ΠΌΠΎΠΆΠ½ΠΎ Ρ€Π΅Π³ΡƒΠ»ΠΈΡ€ΠΎΠ²Π°Ρ‚ΡŒ, измСняя ΡΡ„Ρ„Π΅ΠΊΡ‚ΠΈΠ²Π½ΡƒΡŽ ΠΏΠ»ΠΎΡ‰Π°Π΄ΡŒ пластин. (ΠΊΡ€Π΅Π΄ΠΈΡ‚: модификация Ρ€Π°Π±ΠΎΡ‚Ρ‹ Π ΠΎΠ±Π±ΠΈ Π‘ΠΏΡ€ΡƒΠ»Π°)

Π‘ΠΈΠΌΠ²ΠΎΠ»Ρ‹, ΠΏΠΎΠΊΠ°Π·Π°Π½Π½Ρ‹Π΅ Π½Π° рисункС 8.9, ΠΏΡ€Π΅Π΄ΡΡ‚Π°Π²Π»ΡΡŽΡ‚ собой схСмныС изобраТСния Ρ€Π°Π·Π»ΠΈΡ‡Π½Ρ‹Ρ… Ρ‚ΠΈΠΏΠΎΠ² кондСнсаторов.ΠžΠ±Ρ‹Ρ‡Π½ΠΎ ΠΌΡ‹ ΠΈΡΠΏΠΎΠ»ΡŒΠ·ΡƒΠ΅ΠΌ символ, ΠΏΠΎΠΊΠ°Π·Π°Π½Π½Ρ‹ΠΉ Π½Π° рис. 8.9 (Π°). Π‘ΠΈΠΌΠ²ΠΎΠ» Π½Π° РисункС 8.9 (c) прСдставляСт кондСнсатор ΠΏΠ΅Ρ€Π΅ΠΌΠ΅Π½Π½ΠΎΠΉ Смкости. ΠžΠ±Ρ€Π°Ρ‚ΠΈΡ‚Π΅ Π²Π½ΠΈΠΌΠ°Π½ΠΈΠ΅ Π½Π° сходство этих символов с симмСтриСй кондСнсатора с ΠΏΠ°Ρ€Π°Π»Π»Π΅Π»ΡŒΠ½Ρ‹ΠΌΠΈ пластинами. ЭлСктролитичСский кондСнсатор прСдставлСн символом Π½Π° рис. 8.9 (b), Π³Π΄Π΅ изогнутая пластина ΠΎΠ±ΠΎΠ·Π½Π°Ρ‡Π°Π΅Ρ‚ ΠΎΡ‚Ρ€ΠΈΡ†Π°Ρ‚Π΅Π»ΡŒΠ½Ρ‹ΠΉ Π²Ρ‹Π²ΠΎΠ΄.

Π€ΠΈΠ³ΡƒΡ€Π°
8.9

Π—Π΄Π΅ΡΡŒ ΠΏΠΎΠΊΠ°Π·Π°Π½Ρ‹ Ρ‚Ρ€ΠΈ Ρ€Π°Π·Π»ΠΈΡ‡Π½Ρ‹Ρ… схСмных прСдставлСния кондСнсаторов. Π‘ΠΈΠΌΠ²ΠΎΠ» Π² (Π°) являСтся Π½Π°ΠΈΠ±ΠΎΠ»Π΅Π΅ часто ΠΈΡΠΏΠΎΠ»ΡŒΠ·ΡƒΠ΅ΠΌΡ‹ΠΌ.Π‘ΠΈΠΌΠ²ΠΎΠ» Π² (b) прСдставляСт собой элСктролитичСский кондСнсатор. Π‘ΠΈΠΌΠ²ΠΎΠ» Π² (c) прСдставляСт кондСнсатор ΠΏΠ΅Ρ€Π΅ΠΌΠ΅Π½Π½ΠΎΠΉ Смкости.

Π˜Π½Ρ‚Π΅Ρ€Π΅ΡΠ½Ρ‹ΠΉ ΠΏΡ€ΠΈΠΊΠ»Π°Π΄Π½ΠΎΠΉ ΠΏΡ€ΠΈΠΌΠ΅Ρ€ ΠΌΠΎΠ΄Π΅Π»ΠΈ кондСнсатора взят ΠΈΠ· ΠΊΠ»Π΅Ρ‚ΠΎΡ‡Π½ΠΎΠΉ Π±ΠΈΠΎΠ»ΠΎΠ³ΠΈΠΈ ΠΈ ΠΈΠΌΠ΅Π΅Ρ‚ Π΄Π΅Π»ΠΎ с элСктричСским ΠΏΠΎΡ‚Π΅Π½Ρ†ΠΈΠ°Π»ΠΎΠΌ Π² плазматичСской ΠΌΠ΅ΠΌΠ±Ρ€Π°Π½Π΅ ΠΆΠΈΠ²ΠΎΠΉ ΠΊΠ»Π΅Ρ‚ΠΊΠΈ (рис. 8.10). ΠšΠ»Π΅Ρ‚ΠΎΡ‡Π½Ρ‹Π΅ ΠΌΠ΅ΠΌΠ±Ρ€Π°Π½Ρ‹ ΠΎΡ‚Π΄Π΅Π»ΡΡŽΡ‚ ΠΊΠ»Π΅Ρ‚ΠΊΠΈ ΠΎΡ‚ ΠΈΡ… окруТСния, Π½ΠΎ ΠΏΠΎΠ·Π²ΠΎΠ»ΡΡŽΡ‚ Π½Π΅ΠΊΠΎΡ‚ΠΎΡ€Ρ‹ΠΌ ΠΎΡ‚ΠΎΠ±Ρ€Π°Π½Π½Ρ‹ΠΌ ΠΈΠΎΠ½Π°ΠΌ ΠΏΡ€ΠΎΡ…ΠΎΠ΄ΠΈΡ‚ΡŒ Π²Π½ΡƒΡ‚Ρ€ΡŒ ΠΈΠ»ΠΈ ΠΈΠ· ΠΊΠ»Π΅Ρ‚ΠΊΠΈ. Π Π°Π·Π½ΠΎΡΡ‚ΡŒ ΠΏΠΎΡ‚Π΅Π½Ρ†ΠΈΠ°Π»ΠΎΠ² Π½Π° ΠΌΠ΅ΠΌΠ±Ρ€Π°Π½Π΅ составляСт ΠΎΠΊΠΎΠ»ΠΎ 70 ΠΌΠ’. ΠšΠ»Π΅Ρ‚ΠΎΡ‡Π½Π°Ρ ΠΌΠ΅ΠΌΠ±Ρ€Π°Π½Π° ΠΌΠΎΠΆΠ΅Ρ‚ ΠΈΠΌΠ΅Ρ‚ΡŒ Ρ‚ΠΎΠ»Ρ‰ΠΈΠ½Ρƒ ΠΎΡ‚ 7 Π΄ΠΎ 10 Π½ΠΌ.Рассматривая ΠΊΠ»Π΅Ρ‚ΠΎΡ‡Π½ΡƒΡŽ ΠΌΠ΅ΠΌΠ±Ρ€Π°Π½Ρƒ ΠΊΠ°ΠΊ Π½Π°Π½ΠΎΡ€Π°Π·ΠΌΠ΅Ρ€Π½Ρ‹ΠΉ кондСнсатор, ΠΎΡ†Π΅Π½ΠΊΠ° наимСньшСй напряТСнности элСктричСского поля Π½Π° Π΅Π³ΠΎ « пластинах » Π΄Π°Π΅Ρ‚ Π·Π½Π°Ρ‡Π΅Π½ΠΈΠ΅ E = Vd = 70 Γ— 10βˆ’3V Β· 10 Β· 10βˆ’9m = 7 Β· 106V / m> 3MV / mE. = Vd = 70 Γ— 10βˆ’3V10 Γ— 10βˆ’9m = 7 Γ— 106V / m> 3MV / m.

Π­Ρ‚ΠΎΠΉ Π²Π΅Π»ΠΈΡ‡ΠΈΠ½Ρ‹ элСктричСского поля достаточно, Ρ‡Ρ‚ΠΎΠ±Ρ‹ Π²Ρ‹Π·Π²Π°Ρ‚ΡŒ ΡΠ»Π΅ΠΊΡ‚Ρ€ΠΈΡ‡Π΅ΡΠΊΡƒΡŽ искру Π² Π²ΠΎΠ·Π΄ΡƒΡ…Π΅.

Π€ΠΈΠ³ΡƒΡ€Π°
8.10

ΠŸΠΎΠ»ΡƒΠΏΡ€ΠΎΠ½ΠΈΡ†Π°Π΅ΠΌΠ°Ρ ΠΌΠ΅ΠΌΠ±Ρ€Π°Π½Π° биологичСской ΠΊΠ»Π΅Ρ‚ΠΊΠΈ ΠΈΠΌΠ΅Π΅Ρ‚ Ρ€Π°Π·Π½Ρ‹Π΅ ΠΊΠΎΠ½Ρ†Π΅Π½Ρ‚Ρ€Π°Ρ†ΠΈΠΈ ΠΈΠΎΠ½ΠΎΠ² Π½Π° Π²Π½ΡƒΡ‚Ρ€Π΅Π½Π½Π΅ΠΉ повСрхности, Ρ‡Π΅ΠΌ Π½Π° внСшнСй.Диффузия ΠΏΠ΅Ρ€Π΅ΠΌΠ΅Ρ‰Π°Π΅Ρ‚ ΠΈΠΎΠ½Ρ‹ K + K + (ΠΊΠ°Π»ΠΈΠΉ) ΠΈ Cl – Cl– (Ρ…Π»ΠΎΡ€ΠΈΠ΄) Π² ΠΏΠΎΠΊΠ°Π·Π°Π½Π½Ρ‹Ρ… направлСниях, ΠΏΠΎΠΊΠ° кулоновская сила Π½Π΅ остановит дальнСйший пСрСнос. Π’Π°ΠΊΠΈΠΌ ΠΎΠ±Ρ€Π°Π·ΠΎΠΌ, внСшняя Ρ‡Π°ΡΡ‚ΡŒ ΠΌΠ΅ΠΌΠ±Ρ€Π°Π½Ρ‹ ΠΏΡ€ΠΈΠΎΠ±Ρ€Π΅Ρ‚Π°Π΅Ρ‚ ΠΏΠΎΠ»ΠΎΠΆΠΈΡ‚Π΅Π»ΡŒΠ½Ρ‹ΠΉ заряд, Π° Π΅Π΅ внутрСнняя ΠΏΠΎΠ²Π΅Ρ€Ρ…Π½ΠΎΡΡ‚ΡŒ ΠΏΡ€ΠΈΠΎΠ±Ρ€Π΅Ρ‚Π°Π΅Ρ‚ ΠΎΡ‚Ρ€ΠΈΡ†Π°Ρ‚Π΅Π»ΡŒΠ½Ρ‹ΠΉ заряд, создавая Ρ€Π°Π·Π½ΠΎΡΡ‚ΡŒ ΠΏΠΎΡ‚Π΅Π½Ρ†ΠΈΠ°Π»ΠΎΠ² Π½Π° ΠΌΠ΅ΠΌΠ±Ρ€Π°Π½Π΅. ΠœΠ΅ΠΌΠ±Ρ€Π°Π½Π° ΠΎΠ±Ρ‹Ρ‡Π½ΠΎ Π½Π΅ΠΏΡ€ΠΎΠ½ΠΈΡ†Π°Π΅ΠΌΠ° для Na + (ΠΈΠΎΠ½ΠΎΠ² натрия).

Π€ΠΈΠ·ΠΈΠΊΠ° для Π½Π°ΡƒΠΊΠΈ ΠΈ Ρ‚Π΅Ρ…Π½ΠΈΠΊΠΈ II

5.5 ЦилиндричСский кондСнсатор ΠΎΡ‚ Office of Academic Technologies Π½Π° Vimeo.

Для дСмонстраций см .:
http://maxwell.uncc.edu/aktas/PHYS2102nline/index2.html

5,05 ЦилиндричСский кондСнсатор

Π’Π΅ΠΏΠ΅Ρ€ΡŒ рассчитаСм Π΅ΠΌΠΊΠΎΡΡ‚ΡŒ цилиндричСского кондСнсатора. Как слСдуСт ΠΈΠ· названия, Ρ‚Π΅ΠΏΠ΅Ρ€ΡŒ ΠΌΡ‹ ΠΈΠΌΠ΅Π΅ΠΌ Π΄Π΅Π»ΠΎ с кондСнсатором, ΠΊΠΎΡ‚ΠΎΡ€Ρ‹ΠΉ состоит ΠΈΠ· Π΄Π²ΡƒΡ… концСнтричСских проводящих цилиндричСских повСрхностСй, скаТСм Ρ‚Π°ΠΊ, это большая ΠΏΠΎΠ²Π΅Ρ€Ρ…Π½ΠΎΡΡ‚ΡŒ ΠΈΠ»ΠΈ внСшняя ΠΏΠΎΠ²Π΅Ρ€Ρ…Π½ΠΎΡΡ‚ΡŒ ΠΈ мСньшая концСнтричСская внутрСнняя ΠΏΠΎΠ²Π΅Ρ€Ρ…Π½ΠΎΡΡ‚ΡŒ. Π₯ΠΎΡ€ΠΎΡˆΠΎ. ΠŸΠΎΡΡ‚ΠΎΠΌΡƒ наш цилиндричСский кондСнсатор выглядит ΠΏΡ€ΠΈΠΌΠ΅Ρ€Π½ΠΎ Ρ‚Π°ΠΊ.И Π΄Π°Π²Π°ΠΉΡ‚Π΅ Ρ‚Π°ΠΊΠΆΠ΅ Π΄Π°Π΄ΠΈΠΌ Π½Π΅ΠΊΠΎΡ‚ΠΎΡ€Ρ‹Π΅ Ρ€Π°Π·ΠΌΠ΅Ρ€Ρ‹ Π½Π°ΡˆΠ΅ΠΌΡƒ кондСнсатору.

ΠŸΡ€Π΅Π΄ΠΏΠΎΠ»ΠΎΠΆΠΈΠΌ, Ρ‡Ρ‚ΠΎ Π΄Π»ΠΈΠ½Π° кондСнсатора Ρ€Π°Π²Π½Π° h, Π° Π²Π½ΡƒΡ‚Ρ€Π΅Π½Π½ΠΈΠΉ радиус Ρ€Π°Π²Π΅Π½ a, Π΄Ρ€ΡƒΠ³ΠΎΠΉ радиус Ρ€Π°Π²Π΅Π½ b. И ΠΌΡ‹ заряТаСм наш кондСнсатор Ρ‚Π°ΠΊ, Ρ‡Ρ‚ΠΎΠ±Ρ‹ ΠΌΡ‹ ΠΏΠΎΠ΄ΠΊΠ»ΡŽΡ‡Π°Π»ΠΈ Π²Π½ΡƒΡ‚Ρ€Π΅Π½Π½ΡŽΡŽ ΠΏΠΎΠ²Π΅Ρ€Ρ…Π½ΠΎΡΡ‚ΡŒ ΠΊ ΠΏΠΎΠ»ΠΎΠΆΠΈΡ‚Π΅Π»ΡŒΠ½ΠΎΠΉ ΠΊΠ»Π΅ΠΌΠΌΠ΅ источника питания, Π° внСшнюю ΠΏΠΎΠ²Π΅Ρ€Ρ…Π½ΠΎΡΡ‚ΡŒ — ΠΊ ΠΎΡ‚Ρ€ΠΈΡ†Π°Ρ‚Π΅Π»ΡŒΠ½ΠΎΠΉ ΠΊΠ»Π΅ΠΌΠΌΠ΅ источника питания нашСй Π±Π°Ρ‚Π°Ρ€Π΅ΠΈ, скаТСм. Π’Π°ΠΊΠΈΠΌ ΠΎΠ±Ρ€Π°Π·ΠΎΠΌ, внутрСнняя пластина Π±ΡƒΠ΄Π΅Ρ‚ Π·Π°Ρ€ΡΠΆΠ°Ρ‚ΡŒΡΡ Π²Π΅Π·Π΄Π΅ ΠΏΠΎΠ»ΠΎΠΆΠΈΡ‚Π΅Π»ΡŒΠ½ΠΎ, Π° внСшняя пластина Π±ΡƒΠ΄Π΅Ρ‚ Π·Π°Ρ€ΡΠΆΠ°Ρ‚ΡŒΡΡ ΠΎΡ‚Ρ€ΠΈΡ†Π°Ρ‚Π΅Π»ΡŒΠ½ΠΎ. Допустим, Π²Π΅Π»ΠΈΡ‡ΠΈΠ½Π° этого заряда Ρ€Π°Π²Π½Π° плюс q, Π° внутрСнняя пластина, минус q, ΠΏΡ€ΠΎΡ…ΠΎΠ΄ΠΈΡ‚ вдоль внСшнСй пластины.

Π‘Π»Π΅Π΄ΠΎΠ²Π°Ρ‚Π΅Π»ΡŒΠ½ΠΎ, заряТая кондСнсатор, ΠΌΡ‹ ΡƒΠΆΠ΅ Π²Ρ‹ΠΏΠΎΠ»Π½ΠΈΠ»ΠΈ ΠΏΠ΅Ρ€Π²Ρ‹ΠΉ шаг ΠΏΠΎ Π²Ρ‹Ρ‡ΠΈΡΠ»Π΅Π½ΠΈΡŽ Смкости ΠΈ Π³ΠΎΠ²ΠΎΡ€ΠΈΠΌ, Ρ‡Ρ‚ΠΎ кондСнсатор заряТСн Π΄ΠΎ Π½Π΅ΡΠΊΠΎΠ»ΡŒΠΊΠΈΡ… q ΠΊΡƒΠ»ΠΎΠ½ΠΎΠ². И Π² качСствС Π²Ρ‚ΠΎΡ€ΠΎΠ³ΠΎ шага ΠΌΡ‹ рассчитаСм элСктричСскоС ΠΏΠΎΠ»Π΅ ΠΌΠ΅ΠΆΠ΄Ρƒ пластинами этого кондСнсатора, ΠΏΡ€ΠΈΠΌΠ΅Π½ΠΈΠ² Π·Π°ΠΊΠΎΠ½ Гаусса, ΠΊΠΎΡ‚ΠΎΡ€Ρ‹ΠΉ Π±Ρ‹Π» ΠΈΠ½Ρ‚Π΅Π³Ρ€Π°Π»ΠΎΠΌ ΠΎΡ‚ E dot dA Π½Π° Π·Π°ΠΌΠΊΠ½ΡƒΡ‚ΠΎΠΉ повСрхности, Ρ€Π°Π²Π½ΠΎΠΌ чистому заряду Π²Π½ΡƒΡ‚Ρ€ΠΈ объСма, ΠΎΠΊΡ€ΡƒΠΆΠ΅Π½Π½ΠΎΠ³ΠΎ этой Π·Π°ΠΌΠΊΠ½ΡƒΡ‚ΠΎΠΉ ΠΏΠΎΠ²Π΅Ρ€Ρ…Π½ΠΎΡΡ‚ΡŒΡŽ, q, Π·Π°ΠΊΠ»ΡŽΡ‡Π΅Π½Π½Ρ‹ΠΉ Π½Π°Π΄ Эпсилоном 0.

ΠžΠΏΡΡ‚ΡŒ ΠΆΠ΅, ΠΈΠ·ΠΎΠ»ΠΈΡ€ΡƒΡŽΡ‰Π°Ρ срСда ΠΌΠ΅ΠΆΠ΄Ρƒ этими двумя проводящими пластинами — Π²ΠΎΠ·Π΄ΡƒΡ…, поэтому это ΠΏΡ€ΠΎΠ²ΠΎΠ΄Π½ΠΈΠΊ, Π° это ΠΏΡ€ΠΎΠ²ΠΎΠ΄Π½ΠΈΠΊ, Π½Π°ΠΏΡ€ΠΈΠΌΠ΅Ρ€, ΠΊΠ°ΠΊ Π°Π»ΡŽΠΌΠΈΠ½ΠΈΠ΅Π²Ρ‹Π΅ пластины. Π₯ΠΎΡ€ΠΎΡˆΠΎ. Π˜Ρ‚Π°ΠΊ, Ρ‚Π΅ΠΏΠ΅Ρ€ΡŒ нас интСрСсуСт элСктричСскоС ΠΏΠΎΠ»Π΅ ΠΌΠ΅ΠΆΠ΄Ρƒ пластинами. Если ΠΌΡ‹ посмотрим Π½Π° Π³Π΅ΠΎΠΌΠ΅Ρ‚Ρ€ΠΈΡŽ пластин, ΠΌΡ‹ ΡƒΠ²ΠΈΠ΄ΠΈΠΌ, Ρ‡Ρ‚ΠΎ элСктричСскоС ΠΏΠΎΠ»Π΅ Π±ΡƒΠ΄Π΅Ρ‚ ΠΈΡΡ…ΠΎΠ΄ΠΈΡ‚ΡŒ ΠΎΡ‚ Π²Π½ΡƒΡ‚Ρ€Π΅Π½Π½Π΅ΠΉ ΠΏΠΎΠ»ΠΎΠΆΠΈΡ‚Π΅Π»ΡŒΠ½ΠΎ заряТСнной пластины ΠΈ Π²ΠΎΠΉΠ΄Π΅Ρ‚ Π² ΠΎΡ‚Ρ€ΠΈΡ†Π°Ρ‚Π΅Π»ΡŒΠ½ΠΎ Π·Π°Ρ€ΡΠΆΠ΅Π½Π½ΡƒΡŽ внСшнюю пластину, Π² Π΄Π°Π½Π½ΠΎΠΌ случаС Ρ€Π°Π΄ΠΈΠ°Π»ΡŒΠ½ΠΎ Π½Π°Ρ€ΡƒΠΆΡƒ. Π‘Π»Π΅Π΄ΠΎΠ²Π°Ρ‚Π΅Π»ΡŒΠ½ΠΎ, элСктричСскоС ΠΏΠΎΠ»Π΅ Π±ΡƒΠ΄Π΅Ρ‚ Π·Π°ΠΏΠΎΠ»Π½ΡΡ‚ΡŒ пространство ΠΎΡ‚ ΠΏΠΎΠ»ΠΎΠΆΠΈΡ‚Π΅Π»ΡŒΠ½ΠΎΠΉ пластины Π΄ΠΎ ΠΎΡ‚Ρ€ΠΈΡ†Π°Ρ‚Π΅Π»ΡŒΠ½ΠΎΠΉ Π² Ρ€Π°Π΄ΠΈΠ°Π»ΡŒΠ½ΠΎΠΌ Π½Π°ΠΏΡ€Π°Π²Π»Π΅Π½ΠΈΠΈ Π½Π°Ρ€ΡƒΠΆΡƒ.

Вся ΠΎΠ±Π»Π°ΡΡ‚ΡŒ ΠΌΠ΅ΠΆΠ΄Ρƒ пластинами Π±ΡƒΠ΄Π΅Ρ‚ Π·Π°ΠΏΠΎΠ»Π½Π΅Π½Π° элСктричСским ΠΏΠΎΠ»Π΅ΠΌ, создаваСмым этими двумя заряТСнными пластинами. Π˜Ρ‚Π°ΠΊ, ΠΏΠΎΡΠΊΠΎΠ»ΡŒΠΊΡƒ ΠΌΡ‹ ΠΈΠΌΠ΅Π΅ΠΌ Π΄Π΅Π»ΠΎ с цилиндричСской Π³Π΅ΠΎΠΌΠ΅Ρ‚Ρ€ΠΈΠ΅ΠΉ, ΠΌΡ‹ собираСмся Π²Ρ‹Π±Ρ€Π°Ρ‚ΡŒ Π½Π°ΡˆΡƒ гауссову ΠΏΠΎΠ²Π΅Ρ€Ρ…Π½ΠΎΡΡ‚ΡŒ Π² Ρ„ΠΎΡ€ΠΌΠ΅ Ρ†ΠΈΠ»ΠΈΠ½Π΄Ρ€Π°, Ρ‚Π°ΠΊ Ρ‡Ρ‚ΠΎΠ±Ρ‹ Π΅Π³ΠΎ боковая ΠΏΠΎΠ²Π΅Ρ€Ρ…Π½ΠΎΡΡ‚ΡŒ ΠΏΡ€ΠΎΡ…ΠΎΠ΄ΠΈΠ»Π° Ρ‡Π΅Ρ€Π΅Π· ΠΈΠ½Ρ‚Π΅Ρ€Π΅ΡΡƒΡŽΡ‰ΡƒΡŽ Ρ‚ΠΎΡ‡ΠΊΡƒ. НашСй Ρ‚ΠΎΡ‡ΠΊΠΎΠΉ интСрСса Π² Π΄Π°Π½Π½ΠΎΠΌ случаС Π±ΡƒΠ΄Π΅Ρ‚ любая Ρ‚ΠΎΡ‡ΠΊΠ° ΠΌΠ΅ΠΆΠ΄Ρƒ пластинами. Π”Π°Π²Π°ΠΉΡ‚Π΅ Π²Ρ‹Π±Π΅Ρ€Π΅ΠΌ эту Ρ‚ΠΎΡ‡ΠΊΡƒ ΠΊΠ°ΠΊ-Π½ΠΈΠ±ΡƒΠ΄ΡŒ подальшС ΠΎΡ‚ ΠΊΠΎΠ½Ρ†ΠΎΠ² этого Ρ†ΠΈΠ»ΠΈΠ½Π΄Ρ€Π°, Π³Π΄Π΅-Π½ΠΈΠ±ΡƒΠ΄ΡŒ ΠΏΡ€ΠΈΠΌΠ΅Ρ€Π½ΠΎ посСрСдинС Π² этой Ρ‚ΠΎΡ‡ΠΊΠ΅, стр.Π­Ρ‚ΠΎ ΠΎΠ±Π»Π°ΡΡ‚ΡŒ ΠΌΠ΅ΠΆΠ΄Ρƒ пластинами, ΠΈ ΠΌΡ‹ Π²Ρ‹Π±ΠΈΡ€Π°Π΅ΠΌ наш гауссовский Ρ†ΠΈΠ»ΠΈΠ½Π΄Ρ€, гауссову ΠΏΠΎΠ²Π΅Ρ€Ρ…Π½ΠΎΡΡ‚ΡŒ Π² Ρ„ΠΎΡ€ΠΌΠ΅ Ρ†ΠΈΠ»ΠΈΠ½Π΄Ρ€Π°, Ρ‚Π°ΠΊ Ρ‡Ρ‚ΠΎΠ±Ρ‹ Π΅Π³ΠΎ боковая ΠΏΠΎΠ²Π΅Ρ€Ρ…Π½ΠΎΡΡ‚ΡŒ ΠΏΡ€ΠΎΡ…ΠΎΠ΄ΠΈΠ»Π° Ρ‡Π΅Ρ€Π΅Π· ΠΈΠ½Ρ‚Π΅Ρ€Π΅ΡΡƒΡŽΡ‰ΡƒΡŽ Ρ‚ΠΎΡ‡ΠΊΡƒ. Π‘Π»Π΅Π΄ΠΎΠ²Π°Ρ‚Π΅Π»ΡŒΠ½ΠΎ, это Π±ΡƒΠ΄Π΅Ρ‚ ΠΏΠΎΠ²Π΅Ρ€Ρ…Π½ΠΎΡΡ‚ΡŒ, похоТая Π½Π° эту, Ρ†ΠΈΠ»ΠΈΠ½Π΄Ρ€ΠΈΡ‡Π΅ΡΠΊΡƒΡŽ ΠΏΠΎΠ²Π΅Ρ€Ρ…Π½ΠΎΡΡ‚ΡŒ.

Π’Π΅ΠΏΠ΅Ρ€ΡŒ, ΠΊΠΎΠ³Π΄Π° ΠΌΡ‹ примСняСм этот Π·Π°ΠΊΠΎΠ½ Гаусса, Ρ‡Ρ‚ΠΎΠ±Ρ‹ Π²Ρ‹Ρ‡ΠΈΡΠ»ΠΈΡ‚ΡŒ элСктричСскоС ΠΏΠΎΠ»Π΅ Π² этой Ρ‚ΠΎΡ‡ΠΊΠ΅ ΠΌΠ΅ΠΆΠ΄Ρƒ пластинами, ΠΌΡ‹ собираСмся ΡΠ΄Π΅Π»Π°Ρ‚ΡŒ ΠΏΡ€Π΅Π΄ΠΏΠΎΠ»ΠΎΠΆΠ΅Π½ΠΈΠ΅, Ρ‡Ρ‚ΠΎ ΠΌΡ‹ Π΄Π°Π»Π΅ΠΊΠΎ ΠΎΡ‚ ΠΊΠΎΠ½Π΅Ρ‡Π½Ρ‹Ρ… Ρ‚ΠΎΡ‡Π΅ΠΊ, ΠΈ ΠΌΡ‹ собираСмся ΠΏΡ€Π΅Π½Π΅Π±Ρ€Π΅Ρ‡ΡŒ ΠΊΠΎΠ½Π΅Ρ‡Π½Ρ‹Π΅ эффСкты.ΠŸΡ€Π΅Π½Π΅Π±Ρ€Π΅ΠΆΠ΅Π½ΠΈΠ΅ ΠΊΠΎΠ½Π΅Ρ‡Π½Ρ‹ΠΌΠΈ эффСктами ΠΎΠ·Π½Π°Ρ‡Π°Π΅Ρ‚, Ρ‡Ρ‚ΠΎ, ΠΊΠΎΠ½Π΅Ρ‡Π½ΠΎ, ΠΊΠΎΠ³Π΄Π° ΠΌΡ‹ ΠΏΠ΅Ρ€Π΅Ρ…ΠΎΠ΄ΠΈΠΌ ΠΊ этой Π²Π΅Ρ€Ρ…Π½Π΅ΠΉ Π³Ρ€Π°Π½ΠΈΡ†Π΅, элСктричСскоС ΠΏΠΎΠ»Π΅ здСсь Π½Π΅ Π±ΡƒΠ΄Π΅Ρ‚ Π½Π°ΠΏΡ€Π°Π²Π»Π΅Π½ΠΎ Ρ€Π°Π΄ΠΈΠ°Π»ΡŒΠ½ΠΎ Π½Π°Ρ€ΡƒΠΆΡƒ, ΠΏΠΎΡ‚ΠΎΠΌΡƒ Ρ‡Ρ‚ΠΎ Ρƒ нас Π±ΡƒΠ΄Π΅Ρ‚ элСктричСскоС ΠΏΠΎΠ»Π΅, Π½Π°ΠΏΡ€Π°Π²Π»Π΅Π½Π½ΠΎΠ΅ Π²Π²Π΅Ρ€Ρ…, Π° Ρ€Π΅Π·ΡƒΠ»ΡŒΡ‚ΠΈΡ€ΡƒΡŽΡ‰Π΅Π΅ ΠΏΠΎΠ»Π΅ Π±ΡƒΠ΄Π΅Ρ‚ Π²Π΅ΠΊΡ‚ΠΎΡ€Π½ΠΎΠΉ суммой Ρ‚Π΅ Π΄Π²Π°. Но ΠΌΡ‹ Π½Π΅ Π±ΡƒΠ΄Π΅ΠΌ ΠΎΠ±Ρ€Π°Ρ‰Π°Ρ‚ΡŒ внимания Π½Π° эти ΠΊΠΎΠ½Π΅Ρ‡Π½Ρ‹Π΅ эффСкты. Π’ ΠΏΡ€ΠΎΡ‚ΠΈΠ²Π½ΠΎΠΌ случаС ΠΌΡ‹ Π½Π΅ смоТСм ΠΏΡ€ΠΈΠΌΠ΅Π½ΠΈΡ‚ΡŒ Π·Π°ΠΊΠΎΠ½ Гаусса, ΠΏΠΎΡ‚ΠΎΠΌΡƒ Ρ‡Ρ‚ΠΎ эта цилиндричСская ΠΏΠΎΠ²Π΅Ρ€Ρ…Π½ΠΎΡΡ‚ΡŒ Π² этом случаС Π½Π΅ Π±ΡƒΠ΄Π΅Ρ‚ ΡƒΠ΄ΠΎΠ²Π»Π΅Ρ‚Π²ΠΎΡ€ΡΡ‚ΡŒ условиям для примСнСния Π·Π°ΠΊΠΎΠ½Π° Гаусса. Π’Π°ΠΊΠΈΠΌ ΠΎΠ±Ρ€Π°Π·ΠΎΠΌ, ΠΌΡ‹ Π΄ΠΎΠ»ΠΆΠ½Ρ‹ ΡΠ΄Π΅Π»Π°Ρ‚ΡŒ это ΠΏΡ€Π΅Π΄ΠΏΠΎΠ»ΠΎΠΆΠ΅Π½ΠΈΠ΅, ΠΈ ΠΌΡ‹ скаТСм, Ρ‡Ρ‚ΠΎ ΠΏΡ€Π΅Π½Π΅Π±Ρ€Π΅Π³Π°Π΅ΠΌ ΠΊΠΎΠ½Ρ†Π΅Π²Ρ‹ΠΌΠΈ эффСктами ΠΈ ΠΏΡ€Π΅Π΄ΠΏΠΎΠ»ΠΎΠΆΠΈΠΌ, Ρ‡Ρ‚ΠΎ элСктричСскоС ΠΏΠΎΠ»Π΅ находится ΠΌΠ΅ΠΆΠ΄Ρƒ этими двумя пластинами Π² Ρ€Π°Π΄ΠΈΠ°Π»ΡŒΠ½ΠΎΠΌ Π½Π°ΠΏΡ€Π°Π²Π»Π΅Π½ΠΈΠΈ Π½Π°Ρ€ΡƒΠΆΡƒ.

Π˜Ρ‚Π°ΠΊ, учитывая это, ΠΌΡ‹ ΠΌΠΎΠΆΠ΅ΠΌ Ρ€Π°Π·Π΄Π΅Π»ΠΈΡ‚ΡŒ этот ΠΈΠ½Ρ‚Π΅Π³Ρ€Π°Π» Π·Π°ΠΌΠΊΠ½ΡƒΡ‚ΠΎΠΉ повСрхности Π½Π° сумму ΠΎΡ‚ΠΊΡ€Ρ‹Ρ‚Ρ‹Ρ… повСрхностСй, которая постСпСнно составляСт вСсь этот Ρ†ΠΈΠ»ΠΈΠ½Π΄Ρ€. И, ΠΊΠ°ΠΊ Π²Ρ‹ ΠΏΠΎΠΌΠ½ΠΈΡ‚Π΅ ΠΈΠ· цилиндричСской Π³Π΅ΠΎΠΌΠ΅Ρ‚Ρ€ΠΈΠΈ, ΠΊΠΎΠ³Π΄Π° ΠΌΡ‹ примСняСм Π·Π°ΠΊΠΎΠ½ Гаусса ΠΊ Π·Π°Π΄Π°Ρ‡Π΅ ΠΎ бСсконСчном прямом ΠΌΠ°Ρ€ΡˆΡ€ΡƒΡ‚Π΅, ΠΏΠΎΡΠΊΠΎΠ»ΡŒΠΊΡƒ открытая Ρ„ΠΎΡ€ΠΌΠ° этого Ρ†ΠΈΠ»ΠΈΠ½Π΄Ρ€Π° состоит ΠΈΠ· повСрхности ΠΏΡ€ΡΠΌΠΎΡƒΠ³ΠΎΠ»ΡŒΠ½ΠΎΠ³ΠΎ Ρ€Π°Π·ΠΌΠ΅Ρ€Π° ΠΈ ΠΊΡ€ΡƒΠ³Π»Ρ‹Ρ… Π²Π΅Ρ€Ρ…Π½Π΅ΠΉ ΠΈ Π½ΠΈΠΆΠ½Π΅ΠΉ повСрхностСй, ΠΊΠΎΡ‚ΠΎΡ€Ρ‹Π΅ эта ΠΏΡ€ΡΠΌΠΎΡƒΠ³ΠΎΠ»ΡŒΠ½Π°Ρ ΠΏΠΎΠ²Π΅Ρ€Ρ…Π½ΠΎΡΡ‚ΡŒ ΠΎΡ…Π²Π°Ρ‚Ρ‹Π²Π°Π΅Ρ‚, ΠΌΠΎΠΆΠ½ΠΎ ΡΠΊΠ°Π·Π°Ρ‚ΡŒ, Ρ‡Ρ‚ΠΎ этот ΠΈΠ½Ρ‚Π΅Π³Ρ€Π°Π» ΠΌΠΎΠΆΠ½ΠΎ Π²Ρ‹Ρ€Π°Π·ΠΈΡ‚ΡŒ ΠΊΠ°ΠΊ ΠΈΠ½Ρ‚Π΅Π³Ρ€Π°Π» ΠΏΠΎ Π²Π΅Ρ€Ρ…Π½Π΅ΠΉ повСрхности Ρ†ΠΈΠ»ΠΈΠ½Π΄Ρ€Π°, ΠΊΠΎΡ‚ΠΎΡ€Ρ‹ΠΌ являСтся этот.И Π·Π°Ρ‚Π΅ΠΌ, Ρ‚ΠΎ Π΅ΡΡ‚ΡŒ Π²Π΅Π»ΠΈΡ‡ΠΈΠ½Π° E dA Π²Π΅Π»ΠΈΡ‡ΠΈΠ½Π°.

И Ссли ΠΌΡ‹ посмотрим Π½Π° ΡƒΠ³ΠΎΠ» ΠΌΠ΅ΠΆΠ΄Ρƒ E ΠΈ Π²Π΅ΠΊΡ‚ΠΎΡ€ΠΎΠΌ dA для Π²Π΅Ρ€Ρ…Π½Π΅ΠΉ повСрхности, ΠΈ Π³Π΄Π΅ элСктричСскоС ΠΏΠΎΠ»Π΅ находится Π² этой области прямо здСсь, Ρ‚Π°ΠΊ ΠΊΠ°ΠΊ для Π²Π΅Ρ€Ρ…Π½Π΅ΠΉ повСрхности dA пСрпСндикулярно повСрхности Π²ΠΎΡ‚ Ρ‚Π°ΠΊ, Π° элСктричСскоС ΠΏΠΎΠ»Π΅ Ρ€Π°Π΄ΠΈΠ°Π»ΡŒΠ½ΠΎ Π½Π°Ρ€ΡƒΠΆΡƒ ΡƒΠ³ΠΎΠ» ΠΌΠ΅ΠΆΠ΄Ρƒ Π½ΠΈΠΌΠΈ для Π²Π΅Ρ€Ρ…Π½Π΅ΠΉ повСрхности составляСт 90 градусов.

Π˜Ρ‚Π°ΠΊ, Ρƒ нас Π±ΡƒΠ΄Π΅Ρ‚ косинус 90. А ΠΏΠΎΡΠΊΠΎΠ»ΡŒΠΊΡƒ косинус 90 Ρ€Π°Π²Π΅Π½ Π½ΡƒΠ»ΡŽ, этот ΠΈΠ½Ρ‚Π΅Π³Ρ€Π°Π» Π½Π΅ Π±ΡƒΠ΄Π΅Ρ‚ Π²Π½ΠΎΡΠΈΡ‚ΡŒ Π½ΠΈΠΊΠ°ΠΊΠΎΠ³ΠΎ Π²ΠΊΠ»Π°Π΄Π°. И плюс Ρ‚Π°ΠΊΠΎΠΉ ΠΆΠ΅ для Π½ΠΈΠΆΠ½Π΅ΠΉ повСрхности; интСгрируя ΠΏΠΎ Π½ΠΈΠΆΠ½Π΅ΠΉ повСрхности, снова dA пСрпСндикулярно повСрхности, Π½Π°ΠΏΡ€Π°Π²Π»Π΅Π½ΠΎ Π²Π½ΠΈΠ·, Π° элСктричСскоС ΠΏΠΎΠ»Π΅ Π²Ρ‹Ρ…ΠΎΠ΄ΠΈΡ‚ Ρ€Π°Π΄ΠΈΠ°Π»ΡŒΠ½ΠΎ Π½Π°Ρ€ΡƒΠΆΡƒ, заполняя всС это пространство.Π’ этой части ΠΎΠ½ Π±ΡƒΠ΄Π΅Ρ‚ ΡƒΠΊΠ°Π·Ρ‹Π²Π°Ρ‚ΡŒ Π²ΠΎΡ‚ Ρ‚Π°ΠΊ, ΠΈ снова ΡƒΠ³ΠΎΠ» ΠΌΠ΅ΠΆΠ΄Ρƒ Π½ΠΈΠΌΠΈ Π±ΡƒΠ΄Π΅Ρ‚ 90 градусов. Π’Π°ΠΊ Ρ‡Ρ‚ΠΎ это Ρ‚ΠΎΠΆΠ΅ Π½Π΅ ΠΏΠΎΠΌΠΎΠΆΠ΅Ρ‚. Π£ нас Π±ΡƒΠ΄Π΅Ρ‚ Π²Π΅Π»ΠΈΡ‡ΠΈΠ½Π° E dA, умноТСнная Π½Π° косинус 90, Ρ‡Ρ‚ΠΎ даст Π½Π°ΠΌ ноль.

И СдинствСнная ΠΎΡΡ‚Π°Π²ΡˆΠ°ΡΡΡ ΠΏΠΎΠ²Π΅Ρ€Ρ…Π½ΠΎΡΡ‚ΡŒ — это ΠΈΠ½Ρ‚Π΅Π³Ρ€Π°Π» Π±ΠΎΠΊΠΎΠ²ΠΎΠΉ повСрхности, для Π±ΠΎΠΊΠΎΠ²ΠΎΠΉ повСрхности, Ссли ΠΌΡ‹ посмотрим Π½Π° Π½Π°ΡˆΡƒ Ρ‚ΠΎΡ‡ΠΊΡƒ интСрСса, элСктричСскоС ΠΏΠΎΠ»Π΅ здСсь Ρ€Π°Π΄ΠΈΠ°Π»ΡŒΠ½ΠΎ, ΠΈ dA снова пСрпСндикулярно повСрхности. Π‘Π»Π΅Π΄ΠΎΠ²Π°Ρ‚Π΅Π»ΡŒΠ½ΠΎ, для этой Ρ‚ΠΎΡ‡ΠΊΠΈ ΠΈ для всСй Π±ΠΎΠΊΠΎΠ²ΠΎΠΉ повСрхности ΡƒΠ³ΠΎΠ» ΠΌΠ΅ΠΆΠ΄Ρƒ Π²Π΅ΠΊΡ‚ΠΎΡ€ΠΎΠΌ элСктричСского поля ΠΈ Π²Π΅ΠΊΡ‚ΠΎΡ€ΠΎΠΌ ΡƒΠ²Π΅Π»ΠΈΡ‡ΠΈΠ²Π°ΡŽΡ‰Π΅ΠΉΡΡ ΠΏΠ»ΠΎΡ‰Π°Π΄ΠΈ повСрхности Π±ΡƒΠ΄Π΅Ρ‚ просто Π½ΡƒΠ»Π΅Π²Ρ‹ΠΌ градусом.Π’Π°ΠΊΠΈΠΌ ΠΎΠ±Ρ€Π°Π·ΠΎΠΌ, для ΠΈΠ½Ρ‚Π΅Π³Ρ€Π°Π»Π° Π±ΠΎΠΊΠΎΠ²ΠΎΠΉ повСрхности Ρƒ нас Π±ΡƒΠ΄Π΅Ρ‚ EdA, ΡƒΠΌΠ½ΠΎΠΆΠ΅Π½Π½ΠΎΠ΅ Π½Π° косинус 0,

.

Когда ΠΌΡ‹ слоТим всС эти ΠΈΠ½Ρ‚Π΅Π³Ρ€Π°Π»Ρ‹ ΠΏΠΎ ΠΎΡ‚ΠΊΡ€Ρ‹Ρ‚ΠΎΠΉ повСрхности, Ρ‚ΠΎ получится ΠΈΠ½Ρ‚Π΅Π³Ρ€Π°Π» ΠΏΠΎ всСй Π·Π°ΠΌΠΊΠ½ΡƒΡ‚ΠΎΠΉ повСрхности Ρ†ΠΈΠ»ΠΈΠ½Π΄Ρ€Π°. А Π² ΠΏΡ€Π°Π²ΠΎΠΉ части Ρƒ нас Π±ΡƒΠ΄Π΅Ρ‚ q, Π·Π°ΠΊΠ»ΡŽΡ‡Π΅Π½Π½ΠΎΠ΅ Π² эпсилон 0. ΠšΠΎΡΠΈΠ½ΡƒΡ 0, ΠΎΠΏΡΡ‚ΡŒ ΠΆΠ΅, Ρ€Π°Π²Π΅Π½ 1. И ΠΏΠΎΠΊΠ° ΠΌΡ‹ находимся Π½Π° Π±ΠΎΠΊΠΎΠ²ΠΎΠΉ повСрхности этого гауссовского Ρ†ΠΈΠ»ΠΈΠ½Π΄Ρ€Π°, ΠΌΡ‹ Π±ΡƒΠ΄Π΅ΠΌ Π½Π° Ρ‚Π°ΠΊΠΎΠΌ ΠΆΠ΅ расстоянии ΠΎΡ‚ заряд, ΠΊΠΎΡ‚ΠΎΡ€Ρ‹ΠΉ ΠΎΠ½ Π²ΠΌΠ΅Ρ‰Π°Π΅Ρ‚, ΠΊΠΎΡ‚ΠΎΡ€Ρ‹ΠΉ здСсь являСтся Π²Π½ΡƒΡ‚Ρ€Π΅Π½Π½ΠΈΠΌ Ρ†ΠΈΠ»ΠΈΠ½Π΄Ρ€ΠΎΠΌ. Π’Π°ΠΊΠΈΠΌ ΠΎΠ±Ρ€Π°Π·ΠΎΠΌ, элСктричСскоС ΠΏΠΎΠ»Π΅ Π½Π° Π±ΠΎΠΊΠΎΠ²ΠΎΠΉ повСрхности Π±ΡƒΠ΄Π΅Ρ‚ постоянным.Π’ΠΎΠ³Π΄Π° ΠΌΡ‹ ΠΌΠΎΠΆΠ΅ΠΌ вынСсти Π΅Π³ΠΎ Π·Π° ΠΏΡ€Π΅Π΄Π΅Π»Ρ‹ ΠΈΠ½Ρ‚Π΅Π³Ρ€Π°Π»Π°.

Π§Ρ‚ΠΎ ΠΆ, двигаясь дальшС, Ρƒ нас Π±ΡƒΠ΄Π΅Ρ‚ E-ΠΊΡ€Π°Ρ‚Π½Ρ‹ΠΉ ΠΈΠ½Ρ‚Π΅Π³Ρ€Π°Π» ΠΏΠΎ Π±ΠΎΠΊΠΎΠ²ΠΎΠΉ повСрхности Ρ†ΠΈΠ»ΠΈΠ½Π΄Ρ€Π°, Π³Π΄Π΅ dA Ρ€Π°Π²Π½ΠΎ q, Π·Π°ΠΊΠ»ΡŽΡ‡Π΅Π½Π½ΠΎΠΌΡƒ Π½Π°Π΄ эпсилоном 0. Π§Ρ‚ΠΎ ΠΆ, Ссли ΠΌΡ‹ посмотрим Π½Π° Π½Π°ΡˆΡƒ Ρ„ΠΎΡ€ΠΌΡƒ ΠΎΡ‚ΠΊΡ€Ρ‹Ρ‚ΠΎΠΉ повСрхности этого гауссовского Ρ†ΠΈΠ»ΠΈΠ½Π΄Ρ€Π° здСсь, ΠΎΠ½ ΠΈΠΌΠ΅Π΅Ρ‚ радиус r; это располоТСниС Ρ‚ΠΎΡ‡ΠΊΠΈ ΠΎΡ‚Π½ΠΎΡΠΈΡ‚Π΅Π»ΡŒΠ½ΠΎ Ρ†Π΅Π½Ρ‚Ρ€Π°. Π’Π°ΠΊΠΈΠΌ ΠΎΠ±Ρ€Π°Π·ΠΎΠΌ, эта сторона Π±ΡƒΠ΄Π΅Ρ‚ Ρ€Π°Π²Π½Π° Π΄Π»ΠΈΠ½Π΅ окруТности Π»ΠΈΠ±ΠΎ Π²Π΅Ρ€Ρ…Π½Π΅Π³ΠΎ ΠΊΡ€ΡƒΠ³Π°, Π»ΠΈΠ±ΠΎ Π½ΠΈΠΆΠ½Π΅Π³ΠΎ ΠΊΡ€ΡƒΠ³Π°, Ρ‡Ρ‚ΠΎ Ρ‚ΠΎΠ³Π΄Π° Π±ΡƒΠ΄Π΅Ρ‚ Ρ€Π°Π²Π½ΠΎ 2 Pi r, ΠΏΠΎΡΠΊΠΎΠ»ΡŒΠΊΡƒ ΠΎΠ½ΠΎ оборачиваСтся Π²ΠΎΠΊΡ€ΡƒΠ³ этих ΠΊΡ€ΡƒΠ³ΠΎΠ².И ΠΌΡ‹ Π΄Π°Π»ΠΈ Ρ€Π°Π·ΠΌΠ΅Ρ€ h для высоты ΠΈΠ»ΠΈ Π΄Π»ΠΈΠ½Ρ‹ Ρ†ΠΈΠ»ΠΈΠ½Π΄Ρ€Π°. Π’Π°ΠΊΠΈΠΌ ΠΎΠ±Ρ€Π°Π·ΠΎΠΌ, ΠΈΠ½Ρ‚Π΅Π³Ρ€Π°Π» dA ΠΏΠΎ Π±ΠΎΠΊΠΎΠ²ΠΎΠΉ повСрхности, ΠΊΠΎΡ‚ΠΎΡ€Ρ‹ΠΉ даст Π½Π°ΠΌ ΠΏΠ»ΠΎΡ‰Π°Π΄ΡŒ Π±ΠΎΠΊΠΎΠ²ΠΎΠΉ повСрхности, ΠΊΠΎΡ‚ΠΎΡ€Ρ‹ΠΉ Π±ΡƒΠ΄Π΅Ρ‚ Ρ€Π°Π²Π΅Π½ e, ΡƒΠΌΠ½ΠΎΠΆΠ΅Π½Π½ΠΎΠΌΡƒ Π½Π° ΠΏΠ»ΠΎΡ‰Π°Π΄ΡŒ Π±ΠΎΠΊΠΎΠ²ΠΎΠΉ повСрхности, Ρ‡Ρ‚ΠΎ составляСт 2 Pi r h. А Π² ΠΏΡ€Π°Π²ΠΎΠΉ части Ρƒ нас Π΅ΡΡ‚ΡŒ q, Π·Π°ΠΊΠ»ΡŽΡ‡Π΅Π½Π½ΠΎΠ΅ Π½Π°Π΄ Эпсилоном 0.

ΠžΠΏΡΡ‚ΡŒ ΠΆΠ΅, q ΠΎΡ…Π²Π°Ρ‚Ρ‹Π²Π°Π΅Ρ‚ чистый заряд Π²Π½ΡƒΡ‚Ρ€ΠΈ области, ΠΎΠΊΡ€ΡƒΠΆΠ΅Π½Π½ΠΎΠΉ гауссовой ΠΏΠΎΠ²Π΅Ρ€Ρ…Π½ΠΎΡΡ‚ΡŒΡŽ, Π² Π΄Π°Π½Π½ΠΎΠΌ случаС это Ρ„ΠΈΠΎΠ»Π΅Ρ‚ΠΎΠ²Ρ‹ΠΉ Ρ†ΠΈΠ»ΠΈΠ½Π΄Ρ€. Когда ΠΌΡ‹ смотрим Π½Π° Π²Π½ΡƒΡ‚Ρ€Π΅Π½Π½ΡŽΡŽ Ρ‡Π°ΡΡ‚ΡŒ этой области, ΠΌΡ‹ Π²ΠΈΠ΄ΠΈΠΌ, Ρ‡Ρ‚ΠΎ ΠΎΠ½Π° ΠΎΡ…Π²Π°Ρ‚Ρ‹Π²Π°Π΅Ρ‚ всю Π²Π½ΡƒΡ‚Ρ€Π΅Π½Π½ΡŽΡŽ ΠΏΠΎΠ²Π΅Ρ€Ρ…Π½ΠΎΡΡ‚ΡŒ ΠΈ, ΡΠ»Π΅Π΄ΠΎΠ²Π°Ρ‚Π΅Π»ΡŒΠ½ΠΎ, Π²ΠΊΠ»ΡŽΡ‡Π°Π΅Ρ‚ Π² сСбя любой заряд, распрСдСлСнный ΠΏΠΎ этой Π²Π½ΡƒΡ‚Ρ€Π΅Π½Π½Π΅ΠΉ повСрхности этого кондСнсатора, ΠΈ ΠΊΠΎΡ‚ΠΎΡ€Ρ‹ΠΉ Ρ€Π°Π²Π΅Π½ ΠΎΠ±Ρ‰Π΅ΠΌΡƒ заряду вдоль этой повСрхности, ΠΊΠΎΡ‚ΠΎΡ€Ρ‹ΠΉ это q.

Π˜Ρ‚Π°ΠΊ, вычисляя элСктричСскоС ΠΏΠΎΠ»Π΅, Π²Π΅Π»ΠΈΡ‡ΠΈΠ½Π° элСктричСского поля становится Ρ€Π°Π²Π½ΠΎΠΉ q Π² Ρ‚Π΅Ρ‡Π΅Π½ΠΈΠ΅ 2 Pi Epsilon 0 h, ΡƒΠΌΠ½ΠΎΠΆΠ΅Π½Π½Ρ‹Ρ… Π½Π° r. Когда ΠΌΡ‹ смотрим Π½Π° это Π²Ρ‹Ρ€Π°ΠΆΠ΅Π½ΠΈΠ΅ ΠΊΠ°ΠΊ Π½Π° ΠΎΡ‚Π»ΠΈΡ‡Π½ΠΎΠ΅ ΠΎΡ‚ кондСнсатора с ΠΏΠ°Ρ€Π°Π»Π»Π΅Π»ΡŒΠ½Ρ‹ΠΌΠΈ пластинами, ΠΌΡ‹ Π²ΠΈΠ΄ΠΈΠΌ, Ρ‡Ρ‚ΠΎ это Π½Π΅ постоянная Π²Π΅Π»ΠΈΡ‡ΠΈΠ½Π°; ΠΎΠ½ мСняСтся Π½Π° 1 ΠΏΠΎ r. Π”Ρ€ΡƒΠ³ΠΈΠΌΠΈ словами, ΠΏΠΎ ΠΌΠ΅Ρ€Π΅ Ρ‚ΠΎΠ³ΠΎ, ΠΊΠ°ΠΊ ΠΌΡ‹ ΠΈΠ΄Π΅ΠΌ ΠΎΡ‚ Π²Π½ΡƒΡ‚Ρ€Π΅Π½Π½Π΅ΠΉ повСрхности ΠΊ внСшнСй повСрхности, ΠΎΡ‚ Π²Π½ΡƒΡ‚Ρ€Π΅Π½Π½Π΅ΠΉ повСрхности ΠΊ внСшнСй повСрхности, Π½Π°ΠΏΡ€ΡΠΆΠ΅Π½Π½ΠΎΡΡ‚ΡŒ элСктричСского поля ΡƒΠΌΠ΅Π½ΡŒΡˆΠ°Π΅Ρ‚ΡΡ с 1 ΠΏΠΎ r, с 1 ΠΏΠΎ Ρ€Π°ΡΡΡ‚ΠΎΡΠ½ΠΈΡŽ ΠΎΡ‚Π½ΠΎΡΠΈΡ‚Π΅Π»ΡŒΠ½ΠΎ оси Ρ†ΠΈΠ»ΠΈΠ½Π΄Ρ€Π°.

Π’Π΅ΠΏΠ΅Ρ€ΡŒ, ΠΊΠΎΠ³Π΄Π° ΠΌΡ‹ ΠΎΠΏΡ€Π΅Π΄Π΅Π»ΠΈΠΌ Π²Π΅Π»ΠΈΡ‡ΠΈΠ½Ρƒ элСктричСского поля, ΠΊΠΎΠ½Π΅Ρ‡Π½ΠΎ, Π΅Π³ΠΎ Π½Π°ΠΏΡ€Π°Π²Π»Π΅Π½ΠΈΠ΅ Ρ€Π°Π΄ΠΈΠ°Π»ΡŒΠ½ΠΎ Π½Π°Ρ€ΡƒΠΆΡƒ ΠΎΡ‚ ΠΏΠΎΠ»ΠΎΠΆΠΈΡ‚Π΅Π»ΡŒΠ½ΠΎΠΉ пластины ΠΊ ΠΎΡ‚Ρ€ΠΈΡ†Π°Ρ‚Π΅Π»ΡŒΠ½ΠΎΠΉ, Ρ‚Π΅ΠΏΠ΅Ρ€ΡŒ ΠΌΡ‹ ΠΌΠΎΠΆΠ΅ΠΌ ΠΏΠ΅Ρ€Π΅ΠΉΡ‚ΠΈ ΠΊ Ρ‚Ρ€Π΅Ρ‚ΡŒΠ΅ΠΌΡƒ этапу, ΠΊΠΎΡ‚ΠΎΡ€Ρ‹ΠΉ вычисляСт Ρ€Π°Π·Π½ΠΎΡΡ‚ΡŒ ΠΏΠΎΡ‚Π΅Π½Ρ†ΠΈΠ°Π»ΠΎΠ² ΠΌΠ΅ΠΆΠ΄Ρƒ пластинами ΠΏΡƒΡ‚Π΅ΠΌ взятия ΠΈΠ½Ρ‚Π΅Π³Ρ€Π°Π»Π°, ΠΈΠ½Ρ‚Π΅Π³Ρ€Π°Π»ΡŒΠ½Π°Ρ линия, ΠΎΡ‚ ΠΏΠΎΠ»ΠΎΠΆΠΈΡ‚Π΅Π»ΡŒΠ½ΠΎΠΉ ΠΊ ΠΎΡ‚Ρ€ΠΈΡ†Π°Ρ‚Π΅Π»ΡŒΠ½ΠΎΠΉ пластинС e dot dl. ΠžΠΏΡΡ‚ΡŒ ΠΆΠ΅, это ΠΈΠ½Ρ‚Π΅Π³Ρ€Π°Π» ΠΏΠΎ путям, ΠΈ ΠΌΡ‹ Π²Ρ‹Π±Π΅Ρ€Π΅ΠΌ ΠΏΡ€ΠΎΡΡ‚Π΅ΠΉΡˆΠΈΠΉ ΠΏΡƒΡ‚ΡŒ ΠΎΡ‚Π½ΠΎΡΠΈΡ‚Π΅Π»ΡŒΠ½ΠΎ нашСго Π²Π΅ΠΊΡ‚ΠΎΡ€Π° элСктричСского поля. И Π² этом случаС ΠΏΡ€ΠΎΡΡ‚Π΅ΠΉΡˆΠΈΠΉ ΠΏΡƒΡ‚ΡŒ — это, ΠΏΠΎ сути, ΠΏΡƒΡ‚ΡŒ, ΡΠΎΠ²ΠΏΠ°Π΄Π°ΡŽΡ‰ΠΈΠΉ с Π²Π΅ΠΊΡ‚ΠΎΡ€ΠΎΠΌ элСктричСского поля.

Ну, это Π±ΡƒΠ΄Π΅Ρ‚ Ρ€Π°Π΄ΠΈΠ°Π»ΡŒΠ½Ρ‹ΠΉ ΠΏΡƒΡ‚ΡŒ, Ρ€Π°Π΄ΠΈΠ°Π»ΡŒΠ½ΠΎ Π½Π°Ρ€ΡƒΠΆΡƒ, ΠΈ поэтому я ΡΠΎΠ±ΠΈΡ€Π°ΡŽΡΡŒ Π²Ρ‹Π±Ρ€Π°Ρ‚ΡŒ этот ΠΏΡƒΡ‚ΡŒ, Π²ΠΎΡ‚ Ρ‚Π°ΠΊ, ΠΈ ΠΎΠ½ Π±ΡƒΠ΄Π΅Ρ‚ ΡΠΎΠ²ΠΏΠ°Π΄Π°Ρ‚ΡŒ с Π²Π΅ΠΊΡ‚ΠΎΡ€ΠΎΠΌ элСктричСского поля. ΠŸΡ€ΠΈ этом ΡƒΠ³ΠΎΠ» ΠΌΠ΅ΠΆΠ΄Ρƒ этими двумя Π²Π΅ΠΊΡ‚ΠΎΡ€Π°ΠΌΠΈ становится Ρ€Π°Π²Π½Ρ‹ΠΌ Π½ΡƒΠ»ΡŽ. И Π΅Ρ‰Π΅ ΠΊΠΎΠ΅-Ρ‡Ρ‚ΠΎ, Ссли ΠΏΠΎΡΠΌΠΎΡ‚Ρ€Π΅Ρ‚ΡŒ Π½Π° Π³Π΅ΠΎΠΌΠ΅Ρ‚Ρ€ΠΈΡŽ, это Ρ€Π°Π΄ΠΈΠ°Π»ΡŒΠ½ΠΎΠ΅ расстояниС; элСктричСскоС ΠΏΠΎΠ»Π΅ Π½Π°ΠΏΡ€Π°Π²Π»Π΅Π½ΠΎ Ρ€Π°Π΄ΠΈΠ°Π»ΡŒΠ½ΠΎ Π½Π°Ρ€ΡƒΠΆΡƒ, ΠΈ ΠΌΡ‹ Π²Ρ‹Π±ΠΈΡ€Π°Π΅ΠΌ этот ΠΏΡƒΡ‚ΡŒ Ρ‚Π°ΠΊΠΆΠ΅ Π² Ρ€Π°Π΄ΠΈΠ°Π»ΡŒΠ½ΠΎΠΌ Π½Π°ΠΏΡ€Π°Π²Π»Π΅Π½ΠΈΠΈ. Π’ΠΎΠ³Π΄Π° Π²Π΅ΠΊΡ‚ΠΎΡ€ приращСния смСщСния вдоль этого ΠΏΡƒΡ‚ΠΈ Π±ΡƒΠ΄Π΅Ρ‚ dr, поэтому ΠΌΡ‹ Π·Π°ΠΌΠ΅Π½ΠΈΠΌ dl Π½Π° dr.

Π₯ΠΎΡ€ΠΎΡˆΠΎ. Π’Ρ‹Π±Π΅Ρ€ΠΈΡ‚Π΅ ΠΏΡƒΡ‚ΡŒ Ρ€Π°Π΄ΠΈΠ°Π»ΡŒΠ½ΠΎ Π½Π°Ρ€ΡƒΠΆΡƒ, Ρ‚ΠΎΠ³Π΄Π° dl станСт Ρ€Π°Π²Π½Ρ‹ΠΌ dr. Π’Π΅ΠΏΠ΅Ρ€ΡŒ рассчитаСм Ρ€Π°Π·Π½ΠΎΡΡ‚ΡŒ ΠΏΠΎΡ‚Π΅Π½Ρ†ΠΈΠ°Π»ΠΎΠ² ΠΌΠ΅ΠΆΠ΄Ρƒ пластинами цилиндричСского кондСнсатора. V становится Ρ€Π°Π²Π½Ρ‹ΠΌ ΠΈΠ½Ρ‚Π΅Π³Ρ€Π°Π»Ρƒ ΠΎΡ‚ Π²Π΅Π»ΠΈΡ‡ΠΈΠ½Ρ‹ E, которая Ρ€Π°Π²Π½Π° q Π·Π° 2 Pi Epsilon 0 hr, ΠΊΠΎΡ‚ΠΎΡ€ΡƒΡŽ ΠΌΡ‹ ΠΎΠΏΡ€Π΅Π΄Π΅Π»ΠΈΠ»ΠΈ ΠΈΠ· шага ΠΈΠ»ΠΈ части Π΄Π²Π°. q Π±ΠΎΠ»Π΅Π΅ 2 Pi Epsilon 0 Ρ‡ ΡƒΠΌΠ½ΠΎΠΆΠΈΡ‚ΡŒ Π½Π° r. И вмСсто dl ΠΌΡ‹ Π·Π°ΠΌΠ΅Π½ΠΈΠΌ ΠΈΠ»ΠΈ Π²ΠΎΡΠΏΠΎΠ»ΡŒΠ·ΡƒΠ΅ΠΌΡΡ dr, Π²Π΅ΠΊΡ‚ΠΎΡ€ приращСния смСщСния Π² Ρ€Π°Π΄ΠΈΠ°Π»ΡŒΠ½ΠΎΠΌ Π½Π°ΠΏΡ€Π°Π²Π»Π΅Π½ΠΈΠΈ. Π˜Ρ‚Π°ΠΊ, Π²Π΅Π»ΠΈΡ‡ΠΈΠ½Π° r, ΠΏΠΎΡΠΊΠΎΠ»ΡŒΠΊΡƒ E ΠΈ dr находятся Π² ΠΎΠ΄Π½ΠΎΠΌ Π½Π°ΠΏΡ€Π°Π²Π»Π΅Π½ΠΈΠΈ, ΡΠ»Π΅Π΄ΠΎΠ²Π°Ρ‚Π΅Π»ΡŒΠ½ΠΎ, ΡƒΠ³ΠΎΠ» ΠΌΠ΅ΠΆΠ΄Ρƒ Π½ΠΈΠΌΠΈ Ρ€Π°Π²Π΅Π½ 0, косинусу 0.ΠšΠΎΡΠΈΠ½ΡƒΡ 0 Ρ€Π°Π²Π΅Π½ 1, Π° наша пСрСмСнная — r. И r мСняСтся, ΠΊΠΎΠ³Π΄Π° ΠΌΡ‹ вСрнСмся ΠΊ нашСй Π΄ΠΈΠ°Π³Ρ€Π°ΠΌΠΌΠ΅. Π˜Π½Ρ‚Π΅Π³Ρ€Π°Π» бСрСтся ΠΎΡ‚ ΠΏΠΎΠ»ΠΎΠΆΠΈΡ‚Π΅Π»ΡŒΠ½ΠΎΠΉ пластины ΠΊ ΠΎΡ‚Ρ€ΠΈΡ†Π°Ρ‚Π΅Π»ΡŒΠ½ΠΎΠΉ пластинС, поэтому r Π±ΡƒΠ΄Π΅Ρ‚ ΠΈΠ·ΠΌΠ΅Π½ΡΡ‚ΡŒΡΡ ΠΎΡ‚ Π²Π½ΡƒΡ‚Ρ€Π΅Π½Π½Π΅Π³ΠΎ радиуса a Π΄ΠΎ внСшнСго радиуса b.

Π˜Ρ‚Π°ΠΊ, Π³Ρ€Π°Π½ΠΈΡ†Ρ‹ ΠΈΠ½Ρ‚Π΅Π³Ρ€Π°Π»Π° ΠΏΠΎΠΉΠ΄ΡƒΡ‚ ΠΎΡ‚ a ΠΊ b. И здСсь q 2 Pi Epsilon 0 ΠΈ h постоянны. ΠœΡ‹ ΠΌΠΎΠΆΠ΅ΠΌ вынСсти это Π·Π° ΠΏΡ€Π΅Π΄Π΅Π»Ρ‹ ΠΈΠ½Ρ‚Π΅Π³Ρ€Π°Π»Π°, ΠΈ это оставляСт нас, Ρ€Π°Π·Π½ΠΎΡΡ‚ΡŒ ΠΏΠΎΡ‚Π΅Π½Ρ†ΠΈΠ°Π»ΠΎΠ² ΠΌΠ΅ΠΆΠ΄Ρƒ пластинами Ρ€Π°Π²Π½Π° q Π½Π° 2 Pi Эпсилон 0 h, ΡƒΠΌΠ½ΠΎΠΆΠ΅Π½Π½Ρ‹ΠΉ Π½Π° ΠΈΠ½Ρ‚Π΅Π³Ρ€Π°Π» dr ΠΏΠΎ r, ΠΏΡ€ΠΎΠΈΠ½Ρ‚Π΅Π³Ρ€ΠΈΡ€ΠΎΠ²Π°Π½Π½Ρ‹ΠΉ ΠΎΡ‚ a Π΄ΠΎ b.Π˜Π½Ρ‚Π΅Π³Ρ€Π°Π» ΠΎΡ‚ dr ΠΏΠΎ r Ρ€Π°Π²Π΅Π½ ln ΠΎΡ‚ r. Π”Π²ΠΈΠ³Π°ΡΡΡŒ дальшС, Ρƒ нас Π±ΡƒΠ΄Π΅Ρ‚ q большС 2 Pi Epsilon 0 h, ΡƒΠΌΠ½ΠΎΠΆΠ΅Π½Π½ΠΎΠ΅ Π½Π° ln ΠΈΠ· r, вычислСнноС Π² a ΠΈ d, Ρ‡Ρ‚ΠΎ Π±ΡƒΠ΄Π΅Ρ‚ Ρ€Π°Π²Π½ΠΎ q для 2 Pi Epsilon 0 h, ΡƒΠΌΠ½ΠΎΠΆΠ΅Π½Π½ΠΎΠ΅ Π½Π° ln ΠΈΠ· b минус ln ΠΈΠ· a, ΠΏΡƒΡ‚Π΅ΠΌ Π·Π°ΠΌΠ΅Π½Ρ‹ Π³Ρ€Π°Π½ΠΈΡ†Ρ‹ для r. И Ρ‚Π°ΠΊ ΠΊΠ°ΠΊ ln b минус ln a Ρ€Π°Π²Π½ΠΎ ln b Π½Π°Π΄ a, ΠΌΡ‹ ΠΌΠΎΠΆΠ΅ΠΌ, Π½Π°ΠΊΠΎΠ½Π΅Ρ†, Π²Ρ‹Ρ€Π°Π·ΠΈΡ‚ΡŒ эту Ρ€Π°Π·Π½ΠΎΡΡ‚ΡŒ ΠΏΠΎΡ‚Π΅Π½Ρ†ΠΈΠ°Π»ΠΎΠ² ΠΊΠ°ΠΊ q Π½Π° 2 Pi Epsilon 0 h ΡƒΠΌΠ½ΠΎΠΆΠΈΡ‚ΡŒ Π½Π° ln b Π½Π°Π΄ a.

ПослС Ρ‚ΠΎΠ³ΠΎ, ΠΊΠ°ΠΊ ΠΌΡ‹ ΠΎΠΏΡ€Π΅Π΄Π΅Π»ΠΈΠΌ Ρ€Π°Π·Π½ΠΎΡΡ‚ΡŒ ΠΏΠΎΡ‚Π΅Π½Ρ†ΠΈΠ°Π»ΠΎΠ² ΠΌΠ΅ΠΆΠ΄Ρƒ пластинами, Π² качСствС послСднСго шага, ΠΌΡ‹ ΠΌΠΎΠΆΠ΅ΠΌ Π²Ρ‹Ρ‡ΠΈΡΠ»ΠΈΡ‚ΡŒ Π΅ΠΌΠΊΠΎΡΡ‚ΡŒ цилиндричСского кондСнсатора ΠΏΠΎ Π΅Π³ΠΎ ΠΎΠΏΡ€Π΅Π΄Π΅Π»Π΅Π½ΠΈΡŽ, ΠΊΠΎΡ‚ΠΎΡ€ΠΎΠ΅ прСдставляСт собой ΠΎΡ‚Π½ΠΎΡˆΠ΅Π½ΠΈΠ΅ Π²Π΅Π»ΠΈΡ‡ΠΈΠ½Ρ‹ заряда, хранящСгося Π² пластинах, Π΄Π΅Π»Π΅Π½Π½ΠΎΠ³ΠΎ Π½Π°, ΠΈΠ»ΠΈ ΠΊ разности ΠΏΠΎΡ‚Π΅Π½Ρ†ΠΈΠ°Π»ΠΎΠ². ΠΌΠ΅ΠΆΠ΄Ρƒ пластинами, Π½Π° ΠΊΠΎΡ‚ΠΎΡ€ΠΎΠΌ стоит Π’.Π’Π°ΠΊΠΈΠΌ ΠΎΠ±Ρ€Π°Π·ΠΎΠΌ, Ρƒ нас Π±ΡƒΠ΄Π΅Ρ‚ q, Ρ€Π°Π·Π΄Π΅Π»Π΅Π½Π½ΠΎΠ΅ Π½Π° Ρ€Π°Π·Π½ΠΎΡΡ‚ΡŒ ΠΏΠΎΡ‚Π΅Π½Ρ†ΠΈΠ°Π»ΠΎΠ², которая Ρ€Π°Π²Π½Π° q Π½Π° 2 Pi Epsilon 0 h, ΡƒΠΌΠ½ΠΎΠΆΠ΅Π½Π½ΠΎΠ΅ Π½Π° ln b Π½Π° a. Заряды сократятся Π² числитСлС ΠΈ Π·Π½Π°ΠΌΠ΅Π½Π°Ρ‚Π΅Π»Π΅, Π² Ρ€Π΅Π·ΡƒΠ»ΡŒΡ‚Π°Ρ‚Π΅ Ρ‡Π΅Π³ΠΎ Π΅ΠΌΠΊΠΎΡΡ‚ΡŒ цилиндричСского кондСнсатора Π±ΡƒΠ΄Π΅Ρ‚ Ρ€Π°Π²Π½Π° 2 Pi Epsilon 0 h, ΡƒΠΌΠ½ΠΎΠΆΠ΅Π½Π½ΠΎΠΌΡƒ Π½Π° 1 Π½Π° ln ΠΈΠ»ΠΈ b Π½Π° a.

Π­Ρ‚ΠΎ Π»Π΅Π³ΠΊΠΎ ΡƒΠ²ΠΈΠ΄Π΅Ρ‚ΡŒ, ΠΊΠ°ΠΊ ΠΈ Π² случаС кондСнсатора с ΠΏΠ°Ρ€Π°Π»Π»Π΅Π»ΡŒΠ½Ρ‹ΠΌΠΈ пластинами. Π’ цилиндричСском кондСнсаторС Π΅ΠΌΠΊΠΎΡΡ‚ΡŒ Ρ‚Π°ΠΊΠΆΠ΅ зависит ΠΎΡ‚ физичСских свойств кондСнсатора. Π’ Π΄Π°Π½Π½ΠΎΠΌ случаС Π΄Π»ΠΈΠ½Π° Ρ€Π°Π²Π½Π° высотС цилиндричСского кондСнсатора, Π° Ρ‚Π°ΠΊΠΆΠ΅ Π΅Π³ΠΎ Π²Π½ΡƒΡ‚Ρ€Π΅Π½Π½ΠΈΠΉ ΠΈ внСшний радиус.

Π€ΠΈΠ·ΠΈΠΊΠ° для Π½Π°ΡƒΠΊΠΈ ΠΈ Ρ‚Π΅Ρ…Π½ΠΈΠΊΠΈ II

5.11 ΠŸΡ€ΠΈΠΌΠ΅Ρ€ ΠΈΠ· ΠΎΡ‚Π΄Π΅Π»Π° акадСмичСских Ρ‚Π΅Ρ…Π½ΠΎΠ»ΠΎΠ³ΠΈΠΉ Π½Π° Vimeo.

5.11 ΠŸΡ€ΠΈΠΌΠ΅Ρ€

Π”Π°Π²Π°ΠΉΡ‚Π΅ рассмотрим ΠΏΡ€ΠΈΠΌΠ΅Ρ€, связанный с ΠΊΠΎΠ½Ρ†Π΅ΠΏΡ†ΠΈΠ΅ΠΉ плотности энСргии. ΠŸΡ€Π΅Π΄ΠΏΠΎΠ»ΠΎΠΆΠΈΠΌ, Ρ‡Ρ‚ΠΎ Ρƒ нас Π΅ΡΡ‚ΡŒ цилиндричСский кондСнсатор радиусами a ΠΈ b. Π― Ρ…ΠΎΡ‚Π΅Π» Π±Ρ‹ ΠΏΠΎΠΊΠ°Π·Π°Ρ‚ΡŒ, Ρ‡Ρ‚ΠΎ ΠΏΠΎΠ»ΠΎΠ²ΠΈΠ½Π° Π½Π°ΠΊΠΎΠΏΠ»Π΅Π½Π½ΠΎΠΉ элСктричСской ΠΏΠΎΡ‚Π΅Π½Ρ†ΠΈΠ°Π»ΡŒΠ½ΠΎΠΉ энСргии находится Π²Π½ΡƒΡ‚Ρ€ΠΈ Ρ†ΠΈΠ»ΠΈΠ½Π΄Ρ€Π°, радиус r ΠΊΠΎΡ‚ΠΎΡ€ΠΎΠ³ΠΎ Ρ€Π°Π²Π΅Π½ ΠΊΠ²Π°Π΄Ρ€Π°Ρ‚Π½ΠΎΠΌΡƒ ΠΊΠΎΡ€Π½ΡŽ ΠΈΠ· Π²Π½ΡƒΡ‚Ρ€Π΅Π½Π½Π΅Π³ΠΎ радиуса, ΡƒΠΌΠ½ΠΎΠΆΠ΅Π½Π½ΠΎΠ³ΠΎ Π½Π° внСшний радиус.

Π’Π°ΠΊΠΈΠΌ ΠΎΠ±Ρ€Π°Π·ΠΎΠΌ, Ρƒ нас Π΅ΡΡ‚ΡŒ цилиндричСский кондСнсатор, ΠΈ Π΄Π°Π²Π°ΠΉΡ‚Π΅ посмотрим, это внСшняя ΠΏΠΎΠ²Π΅Ρ€Ρ…Π½ΠΎΡΡ‚ΡŒ цилиндричСского кондСнсатора, ΠΈ это внутрСнняя ΠΏΠΎΠ²Π΅Ρ€Ρ…Π½ΠΎΡΡ‚ΡŒ цилиндричСского кондСнсатора. И Π²Π½ΡƒΡ‚Ρ€Π΅Π½Π½ΠΈΠΉ радиус этого кондСнсатора Π·Π°Π΄Π°Π½ ΠΊΠ°ΠΊ a, Π° внСшний радиус Ρ€Π°Π²Π΅Π½ b. Вопрос Π·Π°ΠΊΠ»ΡŽΡ‡Π°Π΅Ρ‚ΡΡ Π² Ρ‚ΠΎΠΌ, Ρ‡Ρ‚ΠΎΠ±Ρ‹ Π½Π°ΠΉΡ‚ΠΈ Ρ†ΠΈΠ»ΠΈΠ½Π΄Ρ€ΠΈΡ‡Π΅ΡΠΊΡƒΡŽ ΠΎΠ±Π»Π°ΡΡ‚ΡŒ Π²Π½ΡƒΡ‚Ρ€ΠΈ этого цилиндричСского кондСнсатора, Ρ‚Π°ΠΊΡƒΡŽ, Ρ‡Ρ‚ΠΎΠ±Ρ‹ количСство Ρ…Ρ€Π°Π½ΠΈΠΌΠΎΠΉ элСктричСской ΠΏΠΎΡ‚Π΅Π½Ρ†ΠΈΠ°Π»ΡŒΠ½ΠΎΠΉ энСргии Π² этой области составляло ΠΏΠΎΠ»ΠΎΠ²ΠΈΠ½Ρƒ ΠΎΠ±Ρ‰Π΅ΠΉ энСргии, хранящСйся Π² элСктричСском ΠΏΠΎΠ»Π΅ этого кондСнсатора.

Π’Π΅ΠΏΠ΅Ρ€ΡŒ, ΠΊΠ°ΠΊ Π²Ρ‹ ΠΏΠΎΠΌΠ½ΠΈΡ‚Π΅, ΠΏΠ»ΠΎΡ‚Π½ΠΎΡΡ‚ΡŒ энСргии задаСтся ΠΊΠ°ΠΊ ΠΏΠΎΠ»ΠΎΠ²ΠΈΠ½Π° эпсилона, равная Π½ΡƒΠ»ΡŽ, умноТСнная Π½Π° ΠΊΠ²Π°Π΄Ρ€Π°Ρ‚ элСктричСского поля ΠΌΠ΅ΠΆΠ΄Ρƒ пластинами кондСнсатора. ΠŸΡ€Π΅Π΄ΠΏΠΎΠ»ΠΎΠΆΠΈΠΌ, Ρ‡Ρ‚ΠΎ ΠΈΠ½Ρ‚Π΅Ρ€Π΅ΡΡƒΡŽΡ‰Π°Ρ нас цилиндричСская ΠΎΠ±Π»Π°ΡΡ‚ΡŒ — это ΠΎΠ±Π»Π°ΡΡ‚ΡŒ с радиусом r.

Π’Π΅ΠΏΠ΅Ρ€ΡŒ, ΠΊΠΎΠ³Π΄Π° ΠΌΡ‹ заряТаСм этот кондСнсатор, ΠΌΡ‹ Π·Π½Π°Π΅ΠΌ, Ρ‡Ρ‚ΠΎ Ссли ΠΌΡ‹ заряТаСм Π²Π½ΡƒΡ‚Ρ€Π΅Π½Π½ΠΈΠΉ ΠΏΠΎΠ»ΠΎΠΆΠΈΡ‚Π΅Π»ΡŒΠ½ΠΎ, Π° внСшний ΠΎΡ‚Ρ€ΠΈΡ†Π°Ρ‚Π΅Π»ΡŒΠ½ΠΎ, ΠΏΡƒΡ‚Π΅ΠΌ ΠΏΠΎΠ΄ΠΊΠ»ΡŽΡ‡Π΅Π½ΠΈΡ ΠΊ ΠΊΠ»Π΅ΠΌΠΌΠ°ΠΌ источника питания, ΠΌΡ‹ собираСмся ΡΠΎΠ·Π΄Π°Ρ‚ΡŒ Ρ€Π°Π·Π½ΠΎΡΡ‚ΡŒ ΠΏΠΎΡ‚Π΅Π½Ρ†ΠΈΠ°Π»ΠΎΠ² ΠΌΠ΅ΠΆΠ΄Ρƒ этими двумя пластинами, ΠΈ ΠΌΡ‹ Ρ‚Π°ΠΊΠΆΠ΅ создадим элСктричСскоС ΠΏΠΎΠ»Π΅, исходящСС ΠΎΡ‚ ΠΏΠΎΠ»ΠΎΠΆΠΈΡ‚Π΅Π»ΡŒΠ½ΠΎΠΉ пластины ΠΈ входящСС Π² ΠΎΡ‚Ρ€ΠΈΡ†Π°Ρ‚Π΅Π»ΡŒΠ½ΡƒΡŽ пластину Π² Ρ€Π°Π΄ΠΈΠ°Π»ΡŒΠ½ΠΎΠΌ Π½Π°ΠΏΡ€Π°Π²Π»Π΅Π½ΠΈΠΈ Π½Π°Ρ€ΡƒΠΆΡƒ, заполняя ΠΎΠ±Π»Π°ΡΡ‚ΡŒ ΠΌΠ΅ΠΆΠ΄Ρƒ этими пластинами.

И ΠΌΡ‹ ΠΌΠΎΠΆΠ΅ΠΌ Π²Ρ‹Ρ‡ΠΈΡΠ»ΠΈΡ‚ΡŒ Π½Π°ΠΏΡ€ΡΠΆΠ΅Π½Π½ΠΎΡΡ‚ΡŒ этого элСктричСского поля, Ρ‡Ρ‚ΠΎ ΠΌΡ‹ сдСлали Ρ€Π°Π½Π΅Π΅, ΠΏΡ€ΠΈΠΌΠ΅Π½ΠΈΠ² Π·Π°ΠΊΠΎΠ½ Гаусса, ΠΈ это Π±Ρ‹Π»ΠΎ e dot da, ΠΈΠ½Ρ‚Π΅Π³Ρ€ΠΈΡ€ΠΎΠ²Π°Π½Π½ΠΎΠ΅ ΠΏΠΎ этому гауссовскому Ρ†ΠΈΠ»ΠΈΠ½Π΄Ρ€Ρƒ, ΠΊΠΎΡ‚ΠΎΡ€Ρ‹ΠΉ ΠΌΡ‹ Π²Ρ‹Π±ΠΈΡ€Π°Π΅ΠΌ, ΠΈΡΠΏΠΎΠ»ΡŒΠ·ΡƒΡ ΡΠΈΠΌΠΌΠ΅Ρ‚Ρ€ΠΈΡŽ Π·Π°Π΄Π°Ρ‡ΠΈ, Ρ‚Π°ΠΊ Ρ‡Ρ‚ΠΎ Π΅Π³ΠΎ сторона ΠŸΠΎΠ²Π΅Ρ€Ρ…Π½ΠΎΡΡ‚ΡŒ ΠΏΡ€ΠΎΡ…ΠΎΠ΄ΠΈΡ‚ Ρ‡Π΅Ρ€Π΅Π· Ρ‚ΠΎΡ‡ΠΊΡƒ интСрСса, Ρ€Π°ΡΠΏΠΎΠ»ΠΎΠΆΠ΅Π½Π½ΡƒΡŽ ΠΌΠ΅ΠΆΠ΄Ρƒ пластинами, ΠΈ ΠΌΡ‹ Π²Ρ‹Π±ΠΈΡ€Π°Π΅ΠΌ это мСсто подальшС ΠΎΡ‚ ΠΊΠΎΠ½Ρ†ΠΎΠ² кондСнсатора, Π²Ρ‹ Π·Π½Π°Π΅Ρ‚Π΅, Ρ‡Ρ‚ΠΎΠ±Ρ‹ ΠΈΠΌΠ΅Ρ‚ΡŒ Π²ΠΎΠ·ΠΌΠΎΠΆΠ½ΠΎΡΡ‚ΡŒ ΠΏΡ€ΠΈΠΌΠ΅Π½ΠΈΡ‚ΡŒ Π·Π°ΠΊΠΎΠ½ Гаусса, прСнСбрСгая ΠΊΠΎΠ½Ρ†Π΅Π²Ρ‹ΠΌΠΈ эффСктами, ΠΊΠΎΡ‚ΠΎΡ€Ρ‹Π΅ ΠΌΡ‹ сдСлали это Ρ€Π°Π½Π΅Π΅. А правая Ρ‡Π°ΡΡ‚ΡŒ Π·Π°ΠΊΠΎΠ½Π° Гаусса, ΠΊΠ°ΠΊ Π²Ρ‹ ΠΏΠΎΠΌΠ½ΠΈΡ‚Π΅, Π±ΡƒΠ΄Π΅Ρ‚ Ρ€Π°Π²Π½Π° q, Π·Π°ΠΊΠ»ΡŽΡ‡Π΅Π½Π½ΠΎΠΌΡƒ Π² эпсилон-ноль, ΠΊΠΎΡ‚ΠΎΡ€Ρ‹ΠΉ прСдставляСт собой чистый заряд, Π·Π°ΠΊΠ»ΡŽΡ‡Π΅Π½Π½Ρ‹ΠΉ Π² области, ΠΎΠΊΡ€ΡƒΠΆΠ΅Π½Π½ΠΎΠΉ гауссовым Ρ†ΠΈΠ»ΠΈΠ½Π΄Ρ€ΠΎΠΌ, Π΄Π΅Π»Π΅Π½Π½Ρ‹ΠΉ Π½Π° эпсилон-ноль.

И снова, ΠΊΠ°ΠΊ Π²Ρ‹ ΠΏΠΎΠΌΠ½ΠΈΡ‚Π΅, ΠΌΡ‹ Ρ€Π°Π·Π΄Π΅Π»ΠΈΠ»ΠΈ это ΠΈΠ½Ρ‚Π΅Π³Ρ€ΠΈΡ€ΠΎΠ²Π°Π½ΠΈΠ΅ Π·Π°ΠΌΠΊΠ½ΡƒΡ‚ΠΎΠΉ повСрхности Π½Π° сумму Ρ‚Ρ€Π΅Ρ… ΠΈΠ½Ρ‚Π΅Π³Ρ€Π°Π»ΠΎΠ², взятых ΠΏΠΎ Π²Π΅Ρ€Ρ…Π½Π΅ΠΉ, Π½ΠΈΠΆΠ½Π΅ΠΉ ΠΈ Π±ΠΎΠΊΠΎΠ²Ρ‹ΠΌ повСрхностям Ρ†ΠΈΠ»ΠΈΠ½Π΄Ρ€Π°. И ΠΊΠΎΠ³Π΄Π° ΠΌΡ‹ ΠΈΡ… слоТили, ΠΌΡ‹ ΠΏΠΎΠ»ΡƒΡ‡ΠΈΠ»ΠΈ ΠΈΠ½Ρ‚Π΅Π³Ρ€Π°Π» ΠΏΠΎ Π·Π°ΠΌΠΊΠ½ΡƒΡ‚ΠΎΠΉ повСрхности. Π’Π°ΠΊΠΈΠΌ ΠΎΠ±Ρ€Π°Π·ΠΎΠΌ, ΠΈΠ½Ρ‚Π΅Π³Ρ€Π°Π»ΡŒΠ½Π°Ρ Π½Π°Π΄ Π²Π΅Ρ€Ρ…Π½Π΅ΠΉ ΠΏΠΎΠ²Π΅Ρ€Ρ…Π½ΠΎΡΡ‚ΡŒΡŽ EdA, ΠΊΠ°ΠΊ ΠΌΡ‹ ΠΏΠΎΠΌΠ½ΠΈΠΌ, здСсь dA Π±Ρ‹Π»Π° пСрпСндикулярна Π²Π΅Ρ€Ρ…Π½Π΅ΠΉ повСрхности, поэтому ΡƒΠ³ΠΎΠ» ΠΌΠ΅ΠΆΠ΄Ρƒ Π½ΠΈΠΌΠΈ, E ΠΈ dA составлял 90 градусов, косинус 90 ΠΎΡ‚ этого ΠΏΡ€ΠΎΠ΄ΡƒΠΊΡ‚Π°.

И интСгрируя с Π½ΠΈΠΆΠ½Π΅ΠΉ ΠΏΠΎΠ²Π΅Ρ€Ρ…Π½ΠΎΡΡ‚ΡŒΡŽ EdA, ΠΌΡ‹ снова ΠΈΠΌΠ΅Π΅ΠΌ ΡΠΈΡ‚ΡƒΠ°Ρ†ΠΈΡŽ Π°Π½Π°Π»ΠΎΠ³ΠΈΡ‡Π½ΠΎΠ³ΠΎ Ρ‚ΠΈΠΏΠ°, ΠΊΠΎΠ³Π΄Π° dA Π½Π°ΠΏΡ€Π°Π²Π»Π΅Π½ Π²Π½ΠΈΠ·, элСктричСскоС ΠΏΠΎΠ»Π΅ Π²Ρ‹Ρ…ΠΎΠ΄ΠΈΡ‚ Ρ€Π°Π΄ΠΈΠ°Π»ΡŒΠ½ΠΎ, Π±Π΅Π· ΡƒΡ‡Π΅Ρ‚Π° ΠΊΠΎΠ½Ρ†Π΅Π²Ρ‹Ρ… эффСктов.Π‘Π»Π΅Π΄ΠΎΠ²Π°Ρ‚Π΅Π»ΡŒΠ½ΠΎ, ΡƒΠ³ΠΎΠ» ΠΌΠ΅ΠΆΠ΄Ρƒ Π½ΠΈΠΌΠΈ снова составляСт 90 градусов. Плюс ΠΈΠ½Ρ‚Π΅Π³Ρ€Π°Π» ΠΏΠΎ Π±ΠΎΠΊΠΎΠ²ΠΎΠΉ повСрхности, EdA, ΠΈ для Π±ΠΎΠΊΠΎΠ²ΠΎΠΉ повСрхности, ΠΊΠ°ΠΊ ΠΌΡ‹ снова Π²ΠΈΠ΄Π΅Π»ΠΈ Ρ€Π°Π½Π΅Π΅, e Ρ€Π°Π΄ΠΈΠ°Π»ΡŒΠ½ΠΎ Π²Ρ‹Ρ…ΠΎΠ΄ΠΈΡ‚ Π½Π°Ρ€ΡƒΠΆΡƒ, Π° dA Ρ‚Π°ΠΊΠΆΠ΅ пСрпСндикулярно повСрхности. Π‘Π»Π΅Π΄ΠΎΠ²Π°Ρ‚Π΅Π»ΡŒΠ½ΠΎ, ΡƒΠ³ΠΎΠ» Π±ΠΎΠΊΠΎΠ²ΠΎΠΉ повСрхности ΠΌΠ΅ΠΆΠ΄Ρƒ e ΠΈ dA всСгда Ρ€Π°Π²Π΅Π½ Π½ΡƒΠ»ΡŽ градусов, Π° правая Ρ‡Π°ΡΡ‚ΡŒ снова Ρ€Π°Π²Π½Π° q, Π·Π°ΠΊΠ»ΡŽΡ‡Π΅Π½Π½ΠΎΠΌΡƒ Π½Π°Π΄ Π½ΡƒΠ»Π΅Π²Ρ‹ΠΌ эпсилон. ΠŸΠΎΡΠΊΠΎΠ»ΡŒΠΊΡƒ косинус 90 Ρ€Π°Π²Π΅Π½ Π½ΡƒΠ»ΡŽ, Π½Π΅ Π±ΡƒΠ΄Π΅Ρ‚ Π½ΠΈΠΊΠ°ΠΊΠΎΠ³ΠΎ Π²ΠΊΠ»Π°Π΄Π° ΠΎΡ‚ интСгрирования Π²Π΅Ρ€Ρ…Π½Π΅ΠΉ ΠΈ Π½ΠΈΠΆΠ½Π΅ΠΉ повСрхностСй, ΠΊΡ€ΠΎΠΌΠ΅ Π²ΠΊΠ»Π°Π΄Π°, ΠΊΠΎΡ‚ΠΎΡ€Ρ‹ΠΉ Π±ΡƒΠ΄Π΅Ρ‚ ΠΈΡΡ…ΠΎΠ΄ΠΈΡ‚ΡŒ ΠΎΡ‚ Π±ΠΎΠΊΠΎΠ²ΠΎΠΉ повСрхности, ΠΏΠΎΡΠΊΠΎΠ»ΡŒΠΊΡƒ косинус нуля Ρ€Π°Π²Π΅Π½ 1, ΠΈ ΠΏΠΎΠΊΠ° ΠΌΡ‹ находимся вдоль Π±ΠΎΠΊΠΎΠ²ΠΎΠΉ повСрхности Ρ†ΠΈΠ»ΠΈΠ½Π΄Ρ€Π°, элСктричСскоС ΠΏΠΎΠ»Π΅ постоянно, Π΅Π³ΠΎ ΠΌΠΎΠΆΠ½ΠΎ вынСсти Π·Π° ΠΏΡ€Π΅Π΄Π΅Π»Ρ‹ ΠΈΠ½Ρ‚Π΅Π³Ρ€Π°Π»Π°.А Ρ‚Π°ΠΊΠΆΠ΅ ΠΌΡ‹ ΠΌΠΎΠΆΠ΅ΠΌ Π»Π΅Π³ΠΊΠΎ ΡƒΠ²ΠΈΠ΄Π΅Ρ‚ΡŒ, Ρ‡Ρ‚ΠΎ, ΠΏΠΎΡΠΊΠΎΠ»ΡŒΠΊΡƒ этот гауссов Ρ†ΠΈΠ»ΠΈΠ½Π΄Ρ€ ΠΏΠΎΠ»Π½ΠΎΡΡ‚ΡŒΡŽ ΠΎΡ…Π²Π°Ρ‚Ρ‹Π²Π°Π» Π²Π½ΡƒΡ‚Ρ€Π΅Π½Π½ΡŽΡŽ Π·Π°Ρ€ΡΠΆΠ΅Π½Π½ΡƒΡŽ ΠΏΠΎΠ²Π΅Ρ€Ρ…Π½ΠΎΡΡ‚ΡŒ, ΠΈ Ссли ΠΎΠ±Ρ‰ΠΈΠΉ заряд Π½Π° этой повСрхности Ρ€Π°Π²Π΅Π½ q, Ρ‚ΠΎ Π·Π°ΠΊΠ»ΡŽΡ‡Π΅Π½Π½Ρ‹ΠΉ q просто Ρ€Π°Π²Π΅Π½ q, ΠΈ поэтому лСвая Ρ‡Π°ΡΡ‚ΡŒ, выводящая E Π½Π°Ρ€ΡƒΠΆΡƒ, становится E, ΡƒΠΌΠ½ΠΎΠΆΠ΅Π½Π½Ρ‹ΠΌ Π½Π° ΠΈΠ½Ρ‚Π΅Π³Ρ€Π°Π» dA ΠΏΠΎ Π±ΠΎΠΊΠΎΠ²ΠΎΠΉ повСрхности, Ρ€Π°Π²Π΅Π½ q Π½Π°Π΄ эпсилоном нуля для ΠΏΡ€Π°Π²ΠΎΠΉ части.

Π˜Π½Ρ‚Π΅Π³Ρ€Π°Π» ΠΏΠΎ Π±ΠΎΠΊΠΎΠ²ΠΎΠΉ повСрхности, ΠΊΠ°ΠΊ ΠΌΡ‹ Π²ΠΈΠ΄Π΅Π»ΠΈ Ρ€Π°Π½Π΅Π΅, складываСт всС эти ΠΈΠ½ΠΊΡ€Π΅ΠΌΠ΅Π½Ρ‚Π°Π»ΡŒΠ½Ρ‹Π΅ области Π΄Ρ€ΡƒΠ³ с Π΄Ρ€ΡƒΠ³ΠΎΠΌ ΠΏΠΎ Π±ΠΎΠΊΠΎΠ²ΠΎΠΉ повСрхности ΠΈ Π΄Π°Π΅Ρ‚ Π½Π°ΠΌ ΠΏΠ»ΠΎΡ‰Π°Π΄ΡŒ Π±ΠΎΠΊΠΎΠ²ΠΎΠΉ повСрхности Ρ†ΠΈΠ»ΠΈΠ½Π΄Ρ€Π°.Если ΠΌΡ‹ скаТСм, Ρ‡Ρ‚ΠΎ высота нашСго цилиндричСского кондСнсатора Ρ€Π°Π²Π½Π° h, Π° радиус Ρ†ΠΈΠ»ΠΈΠ½Π΄Ρ€Π° Ρ€Π°Π²Π΅Π½ r, Ρ‚ΠΎ ΠΌΡ‹ ΠΌΠΎΠΆΠ΅ΠΌ Π²Ρ‹Ρ€Π°Π·ΠΈΡ‚ΡŒ ΠΏΠ»ΠΎΡ‰Π°Π΄ΡŒ Π±ΠΎΠΊΠΎΠ²ΠΎΠΉ повСрхности ΠΊΠ°ΠΊ 2 pi r, ΡƒΠΌΠ½ΠΎΠΆΠ΅Π½Π½Ρ‹Π΅ Π½Π° h. Если элСктричСскоС ΠΏΠΎΠ»Π΅ Ρ€Π°Π²Π½ΠΎ q Π½Π°Π΄ Π½ΡƒΠ»Π΅Π²Ρ‹ΠΌ эпсилон, ΠΈ ΠΎΡ‚ΡΡŽΠ΄Π° элСктричСскоС ΠΏΠΎΠ»Π΅ оказываСтся Ρ€Π°Π²Π½Ρ‹ΠΌ q Π½Π° 2 эпсилон ноль h, ΡƒΠΌΠ½ΠΎΠΆΠ΅Π½Π½ΠΎΠ΅ Π½Π° r.

И, ΠΊΠ°ΠΊ ΠΌΡ‹ Π²ΠΈΠ΄ΠΈΠΌ, это элСктричСскоС ΠΏΠΎΠ»Π΅ мСняСтся с Ρ€Π°Π΄ΠΈΠ°Π»ΡŒΠ½Ρ‹ΠΌ расстояниСм. И ΠΏΠ»ΠΎΡ‚Π½ΠΎΡΡ‚ΡŒ энСргии, ΠΏΠΎ ΠΎΠΏΡ€Π΅Π΄Π΅Π»Π΅Π½ΠΈΡŽ, Ρ€Π°Π²Π½Π° ΠΏΠΎΠ»ΠΎΠ²ΠΈΠ½Π΅ эпсилон ноль E Π² ΠΊΠ²Π°Π΄Ρ€Π°Ρ‚Π΅, ΠΈ поэтому ΠΏΠ»ΠΎΡ‚Π½ΠΎΡΡ‚ΡŒ энСргии, энСргия Π½Π° Π΅Π΄ΠΈΠ½ΠΈΡ†Ρƒ объСма этого цилиндричСского кондСнсатора, становится Ρ€Π°Π²Π½ΠΎΠΉ ΠΏΠΎΠ»ΠΎΠ²ΠΈΠ½Π΅ эпсилон ноль E Π² ΠΊΠ²Π°Π΄Ρ€Π°Ρ‚Π΅, ΠΈ это q Π² ΠΊΠ²Π°Π΄Ρ€Π°Ρ‚Π΅ Π½Π° 4 ΠΏΠΈ. Π² ΠΊΠ²Π°Π΄Ρ€Π°Ρ‚Π΅ эпсилон ноль Π² ΠΊΠ²Π°Π΄Ρ€Π°Ρ‚Π΅ h Π² ΠΊΠ²Π°Π΄Ρ€Π°Ρ‚Π΅ ΠΈ r Π² ΠΊΠ²Π°Π΄Ρ€Π°Ρ‚Π΅.

Π˜Ρ‚Π°ΠΊ, Ρ‚Π΅ΠΏΠ΅Ρ€ΡŒ ΠΌΡ‹ Π²ΠΈΠ΄ΠΈΠΌ, Ρ‡Ρ‚ΠΎ ΠΏΠ»ΠΎΡ‚Π½ΠΎΡΡ‚ΡŒ энСргии мСняСтся Π½Π° Π΅Π΄ΠΈΠ½ΠΈΡ†Ρƒ большС r Π² ΠΊΠ²Π°Π΄Ρ€Π°Ρ‚Π΅. ΠŸΠ»ΠΎΡ‚Π½ΠΎΡΡ‚ΡŒ энСргии — это энСргия Π½Π° Π΅Π΄ΠΈΠ½ΠΈΡ†Ρƒ объСма, ΠΈ эта ΠΏΠ»ΠΎΡ‚Π½ΠΎΡΡ‚ΡŒ Π½Π΅ постоянна. Если Π²Ρ‹ вспомнитС, Π½Π°ΠΏΡ€ΠΈΠΌΠ΅Ρ€, Π·Π°Π΄Π°Ρ‡Ρƒ ΠΏΠ΅Ρ€Π΅ΠΌΠ΅Π½Π½ΠΎΠΉ плотности заряда, ΠΊΠΎΡ‚ΠΎΡ€ΡƒΡŽ ΠΌΡ‹ Π΄Π΅Π»Π°Π»ΠΈ Ρ€Π°Π½Π΅Π΅, ΠΌΡ‹ собираСмся ΠΏΠΎΠ΄ΠΎΠΉΡ‚ΠΈ ΠΊ этой ΠΏΡ€ΠΎΠ±Π»Π΅ΠΌΠ΅ Π°Π½Π°Π»ΠΎΠ³ΠΈΡ‡Π½Ρ‹ΠΌ ΠΎΠ±Ρ€Π°Π·ΠΎΠΌ. БСйчас нас интСрСсуСт энСргия. Π˜Ρ‚Π°ΠΊ, ΠΌΡ‹ смотрим Π½Π° количСство энСргии, хранящСйся Π² этой области, Ρ…ΠΎΡ€ΠΎΡˆΠΎ? И ΠΌΡ‹ Π³ΠΎΠ²ΠΎΡ€ΠΈΠΌ, Ρ‡Ρ‚ΠΎ Ссли общая энСргия, накоплСнная ΠΌΠ΅ΠΆΠ΄Ρƒ пластинами этого кондСнсатора, Ρ€Π°Π²Π½Π° U, ΠΌΡ‹ Ρ…ΠΎΡ‚Π΅Π»ΠΈ Π±Ρ‹ Π²Ρ‹Ρ‡ΠΈΡΠ»ΠΈΡ‚ΡŒ радиус Ρ†ΠΈΠ»ΠΈΠ½Π΄Ρ€Π°, Ρ‡Ρ‚ΠΎΠ±Ρ‹ энСргия, запасСнная Π²Π½ΡƒΡ‚Ρ€ΠΈ этого Ρ†ΠΈΠ»ΠΈΠ½Π΄Ρ€Π°, ΠΊΠΎΡ‚ΠΎΡ€Ρ‹ΠΉ являСтся ΠΎΠ±Π»Π°ΡΡ‚ΡŒΡŽ Π²Π½ΡƒΡ‚Ρ€ΠΈ этого Гауссов Ρ†ΠΈΠ»ΠΈΠ½Π΄Ρ€, Π±ΡƒΠ΄Π΅Ρ‚ ΡΠΎΡΡ‚Π°Π²Π»ΡΡ‚ΡŒ ΠΏΠΎΠ»ΠΎΠ²ΠΈΠ½Ρƒ ΠΏΠΎΠ»Π½ΠΎΠΉ энСргии.Π­Ρ‚ΠΎ Ρ‚ΠΎ, Ρ‡Ρ‚ΠΎ Π½Π°ΠΌ Π½ΡƒΠΆΠ½ΠΎ.

И ΠΏΠΎΡΠΊΠΎΠ»ΡŒΠΊΡƒ наша ΠΏΠ»ΠΎΡ‚Π½ΠΎΡΡ‚ΡŒ энСргии измСняСтся с Ρ€Π°Π΄ΠΈΠ°Π»ΡŒΠ½Ρ‹ΠΌ расстояниСм, Ρ‚ΠΎ, Ρ‡Ρ‚ΠΎ ΠΌΡ‹ собираСмся ΡΠ΄Π΅Π»Π°Ρ‚ΡŒ, Π² этом случаС ΠΌΡ‹ Π²Ρ‹Π±Π΅Ρ€Π΅ΠΌ ΠΈΠ½ΠΊΡ€Π΅ΠΌΠ΅Π½Ρ‚Π½ΡƒΡŽ Ρ†ΠΈΠ»ΠΈΠ½Π΄Ρ€ΠΈΡ‡Π΅ΡΠΊΡƒΡŽ ΠΎΠ±ΠΎΠ»ΠΎΡ‡ΠΊΡƒ с ΠΎΡ‡Π΅Π½ΡŒ ΠΌΠ°Π»ΠΎΠΉ Ρ‚ΠΎΠ»Ρ‰ΠΈΠ½ΠΎΠΉ Π² Ρ€Π°Π΄ΠΈΠ°Π»ΡŒΠ½ΠΎΠΌ Π½Π°ΠΏΡ€Π°Π²Π»Π΅Π½ΠΈΠΈ. Π§Ρ‚ΠΎ-Ρ‚ΠΎ Π²Ρ€ΠΎΠ΄Π΅ этого. И эта ΠΎΠ±ΠΎΠ»ΠΎΡ‡ΠΊΠ° Π½Π°ΡΡ‚ΠΎΠ»ΡŒΠΊΠΎ Ρ‚ΠΎΠ½ΠΊΠ°, Ρ‡Ρ‚ΠΎ ΠΌΡ‹ ΠΏΡ€Π΅Π΄ΠΏΠΎΠ»ΠΎΠΆΠΈΠΌ, Ρ‡Ρ‚ΠΎ количСство энСргии, запасСнной вдоль этой сфСричСской ΠΎΠ±ΠΎΠ»ΠΎΡ‡ΠΊΠΈ, ΠΈ объСм этой сфСричСской ΠΎΠ±ΠΎΠ»ΠΎΡ‡ΠΊΠΈ, Π½Π°Π·ΠΎΠ²Π΅ΠΌ этот объСм db, объСмом ΡƒΠ²Π΅Π»ΠΈΡ‡ΠΈΠ²Π°ΡŽΡ‰Π΅ΠΉΡΡ цилиндричСской ΠΎΠ±ΠΎΠ»ΠΎΡ‡ΠΊΠΈ. И ΠΏΡ€Π΅Π΄ΠΏΠΎΠ»ΠΎΠΆΠΈΠΌ, Ρ‡Ρ‚ΠΎ радиус этой ΠΎΠ±ΠΎΠ»ΠΎΡ‡ΠΊΠΈ Ρ€Π°Π²Π΅Π½ s, Π° Ρ‚ΠΎΠ»Ρ‰ΠΈΠ½Π°, Π½Π°ΠΏΡ€ΠΈΠΌΠ΅Ρ€, ds, ΠΈ Ρ‡Ρ‚ΠΎ это du Π½Π°ΡΡ‚ΠΎΠ»ΡŒΠΊΠΎ ΠΌΠ°Π»ΠΎ, Ρ‡Ρ‚ΠΎ ΠΌΡ‹ ΠΌΠΎΠΆΠ΅ΠΌ ΠΏΡ€Π΅Π΄ΠΏΠΎΠ»ΠΎΠΆΠΈΡ‚ΡŒ, Ρ‡Ρ‚ΠΎ эта ΠΏΠ»ΠΎΡ‚Π½ΠΎΡΡ‚ΡŒ энСргии, которая измСняСтся Π² зависимости ΠΎΡ‚ ΠΊΠ²Π°Π΄Ρ€Π°Ρ‚Π° Ρ€Π°Π΄ΠΈΠ°Π»ΡŒΠ½ΠΎΠ³ΠΎ расстояния, остаСтся постоянной Π½Π° этом расстоянии.Π—Π°Ρ‚Π΅ΠΌ ΠΌΡ‹ ΠΌΠΎΠΆΠ΅ΠΌ Π²Ρ‹Ρ‡ΠΈΡΠ»ΠΈΡ‚ΡŒ количСство энСргии, хранящСйся Π²Π½ΡƒΡ‚Ρ€ΠΈ этой цилиндричСской ΠΎΠ±ΠΎΠ»ΠΎΡ‡ΠΊΠΈ.

И ΠΊΠ°ΠΊ это Π²Ρ‹Ρ‡ΠΈΡΠ»ΠΈΡ‚ΡŒ? ЭнСргия, Π½Π°Π·ΠΎΠ²Π΅ΠΌ Π΅Π΅ d sub u, энСргия, запасСнная Π² ΡƒΠ²Π΅Π»ΠΈΡ‡ΠΈΠ²Π°ΡŽΡ‰Π΅ΠΉΡΡ цилиндричСской ΠΎΠ±ΠΎΠ»ΠΎΡ‡ΠΊΠ΅. И это Π±ΡƒΠ΄Π΅Ρ‚ Ρ€Π°Π²Π½ΠΎ плотности энСргии u, Ρ‚ΠΎ Π΅ΡΡ‚ΡŒ энСргии Π½Π° Π΅Π΄ΠΈΠ½ΠΈΡ†Ρƒ объСма, ΠΊΠ°ΠΊ Π²Ρ‹ ΠΏΠΎΠΌΠ½ΠΈΡ‚Π΅, ΠΏΠΎ ΠΎΠΏΡ€Π΅Π΄Π΅Π»Π΅Π½ΠΈΡŽ, ΡƒΠΌΠ½ΠΎΠΆΠ΅Π½Π½ΠΎΠΉ Π½Π° объСм области, которая нас интСрСсуСт, ΠΈ это объСм ΡƒΠ²Π΅Π»ΠΈΡ‡ΠΈΠ²Π°ΡŽΡ‰Π΅ΠΉΡΡ цилиндричСской ΠΎΠ±ΠΎΠ»ΠΎΡ‡ΠΊΠΈ. Π’Π°ΠΊΠΈΠΌ ΠΎΠ±Ρ€Π°Π·ΠΎΠΌ, это Π²Ρ‹Ρ€Π°ΠΆΠ΅Π½ΠΈΠ΅ даст Π½Π°ΠΌ, сколько энСргии находится Π²Π½ΡƒΡ‚Ρ€ΠΈ этой области.Π’ явном Π²ΠΈΠ΄Π΅ это Π±ΡƒΠ΄Π΅Ρ‚ Ρ€Π°Π²Π½ΡΡ‚ΡŒΡΡ ΠΏΠΎΠ»ΠΎΠ²ΠΈΠ½Π΅ эпсилон ноль q Π² ΠΊΠ²Π°Π΄Ρ€Π°Ρ‚Π΅ Π½Π° 4 ΠΏΠΈ Π² ΠΊΠ²Π°Π΄Ρ€Π°Ρ‚Π΅ эпсилон ноль Π² ΠΊΠ²Π°Π΄Ρ€Π°Ρ‚Π΅ h Π² ΠΊΠ²Π°Π΄Ρ€Π°Ρ‚Π΅ r Π² ΠΊΠ²Π°Π΄Ρ€Π°Ρ‚Π΅. По сути, это ΠΏΠ»ΠΎΡ‚Π½ΠΎΡΡ‚ΡŒ энСргии u, ΠΈ Π΄Π°Π²Π°ΠΉΡ‚Π΅ ΠΎΠΏΡ€Π΅Π΄Π΅Π»ΠΈΠΌ Π΅Π΅ ΠΊΠ°ΠΊ dv, объСм ΡƒΠ²Π΅Π»ΠΈΡ‡ΠΈΠ²Π°ΡŽΡ‰Π΅ΠΉΡΡ цилиндричСской ΠΎΠ±ΠΎΠ»ΠΎΡ‡ΠΊΠΈ.

А Π΄Π°Π²Π°ΠΉΡ‚Π΅ посчитаСм этот объСм. Π­Ρ‚ΠΎ инкрСмСнтная цилиндричСская ΠΎΠ±ΠΎΠ»ΠΎΡ‡ΠΊΠ°, Ρ‡Ρ‚ΠΎ-Ρ‚ΠΎ Π²Ρ€ΠΎΠ΄Π΅ этого, ΠΈ нас интСрСсуСт объСм этого Ρ†ΠΈΠ»ΠΈΠ½Π΄Ρ€Π°. Он ΠΈΠΌΠ΅Π΅Ρ‚ радиус s ΠΈ Ρ‚ΠΎΠ»Ρ‰ΠΈΠ½Ρƒ ds. ΠšΠΎΠ½Π΅Ρ‡Π½ΠΎ, ΠΏΠΎΡΠΊΠΎΠ»ΡŒΠΊΡƒ Π΅Π³ΠΎ ΠΏΠ»ΠΎΡ‰Π°Π΄ΡŒ ΠΏΠΎΠΏΠ΅Ρ€Π΅Ρ‡Π½ΠΎΠ³ΠΎ сСчСния постоянна ΠΏΠΎ всСй Π΄Π»ΠΈΠ½Π΅, объСм этой ΠΎΠ±ΠΎΠ»ΠΎΡ‡ΠΊΠΈ, dv, Π±ΡƒΠ΄Π΅Ρ‚ Ρ€Π°Π²Π΅Π½ ΠΏΠ»ΠΎΡ‰Π°Π΄ΠΈ повСрхности, ΠΈ это Π±ΡƒΠ΄Π΅Ρ‚ ΠΏΠ»ΠΎΡ‰Π°Π΄ΡŒ этого ΡƒΠ²Π΅Π»ΠΈΡ‡ΠΈΠ²Π°ΡŽΡ‰Π΅Π³ΠΎΡΡ ΠΊΠΎΠ»ΡŒΡ†Π°, которая Ρ€Π°Π²Π½Π° 2 pi r dr , ΡƒΠΌΠ½ΠΎΠΆΠ΅Π½Π½ΠΎΠ΅ Π½Π° высоту этого Ρ†ΠΈΠ»ΠΈΠ½Π΄Ρ€Π°, ΠΈ это h.ΠŸΠΎΡΠΊΠΎΠ»ΡŒΠΊΡƒ здСсь ΠΌΡ‹ ΠΈΡΠΏΠΎΠ»ΡŒΠ·ΡƒΠ΅ΠΌ ΠΏΠ΅Ρ€Π΅ΠΌΠ΅Π½Π½ΡƒΡŽ s, Π΄Π°Π²Π°ΠΉΡ‚Π΅ Π·Π°ΠΌΠ΅Π½ΠΈΠΌ эти Β«rΒ» Π½Π° ds, Π° Ρ‚Π°ΠΊΠΆΠ΅ r Π² плотности энСргии Π½Π° s. Π₯ΠΎΡ€ΠΎΡˆΠΎ, Ρ‚ΠΎΠ³Π΄Π° явная Ρ„ΠΎΡ€ΠΌΠ° Π΄ΠΎΠΏΠΎΠ»Π½ΠΈΡ‚Π΅Π»ΡŒΠ½ΠΎΠΉ ΠΏΠΎΡ‚Π΅Π½Ρ†ΠΈΠ°Π»ΡŒΠ½ΠΎΠΉ энСргии, хранящСйся Π²Π½ΡƒΡ‚Ρ€ΠΈ объСма этой цилиндричСской ΠΎΠ±ΠΎΠ»ΠΎΡ‡ΠΊΠΈ, Π±ΡƒΠ΄Π΅Ρ‚ Ρ€Π°Π²Π½Π° Π΅Π΄ΠΈΠ½ΠΈΡ†Π΅ Π½Π° 2 эпсилон ноль q Π² ΠΊΠ²Π°Π΄Ρ€Π°Ρ‚Π΅ Π½Π° 4 ΠΏΠΈ Π² ΠΊΠ²Π°Π΄Ρ€Π°Ρ‚Π΅ эпсилон Π² ΠΊΠ²Π°Π΄Ρ€Π°Ρ‚Π΅ нуля h Π² ΠΊΠ²Π°Π΄Ρ€Π°Ρ‚Π΅ s Π² ΠΊΠ²Π°Π΄Ρ€Π°Ρ‚Π΅, ΡƒΠΌΠ½ΠΎΠΆΠ΅Π½Π½ΠΎΠΌ Π½Π° dv, объСм этого приращСния цилиндричСская ΠΎΠ±ΠΎΠ»ΠΎΡ‡ΠΊΠ°, которая Ρ€Π°Π²Π½Π° 2 p s ds Ρ€Π°Π· h.

Π˜Ρ‚Π°ΠΊ, Ρƒ нас Π΅ΡΡ‚ΡŒ энСргия Π½Π° Π΅Π΄ΠΈΠ½ΠΈΡ†Ρƒ объСма, умноТСнная Π½Π° объСм, ΠΊΠΎΡ‚ΠΎΡ€Ρ‹ΠΉ нас интСрСсуСт, поэтому ΠΎΠ±ΡŠΠ΅ΠΌΡ‹ ΡƒΡ€Π°Π²Π½ΡΡŽΡ‚ΡΡ, ΠΈ Π² ΠΈΡ‚ΠΎΠ³Π΅ ΠΌΡ‹ ΠΏΠΎΠ»ΡƒΡ‡ΠΈΠΌ ΡΠ½Π΅Ρ€Π³ΠΈΡŽ Π²Π½ΡƒΡ‚Ρ€ΠΈ этой области, ΡΠ½Π΅Ρ€Π³ΠΈΡŽ, Π·Π°ΠΏΠ°ΡΠ΅Π½Π½ΡƒΡŽ Π² этой области.И здСсь ΠΌΡ‹ ΠΌΠΎΠΆΠ΅ΠΌ ΠΎΡ‚ΠΌΠ΅Π½ΠΈΡ‚ΡŒ ΠΎΠ΄Π½ΠΎ ΠΈΠ· этих s, Ρ‡ΠΈΡΠ»ΠΈΡ‚Π΅Π»ΡŒ ΠΈ Π·Π½Π°ΠΌΠ΅Π½Π°Ρ‚Π΅Π»ΡŒ, ΠΈ ΠΌΡ‹ ΠΌΠΎΠΆΠ΅ΠΌ ΠΎΡ‚ΠΌΠ΅Π½ΠΈΡ‚ΡŒ ΠΎΠ΄Π½ΠΎ ΠΈΠ· этих h Π² ΠΊΠ²Π°Π΄Ρ€Π°Ρ‚Π΅ с h Π² числитСлС, ΠΈ Π°Π½Π°Π»ΠΎΠ³ΠΈΡ‡Π½ΠΎ, эпсилон ноль отмСняСтся с эпсилон ноль Π² ΠΊΠ²Π°Π΄Ρ€Π°Ρ‚Π΅, Π° ΠΊΠ²Π°Π΄Ρ€Π°Ρ‚ ΠΏΠΈ отмСняСтся этим pi, ΠΈ ΠΌΡ‹ ΠΌΠΎΠΆΠ΅ΠΌ ΠΎΡ‚ΠΌΠ΅Π½ΠΈΡ‚ΡŒ 2 с ΠΏΠΎΠΌΠΎΡ‰ΡŒΡŽ этого 2. И ΠΌΡ‹ собираСмся ΠΏΠΎΠ»ΡƒΡ‡ΠΈΡ‚ΡŒ ΠΈΠ½ΠΊΡ€Π΅ΠΌΠ΅Π½Ρ‚Π½ΡƒΡŽ ΠΏΠΎΡ‚Π΅Π½Ρ†ΠΈΠ°Π»ΡŒΠ½ΡƒΡŽ ΡΠ½Π΅Ρ€Π³ΠΈΡŽ, Ρ…Ρ€Π°Π½ΡΡ‰ΡƒΡŽΡΡ Π²Π½ΡƒΡ‚Ρ€ΠΈ объСма этой ΠΈΠ½ΠΊΡ€Π΅ΠΌΠ΅Π½Ρ‚Π½ΠΎΠΉ цилиндричСской ΠΎΠ±ΠΎΠ»ΠΎΡ‡ΠΊΠΈ, ΠΊΠ°ΠΊ q Π² ΠΊΠ²Π°Π΄Ρ€Π°Ρ‚Π΅, Π΄Π΅Π»Π΅Π½Π½ΠΎΠ΅ Π½Π° 4 ΠΏΠΈ-эпсилон ноль hs ds.

Π₯ΠΎΡ€ΠΎΡˆΠΎ. Π§Ρ‚ΠΎ ΠΆ, это количСство энСргии, хранящСйся Π² объСмС этой ΡƒΠ²Π΅Π»ΠΈΡ‡ΠΈΠ²Π°ΡŽΡ‰Π΅ΠΉΡΡ цилиндричСской ΠΎΠ±ΠΎΠ»ΠΎΡ‡ΠΊΠΈ.Π― ΠΌΠΎΠ³Ρƒ просто ΠΏΠΎΠΉΡ‚ΠΈ дальшС ΠΈ Π²Ρ‹Ρ‡ΠΈΡΠ»ΠΈΡ‚ΡŒ ΡΠ½Π΅Ρ€Π³ΠΈΡŽ, Π·Π°ΠΏΠ°ΡΠ΅Π½Π½ΡƒΡŽ Π² ΡΠ»Π΅Π΄ΡƒΡŽΡ‰Π΅ΠΉ ΠΎΠ±ΠΎΠ»ΠΎΡ‡ΠΊΠ΅ ΠΈ Π² ΡΠ»Π΅Π΄ΡƒΡŽΡ‰Π΅ΠΉ ΠΎΠ±ΠΎΠ»ΠΎΡ‡ΠΊΠ΅, ΠΈ Ρ‚Π°ΠΊ Π΄Π°Π»Π΅Π΅ ΠΈ Ρ‚Π°ΠΊ Π΄Π°Π»Π΅Π΅, ΠΏΠΎ всСй области, которая мСня интСрСсуСт. И эта ΠΎΠ±Π»Π°ΡΡ‚ΡŒ Ρ€Π°ΡΡˆΠΈΡ€ΡΠ΅Ρ‚ΡΡ, радиус начинаСтся ΠΎΡ‚ a Π΄ΠΎ этого ΠΊΠΎΠ½ΠΊΡ€Π΅Ρ‚Π½ΠΎΠ³ΠΎ радиуса r, ΠΊΠΎΡ‚ΠΎΡ€Ρ‹ΠΉ я ΠΏΡ‹Ρ‚Π°ΡŽΡΡŒ Π²Ρ‹Ρ‡ΠΈΡΠ»ΠΈΡ‚ΡŒ. Π‘Π»Π΅Π΄ΠΎΠ²Π°Ρ‚Π΅Π»ΡŒΠ½ΠΎ, Ссли я ΠΏΡ€ΠΎΠΈΠ½Ρ‚Π΅Π³Ρ€ΠΈΡ€ΡƒΡŽ это du ΠΎΡ‚ΡΡŽΠ΄Π°, ΠΎΡ‚ a Π΄ΠΎ r, Ρ‚ΠΎΠ³Π΄Π° я ΠΏΠΎΠ»ΡƒΡ‡Ρƒ количСство энСргии, хранящСйся Π²Π½ΡƒΡ‚Ρ€ΠΈ этого ΠΏΡƒΡ€ΠΏΡƒΡ€Π½ΠΎΠ³ΠΎ Ρ†ΠΈΠ»ΠΈΠ½Π΄Ρ€Π°.

Π§Ρ‚ΠΎ ΠΆ, я Ρ…ΠΎΡ‡Ρƒ, Ρ‡Ρ‚ΠΎΠ±Ρ‹ эта энСргия составляла ΠΏΠΎΠ»ΠΎΠ²ΠΈΠ½Ρƒ ΠΎΠ±Ρ‰Π΅ΠΉ энСргии, хранящСйся Π² элСктричСском ΠΏΠΎΠ»Π΅ этого кондСнсатора, ΠΈ общая энСргия ΠΌΠΎΠΆΠ΅Ρ‚ Π±Ρ‹Ρ‚ΡŒ ΠΏΠΎΠ»ΡƒΡ‡Π΅Π½Π° ΠΏΡƒΡ‚Π΅ΠΌ слоТСния этих Β«duΒ», Π΄ΠΎΠΏΠΎΠ»Π½ΠΈΡ‚Π΅Π»ΡŒΠ½Ρ‹Ρ… энСргий, хранящихся Π² этих Π΄ΠΎΠΏΠΎΠ»Π½ΠΈΡ‚Π΅Π»ΡŒΠ½Ρ‹Ρ… цилиндричСских ΠΎΠ±ΠΎΠ»ΠΎΡ‡ΠΊΠ°Ρ…, начиная с этого ΠΎΡ‚ Π²Π½ΡƒΡ‚Ρ€Π΅Π½Π½Π΅Π³ΠΎ радиуса a Π΄ΠΎ внСшнСго радиуса b.Π”Ρ€ΡƒΠ³ΠΈΠΌΠΈ словами, ΠΌΡ‹ ΠΈΠΌΠ΅Π΅ΠΌ здСсь, Ρ‡Ρ‚ΠΎ ΠΈΠ½Ρ‚Π΅Π³Ρ€Π°Π» du, ΠΏΡ€ΠΎΠΈΠ½Ρ‚Π΅Π³Ρ€ΠΈΡ€ΠΎΠ²Π°Π½Π½Ρ‹ΠΉ ΠΎΡ‚ a Π΄ΠΎ r, Π΄ΠΎ радиуса, ΠΊΠΎΡ‚ΠΎΡ€Ρ‹ΠΉ нас интСрСсуСт, ΠΌΡ‹ Ρ…ΠΎΡ‚ΠΈΠΌ, Ρ‡Ρ‚ΠΎΠ±Ρ‹ эта энСргия Π±Ρ‹Π»Π° Ρ€Π°Π²Π½Π° ΠΏΠΎΠ»ΠΎΠ²ΠΈΠ½Π΅ ΠΏΠΎΠ»Π½ΠΎΠΉ энСргии. И полная энСргия являСтся ΠΈΠ½Ρ‚Π΅Π³Ρ€Π°Π»ΠΎΠΌ du, количСства энСргии, хранящСйся Π² этих Π΄ΠΎΠΏΠΎΠ»Π½ΠΈΡ‚Π΅Π»ΡŒΠ½Ρ‹Ρ… цилиндричСских ΠΎΠ±ΠΎΠ»ΠΎΡ‡ΠΊΠ°Ρ…, ΠΈΠ½Ρ‚Π΅Π³Ρ€ΠΈΡ€ΠΎΠ²Π°Π½Π½ΠΎΠΉ ΠΎΡ‚ a Π΄ΠΎ b, которая даст Π½Π°ΠΌ ΠΏΠΎΠ»Π½ΡƒΡŽ ΡΠ½Π΅Ρ€Π³ΠΈΡŽ, ΠΈ я Ρ…ΠΎΡ‡Ρƒ, Ρ‡Ρ‚ΠΎΠ±Ρ‹ эта энСргия составляла ΠΏΠΎΠ»ΠΎΠ²ΠΈΠ½Ρƒ этой энСргии. ΠŸΠΎΡΡ‚ΠΎΠΌΡƒ я Ρ€Π°Π·Π΄Π΅Π»ΡŽ это Π½Π° 2.

Π₯ΠΎΡ€ΠΎΡˆΠΎ. Π”Π°Π²Π°ΠΉΡ‚Π΅ Ρ‚Π΅ΠΏΠ΅Ρ€ΡŒ вычислим эти ΠΈΠ½Ρ‚Π΅Π³Ρ€Π°Π»Ρ‹.Π˜Π½Ρ‚Π΅Π³Ρ€Π°Π» ΠΎΡ‚ a Π΄ΠΎ r ΠΈΠ· q Π² ΠΊΠ²Π°Π΄Ρ€Π°Ρ‚Π΅ Π½Π° 4 ΠΏΠΈ-эпсилон ноль h, ΡƒΠΌΠ½ΠΎΠΆΠ΅Π½Π½Ρ‹ΠΉ Π½Π° s, ΡƒΠΌΠ½ΠΎΠΆΠ΅Π½Π½Ρ‹ΠΉ Π½Π° ds, Π±ΡƒΠ΄Π΅Ρ‚ Ρ€Π°Π²Π΅Π½ ΠΏΠΎΠ»ΠΎΠ²ΠΈΠ½Π΅ ΠΈΠ½Ρ‚Π΅Π³Ρ€Π°Π»Π° ΠΎΡ‚ a Π΄ΠΎ b ΠΈΠ· q, Π²ΠΎΠ·Π²Π΅Π΄Π΅Π½Π½ΠΎΠ³ΠΎ Π² ΠΊΠ²Π°Π΄Ρ€Π°Ρ‚ 4 ΠΏΠΈ-эпсилон нуля h s, ΡƒΠΌΠ½ΠΎΠΆΠ΅Π½Π½ΠΎΠ³ΠΎ Π½Π° ds. А здСсь q Π² ΠΊΠ²Π°Π΄Ρ€Π°Ρ‚Π΅ 4 ΠΏΠΈ эпсилон ноль h, эти Ρ‡Π»Π΅Π½Ρ‹ постоянны, ΠΌΡ‹ ΠΌΠΎΠΆΠ΅ΠΌ вынСсти ΠΈΡ… Π·Π° ΠΏΡ€Π΅Π΄Π΅Π»Ρ‹ ΠΈΠ½Ρ‚Π΅Π³Ρ€Π°Π»Π° для ΠΎΠ±Π΅ΠΈΡ… сторон. И послС Ρ‚ΠΎΠ³ΠΎ, ΠΊΠ°ΠΊ ΠΌΡ‹ вынСсСм ΠΈΡ… Π½Π°Ρ€ΡƒΠΆΡƒ, ΠΌΡ‹ Π»Π΅Π³ΠΊΠΎ ΡƒΠ²ΠΈΠ΄ΠΈΠΌ, Ρ‡Ρ‚ΠΎ, ΠΏΠΎΡΠΊΠΎΠ»ΡŒΠΊΡƒ Ρƒ нас Π΅ΡΡ‚ΡŒ ΠΎΠ΄ΠΈΠ½Π°ΠΊΠΎΠ²Ρ‹Π΅ Ρ‡Π»Π΅Π½Ρ‹ с ΠΎΠ±Π΅ΠΈΡ… сторон, дСля ΠΎΠ±Π΅ части уравнСния Π½Π° ΠΎΠ΄Π½ΠΈ ΠΈ Ρ‚Π΅ ΠΆΠ΅ Ρ‡Π»Π΅Π½Ρ‹, ΠΌΡ‹ ΠΌΠΎΠΆΠ΅ΠΌ ΠΈΡ… ΠΈΡΠΊΠ»ΡŽΡ‡ΠΈΡ‚ΡŒ.

Π˜Ρ‚Π°ΠΊ, ΠΌΡ‹ собираСмся Π·Π°ΠΊΠΎΠ½Ρ‡ΠΈΡ‚ΡŒ Ρ‚Π΅ΠΌ, Ρ‡Ρ‚ΠΎ ΠΈΠ½Ρ‚Π΅Π³Ρ€Π°Π» ΠΎΡ‚ a Π΄ΠΎ r ΠΎΡ‚ ds ΠΏΠΎ s Π±ΡƒΠ΄Π΅Ρ‚ Ρ€Π°Π²Π΅Π½ ΠΏΠΎΠ»ΠΎΠ²ΠΈΠ½Π½ΠΎΠΌΡƒ ΠΈΠ½Ρ‚Π΅Π³Ρ€Π°Π»Ρƒ ΠΎΡ‚ a Π΄ΠΎ b ΠΎΡ‚ ds ΠΏΠΎ s.Π”Π²ΠΈΠ³Π°ΡΡΡŒ дальшС, ΠΈΠ½Ρ‚Π΅Π³Ρ€Π°Π» ds ΠΏΠΎ s Ρ€Π°Π²Π΅Π½ ln ΠΎΡ‚ s, ΠΈ ΠΎΠ½ Π±ΡƒΠ΄Π΅Ρ‚ ΠΎΡ†Π΅Π½ΠΈΠ²Π°Ρ‚ΡŒΡΡ Π² a ΠΈ r, Π±ΡƒΠ΄Π΅Ρ‚ Ρ€Π°Π²Π΅Π½ ΠΏΠΎΠ»ΠΎΠ²ΠΈΠ½Π΅, снова ln ΠΎΡ‚ S, Ρ‚Π΅ΠΏΠ΅Ρ€ΡŒ ΠΎΠ½ оцСниваСтся Π² a ΠΈ b. ΠŸΠΎΠ΄ΡΡ‚Π°Π²Π»ΡΡ Π³Ρ€Π°Π½ΠΈΡ†Ρ‹, ΠΌΡ‹ ΠΏΠΎΠ»ΡƒΡ‡ΠΈΠΌ ln ΠΈΠ· r минус ln ΠΈΠ· a, Π±ΡƒΠ΄Π΅Ρ‚ Ρ€Π°Π²Π½ΠΎ ΠΏΠΎΠ»ΠΎΠ²ΠΈΠ½Π΅ ln ΠΈΠ· b минус ΠΏΠΎΠ»ΠΎΠ²ΠΈΠ½Π° ln ΠΈΠ· a. Если ΠΌΡ‹ оставим ln of r Π² ΠΏΠΎΠΊΠΎΠ΅ Π½Π° ΠΎΠ΄Π½ΠΎΠΉ сторонС уравнСния, пСрСмСстив ln of a Π½Π° Π΄Ρ€ΡƒΠ³ΡƒΡŽ сторону, ΠΌΡ‹ ΠΏΠΎΠ»ΡƒΡ‡ΠΈΠΌ, Ρ‡Ρ‚ΠΎ ΠΏΠΎΠ»ΠΎΠ²ΠΈΠ½Π° ΠΎΠ΄Π½ΠΎΠΉ ΠΏΠΎΠ»ΠΎΠ²ΠΈΠ½Ρ‹ ln a Π±ΡƒΠ΄Π΅Ρ‚ ΠΏΠΎΠ»ΠΎΠΆΠΈΡ‚Π΅Π»ΡŒΠ½ΠΎΠΉ для Π΄Ρ€ΡƒΠ³ΠΎΠΉ стороны, Π° минус ΠΏΠΎΠ»ΠΎΠ²ΠΈΠ½Π° ln a даст Π½Π°ΠΌ ΠΎΠ΄Π½Π° ΠΏΠΎΠ»ΠΎΠ²ΠΈΠ½Π° ln плюс ΠΏΠΎΠ»ΠΎΠ²ΠΈΠ½Π° ln b.

ΠœΡ‹ ΠΌΠΎΠΆΠ΅ΠΌ ΠΏΠ΅Ρ€Π΅ΠΏΠΈΡΠ°Ρ‚ΡŒ это Ρ‚Π°ΠΊ, ΠΊΠ°ΠΊ ln of r Ρ€Π°Π²Π½ΠΎ ln a Π² ΠΏΠΎΠ»ΠΎΠ²ΠΈΠ½Π½Ρ‹Ρ… скобках плюс ΠΏΠΎΠ»ΠΎΠ²ΠΈΠ½Π° ln b, которая Ρ€Π°Π²Π½Π° ΠΏΠΎΠ»ΠΎΠ²ΠΈΠ½Π΅ ln a, ΡƒΠΌΠ½ΠΎΠΆΠ΅Π½Π½ΠΎΠΌΡƒ Π½Π° b. ln числа r Π±ΡƒΠ΄Π΅Ρ‚ Ρ€Π°Π²Π½ΠΎ ln ΡƒΠΌΠ½ΠΎΠΆΠ΅Π½Π½ΠΎΠ³ΠΎ Π½Π° b Π² стСпСни ΠΏΠΎΠ»ΠΎΠ²ΠΈΠ½Ρ‹, ΠΈ Ссли ΠΌΡ‹ возьмСм ΠΏΠΎΠ³Ρ€ΡƒΠΆΠ΅Π½Π½Ρ‹ΠΉ Π»ΠΎΠ³Π°Ρ€ΠΈΡ„ΠΌ ΠΎΠ±Π΅ΠΈΡ… сторон, Ρ‚ΠΎ ΠΏΠΎΠ»ΡƒΡ‡ΠΈΠΌ, Ρ‡Ρ‚ΠΎ r Ρ€Π°Π²Π½ΠΎ ΠΊΠ²Π°Π΄Ρ€Π°Ρ‚Π½ΠΎΠΌΡƒ ΠΊΠΎΡ€Π½ΡŽ ΠΈΠ· умноТСния Π½Π° b. И это Π±Ρ‹Π» Ρ‚ΠΎΡ‚ случай, ΠΊΠΎΡ‚ΠΎΡ€Ρ‹ΠΉ ΠΌΡ‹ Π΄ΠΎΠ»ΠΆΠ½Ρ‹ Π±Ρ‹Π»ΠΈ Π΄ΠΎΠΊΠ°Π·Π°Ρ‚ΡŒ. Π”Π΅ΠΉΡΡ‚Π²ΠΈΡ‚Π΅Π»ΡŒΠ½ΠΎ, Ссли радиус Ρ†ΠΈΠ»ΠΈΠ½Π΄Ρ€Π° Ρ€Π°Π²Π΅Π½ ΠΊΠ²Π°Π΄Ρ€Π°Ρ‚Π½ΠΎΠΌΡƒ ΠΊΠΎΡ€Π½ΡŽ ΠΈΠ· Π²Π½ΡƒΡ‚Ρ€Π΅Π½Π½Π΅Π³ΠΎ радиуса, ΡƒΠΌΠ½ΠΎΠΆΠ΅Π½Π½ΠΎΠ³ΠΎ Π½Π° внСшний радиус, Ρ‚ΠΎ количСство энСргии для этого кондСнсатора, хранящСйся Π² этой области, Π±ΡƒΠ΄Π΅Ρ‚ ΡΠΎΡΡ‚Π°Π²Π»ΡΡ‚ΡŒ ΠΏΠΎΠ»ΠΎΠ²ΠΈΠ½Ρƒ ΠΏΠΎΡ‚Π΅Π½Ρ†ΠΈΠ°Π»ΡŒΠ½ΠΎΠΉ энСргии, хранящСйся ΠΌΠ΅ΠΆΠ΄Ρƒ пластинами этого Ρ†ΠΈΠ»ΠΈΠ½Π΄Ρ€.Π”Ρ€ΡƒΠ³ΠΈΠΌΠΈ словами, ΠΊΠΎΠ³Π΄Π° ΠΌΡ‹ рассматриваСм эту ΠΎΠ±Π»Π°ΡΡ‚ΡŒ с Ρ‚Π°ΠΊΠΈΠΌ радиусом, Ρ‡Ρ‚ΠΎ r Ρ€Π°Π²Π½ΠΎ ΠΊΠ²Π°Π΄Ρ€Π°Ρ‚Π½ΠΎΠΌΡƒ ΠΊΠΎΡ€Π½ΡŽ ΠΈΠ· a, ΡƒΠΌΠ½ΠΎΠΆΠ΅Π½Π½ΠΎΠΌΡƒ Π½Π° b, Π²Π½ΡƒΡ‚Ρ€ΠΈ этого Ρ†ΠΈΠ»ΠΈΠ½Π΄Ρ€Π° количСство запасСнной энСргии просто Ρ€Π°Π²Π½ΠΎ ΠΏΠΎΠ»ΠΎΠ²ΠΈΠ½Π΅ ΠΏΠΎΠ»Π½ΠΎΠΉ энСргии, запасСнной этим цилиндричСским кондСнсатором. .

Π’Π°ΠΊΠΈΠΌ ΠΎΠ±Ρ€Π°Π·ΠΎΠΌ, этот ΠΏΡ€ΠΈΠΌΠ΅Ρ€ дСмонстрируСт использованиС плотности энСргии, ΠΊΠΎΡ‚ΠΎΡ€ΡƒΡŽ ΠΌΡ‹ опрСдСляСм ΠΊΠ°ΠΊ ΡΠ½Π΅Ρ€Π³ΠΈΡŽ Π½Π° Π΅Π΄ΠΈΠ½ΠΈΡ†Ρƒ объСма. Π­Ρ‚ΠΎ Ρ‚Π°ΠΊΠΆΠ΅ ΠΏΠΎΠΊΠ°Π·Ρ‹Π²Π°Π΅Ρ‚, ΠΊΠ°ΠΊ ΠΌΡ‹ ΠΏΠΎΠ΄Ρ…ΠΎΠ΄ΠΈΠΌ ΠΊ случаям ΠΏΠ΅Ρ€Π΅ΠΌΠ΅Π½Π½ΠΎΠΉ плотности. Π­Ρ‚ΠΎ Π² Π½Π΅ΠΊΠΎΡ‚ΠΎΡ€ΠΎΠΌ Ρ€ΠΎΠ΄Π΅ ΠΏΡ€ΠΈΠΌΠ΅Ρ€ Ρ‚ΠΎΠ³ΠΎ ΠΆΠ΅ Ρ‚ΠΈΠΏΠ°, Ρ‡Ρ‚ΠΎ ΠΈ ΠΏΡ€ΠΎΠ±Π»Π΅ΠΌΠ° с ΠΏΠ΅Ρ€Π΅ΠΌΠ΅Π½Π½ΠΎΠΉ ΠΏΠ»ΠΎΡ‚Π½ΠΎΡΡ‚ΡŒΡŽ заряда.Но здСсь вмСсто ΠΏΠ΅Ρ€Π΅ΠΌΠ΅Π½Π½ΠΎΠΉ плотности заряда ΠΌΡ‹ ΠΈΠΌΠ΅Π΅ΠΌ Π΄Π΅Π»ΠΎ с ΠΏΠ΅Ρ€Π΅ΠΌΠ΅Π½Π½ΠΎΠΉ ΠΏΠ»ΠΎΡ‚Π½ΠΎΡΡ‚ΡŒΡŽ энСргии. Но матСматичСский способ Π°Π½Π°Π»ΠΈΠ·Π° Ρ‚Π°ΠΊΠΈΡ… ΠΏΡ€ΠΎΠ±Π»Π΅ΠΌ практичСски ΠΈΠ΄Π΅Π½Ρ‚ΠΈΡ‡Π΅Π½.

Π€ΠΎΡ€ΠΌΡƒΠ»Ρ‹ ΠΈ ΠΊΠ°Π»ΡŒΠΊΡƒΠ»ΡΡ‚ΠΎΡ€Ρ‹ Смкости

На этой страницС прСдставлСны Ρ„ΠΎΡ€ΠΌΡƒΠ»Ρ‹ ΠΈ ΠΊΠ°Π»ΡŒΠΊΡƒΠ»ΡΡ‚ΠΎΡ€Ρ‹ СмкостСй
Ρ€Π°Π·Π»ΠΈΡ‡Π½Ρ‹Π΅ Ρ„ΠΎΡ€ΠΌΡ‹ ΠΈΠ»ΠΈ Ρ‚ΠΈΠΏΡ‹ кондСнсаторов. Π­Ρ‚ΠΎ Ρ‚Π°ΠΊΠΆΠ΅ ΠΏΠΎΠ»Π΅Π·Π½ΠΎ, Ссли Π²Ρ‹
ΡΠΎΠ±ΠΈΡ€Π°Π΅Ρ‚Π΅ΡΡŒ ΠΈΡΠΏΠΎΠ»ΡŒΠ·ΠΎΠ²Π°Ρ‚ΡŒ свой кондСнсатор Π²
Π’Π°Π½ΠΊ LC рСзонансный
схСма.

Π•ΠΌΠΊΠΎΡΡ‚ΡŒ кондСнсаторов с ΠΏΠ°Ρ€Π°Π»Π»Π΅Π»ΡŒΠ½Ρ‹ΠΌΠΈ пластинами

ΠšΠΎΠ½Π΄Π΅Π½ΡΠ°Ρ‚ΠΎΡ€ с ΠΏΠ°Ρ€Π°Π»Π»Π΅Π»ΡŒΠ½Ρ‹ΠΌΠΈ пластинами состоит ΠΈΠ· Π΄Π²ΡƒΡ… плоских ΠΏΠ°Ρ€Π°Π»Π»Π΅Π»ΡŒΠ½Ρ‹Ρ… пластин, ΠΊΠΎΡ‚ΠΎΡ€Ρ‹Π΅
элСктроды, Ρ€Π°Π·Π΄Π΅Π»Π΅Π½Π½Ρ‹Π΅
диэлСктрик
ΠΈΠ»ΠΈ изолятор.Для Ρ„ΠΎΡ€ΠΌΡƒΠ»Ρ‹ ΠΈ ΠΊΠ°Π»ΡŒΠΊΡƒΠ»ΡΡ‚ΠΎΡ€Π° здСсь пластины ΠΌΠΎΠ³ΡƒΡ‚ Π±Ρ‹Ρ‚ΡŒ
любой Ρ„ΠΎΡ€ΠΌΡ‹, Ссли ΠΎΠ½ΠΈ плоскиС, ΠΏΠ°Ρ€Π°Π»Π»Π΅Π»ΡŒΠ½Ρ‹Π΅ ΠΈ Π²Ρ‹ Π·Π½Π°Π΅Ρ‚Π΅ ΠΏΠ»ΠΎΡ‰Π°Π΄ΡŒ
Ρ‚Π°Ρ€Π΅Π»ΠΊΠΈ ΠΈΠ»ΠΈ Ρ‡Ρ‚ΠΎ-Ρ‚ΠΎ Π΅Ρ‰Π΅, Ρ‡Ρ‚ΠΎ Π½ΡƒΠΆΠ½ΠΎ, Ρ‡Ρ‚ΠΎΠ±Ρ‹ Π½Π°ΠΉΡ‚ΠΈ этот Ρ€Π°ΠΉΠΎΠ½.

ΠšΠΎΠ½Π΄Π΅Π½ΡΠ°Ρ‚ΠΎΡ€ с ΠΏΠ°Ρ€Π°Π»Π»Π΅Π»ΡŒΠ½Ρ‹ΠΌΠΈ пластинами — пластины ΠΏΡ€ΡΠΌΠΎΡƒΠ³ΠΎΠ»ΡŒΠ½ΠΎΠΉ Ρ„ΠΎΡ€ΠΌΡ‹.
ΠšΠΎΠ½Π΄Π΅Π½ΡΠ°Ρ‚ΠΎΡ€ с ΠΏΠ°Ρ€Π°Π»Π»Π΅Π»ΡŒΠ½Ρ‹ΠΌΠΈ пластинами — ΠΊΡ€ΡƒΠ³Π»Ρ‹Π΅ пластины.

Π€ΠΎΡ€ΠΌΡƒΠ»Π° Смкости кондСнсатора с ΠΏΠ°Ρ€Π°Π»Π»Π΅Π»ΡŒΠ½Ρ‹ΠΌΠΈ пластинами:

Π“Π΄Π΅:

  • Ξ΅ r = ΠΎΡ‚Π½ΠΎΡΠΈΡ‚Π΅Π»ΡŒΠ½Π°Ρ диэлСктричСская ΠΏΡ€ΠΎΠ½ΠΈΡ†Π°Π΅ΠΌΠΎΡΡ‚ΡŒ диэлСктрика
    (Ρ€Π΅ΠΆΠ΅ К, диэлСктричСская ΠΏΡ€ΠΎΠ½ΠΈΡ†Π°Π΅ΠΌΠΎΡΡ‚ΡŒ)
  • Ξ΅ 0 = 8.854×10 -12 Π€ / ΠΌ (Ρ„Π°Ρ€Π°Π΄ / ΠΌΠ΅Ρ‚Ρ€) =
    диэлСктричСская ΠΏΡ€ΠΎΠ½ΠΈΡ†Π°Π΅ΠΌΠΎΡΡ‚ΡŒ Π²Π°ΠΊΡƒΡƒΠΌΠ° ΠΈΠ»ΠΈ диэлСктричСская ΠΏΡ€ΠΎΠ½ΠΈΡ†Π°Π΅ΠΌΠΎΡΡ‚ΡŒ свободного пространства

На схСмах ΠΏΠΎΠΊΠ°Π·Π°Π½Ρ‹ кондСнсаторы с ΠΏΠ°Ρ€Π°Π»Π»Π΅Π»ΡŒΠ½Ρ‹ΠΌΠΈ пластинами Ρ€Π°Π·Π½ΠΎΠΉ Ρ„ΠΎΡ€ΠΌΡ‹.
пластины, ΠΎΠ΄Π½Π° ΠΏΡ€ΡΠΌΠΎΡƒΠ³ΠΎΠ»ΡŒΠ½Π°Ρ ΠΈ ΠΎΠ΄Π½Π° круглая. Π€ΠΎΡ€ΠΌΡƒΠ»Π° для расчСта
ΠΏΠ»ΠΎΡ‰Π°Π΄ΡŒ ΠΏΡ€ΡΠΌΠΎΡƒΠ³ΠΎΠ»ΡŒΠ½ΠΈΠΊΠ°:

Π° Ρ„ΠΎΡ€ΠΌΡƒΠ»Π° для вычислСния ΠΏΠ»ΠΎΡ‰Π°Π΄ΠΈ ΠΊΡ€ΡƒΠ³Π°:

Π“Π΄Π΅ Ο€ — это число ΠΏΠΈ, Ρ€Π°Π²Π½ΠΎΠ΅ 3,14159.

Π•ΠΌΠΊΠΎΡΡ‚ΡŒ цилиндричСских кондСнсаторов

ЦилиндричСский кондСнсатор состоит ΠΈΠ· Π΄Π²ΡƒΡ… Ρ†ΠΈΠ»ΠΈΠ½Π΄Ρ€ΠΎΠ², Ρ‚Π°ΠΊΠΆΠ΅ Π½Π°Π·Ρ‹Π²Π°Π΅ΠΌΡ‹Ρ…
пластины, ΠΊΠΎΡ‚ΠΎΡ€Ρ‹Π΅ ΡΠ²Π»ΡΡŽΡ‚ΡΡ элСктродами, Ρ€Π°Π·Π΄Π΅Π»Π΅Π½Ρ‹
диэлСктрик
ΠΈΠ»ΠΈ изолятор.

ЦилиндричСский кондСнсатор.

Π€ΠΎΡ€ΠΌΡƒΠ»Π° Смкости цилиндричСского кондСнсатора:

Π“Π΄Π΅:

  • Ξ΅ r = ΠΎΡ‚Π½ΠΎΡΠΈΡ‚Π΅Π»ΡŒΠ½Π°Ρ диэлСктричСская ΠΏΡ€ΠΎΠ½ΠΈΡ†Π°Π΅ΠΌΠΎΡΡ‚ΡŒ диэлСктрика
    (Ρ€Π΅ΠΆΠ΅ К, диэлСктричСская ΠΏΡ€ΠΎΠ½ΠΈΡ†Π°Π΅ΠΌΠΎΡΡ‚ΡŒ)
  • Ξ΅ 0 = 8,854×10 -12 Π€ / ΠΌ (Ρ„Π°Ρ€Π°Π΄ / ΠΌΠ΅Ρ‚Ρ€) =
    диэлСктричСская ΠΏΡ€ΠΎΠ½ΠΈΡ†Π°Π΅ΠΌΠΎΡΡ‚ΡŒ Π²Π°ΠΊΡƒΡƒΠΌΠ° ΠΈΠ»ΠΈ диэлСктричСская ΠΏΡ€ΠΎΠ½ΠΈΡ†Π°Π΅ΠΌΠΎΡΡ‚ΡŒ свободного пространства

Π’ΠΈΠ΄Π΅ΠΎ — Как ΡΠ΄Π΅Π»Π°Ρ‚ΡŒ кондСнсаторы — НизкоС напряТСниС

Π’ этом Π²ΠΈΠ΄Π΅ΠΎ Π½Π΅ Ρ‚ΠΎΠ»ΡŒΠΊΠΎ ΠΏΠΎΠΊΠ°Π·Π°Π½ΠΎ, ΠΊΠ°ΠΊ ΡΠ΄Π΅Π»Π°Ρ‚ΡŒ кондСнсаторы, Π½ΠΎ ΠΈ
Ρ„ΠΎΡ€ΠΌΡƒΠ»Π° Смкости Π² Π±ΠΎΠ»Π΅Π΅ Π΄ΠΈΠ½Π°ΠΌΠΈΡ‡Π½ΠΎΠΌ Ρ„ΠΎΡ€ΠΌΠ°Ρ‚Π΅, Ρ‡Π΅ΠΌ ΡƒΠΊΠ°Π·Π°Π½ΠΎ Π²Ρ‹ΡˆΠ΅.ПослС всСго,
Ссли Π²Ρ‹ Π΄Π΅Π»Π°Π΅Ρ‚Π΅ кондСнсатор, Π²Π°ΠΌ сначала Π½ΡƒΠΆΠ½ΠΎ Π·Π½Π°Ρ‚ΡŒ, ΠΊΠ°ΠΊ
ΡΠΏΡ€ΠΎΠ΅ΠΊΡ‚ΠΈΡ€ΠΎΠ²Π°Ρ‚ΡŒ кондСнсатор.

Π’ΠΈΠ΄Π΅ΠΎ — Как ΡΠ΄Π΅Π»Π°Ρ‚ΡŒ кондСнсаторы — ВысокоС напряТСниС

Π’ этом Π²ΠΈΠ΄Π΅ΠΎ ΠΏΠΎΠΊΠ°Π·Π°Π½ΠΎ, ΠΊΠ°ΠΊ Ρ€Π°Π·Ρ€Π°Π±ΠΎΡ‚Π°Ρ‚ΡŒ кондСнсаторы для высокого напряТСния, объясняя,
измСрСния ΠΈ построСния для напряТСния пробоя / диэлСктричСской прочности, Ρ‡Ρ‚ΠΎΠ±Ρ‹
Ρ‡Ρ‚ΠΎ кондСнсатор ΠΌΠΎΠΆΠ΅Ρ‚ Π²Ρ‹Π΄Π΅Ρ€ΠΆΠΈΠ²Π°Ρ‚ΡŒ ΠΆΠ΅Π»Π°Π΅ΠΌΠΎΠ΅ высокоС напряТСниС.

ΠšΠ°Π»ΡŒΠΊΡƒΠ»ΡΡ‚ΠΎΡ€ ΠΏΠ°Ρ€Π°Π»Π»Π΅Π»ΡŒΠ½Ρ‹Ρ… пластинчатых кондСнсаторов

— Π•ΠΌΠΊΠΎΡΡ‚ΡŒ

Π’Π²Π΅Π΄ΠΈΡ‚Π΅ значСния Π² поля Π½ΠΈΠΆΠ΅, Ρ‡Ρ‚ΠΎΠ±Ρ‹ Ρ€Π°ΡΡΡ‡ΠΈΡ‚Π°Ρ‚ΡŒ Π΅ΠΌΠΊΠΎΡΡ‚ΡŒ ΠΌΠ΅ΠΆΠ΄Ρƒ кондСнсаторами с ΠΏΠ°Ρ€Π°Π»Π»Π΅Π»ΡŒΠ½Ρ‹ΠΌΠΈ пластинами с ΠΏΠΎΠΌΠΎΡ‰ΡŒΡŽ ΠΊΠ°Π»ΡŒΠΊΡƒΠ»ΡΡ‚ΠΎΡ€Π° Смкости.

ΠšΠ°Π»ΡŒΠΊΡƒΠ»ΡΡ‚ΠΎΡ€ кондСнсаторов — это ΠΎΠ½Π»Π°ΠΉΠ½-инструмСнт, ΠΊΠΎΡ‚ΠΎΡ€Ρ‹ΠΉ ΠΈΡΠΏΠΎΠ»ΡŒΠ·ΡƒΠ΅Ρ‚ΡΡ для расчСта Смкости, принимая Π² качСствС Π²Ρ…ΠΎΠ΄Π½Ρ‹Ρ… Π΄Π°Π½Π½Ρ‹Ρ… Π΄ΠΈΡΠ»Π΅ΠΊΡ‚Ρ€ΠΈΡ‡Π΅ΡΠΊΡƒΡŽ ΠΏΡ€ΠΎΠ½ΠΈΡ†Π°Π΅ΠΌΠΎΡΡ‚ΡŒ, расстояниС ΠΌΠ΅ΠΆΠ΄Ρƒ пластинами ΠΈ ΠΏΠ»ΠΎΡ‰Π°Π΄ΡŒ пластин.

Π§Ρ‚ΠΎ Ρ‚Π°ΠΊΠΎΠ΅ кондСнсатор?

ΠšΠΎΠ½Π΄Π΅Π½ΡΠ°Ρ‚ΠΎΡ€ — это устройство, ΠΊΠΎΡ‚ΠΎΡ€ΠΎΠ΅ ΠΌΠΎΠΆΠ΅Ρ‚ Π½Π°ΠΊΠ°ΠΏΠ»ΠΈΠ²Π°Ρ‚ΡŒ элСктричСскиС заряды. НаиболСС распространСнный Ρ‚ΠΈΠΏ кондСнсатора, ΠΏΠΎΠΊΠ°Π·Π°Π½Π½Ρ‹ΠΉ Π½ΠΈΠΆΠ΅, — это ΠΏΠ°Ρ€Π°Π»Π»Π΅Π»ΡŒΠ½Ρ‹ΠΉ кондСнсатор. Π’ этом Ρ‚ΠΈΠΏΠ΅ кондСнсатора пластины кондСнсатора Ρ€Π°Π·Π΄Π΅Π»Π΅Π½Ρ‹ изолятором.

Π•ΠΌΠΊΠΎΡΡ‚ΡŒ

Π•ΠΌΠΊΠΎΡΡ‚ΡŒ — это количСство элСктричСского заряда, ΠΊΠΎΡ‚ΠΎΡ€ΠΎΠ΅ ΠΌΠΎΠΆΠ΅Ρ‚ Π½Π°ΠΊΠ°ΠΏΠ»ΠΈΠ²Π°Ρ‚ΡŒ кондСнсатор.

Π€ΠΎΡ€ΠΌΡƒΠ»Π° Смкости

Π€ΠΎΡ€ΠΌΡƒΠ»Π°, ΠΈΡΠΏΠΎΠ»ΡŒΠ·ΡƒΠ΅ΠΌΠ°Ρ для опрСдСлСния Смкости:

C = Ξ΅ A / S

Π’ этом ΡƒΡ€Π°Π²Π½Π΅Π½ΠΈΠΈ:

A = ΠΏΠ»ΠΎΡ‰Π°Π΄ΡŒ пСрСкрытия пластин

S = расстояниС ΠΌΠ΅ΠΆΠ΄Ρƒ пластинами

Ξ΅ = диэлСктричСская ΠΏΡ€ΠΎΠ½ΠΈΡ†Π°Π΅ΠΌΠΎΡΡ‚ΡŒ ΠΏΡ€ΠΎΠ²ΠΎΠ΄Π½ΠΈΠΊΠ°

Π—Π½Π°Ρ‡Π΅Π½ΠΈΠ΅ диэлСктричСской проницаСмости Π²Π°Ρ€ΡŒΠΈΡ€ΡƒΠ΅Ρ‚ΡΡ ΠΎΡ‚ ΠΏΡ€ΠΎΠ²ΠΎΠ΄Π½ΠΈΠΊΠ° ΠΊ ΠΏΡ€ΠΎΠ²ΠΎΠ΄Π½ΠΈΠΊΡƒ. Π§Ρ‚ΠΎΠ±Ρ‹ Π½Π°ΠΉΡ‚ΠΈ Π΅ΠΌΠΊΠΎΡΡ‚ΡŒ с ΡƒΡ‡Π΅Ρ‚ΠΎΠΌ заряда ΠΈ разности ΠΏΠΎΡ‚Π΅Π½Ρ†ΠΈΠ°Π»ΠΎΠ², Π²ΠΎΡΠΏΠΎΠ»ΡŒΠ·ΡƒΠΉΡ‚Π΅ΡΡŒ Π΄Ρ€ΡƒΠ³ΠΈΠΌ нашим ΠΊΠ°Π»ΡŒΠΊΡƒΠ»ΡΡ‚ΠΎΡ€ΠΎΠΌ Смкости.

Как Ρ€Π°ΡΡΡ‡ΠΈΡ‚Π°Ρ‚ΡŒ Π΅ΠΌΠΊΠΎΡΡ‚ΡŒ?

ΠŸΡ€ΠΈΠΌΠ΅Ρ€:

РассчитайтС Π΅ΠΌΠΊΠΎΡΡ‚ΡŒ кондСнсатора, Ρƒ ΠΊΠΎΡ‚ΠΎΡ€ΠΎΠ³ΠΎ 0,03 ΠΌ 2 ΠΏΠ»ΠΎΡ‰Π°Π΄ΡŒ пластин, Ρ€Π°Π·Π΄Π΅Π»Π΅Π½Π½Ρ‹Ρ… свободным пространством, ΠΈ расстояниС ΠΌΠ΅ΠΆΠ΄Ρƒ пластинами 0,7 ΠΌ.

РСшСниС :

Π¨Π°Π³ 1: ΠžΠΏΡ€Π΅Π΄Π΅Π»ΠΈΡ‚Π΅ значСния.

A = 0,03 ΠΌ 2

S = 0,7 ΠΌ

ΠŸΡ€ΠΎΠ½ΠΈΡ†Π°Π΅ΠΌΠΎΡΡ‚ΡŒ свободного пространства = 8.85418782 Γ— 10 -12 ΠΌ -3 ΠΊΠ³ -1 с 4 A 2

Π¨Π°Π³ 2: Π’Π²Π΅Π΄ΠΈΡ‚Π΅ значСния Π² Ρ„ΠΎΡ€ΠΌΡƒΠ»Ρƒ Смкости.

C = Ξ΅ A / S

= 8,85418782 Γ— 10 -12 Γ— 0,03 / 0,7

= 0,38 F

4.1 ΠšΠΎΠ½Π΄Π΅Π½ΡΠ°Ρ‚ΠΎΡ€Ρ‹ ΠΈ Π΅ΠΌΠΊΠΎΡΡ‚ΡŒ — Π’Π²Π΅Π΄Π΅Π½ΠΈΠ΅ Π² элСктричСство, ΠΌΠ°Π³Π½Π΅Ρ‚ΠΈΠ·ΠΌ ΠΈ схСмы

Π¦Π•Π›Π˜ ΠžΠ‘Π£Π§Π•ΠΠ˜Π―

К ΠΊΠΎΠ½Ρ†Ρƒ этого Ρ€Π°Π·Π΄Π΅Π»Π° Π²Ρ‹ смоТСтС:

  • ΠžΠ±ΡŠΡΡΠ½ΠΈΡ‚Π΅ понятиС кондСнсатора ΠΈ Π΅Π³ΠΎ Смкости
  • ΠžΠΏΠΈΡˆΠΈΡ‚Π΅, ΠΊΠ°ΠΊ ΠΎΡ†Π΅Π½ΠΈΡ‚ΡŒ Π΅ΠΌΠΊΠΎΡΡ‚ΡŒ систСмы ΠΏΡ€ΠΎΠ²ΠΎΠ΄ΠΎΠ²

ΠšΠΎΠ½Π΄Π΅Π½ΡΠ°Ρ‚ΠΎΡ€ — это устройство, ΠΈΡΠΏΠΎΠ»ΡŒΠ·ΡƒΠ΅ΠΌΠΎΠ΅ для хранСния элСктричСского заряда ΠΈ элСктричСской энСргии.Он состоит ΠΊΠ°ΠΊ ΠΌΠΈΠ½ΠΈΠΌΡƒΠΌ ΠΈΠ· Π΄Π²ΡƒΡ… элСктричСских ΠΏΡ€ΠΎΠ²ΠΎΠ΄Π½ΠΈΠΊΠΎΠ², Ρ€Π°Π·Π΄Π΅Π»Π΅Π½Π½Ρ‹Ρ… расстояниСм. (ΠžΠ±Ρ€Π°Ρ‚ΠΈΡ‚Π΅ Π²Π½ΠΈΠΌΠ°Π½ΠΈΠ΅, Ρ‡Ρ‚ΠΎ Ρ‚Π°ΠΊΠΈΠ΅ элСктричСскиС ΠΏΡ€ΠΎΠ²ΠΎΠ΄Π½ΠΈΠΊΠΈ ΠΈΠ½ΠΎΠ³Π΄Π° Π½Π°Π·Ρ‹Π²Π°ΡŽΡ‚ «элСктродами», Π½ΠΎ, Ρ‚ΠΎΡ‡Π½Π΅Π΅, ΠΎΠ½ΠΈ Β«ΠΎΠ±ΠΊΠ»Π°Π΄ΠΊΠΈ кондСнсатора».) ΠŸΡ€ΠΎΡΡ‚Ρ€Π°Π½ΡΡ‚Π²ΠΎ ΠΌΠ΅ΠΆΠ΄Ρƒ кондСнсаторами ΠΌΠΎΠΆΠ΅Ρ‚ Π±Ρ‹Ρ‚ΡŒ просто Π²Π°ΠΊΡƒΡƒΠΌΠΎΠΌ, ΠΈ Π² этом случаС кондСнсатор Π±ΡƒΠ΄Π΅Ρ‚ извСстСн ΠΊΠ°ΠΊ Β«Π’Π°ΠΊΡƒΡƒΠΌΠ½Ρ‹ΠΉ кондСнсатор». Однако пространство ΠΎΠ±Ρ‹Ρ‡Π½ΠΎ заполняСтся ΠΈΠ·ΠΎΠ»ΠΈΡ€ΡƒΡŽΡ‰ΠΈΠΌ ΠΌΠ°Ρ‚Π΅Ρ€ΠΈΠ°Π»ΠΎΠΌ, извСстным ΠΊΠ°ΠΊ диэлСктрик . (Π’Ρ‹ ΡƒΠ·Π½Π°Π΅Ρ‚Π΅ большС ΠΎ диэлСктриках Π² Ρ€Π°Π·Π΄Π΅Π»Π°Ρ…, посвящСнных диэлСктрикам, Π΄Π°Π»Π΅Π΅ Π² этой Π³Π»Π°Π²Π΅.) ОбъСм памяти Π² кондСнсаторС опрСдСляСтся свойством, Π½Π°Π·Ρ‹Π²Π°Π΅ΠΌΡ‹ΠΌ Π΅ΠΌΠΊΠΎΡΡ‚ΡŒΡŽ , , ΠΎ ΠΊΠΎΡ‚ΠΎΡ€ΠΎΠΌ Π²Ρ‹ ΡƒΠ·Π½Π°Π΅Ρ‚Π΅ большС Ρ‡ΡƒΡ‚ΡŒ ΠΏΠΎΠ·ΠΆΠ΅ Π² этом Ρ€Π°Π·Π΄Π΅Π»Π΅.

ΠšΠΎΠ½Π΄Π΅Π½ΡΠ°Ρ‚ΠΎΡ€Ρ‹

ΠΈΠΌΠ΅ΡŽΡ‚ Ρ€Π°Π·Π»ΠΈΡ‡Π½Ρ‹Π΅ примСнСния: ΠΎΡ‚ Ρ„ΠΈΠ»ΡŒΡ‚Ρ€Π°Ρ†ΠΈΠΈ статичСского элСктричСства, Ρ€Π°Π΄ΠΈΠΎΠΏΡ€ΠΈΠ΅ΠΌΠ° Π΄ΠΎ накоплСния энСргии Π² дСфибрилляторах сСрдца. ΠžΠ±Ρ‹Ρ‡Π½ΠΎ Π² ΠΏΡ€ΠΎΠΌΡ‹ΡˆΠ»Π΅Π½Π½Ρ‹Ρ… кондСнсаторах Π΄Π²Π΅ токопроводящиС части располоТСны Π±Π»ΠΈΠ·ΠΊΠΎ Π΄Ρ€ΡƒΠ³ ΠΊ Π΄Ρ€ΡƒΠ³Ρƒ, Π½ΠΎ Π½Π΅ ΡΠΎΠΏΡ€ΠΈΠΊΠ°ΡΠ°ΡŽΡ‚ΡΡ, ΠΊΠ°ΠΊ ΠΏΠΎΠΊΠ°Π·Π°Π½ΠΎ Π½Π° рисункС 4.1.1. Π’ Π±ΠΎΠ»ΡŒΡˆΠΈΠ½ΡΡ‚Π²Π΅ случаСв ΠΌΠ΅ΠΆΠ΄Ρƒ двумя пластинами ΠΈΡΠΏΠΎΠ»ΡŒΠ·ΡƒΠ΅Ρ‚ΡΡ диэлСктрик. Когда ΠΊΠ»Π΅ΠΌΠΌΡ‹ Π±Π°Ρ‚Π°Ρ€Π΅ΠΈ ΠΏΠΎΠ΄ΠΊΠ»ΡŽΡ‡Π΅Π½Ρ‹ ΠΊ ΠΏΠ΅Ρ€Π²ΠΎΠ½Π°Ρ‡Π°Π»ΡŒΠ½ΠΎ нСзаряТСнному кондСнсатору, ΠΏΠΎΡ‚Π΅Π½Ρ†ΠΈΠ°Π» Π±Π°Ρ‚Π°Ρ€Π΅ΠΈ ΠΏΠ΅Ρ€Π΅ΠΌΠ΅Ρ‰Π°Π΅Ρ‚ нСбольшой заряд Π²Π΅Π»ΠΈΡ‡ΠΈΠ½Ρ‹ ΠΎΡ‚ ΠΏΠΎΠ»ΠΎΠΆΠΈΡ‚Π΅Π»ΡŒΠ½ΠΎΠΉ пластины ΠΊ ΠΎΡ‚Ρ€ΠΈΡ†Π°Ρ‚Π΅Π»ΡŒΠ½ΠΎΠΉ.ΠšΠΎΠ½Π΄Π΅Π½ΡΠ°Ρ‚ΠΎΡ€ Π² Ρ†Π΅Π»ΠΎΠΌ остаСтся Π½Π΅ΠΉΡ‚Ρ€Π°Π»ΡŒΠ½Ρ‹ΠΌ, Π½ΠΎ заряТаСтся ΠΈ находится Π½Π° ΠΏΡ€ΠΎΡ‚ΠΈΠ²ΠΎΠΏΠΎΠ»ΠΎΠΆΠ½Ρ‹Ρ… пластинах.

(рисунок 4.1.1)

Рисунок 4.1.1 Оба кондСнсатора, ΠΏΠΎΠΊΠ°Π·Π°Π½Π½Ρ‹Π΅ здСсь, Π±Ρ‹Π»ΠΈ ΠΈΠ·Π½Π°Ρ‡Π°Π»ΡŒΠ½ΠΎ разряТСны ΠΏΠ΅Ρ€Π΅Π΄ ΠΏΠΎΠ΄ΠΊΠ»ΡŽΡ‡Π΅Π½ΠΈΠ΅ΠΌ ΠΊ Π±Π°Ρ‚Π°Ρ€Π΅Π΅. Π’Π΅ΠΏΠ΅Ρ€ΡŒ Ρƒ Π½ΠΈΡ… Π΅ΡΡ‚ΡŒ заряды ΠΈ (соотвСтствСнно) Π½Π° своих Ρ‚Π°Ρ€Π΅Π»ΠΊΠ°Ρ…. (a) ΠšΠΎΠ½Π΄Π΅Π½ΡΠ°Ρ‚ΠΎΡ€ с ΠΏΠ°Ρ€Π°Π»Π»Π΅Π»ΡŒΠ½Ρ‹ΠΌΠΈ пластинами состоит ΠΈΠ· Π΄Π²ΡƒΡ… пластин ΠΏΡ€ΠΎΡ‚ΠΈΠ²ΠΎΠΏΠΎΠ»ΠΎΠΆΠ½ΠΎΠ³ΠΎ заряда с ΠΏΠ»ΠΎΡ‰Π°Π΄ΡŒΡŽ A, Ρ€Π°Π·Π΄Π΅Π»Π΅Π½Π½ΠΎΠΉ расстояниСм d. (b) ΠšΠ°Ρ‚Π°Π½Ρ‹ΠΉ кондСнсатор ΠΈΠΌΠ΅Π΅Ρ‚ диэлСктричСский ΠΌΠ°Ρ‚Π΅Ρ€ΠΈΠ°Π» ΠΌΠ΅ΠΆΠ΄Ρƒ двумя проводящими листами (пластинами).

БистСма, состоящая ΠΈΠ· Π΄Π²ΡƒΡ… ΠΈΠ΄Π΅Π½Ρ‚ΠΈΡ‡Π½Ρ‹Ρ… ΠΏΠ°Ρ€Π°Π»Π»Π΅Π»ΡŒΠ½ΠΎ проводящих пластин, Ρ€Π°Π·Π΄Π΅Π»Π΅Π½Π½Ρ‹Ρ… расстояниСм, называСтся кондСнсатором с ΠΏΠ°Ρ€Π°Π»Π»Π΅Π»ΡŒΠ½Ρ‹ΠΌΠΈ пластинами (рисунок 4.1.2). Π’Π΅Π»ΠΈΡ‡ΠΈΠ½Π° элСктричСского поля Π² пространствС ΠΌΠ΅ΠΆΠ΄Ρƒ ΠΏΠ°Ρ€Π°Π»Π»Π΅Π»ΡŒΠ½Ρ‹ΠΌΠΈ пластинами Ρ€Π°Π²Π½Π°, Π³Π΄Π΅ ΠΎΠ±ΠΎΠ·Π½Π°Ρ‡Π°Π΅Ρ‚ ΠΏΠΎΠ²Π΅Ρ€Ρ…Π½ΠΎΡΡ‚Π½ΡƒΡŽ ΠΏΠ»ΠΎΡ‚Π½ΠΎΡΡ‚ΡŒ заряда Π½Π° ΠΎΠ΄Π½ΠΎΠΉ пластинС (Π½Π°ΠΏΠΎΠΌΠ½ΠΈΠΌ, Ρ‡Ρ‚ΠΎ это заряд Π½Π° ΠΏΠ»ΠΎΡ‰Π°Π΄ΡŒ повСрхности). Π’Π°ΠΊΠΈΠΌ ΠΎΠ±Ρ€Π°Π·ΠΎΠΌ, Π²Π΅Π»ΠΈΡ‡ΠΈΠ½Π° поля прямо ΠΏΡ€ΠΎΠΏΠΎΡ€Ρ†ΠΈΠΎΠ½Π°Π»ΡŒΠ½Π°.

(рисунок 4.1.2)

Рисунок 4.1.2 Π Π°Π·Π΄Π΅Π»Π΅Π½ΠΈΠ΅ зарядов Π² кондСнсаторС ΠΏΠΎΠΊΠ°Π·Ρ‹Π²Π°Π΅Ρ‚, Ρ‡Ρ‚ΠΎ заряды ΠΎΡΡ‚Π°ΡŽΡ‚ΡΡ Π½Π° повСрхности пластин кондСнсатора.Π›ΠΈΠ½ΠΈΠΈ элСктричСского поля Π² кондСнсаторС с ΠΏΠ°Ρ€Π°Π»Π»Π΅Π»ΡŒΠ½Ρ‹ΠΌΠΈ пластинами Π½Π°Ρ‡ΠΈΠ½Π°ΡŽΡ‚ΡΡ с ΠΏΠΎΠ»ΠΎΠΆΠΈΡ‚Π΅Π»ΡŒΠ½Ρ‹Ρ… зарядов ΠΈ Π·Π°ΠΊΠ°Π½Ρ‡ΠΈΠ²Π°ΡŽΡ‚ΡΡ ΠΎΡ‚Ρ€ΠΈΡ†Π°Ρ‚Π΅Π»ΡŒΠ½Ρ‹ΠΌΠΈ зарядами. Π’Π΅Π»ΠΈΡ‡ΠΈΠ½Π° элСктричСского поля Π² пространствС ΠΌΠ΅ΠΆΠ΄Ρƒ пластинами прямо ΠΏΡ€ΠΎΠΏΠΎΡ€Ρ†ΠΈΠΎΠ½Π°Π»ΡŒΠ½Π° количСству заряда Π½Π° кондСнсаторС.

ΠšΠΎΠ½Π΄Π΅Π½ΡΠ°Ρ‚ΠΎΡ€Ρ‹ с Ρ€Π°Π·Π½Ρ‹ΠΌΠΈ физичСскими характСристиками (Ρ‚Π°ΠΊΠΈΠΌΠΈ ΠΊΠ°ΠΊ Ρ„ΠΎΡ€ΠΌΠ° ΠΈ Ρ€Π°Π·ΠΌΠ΅Ρ€ пластин) Π½Π°ΠΊΠ°ΠΏΠ»ΠΈΠ²Π°ΡŽΡ‚ Ρ€Π°Π·Π½ΠΎΠ΅ количСство заряда для ΠΎΠ΄Π½ΠΎΠ³ΠΎ ΠΈ Ρ‚ΠΎΠ³ΠΎ ΠΆΠ΅ ΠΏΡ€ΠΈΠ»ΠΎΠΆΠ΅Π½Π½ΠΎΠ³ΠΎ напряТСния Π½Π° своих пластинах. Π•ΠΌΠΊΠΎΡΡ‚ΡŒ кондСнсатора опрСдСляСтся ΠΊΠ°ΠΊ ΠΎΡ‚Π½ΠΎΡˆΠ΅Π½ΠΈΠ΅ максимального заряда, ΠΊΠΎΡ‚ΠΎΡ€Ρ‹ΠΉ ΠΌΠΎΠΆΠ΅Ρ‚ Ρ…Ρ€Π°Π½ΠΈΡ‚ΡŒΡΡ Π² кондСнсаторС, ΠΊ ΠΏΡ€ΠΈΠ»ΠΎΠΆΠ΅Π½Π½ΠΎΠΌΡƒ Π½Π°ΠΏΡ€ΡΠΆΠ΅Π½ΠΈΡŽ Π½Π° Π΅Π³ΠΎ пластинах.Π”Ρ€ΡƒΠ³ΠΈΠΌΠΈ словами, Π΅ΠΌΠΊΠΎΡΡ‚ΡŒ — это наибольшая Π²Π΅Π»ΠΈΡ‡ΠΈΠ½Π° заряда Π½Π° Π²ΠΎΠ»ΡŒΡ‚, которая ΠΌΠΎΠΆΠ΅Ρ‚ Ρ…Ρ€Π°Π½ΠΈΡ‚ΡŒΡΡ Π½Π° устройствС:

(4.1.1)

Π•Π΄ΠΈΠ½ΠΈΡ†Π° измСрСния Смкости Π² систСмС БИ — Ρ„Π°Ρ€Π°Π΄ (), названная Π² Ρ‡Π΅ΡΡ‚ΡŒ Майкла ЀарадСя (1791–1867). ΠŸΠΎΡΠΊΠΎΠ»ΡŒΠΊΡƒ Π΅ΠΌΠΊΠΎΡΡ‚ΡŒ — это заряд Π½Π° Π΅Π΄ΠΈΠ½ΠΈΡ†Ρƒ напряТСния, ΠΎΠ΄ΠΈΠ½ Ρ„Π°Ρ€Π°Π΄ Ρ€Π°Π²Π΅Π½ ΠΎΠ΄Π½ΠΎΠΌΡƒ ΠΊΡƒΠ»ΠΎΠ½Ρƒ Π½Π° ΠΎΠ΄ΠΈΠ½ Π²ΠΎΠ»ΡŒΡ‚, ΠΈΠ»ΠΈ

.

По ΠΎΠΏΡ€Π΅Π΄Π΅Π»Π΅Π½ΠΈΡŽ, кондСнсатор способСн Π½Π°ΠΊΠ°ΠΏΠ»ΠΈΠ²Π°Ρ‚ΡŒ заряд (ΠΎΡ‡Π΅Π½ΡŒ большой заряд), ΠΊΠΎΠ³Π΄Π° Ρ€Π°Π·Π½ΠΎΡΡ‚ΡŒ ΠΏΠΎΡ‚Π΅Π½Ρ†ΠΈΠ°Π»ΠΎΠ² ΠΌΠ΅ΠΆΠ΄Ρƒ Π΅Π³ΠΎ пластинами Ρ€Π°Π²Π½Π° всСго.Π‘Π»Π΅Π΄ΠΎΠ²Π°Ρ‚Π΅Π»ΡŒΠ½ΠΎ, ΠΎΠ΄Π½Π° Ρ„Π°Ρ€Π°Π΄Π° — это ΠΎΡ‡Π΅Π½ΡŒ большая Π΅ΠΌΠΊΠΎΡΡ‚ΡŒ. Π’ΠΈΠΏΠΈΡ‡Π½Ρ‹Π΅ значСния Смкости находятся Π² Π΄ΠΈΠ°ΠΏΠ°Π·ΠΎΠ½Π΅ ΠΎΡ‚ ΠΏΠΈΠΊΠΎΡ„Π°Ρ€Π°Π΄ () Π΄ΠΎ ΠΌΠΈΠ»Π»ΠΈΡ„Π°Ρ€Π°Π΄ (), Π²ΠΊΠ»ΡŽΡ‡Π°Ρ ΠΌΠΈΠΊΡ€ΠΎΡ„Π°Ρ€Π°Π΄Ρ‹ (). ΠšΠΎΠ½Π΄Π΅Π½ΡΠ°Ρ‚ΠΎΡ€Ρ‹ ΠΌΠΎΠ³ΡƒΡ‚ ΠΈΠ·Π³ΠΎΡ‚Π°Π²Π»ΠΈΠ²Π°Ρ‚ΡŒΡΡ Ρ€Π°Π·Π»ΠΈΡ‡Π½Ρ‹Ρ… Ρ„ΠΎΡ€ΠΌ ΠΈ Ρ€Π°Π·ΠΌΠ΅Ρ€ΠΎΠ² (рисунок 4.1.3).

(рисунок 4.1.3)

Рисунок 4.1.3 Π­Ρ‚ΠΎ Π½Π΅ΠΊΠΎΡ‚ΠΎΡ€Ρ‹Π΅ Ρ‚ΠΈΠΏΠΈΡ‡Π½Ρ‹Π΅ кондСнсаторы, ΠΈΡΠΏΠΎΠ»ΡŒΠ·ΡƒΠ΅ΠΌΡ‹Π΅ Π² элСктронных устройствах. Π Π°Π·ΠΌΠ΅Ρ€ кондСнсатора Π½Π΅ ΠΎΠ±ΡΠ·Π°Ρ‚Π΅Π»ΡŒΠ½ΠΎ зависит ΠΎΡ‚ Π΅Π³ΠΎ Смкости.

РасчСт Смкости

ΠœΡ‹ ΠΌΠΎΠΆΠ΅ΠΌ Ρ€Π°ΡΡΡ‡ΠΈΡ‚Π°Ρ‚ΡŒ Π΅ΠΌΠΊΠΎΡΡ‚ΡŒ ΠΏΠ°Ρ€Ρ‹ ΠΏΡ€ΠΎΠ²ΠΎΠ΄ΠΎΠ² с ΠΏΠΎΠΌΠΎΡ‰ΡŒΡŽ ΡΠ»Π΅Π΄ΡƒΡŽΡ‰Π΅Π³ΠΎ стандартного ΠΏΠΎΠ΄Ρ…ΠΎΠ΄Π°.


БтратСгия Ρ€Π΅ΡˆΠ΅Π½ΠΈΡ ΠΏΡ€ΠΎΠ±Π»Π΅ΠΌ: расчСт Смкости

Π§Ρ‚ΠΎΠ±Ρ‹ ΠΏΠΎΠΊΠ°Π·Π°Ρ‚ΡŒ, ΠΊΠ°ΠΊ Ρ€Π°Π±ΠΎΡ‚Π°Π΅Ρ‚ эта ΠΏΡ€ΠΎΡ†Π΅Π΄ΡƒΡ€Π°, ΠΌΡ‹ Ρ‚Π΅ΠΏΠ΅Ρ€ΡŒ вычисляСм Смкости ΠΏΠ°Ρ€Π°Π»Π»Π΅Π»ΡŒΠ½Ρ‹Ρ… пластин, сфСричСских ΠΈ цилиндричСских кондСнсаторов. Π’ΠΎ всСх случаях ΠΌΡ‹ ΠΏΡ€Π΅Π΄ΠΏΠΎΠ»Π°Π³Π°Π΅ΠΌ Π²Π°ΠΊΡƒΡƒΠΌΠ½Ρ‹Π΅ кондСнсаторы (пустыС кондСнсаторы) Π±Π΅Π· диэлСктричСского вСщСства Π² пространствС ΠΌΠ΅ΠΆΠ΄Ρƒ ΠΏΡ€ΠΎΠ²ΠΎΠ΄Π½ΠΈΠΊΠ°ΠΌΠΈ.

ΠšΠΎΠ½Π΄Π΅Π½ΡΠ°Ρ‚ΠΎΡ€ с ΠΏΠ°Ρ€Π°Π»Π»Π΅Π»ΡŒΠ½Ρ‹ΠΌΠΈ пластинами

ΠšΠΎΠ½Π΄Π΅Π½ΡΠ°Ρ‚ΠΎΡ€ с ΠΏΠ°Ρ€Π°Π»Π»Π΅Π»ΡŒΠ½Ρ‹ΠΌΠΈ пластинами (рисунок 4.1.4) ΠΈΠΌΠ΅Π΅Ρ‚ Π΄Π²Π΅ ΠΈΠ΄Π΅Π½Ρ‚ΠΈΡ‡Π½Ρ‹Π΅ проводящиС пластины, каТдая ΠΈΠ· ΠΊΠΎΡ‚ΠΎΡ€Ρ‹Ρ… ΠΈΠΌΠ΅Π΅Ρ‚ ΠΏΠ»ΠΎΡ‰Π°Π΄ΡŒ повСрхности, Ρ€Π°Π·Π΄Π΅Π»Π΅Π½Π½ΡƒΡŽ расстояниСм.Когда Π½Π° кондСнсатор подаСтся напряТСниС, ΠΎΠ½ Π½Π°ΠΊΠ°ΠΏΠ»ΠΈΠ²Π°Π΅Ρ‚ заряд, ΠΊΠ°ΠΊ ΠΏΠΎΠΊΠ°Π·Π°Π½ΠΎ Π½Π° рисункС. ΠœΡ‹ ΠΌΠΎΠΆΠ΅ΠΌ ΡƒΠ²ΠΈΠ΄Π΅Ρ‚ΡŒ, ΠΊΠ°ΠΊ Π΅Π³ΠΎ Π΅ΠΌΠΊΠΎΡΡ‚ΡŒ ΠΌΠΎΠΆΠ΅Ρ‚ Π·Π°Π²ΠΈΡΠ΅Ρ‚ΡŒ ΠΎΡ‚ ΠΈ , рассматривая характСристики кулоновской силы. ΠœΡ‹ Π·Π½Π°Π΅ΠΌ, Ρ‡Ρ‚ΠΎ сила ΠΌΠ΅ΠΆΠ΄Ρƒ зарядами увСличиваСтся с ΡƒΠ²Π΅Π»ΠΈΡ‡Π΅Π½ΠΈΠ΅ΠΌ заряда ΠΈ ΡƒΠΌΠ΅Π½ΡŒΡˆΠ°Π΅Ρ‚ΡΡ с расстояниСм ΠΌΠ΅ΠΆΠ΄Ρƒ Π½ΠΈΠΌΠΈ. Π‘Π»Π΅Π΄ΡƒΠ΅Ρ‚ ΠΎΠΆΠΈΠ΄Π°Ρ‚ΡŒ, Ρ‡Ρ‚ΠΎ Ρ‡Π΅ΠΌ большС пластины, Ρ‚Π΅ΠΌ большС заряда ΠΎΠ½ΠΈ ΠΌΠΎΠ³ΡƒΡ‚ Ρ…Ρ€Π°Π½ΠΈΡ‚ΡŒ. Π’Π°ΠΊΠΈΠΌ ΠΎΠ±Ρ€Π°Π·ΠΎΠΌ, Π΄ΠΎΠ»ΠΆΠ½ΠΎ Π±Ρ‹Ρ‚ΡŒ большС для большСго значСния. Π’ΠΎΡ‡Π½ΠΎ Ρ‚Π°ΠΊ ΠΆΠ΅, Ρ‡Π΅ΠΌ Π±Π»ΠΈΠΆΠ΅ пластины Π΄Ρ€ΡƒΠ³ ΠΊ Π΄Ρ€ΡƒΠ³Ρƒ, Ρ‚Π΅ΠΌ сильнСС Π½Π° Π½ΠΈΡ… притяТСниС ΠΏΡ€ΠΎΡ‚ΠΈΠ²ΠΎΠΏΠΎΠ»ΠΎΠΆΠ½Ρ‹Ρ… зарядов.Π‘Π»Π΅Π΄ΠΎΠ²Π°Ρ‚Π΅Π»ΡŒΠ½ΠΎ, Π΄ΠΎΠ»ΠΆΠ½ΠΎ Π±Ρ‹Ρ‚ΡŒ большС Π·Π° мСньшСС.

(рисунок 4.1.4)

Рис. 4.1.4 Π’ кондСнсаторС с ΠΏΠ°Ρ€Π°Π»Π»Π΅Π»ΡŒΠ½Ρ‹ΠΌΠΈ пластинами, ΠΎΠ±ΠΊΠ»Π°Π΄ΠΊΠΈ ΠΊΠΎΡ‚ΠΎΡ€Ρ‹Ρ… разнСсСны Π½Π° расстояниС, каТдая пластина ΠΈΠΌΠ΅Π΅Ρ‚ ΠΎΠ΄ΠΈΠ½Π°ΠΊΠΎΠ²ΡƒΡŽ ΠΏΠ»ΠΎΡ‰Π°Π΄ΡŒ повСрхности.

ΠžΠΏΡ€Π΅Π΄Π΅Π»ΠΈΠΌ ΠΏΠΎΠ²Π΅Ρ€Ρ…Π½ΠΎΡΡ‚Π½ΡƒΡŽ ΠΏΠ»ΠΎΡ‚Π½ΠΎΡΡ‚ΡŒ заряда Π½Π° пластинах ΠΊΠ°ΠΊ

Из ΠΏΡ€Π΅Π΄Ρ‹Π΄ΡƒΡ‰ΠΈΡ… Π³Π»Π°Π² ΠΌΡ‹ Π·Π½Π°Π΅ΠΌ, Ρ‡Ρ‚ΠΎ ΠΊΠΎΠ³Π΄Π° ΠΎΠ½ΠΎ ΠΌΠ°Π»ΠΎ, элСктричСскоС ΠΏΠΎΠ»Π΅ ΠΌΠ΅ΠΆΠ΄Ρƒ пластинами довольно ΠΎΠ΄Π½ΠΎΡ€ΠΎΠ΄Π½ΠΎ (Π±Π΅Π· ΡƒΡ‡Π΅Ρ‚Π° ΠΊΡ€Π°Π΅Π²Ρ‹Ρ… эффСктов) ΠΈ Ρ‡Ρ‚ΠΎ Π΅Π³ΠΎ Π²Π΅Π»ΠΈΡ‡ΠΈΠ½Π° опрСдСляСтся ΠΊΠ°ΠΊ

.

Π³Π΄Π΅ константа — диэлСктричСская ΠΏΡ€ΠΎΠ½ΠΈΡ†Π°Π΅ΠΌΠΎΡΡ‚ΡŒ свободного пространства,.Π•Π΄ΠΈΠ½ΠΈΡ†Π° БИ эквивалСнтна. ΠŸΠΎΡΠΊΠΎΠ»ΡŒΠΊΡƒ элСктричСскоС ΠΏΠΎΠ»Π΅ ΠΌΠ΅ΠΆΠ΄Ρƒ пластинами ΠΎΠ΄Π½ΠΎΡ€ΠΎΠ΄Π½ΠΎ, Ρ€Π°Π·Π½ΠΎΡΡ‚ΡŒ ΠΏΠΎΡ‚Π΅Π½Ρ†ΠΈΠ°Π»ΠΎΠ² ΠΌΠ΅ΠΆΠ΄Ρƒ пластинами составляСт

.

Π‘Π»Π΅Π΄ΠΎΠ²Π°Ρ‚Π΅Π»ΡŒΠ½ΠΎ, ΡƒΡ€Π°Π²Π½Π΅Π½ΠΈΠ΅ 4.1.3 Π΄Π°Π΅Ρ‚ Π΅ΠΌΠΊΠΎΡΡ‚ΡŒ кондСнсатора с ΠΏΠ°Ρ€Π°Π»Π»Π΅Π»ΡŒΠ½Ρ‹ΠΌΠΈ пластинами ΠΊΠ°ΠΊ

(4.1.3)

ΠžΠ±Ρ€Π°Ρ‚ΠΈΡ‚Π΅ Π²Π½ΠΈΠΌΠ°Π½ΠΈΠ΅ Π½Π° это ΡƒΡ€Π°Π²Π½Π΅Π½ΠΈΠ΅, Ρ‡Ρ‚ΠΎ Π΅ΠΌΠΊΠΎΡΡ‚ΡŒ являСтся Ρ„ΡƒΠ½ΠΊΡ†ΠΈΠ΅ΠΉ Ρ‚ΠΎΠ»ΡŒΠΊΠΎ Π³Π΅ΠΎΠΌΠ΅Ρ‚Ρ€ΠΈΠΈ ΠΈ Ρ‚ΠΎΠ³ΠΎ, ΠΊΠ°ΠΊΠΎΠΉ ΠΌΠ°Ρ‚Π΅Ρ€ΠΈΠ°Π» заполняСт пространство ΠΌΠ΅ΠΆΠ΄Ρƒ пластинами (Π² Π΄Π°Π½Π½ΠΎΠΌ случаС Π²Π°ΠΊΡƒΡƒΠΌ) этого кондСнсатора. ЀактичСски, это Π²Π΅Ρ€Π½ΠΎ Π½Π΅ Ρ‚ΠΎΠ»ΡŒΠΊΠΎ для кондСнсатора с ΠΏΠ°Ρ€Π°Π»Π»Π΅Π»ΡŒΠ½Ρ‹ΠΌΠΈ пластинами, Π½ΠΎ ΠΈ для всСх кондСнсаторов: Π΅ΠΌΠΊΠΎΡΡ‚ΡŒ Π½Π΅ зависит ΠΎΡ‚ ΠΈΠ»ΠΈ.Если заряд измСняСтся, соотвСтствСнно измСняСтся ΠΈ ΠΏΠΎΡ‚Π΅Π½Ρ†ΠΈΠ°Π», Ρ‚Π°ΠΊ Ρ‡Ρ‚ΠΎ ΠΎΠ½ остаСтся постоянным.

ΠŸΠ Π˜ΠœΠ•Π  4.1.1


Π•ΠΌΠΊΠΎΡΡ‚ΡŒ ΠΈ заряд Π² кондСнсаторС с ΠΏΠ°Ρ€Π°Π»Π»Π΅Π»ΡŒΠ½Ρ‹ΠΌΠΈ пластинами

(a) Какова Π΅ΠΌΠΊΠΎΡΡ‚ΡŒ пустого кондСнсатора с ΠΏΠ°Ρ€Π°Π»Π»Π΅Π»ΡŒΠ½Ρ‹ΠΌΠΈ пластинами с мСталличСскими пластинами, каТдая ΠΈΠ· ΠΊΠΎΡ‚ΠΎΡ€Ρ‹Ρ… ΠΈΠΌΠ΅Π΅Ρ‚ ΠΏΠ»ΠΎΡ‰Π°Π΄ΡŒ, Ρ€Π°Π·Π΄Π΅Π»Π΅Π½Π½ΡƒΡŽ Π½Π°? (Π±) Бколько заряда хранится Π² этом кондСнсаторС, Ссли ΠΊ Π½Π΅ΠΌΡƒ ΠΏΡ€ΠΈΠ»ΠΎΠΆΠ΅Π½ΠΎ напряТСниС?

БтратСгия

ΠžΠΏΡ€Π΅Π΄Π΅Π»Π΅Π½ΠΈΠ΅ Смкости — это прямоС ΠΏΡ€ΠΈΠ»ΠΎΠΆΠ΅Π½ΠΈΠ΅ уравнСния 4.1.3. Как Ρ‚ΠΎΠ»ΡŒΠΊΠΎ ΠΌΡ‹ Π½Π°ΠΉΠ΄Π΅ΠΌ, ΠΌΡ‹ смоТСм Π½Π°ΠΉΡ‚ΠΈ Π½Π°ΠΊΠΎΠΏΠ»Π΅Π½Π½Ρ‹ΠΉ заряд, ΠΈΡΠΏΠΎΠ»ΡŒΠ·ΡƒΡ ΡƒΡ€Π°Π²Π½Π΅Π½ΠΈΠ΅ 4.1.1.

РСшСниС

Π°. Π’Π²ΠΎΠ΄ Π΄Π°Π½Π½Ρ‹Ρ… Π·Π½Π°Ρ‡Π΅Π½ΠΈΠΉ Π² ΡƒΡ€Π°Π²Π½Π΅Π½ΠΈΠ΅ 4.1.3 Π΄Π°Π΅Ρ‚

Π­Ρ‚ΠΎ нСбольшоС Π·Π½Π°Ρ‡Π΅Π½ΠΈΠ΅ Смкости ΡƒΠΊΠ°Π·Ρ‹Π²Π°Π΅Ρ‚ Π½Π° Ρ‚ΠΎ, насколько слоТно ΡΠ΄Π΅Π»Π°Ρ‚ΡŒ устройство с большой Π΅ΠΌΠΊΠΎΡΡ‚ΡŒΡŽ.

Π³. ΠžΠ±Ρ€Π°Ρ‰Π΅Π½ΠΈΠ΅ уравнСния 4.1.1 ΠΈ Π²Π²ΠΎΠ΄ извСстных Π·Π½Π°Ρ‡Π΅Π½ΠΈΠΉ Π² это ΡƒΡ€Π°Π²Π½Π΅Π½ΠΈΠ΅ Π΄Π°Π΅Ρ‚

Π—Π½Π°Ρ‡Π΅Π½ΠΈΠ΅

Π­Ρ‚ΠΎΡ‚ заряд лишь Π½Π΅ΠΌΠ½ΠΎΠ³ΠΎ большС, Ρ‡Π΅ΠΌ Π² Ρ‚ΠΈΠΏΠΈΡ‡Π½Ρ‹Ρ… прилоТСниях статичСского элСктричСства.ΠŸΠΎΡΠΊΠΎΠ»ΡŒΠΊΡƒ Π²ΠΎΠ·Π΄ΡƒΡ… Ρ€Π°Π·Ρ€ΡƒΡˆΠ°Π΅Ρ‚ΡΡ (становится проводящим) ΠΏΡ€ΠΈ напряТСнности элСктричСского поля ΠΎΠΊΠΎΠ»ΠΎ, Π½Π° этом кондСнсаторС большС Π½Π΅ ΠΌΠΎΠΆΠ΅Ρ‚ Ρ…Ρ€Π°Π½ΠΈΡ‚ΡŒΡΡ заряд ΠΏΡ€ΠΈ ΡƒΠ²Π΅Π»ΠΈΡ‡Π΅Π½ΠΈΠΈ напряТСния.

ΠŸΠ ΠžΠ’Π•Π Π¬Π’Π• ΠŸΠžΠΠ˜ΠœΠΠΠ˜Π• 4.1


Π•ΠΌΠΊΠΎΡΡ‚ΡŒ кондСнсатора с ΠΏΠ°Ρ€Π°Π»Π»Π΅Π»ΡŒΠ½Ρ‹ΠΌΠΈ пластинами составляСт. Если ΠΏΠ»ΠΎΡ‰Π°Π΄ΡŒ ΠΊΠ°ΠΆΠ΄ΠΎΠΉ пластины Ρ€Π°Π²Π½Π°, ΠΊΠ°ΠΊΠΎΠ²ΠΎ расстояниС ΠΌΠ΅ΠΆΠ΄Ρƒ пластинами?

ΠŸΠ ΠžΠ’Π•Π Π¬Π’Π• ΠŸΠžΠΠ˜ΠœΠΠΠ˜Π• 4.2


Π£Π±Π΅Π΄ΠΈΡ‚Π΅ΡΡŒ, Ρ‡Ρ‚ΠΎ Ρƒ вас Ρ‚Π°ΠΊΠΈΠ΅ ΠΆΠ΅ физичСскиС Π΅Π΄ΠΈΠ½ΠΈΡ†Ρ‹.

ΠšΠΎΠ½Π΄Π΅Π½ΡΠ°Ρ‚ΠΎΡ€ сфСричСский

БфСричСский кондСнсатор — это Π΅Ρ‰Π΅ ΠΎΠ΄ΠΈΠ½ Π½Π°Π±ΠΎΡ€ ΠΏΡ€ΠΎΠ²ΠΎΠ΄Π½ΠΈΠΊΠΎΠ², Π΅ΠΌΠΊΠΎΡΡ‚ΡŒ ΠΊΠΎΡ‚ΠΎΡ€Ρ‹Ρ… ΠΌΠΎΠΆΠ½ΠΎ Π»Π΅Π³ΠΊΠΎ ΠΎΠΏΡ€Π΅Π΄Π΅Π»ΠΈΡ‚ΡŒ (Рисунок 4.1.5). Он состоит ΠΈΠ· Π΄Π²ΡƒΡ… концСнтричСских проводящих сфСричСских ΠΎΠ±ΠΎΠ»ΠΎΡ‡Π΅ΠΊ радиусов (внутрСнняя ΠΎΠ±ΠΎΠ»ΠΎΡ‡ΠΊΠ°) ΠΈ (внСшняя ΠΎΠ±ΠΎΠ»ΠΎΡ‡ΠΊΠ°). Бнарядам ΠΏΡ€ΠΈΠ΄Π°ΡŽΡ‚ΡΡ Ρ€Π°Π²Π½Ρ‹Π΅ ΠΈ ΠΏΡ€ΠΎΡ‚ΠΈΠ²ΠΎΠΏΠΎΠ»ΠΎΠΆΠ½Ρ‹Π΅ заряды ΠΈ соотвСтствСнно. Из-Π·Π° симмСтрии элСктричСскоС ΠΏΠΎΠ»Π΅ ΠΌΠ΅ΠΆΠ΄Ρƒ ΠΎΠ±ΠΎΠ»ΠΎΡ‡ΠΊΠ°ΠΌΠΈ Π½Π°ΠΏΡ€Π°Π²Π»Π΅Π½ΠΎ Ρ€Π°Π΄ΠΈΠ°Π»ΡŒΠ½ΠΎ Π½Π°Ρ€ΡƒΠΆΡƒ. ΠœΡ‹ ΠΌΠΎΠΆΠ΅ΠΌ ΠΏΠΎΠ»ΡƒΡ‡ΠΈΡ‚ΡŒ Π²Π΅Π»ΠΈΡ‡ΠΈΠ½Ρƒ поля, ΠΏΡ€ΠΈΠΌΠ΅Π½ΠΈΠ² Π·Π°ΠΊΠΎΠ½ Гаусса ΠΊ сфСричСской гауссовой повСрхности радиусом r , ΠΊΠΎΠ½Ρ†Π΅Π½Ρ‚Ρ€ΠΈΡ‡Π½ΠΎΠΉ ΠΎΠ±ΠΎΠ»ΠΎΡ‡ΠΊΠ°ΠΌ. ВлоТСнная ΠΏΠ»Π°Ρ‚Π° Π΅ΡΡ‚ΡŒ; ΡΠ»Π΅Π΄ΠΎΠ²Π°Ρ‚Π΅Π»ΡŒΠ½ΠΎ, Ρƒ нас Π΅ΡΡ‚ΡŒ

Π’Π°ΠΊΠΈΠΌ ΠΎΠ±Ρ€Π°Π·ΠΎΠΌ, элСктричСскоС ΠΏΠΎΠ»Π΅ ΠΌΠ΅ΠΆΠ΄Ρƒ ΠΏΡ€ΠΎΠ²ΠΎΠ΄Π½ΠΈΠΊΠ°ΠΌΠΈ Ρ€Π°Π²Π½ΠΎ

ΠœΡ‹ подставляСм это Π² ΡƒΡ€Π°Π²Π½Π΅Π½ΠΈΠ΅ 4.1.2 ΠΈ ΠΈΠ½Ρ‚Π΅Π³Ρ€ΠΈΡ€ΠΎΠ²Π°Ρ‚ΡŒ ΠΏΠΎ Ρ€Π°Π΄ΠΈΠ°Π»ΡŒΠ½ΠΎΠΌΡƒ ΠΏΡƒΡ‚ΠΈ ΠΌΠ΅ΠΆΠ΄Ρƒ ΠΎΠ±ΠΎΠ»ΠΎΡ‡ΠΊΠ°ΠΌΠΈ:

Π’ этом ΡƒΡ€Π°Π²Π½Π΅Π½ΠΈΠΈ Ρ€Π°Π·Π½ΠΎΡΡ‚ΡŒ ΠΏΠΎΡ‚Π΅Π½Ρ†ΠΈΠ°Π»ΠΎΠ² ΠΌΠ΅ΠΆΠ΄Ρƒ пластинами Ρ€Π°Π²Π½Π°. ΠŸΠΎΠ΄ΡΡ‚Π°Π²Π»ΡΠ΅ΠΌ этот Ρ€Π΅Π·ΡƒΠ»ΡŒΡ‚Π°Ρ‚ Π² ΡƒΡ€Π°Π²Π½Π΅Π½ΠΈΠ΅ 4.1.1, Ρ‡Ρ‚ΠΎΠ±Ρ‹ Π½Π°ΠΉΡ‚ΠΈ Π΅ΠΌΠΊΠΎΡΡ‚ΡŒ сфСричСского кондСнсатора:

(4.1.4)

(рисунок 4.1.5)

Рисунок 4.1.5 БфСричСский кондСнсатор состоит ΠΈΠ· Π΄Π²ΡƒΡ… концСнтричСских проводящих сфСр. ΠžΠ±Ρ€Π°Ρ‚ΠΈΡ‚Π΅ Π²Π½ΠΈΠΌΠ°Π½ΠΈΠ΅, Ρ‡Ρ‚ΠΎ заряды Π½Π° ΠΏΡ€ΠΎΠ²ΠΎΠ΄Π½ΠΈΠΊΠ΅ находятся Π½Π° Π΅Π³ΠΎ повСрхности.

ΠŸΠ Π˜ΠœΠ•Π  4.1,3


Π•ΠΌΠΊΠΎΡΡ‚ΡŒ ΠΈΠ·ΠΎΠ»ΠΈΡ€ΠΎΠ²Π°Π½Π½ΠΎΠΉ сфСры

РассчитайтС Π΅ΠΌΠΊΠΎΡΡ‚ΡŒ ΠΎΠ΄ΠΈΠ½ΠΎΡ‡Π½ΠΎΠΉ ΠΈΠ·ΠΎΠ»ΠΈΡ€ΠΎΠ²Π°Π½Π½ΠΎΠΉ проводящСй сфСры радиуса ΠΈ сравнитС Π΅Π΅ с ΡƒΡ€Π°Π²Π½Π΅Π½ΠΈΠ΅ΠΌ 4.1.4 Π² ΠΏΡ€Π΅Π΄Π΅Π»Π΅ ΠΊΠ°ΠΊ.

БтратСгия

ΠœΡ‹ ΠΏΡ€Π΅Π΄ΠΏΠΎΠ»Π°Π³Π°Π΅ΠΌ, Ρ‡Ρ‚ΠΎ Π½Π° сфСрС Π΅ΡΡ‚ΡŒ заряд, ΠΈ поэтому выполняСм Ρ‡Π΅Ρ‚Ρ‹Ρ€Π΅ шага, описанныС Ρ€Π°Π½Π΅Π΅. ΠœΡ‹ Ρ‚Π°ΠΊΠΆΠ΅ ΠΏΡ€Π΅Π΄ΠΏΠΎΠ»Π°Π³Π°Π΅ΠΌ, Ρ‡Ρ‚ΠΎ Π΄Ρ€ΡƒΠ³ΠΎΠΉ ΠΏΡ€ΠΎΠ²ΠΎΠ΄Π½ΠΈΠΊ прСдставляСт собой ΠΊΠΎΠ½Ρ†Π΅Π½Ρ‚Ρ€ΠΈΡ‡Π΅ΡΠΊΡƒΡŽ ΠΏΠΎΠ»ΡƒΡŽ сфСру бСсконСчного радиуса.

РСшСниС

На внСшнСй сторонС ΠΈΠ·ΠΎΠ»ΠΈΡ€ΠΎΠ²Π°Π½Π½ΠΎΠΉ проводящСй сфСры элСктричСскоС ΠΏΠΎΠ»Π΅ задаСтся ΡƒΡ€Π°Π²Π½Π΅Π½ΠΈΠ΅ΠΌ 4.1.2. Π’Π΅Π»ΠΈΡ‡ΠΈΠ½Π° разности ΠΏΠΎΡ‚Π΅Π½Ρ†ΠΈΠ°Π»ΠΎΠ² ΠΌΠ΅ΠΆΠ΄Ρƒ ΠΏΠΎΠ²Π΅Ρ€Ρ…Π½ΠΎΡΡ‚ΡŒΡŽ ΠΈΠ·ΠΎΠ»ΠΈΡ€ΠΎΠ²Π°Π½Π½ΠΎΠΉ сфСры ΠΈ Π±Π΅ΡΠΊΠΎΠ½Π΅Ρ‡Π½ΠΎΡΡ‚ΡŒΡŽ составляСт

.

Π’Π°ΠΊΠΈΠΌ ΠΎΠ±Ρ€Π°Π·ΠΎΠΌ, Π΅ΠΌΠΊΠΎΡΡ‚ΡŒ ΠΈΠ·ΠΎΠ»ΠΈΡ€ΠΎΠ²Π°Π½Π½ΠΎΠΉ сфСры Ρ€Π°Π²Π½Π°

.

Π—Π½Π°Ρ‡Π΅Π½ΠΈΠ΅

Π’ΠΎΡ‚ ΠΆΠ΅ Ρ€Π΅Π·ΡƒΠ»ΡŒΡ‚Π°Ρ‚ ΠΌΠΎΠΆΠ΅Ρ‚ Π±Ρ‹Ρ‚ΡŒ ΠΏΠΎΠ»ΡƒΡ‡Π΅Π½, Ссли Π²Π·ΡΡ‚ΡŒ ΠΏΡ€Π΅Π΄Π΅Π» уравнСния 4.1.4 ΠΊΠ°ΠΊ. Π’Π°ΠΊΠΈΠΌ ΠΎΠ±Ρ€Π°Π·ΠΎΠΌ, одиночная изолированная сфСра эквивалСнтна сфСричСскому кондСнсатору, внСшняя ΠΎΠ±ΠΎΠ»ΠΎΡ‡ΠΊΠ° ΠΊΠΎΡ‚ΠΎΡ€ΠΎΠ³ΠΎ ΠΈΠΌΠ΅Π΅Ρ‚ бСсконСчно большой радиус.

ΠŸΠ ΠžΠ’Π•Π Π¬Π’Π• ΠŸΠžΠΠ˜ΠœΠΠΠ˜Π• 4.3

Радиус внСшнСй сфСры сфСричСского кондСнсатора Π² ΠΏΡΡ‚ΡŒ Ρ€Π°Π· большС радиуса Π΅Π³ΠΎ Π²Π½ΡƒΡ‚Ρ€Π΅Π½Π½Π΅ΠΉ ΠΎΠ±ΠΎΠ»ΠΎΡ‡ΠΊΠΈ.КакиС Ρ€Π°Π·ΠΌΠ΅Ρ€Ρ‹ Ρƒ этого кондСнсатора, Ссли Π΅Π³ΠΎ Π΅ΠΌΠΊΠΎΡΡ‚ΡŒ?

ЦилиндричСский кондСнсатор

ЦилиндричСский кондСнсатор состоит ΠΈΠ· Π΄Π²ΡƒΡ… концСнтричСских проводящих Ρ†ΠΈΠ»ΠΈΠ½Π΄Ρ€ΠΎΠ² (рисунок 4.1.6). Π’Π½ΡƒΡ‚Ρ€Π΅Π½Π½ΠΈΠΉ Ρ†ΠΈΠ»ΠΈΠ½Π΄Ρ€ радиуса ΠΌΠΎΠΆΠ΅Ρ‚ Π±Ρ‹Ρ‚ΡŒ Π»ΠΈΠ±ΠΎ ΠΎΠ±ΠΎΠ»ΠΎΡ‡ΠΊΠΎΠΉ, Π»ΠΈΠ±ΠΎ ΠΏΠΎΠ»Π½ΠΎΡΡ‚ΡŒΡŽ Ρ‚Π²Π΅Ρ€Π΄Ρ‹ΠΌ. Π’Π½Π΅ΡˆΠ½ΠΈΠΉ Ρ†ΠΈΠ»ΠΈΠ½Π΄Ρ€ прСдставляСт собой ΠΎΠ±ΠΎΠ»ΠΎΡ‡ΠΊΡƒ Π²Π½ΡƒΡ‚Ρ€Π΅Π½Π½Π΅Π³ΠΎ радиуса. ΠœΡ‹ ΠΏΡ€Π΅Π΄ΠΏΠΎΠ»Π°Π³Π°Π΅ΠΌ, Ρ‡Ρ‚ΠΎ Π΄Π»ΠΈΠ½Π° ΠΊΠ°ΠΆΠ΄ΠΎΠ³ΠΎ Ρ†ΠΈΠ»ΠΈΠ½Π΄Ρ€Π° Ρ€Π°Π²Π½Π° ΠΈ Ρ‡Ρ‚ΠΎ ΠΈΠ·Π±Ρ‹Ρ‚ΠΎΡ‡Π½Ρ‹Π΅ заряды ΠΈ находятся Π½Π° Π²Π½ΡƒΡ‚Ρ€Π΅Π½Π½Π΅ΠΌ ΠΈ внСшнСм Ρ†ΠΈΠ»ΠΈΠ½Π΄Ρ€Π°Ρ… соотвСтствСнно.

(рисунок 4.1.6)

Рисунок 4.1.6 ЦилиндричСский кондСнсатор состоит ΠΈΠ· Π΄Π²ΡƒΡ… концСнтричСских проводящих Ρ†ΠΈΠ»ΠΈΠ½Π΄Ρ€ΠΎΠ². Π—Π΄Π΅ΡΡŒ заряд Π½Π° внСшнСй повСрхности Π²Π½ΡƒΡ‚Ρ€Π΅Π½Π½Π΅Π³ΠΎ Ρ†ΠΈΠ»ΠΈΠ½Π΄Ρ€Π° ΠΏΠΎΠ»ΠΎΠΆΠΈΡ‚Π΅Π»ΡŒΠ½Ρ‹ΠΉ (ΠΎΠ±ΠΎΠ·Π½Π°Ρ‡Π΅Π½ Π·Π½Π°Ρ‡ΠΊΠΎΠΌ), Π° заряд Π½Π° Π²Π½ΡƒΡ‚Ρ€Π΅Π½Π½Π΅ΠΉ повСрхности внСшнСго Ρ†ΠΈΠ»ΠΈΠ½Π΄Ρ€Π° ΠΎΡ‚Ρ€ΠΈΡ†Π°Ρ‚Π΅Π»ΡŒΠ½Ρ‹ΠΉ (ΠΎΠ±ΠΎΠ·Π½Π°Ρ‡Π΅Π½ Π·Π½Π°Ρ‡ΠΊΠΎΠΌ).

Π‘Π΅Π· ΡƒΡ‡Π΅Ρ‚Π° ΠΊΡ€Π°Π΅Π²Ρ‹Ρ… эффСктов элСктричСскоС ΠΏΠΎΠ»Π΅ ΠΌΠ΅ΠΆΠ΄Ρƒ ΠΏΡ€ΠΎΠ²ΠΎΠ΄Π½ΠΈΠΊΠ°ΠΌΠΈ Π½Π°ΠΏΡ€Π°Π²Π»Π΅Π½ΠΎ Ρ€Π°Π΄ΠΈΠ°Π»ΡŒΠ½ΠΎ Π½Π°Ρ€ΡƒΠΆΡƒ ΠΎΡ‚ ΠΎΠ±Ρ‰Π΅ΠΉ оси Ρ†ΠΈΠ»ΠΈΠ½Π΄Ρ€ΠΎΠ². Π˜ΡΠΏΠΎΠ»ΡŒΠ·ΡƒΡ гауссову ΠΏΠΎΠ²Π΅Ρ€Ρ…Π½ΠΎΡΡ‚ΡŒ, ΠΏΠΎΠΊΠ°Π·Π°Π½Π½ΡƒΡŽ Π½Π° рисункС 4.1.6, ΠΌΡ‹ ΠΈΠΌΠ΅Π΅ΠΌ

Π‘Π»Π΅Π΄ΠΎΠ²Π°Ρ‚Π΅Π»ΡŒΠ½ΠΎ, элСктричСскоС ΠΏΠΎΠ»Π΅ ΠΌΠ΅ΠΆΠ΄Ρƒ Ρ†ΠΈΠ»ΠΈΠ½Π΄Ρ€Π°ΠΌΠΈ Ρ€Π°Π²Π½ΠΎ

(4.1,5)

Π—Π΄Π΅ΡΡŒ \ hat {\ mathrm {r}} — Π΅Π΄ΠΈΠ½ΠΈΡ‡Π½Ρ‹ΠΉ Ρ€Π°Π΄ΠΈΠ°Π»ΡŒΠ½Ρ‹ΠΉ Π²Π΅ΠΊΡ‚ΠΎΡ€ ΠΏΠΎ радиусу Ρ†ΠΈΠ»ΠΈΠ½Π΄Ρ€Π°. ΠœΡ‹ ΠΌΠΎΠΆΠ΅ΠΌ ΠΏΠΎΠ΄ΡΡ‚Π°Π²ΠΈΡ‚ΡŒ Π² ΡƒΡ€Π°Π²Π½Π΅Π½ΠΈΠ΅ 4.1.2 ΠΈ Π½Π°ΠΉΡ‚ΠΈ Ρ€Π°Π·Π½ΠΎΡΡ‚ΡŒ ΠΏΠΎΡ‚Π΅Π½Ρ†ΠΈΠ°Π»ΠΎΠ² ΠΌΠ΅ΠΆΠ΄Ρƒ Ρ†ΠΈΠ»ΠΈΠ½Π΄Ρ€Π°ΠΌΠΈ:

Π’Π°ΠΊΠΈΠΌ ΠΎΠ±Ρ€Π°Π·ΠΎΠΌ, Π΅ΠΌΠΊΠΎΡΡ‚ΡŒ цилиндричСского кондСнсатора

(4.1.6)

Как ΠΈ Π² Π΄Ρ€ΡƒΠ³ΠΈΡ… случаях, эта Π΅ΠΌΠΊΠΎΡΡ‚ΡŒ зависит Ρ‚ΠΎΠ»ΡŒΠΊΠΎ ΠΎΡ‚ Π³Π΅ΠΎΠΌΠ΅Ρ‚Ρ€ΠΈΠΈ располоТСния ΠΏΡ€ΠΎΠ²ΠΎΠ΄Π½ΠΈΠΊΠΎΠ². Π’Π°ΠΆΠ½Ρ‹ΠΌ ΠΏΡ€ΠΈΠΌΠ΅Π½Π΅Π½ΠΈΠ΅ΠΌ уравнСния 4.1.6 являСтся ΠΎΠΏΡ€Π΅Π΄Π΅Π»Π΅Π½ΠΈΠ΅ Смкости Π½Π° Π΅Π΄ΠΈΠ½ΠΈΡ†Ρƒ Π΄Π»ΠΈΠ½Ρ‹ коаксиального кабСля , ΠΊΠΎΡ‚ΠΎΡ€Ρ‹ΠΉ ΠΎΠ±Ρ‹Ρ‡Π½ΠΎ ΠΈΡΠΏΠΎΠ»ΡŒΠ·ΡƒΠ΅Ρ‚ΡΡ для ΠΏΠ΅Ρ€Π΅Π΄Π°Ρ‡ΠΈ ΠΈΠ·ΠΌΠ΅Π½ΡΡŽΡ‰ΠΈΡ…ΡΡ Π²ΠΎ Π²Ρ€Π΅ΠΌΠ΅Π½ΠΈ элСктричСских сигналов.ΠšΠΎΠ°ΠΊΡΠΈΠ°Π»ΡŒΠ½Ρ‹ΠΉ кабСль состоит ΠΈΠ· Π΄Π²ΡƒΡ… концСнтричСских цилиндричСских ΠΏΡ€ΠΎΠ²ΠΎΠ΄Π½ΠΈΠΊΠΎΠ², Ρ€Π°Π·Π΄Π΅Π»Π΅Π½Π½Ρ‹Ρ… изоляционным ΠΌΠ°Ρ‚Π΅Ρ€ΠΈΠ°Π»ΠΎΠΌ. (Π—Π΄Π΅ΡΡŒ ΠΌΡ‹ ΠΏΡ€Π΅Π΄ΠΏΠΎΠ»Π°Π³Π°Π΅ΠΌ Π½Π°Π»ΠΈΡ‡ΠΈΠ΅ Π²Π°ΠΊΡƒΡƒΠΌΠ° ΠΌΠ΅ΠΆΠ΄Ρƒ ΠΏΡ€ΠΎΠ²ΠΎΠ΄Π½ΠΈΠΊΠ°ΠΌΠΈ, Π½ΠΎ Ρ„ΠΈΠ·ΠΈΠΊΠ° качСствСнно ΠΏΠΎΡ‡Ρ‚ΠΈ такая ΠΆΠ΅, ΠΊΠΎΠ³Π΄Π° пространство ΠΌΠ΅ΠΆΠ΄Ρƒ ΠΏΡ€ΠΎΠ²ΠΎΠ΄Π½ΠΈΠΊΠ°ΠΌΠΈ Π·Π°ΠΏΠΎΠ»Π½Π΅Π½ΠΎ диэлСктриком.) Π­Ρ‚Π° конфигурация экранируСт элСктричСский сигнал, Ρ€Π°ΡΠΏΡ€ΠΎΡΡ‚Ρ€Π°Π½ΡΡŽΡ‰ΠΈΠΉΡΡ ΠΏΠΎ Π²Π½ΡƒΡ‚Ρ€Π΅Π½Π½Π΅ΠΌΡƒ ΠΏΡ€ΠΎΠ²ΠΎΠ΄Π½ΠΈΠΊΡƒ, ΠΎΡ‚ ΠΏΠ°Ρ€Π°Π·ΠΈΡ‚Π½Ρ‹Ρ… элСктричСских ΠΏΠΎΠ»Π΅ΠΉ, Π²Π½Π΅ΡˆΠ½ΠΈΡ… ΠΏΠΎ ΠΎΡ‚Π½ΠΎΡˆΠ΅Π½ΠΈΡŽ ΠΊ ΠΏΡ€ΠΎΠ²ΠΎΠ΄Π½ΠΈΠΊΡƒ. кабСль. Π’ΠΎΠΊ Ρ‚Π΅Ρ‡Π΅Ρ‚ Π² ΠΏΡ€ΠΎΡ‚ΠΈΠ²ΠΎΠΏΠΎΠ»ΠΎΠΆΠ½Ρ‹Ρ… направлСниях Π²ΠΎ Π²Π½ΡƒΡ‚Ρ€Π΅Π½Π½Π΅ΠΌ ΠΈ внСшнСм ΠΏΡ€ΠΎΠ²ΠΎΠ΄Π½ΠΈΠΊΠ°Ρ…, ΠΏΡ€ΠΈ этом внСшний ΠΏΡ€ΠΎΠ²ΠΎΠ΄ ΠΎΠ±Ρ‹Ρ‡Π½ΠΎ Π·Π°Π·Π΅ΠΌΠ»Π΅Π½.Π’Π΅ΠΏΠ΅Ρ€ΡŒ ΠΈΠ· уравнСния 4.1.6 Π΅ΠΌΠΊΠΎΡΡ‚ΡŒ коаксиального кабСля Π½Π° Π΅Π΄ΠΈΠ½ΠΈΡ†Ρƒ Π΄Π»ΠΈΠ½Ρ‹ Ρ€Π°Π²Π½Π°

.

Π’ практичСских прилоТСниях Π²Π°ΠΆΠ½ΠΎ Π²Ρ‹Π±ΠΈΡ€Π°Ρ‚ΡŒ ΠΊΠΎΠ½ΠΊΡ€Π΅Ρ‚Π½Ρ‹Π΅ значСния. Π­Ρ‚ΠΎ ΠΌΠΎΠΆΠ΅Ρ‚ Π±Ρ‹Ρ‚ΡŒ достигнуто Π·Π° счСт ΡΠΎΠΎΡ‚Π²Π΅Ρ‚ΡΡ‚Π²ΡƒΡŽΡ‰Π΅Π³ΠΎ Π²Ρ‹Π±ΠΎΡ€Π° радиусов ΠΏΡ€ΠΎΠ²ΠΎΠ΄Π½ΠΈΠΊΠΎΠ² ΠΈ изоляционного ΠΌΠ°Ρ‚Π΅Ρ€ΠΈΠ°Π»Π° ΠΌΠ΅ΠΆΠ΄Ρƒ Π½ΠΈΠΌΠΈ.

ΠŸΠ ΠžΠ’Π•Π Π¬Π’Π• ΠŸΠžΠΠ˜ΠœΠΠΠ˜Π• 4.4


Когда цилиндричСский кондСнсатор заряТаСтся, ΠΌΠ΅ΠΆΠ΄Ρƒ Ρ†ΠΈΠ»ΠΈΠ½Π΄Ρ€Π°ΠΌΠΈ измСряСтся Ρ€Π°Π·Π½ΠΎΡΡ‚ΡŒ ΠΏΠΎΡ‚Π΅Π½Ρ†ΠΈΠ°Π»ΠΎΠ².Π°) Какова Π΅ΠΌΠΊΠΎΡΡ‚ΡŒ этой систСмы? Π±) Если Ρ†ΠΈΠ»ΠΈΠ½Π΄Ρ€Ρ‹ Π΄Π»ΠΈΠ½Π½Ρ‹Π΅, ΠΊΠ°ΠΊΠΎΠ²ΠΎ ΠΎΡ‚Π½ΠΎΡˆΠ΅Π½ΠΈΠ΅ ΠΈΡ… радиусов?

НСсколько Ρ‚ΠΈΠΏΠΎΠ² кондСнсаторов, ΠΊΠΎΡ‚ΠΎΡ€Ρ‹Π΅ ΠΌΠΎΠΆΠ½ΠΎ ΠΈΡΠΏΠΎΠ»ΡŒΠ·ΠΎΠ²Π°Ρ‚ΡŒ Π½Π° ΠΏΡ€Π°ΠΊΡ‚ΠΈΠΊΠ΅, ΠΏΠΎΠΊΠ°Π·Π°Π½Ρ‹ Π½Π° рисункС 4.1.3. ΠžΠ±Ρ‹Ρ‡Π½Ρ‹Π΅ кондСнсаторы часто состоят ΠΈΠ· Π΄Π²ΡƒΡ… Π½Π΅Π±ΠΎΠ»ΡŒΡˆΠΈΡ… кусочков мСталличСской Ρ„ΠΎΠ»ΡŒΠ³ΠΈ, Ρ€Π°Π·Π΄Π΅Π»Π΅Π½Π½Ρ‹Ρ… двумя нСбольшими кусочками изоляции (см. Рисунок 4.1.1 (b)). ΠœΠ΅Ρ‚Π°Π»Π»ΠΈΡ‡Π΅ΡΠΊΠ°Ρ Ρ„ΠΎΠ»ΡŒΠ³Π° ΠΈ изоляция ΠΏΠΎΠΊΡ€Ρ‹Ρ‚Ρ‹ Π·Π°Ρ‰ΠΈΡ‚Π½Ρ‹ΠΌ ΠΏΠΎΠΊΡ€Ρ‹Ρ‚ΠΈΠ΅ΠΌ, Π° Π΄Π²Π° мСталличСских Π²Ρ‹Π²ΠΎΠ΄Π° ΠΈΡΠΏΠΎΠ»ΡŒΠ·ΡƒΡŽΡ‚ΡΡ для ΠΏΠΎΠ΄ΠΊΠ»ΡŽΡ‡Π΅Π½ΠΈΡ Ρ„ΠΎΠ»ΡŒΠ³ΠΈ ΠΊ внСшнСй Ρ†Π΅ΠΏΠΈ. НСкоторыС распространСнныС изоляционныС ΠΌΠ°Ρ‚Π΅Ρ€ΠΈΠ°Π»Ρ‹ — это слюда, ΠΊΠ΅Ρ€Π°ΠΌΠΈΠΊΠ°, Π±ΡƒΠΌΠ°Π³Π° ΠΈ Π°Π½Ρ‚ΠΈΠΏΡ€ΠΈΠ³Π°Ρ€Π½ΠΎΠ΅ ΠΏΠΎΠΊΡ€Ρ‹Ρ‚ΠΈΠ΅ Teflon β„’.

Π”Ρ€ΡƒΠ³ΠΎΠΉ популярный Ρ‚ΠΈΠΏ кондСнсатора — элСктролитичСский кондСнсатор . Он состоит ΠΈΠ· окислСнного ΠΌΠ΅Ρ‚Π°Π»Π»Π° Π² проводящСй пастС. ΠžΡΠ½ΠΎΠ²Π½Ρ‹ΠΌ прСимущСством элСктролитичСского кондСнсатора являСтся Π΅Π³ΠΎ высокая Π΅ΠΌΠΊΠΎΡΡ‚ΡŒ ΠΏΠΎ ΡΡ€Π°Π²Π½Π΅Π½ΠΈΡŽ с Π΄Ρ€ΡƒΠ³ΠΈΠΌΠΈ распространСнными Ρ‚ΠΈΠΏΠ°ΠΌΠΈ кондСнсаторов. НапримСр, Π΅ΠΌΠΊΠΎΡΡ‚ΡŒ ΠΎΠ΄Π½ΠΎΠ³ΠΎ Ρ‚ΠΈΠΏΠ° алюминиСвого элСктролитичСского кондСнсатора ΠΌΠΎΠΆΠ΅Ρ‚ Π΄ΠΎΡΡ‚ΠΈΠ³Π°Ρ‚ΡŒ. Однако Π²Ρ‹ Π΄ΠΎΠ»ΠΆΠ½Ρ‹ Π±Ρ‹Ρ‚ΡŒ остороТны ΠΏΡ€ΠΈ использовании элСктролитичСского кондСнсатора Π² Ρ†Π΅ΠΏΠΈ, ΠΏΠΎΡ‚ΠΎΠΌΡƒ Ρ‡Ρ‚ΠΎ ΠΎΠ½ Ρ€Π°Π±ΠΎΡ‚Π°Π΅Ρ‚ ΠΏΡ€Π°Π²ΠΈΠ»ΡŒΠ½ΠΎ Ρ‚ΠΎΠ»ΡŒΠΊΠΎ Ρ‚ΠΎΠ³Π΄Π°, ΠΊΠΎΠ³Π΄Π° мСталличСская Ρ„ΠΎΠ»ΡŒΠ³Π° находится ΠΏΠΎΠ΄ Π±ΠΎΠ»Π΅Π΅ высоким ΠΏΠΎΡ‚Π΅Π½Ρ†ΠΈΠ°Π»ΠΎΠΌ, Ρ‡Π΅ΠΌ проводящая паста.Когда Π²ΠΎΠ·Π½ΠΈΠΊΠ°Π΅Ρ‚ обратная поляризация, элСктролитичСскоС дСйствиС Ρ€Π°Π·Ρ€ΡƒΡˆΠ°Π΅Ρ‚ ΠΎΠΊΡΠΈΠ΄Π½ΡƒΡŽ ΠΏΠ»Π΅Π½ΠΊΡƒ. Π­Ρ‚ΠΎΡ‚ Ρ‚ΠΈΠΏ кондСнсатора Π½Π΅ ΠΌΠΎΠΆΠ΅Ρ‚ Π±Ρ‹Ρ‚ΡŒ ΠΏΠΎΠ΄ΠΊΠ»ΡŽΡ‡Π΅Π½ ΠΊ источнику ΠΏΠ΅Ρ€Π΅ΠΌΠ΅Π½Π½ΠΎΠ³ΠΎ Ρ‚ΠΎΠΊΠ°, ΠΏΠΎΡ‚ΠΎΠΌΡƒ Ρ‡Ρ‚ΠΎ Π² ΠΏΠΎΠ»ΠΎΠ²ΠΈΠ½Π΅ случаСв ΠΏΠ΅Ρ€Π΅ΠΌΠ΅Π½Π½ΠΎΠ΅ напряТСниС Π±ΡƒΠ΄Π΅Ρ‚ ΠΈΠΌΠ΅Ρ‚ΡŒ Π½Π΅ΠΏΡ€Π°Π²ΠΈΠ»ΡŒΠ½ΡƒΡŽ ΠΏΠΎΠ»ΡΡ€Π½ΠΎΡΡ‚ΡŒ, ΠΏΠΎΡΠΊΠΎΠ»ΡŒΠΊΡƒ ΠΏΠ΅Ρ€Π΅ΠΌΠ΅Π½Π½Ρ‹ΠΉ Ρ‚ΠΎΠΊ мСняСт свою ΠΏΠΎΠ»ΡΡ€Π½ΠΎΡΡ‚ΡŒ (см. Π‘Ρ…Π΅ΠΌΡ‹ ΠΏΠ΅Ρ€Π΅ΠΌΠ΅Π½Π½ΠΎΠ³ΠΎ Ρ‚ΠΎΠΊΠ° Π² цСпях ΠΏΠ΅Ρ€Π΅ΠΌΠ΅Π½Π½ΠΎΠ³ΠΎ Ρ‚ΠΎΠΊΠ°).

ΠŸΠ΅Ρ€Π΅ΠΌΠ΅Π½Π½Ρ‹ΠΉ Π²ΠΎΠ·Π΄ΡƒΡˆΠ½Ρ‹ΠΉ кондСнсатор (рисунок 4.1.7) ΠΈΠΌΠ΅Π΅Ρ‚ Π΄Π²Π° Π½Π°Π±ΠΎΡ€Π° ΠΏΠ°Ρ€Π°Π»Π»Π΅Π»ΡŒΠ½Ρ‹Ρ… пластин. Один Π½Π°Π±ΠΎΡ€ пластин Π·Π°ΠΊΡ€Π΅ΠΏΠ»Π΅Π½ (ΠΎΠ±ΠΎΠ·Π½Π°Ρ‡Π΅Π½ ΠΊΠ°ΠΊ «статор»), Π° Π΄Ρ€ΡƒΠ³ΠΎΠΉ Π½Π°Π±ΠΎΡ€ пластин ΠΏΡ€ΠΈΠΊΡ€Π΅ΠΏΠ»Π΅Π½ ΠΊ Π²Π°Π»Ρƒ, ΠΊΠΎΡ‚ΠΎΡ€Ρ‹ΠΉ ΠΌΠΎΠΆΠ΅Ρ‚ Π²Ρ€Π°Ρ‰Π°Ρ‚ΡŒΡΡ (обозначаСтся ΠΊΠ°ΠΊ Β«Ρ€ΠΎΡ‚ΠΎΡ€Β»).ΠŸΠΎΠ²ΠΎΡ€Π°Ρ‡ΠΈΠ²Π°Ρ Π²Π°Π», ΠΌΠΎΠΆΠ½ΠΎ ΠΈΠ·ΠΌΠ΅Π½ΡΡ‚ΡŒ ΠΏΠ»ΠΎΡ‰Π°Π΄ΡŒ ΠΏΠΎΠΏΠ΅Ρ€Π΅Ρ‡Π½ΠΎΠ³ΠΎ сСчСния Π² ΠΏΠ΅Ρ€Π΅ΠΊΡ€Ρ‹Ρ‚ΠΈΠΈ пластин; ΡΠ»Π΅Π΄ΠΎΠ²Π°Ρ‚Π΅Π»ΡŒΠ½ΠΎ, Π΅ΠΌΠΊΠΎΡΡ‚ΡŒ этой систСмы ΠΌΠΎΠΆΠ΅Ρ‚ Π±Ρ‹Ρ‚ΡŒ настроСна Π½Π° ΠΆΠ΅Π»Π°Π΅ΠΌΠΎΠ΅ Π·Π½Π°Ρ‡Π΅Π½ΠΈΠ΅. Настройка кондСнсатора Π½Π°Ρ…ΠΎΠ΄ΠΈΡ‚ ΠΏΡ€ΠΈΠΌΠ΅Π½Π΅Π½ΠΈΠ΅ Π² любом Ρ‚ΠΈΠΏΠ΅ Ρ€Π°Π΄ΠΈΠΎΠΏΠ΅Ρ€Π΅Π΄Π°Ρ‡ΠΈ ΠΈ ΠΏΡ€ΠΈ ΠΏΡ€ΠΈΠ΅ΠΌΠ΅ радиосигналов ΠΎΡ‚ элСктронных устройств. ΠšΠ°ΠΆΠ΄Ρ‹ΠΉ Ρ€Π°Π·, ΠΊΠΎΠ³Π΄Π° Π²Ρ‹ настраиваСтС Π°Π²Ρ‚ΠΎΠΌΠΎΠ±ΠΈΠ»ΡŒΠ½ΠΎΠ΅ Ρ€Π°Π΄ΠΈΠΎ Π½Π° Π»ΡŽΠ±ΠΈΠΌΡƒΡŽ ΡΡ‚Π°Π½Ρ†ΠΈΡŽ, Π΄ΡƒΠΌΠ°ΠΉΡ‚Π΅ ΠΎ Смкости.

(рисунок 4.1.7)

Рисунок 4.1.7 Π’ кондСнсаторС ΠΏΠ΅Ρ€Π΅ΠΌΠ΅Π½Π½ΠΎΠ³ΠΎ Ρ‚ΠΎΠΊΠ° Π΅ΠΌΠΊΠΎΡΡ‚ΡŒ ΠΌΠΎΠΆΠ½ΠΎ Ρ€Π΅Π³ΡƒΠ»ΠΈΡ€ΠΎΠ²Π°Ρ‚ΡŒ, измСняя ΡΡ„Ρ„Π΅ΠΊΡ‚ΠΈΠ²Π½ΡƒΡŽ ΠΏΠ»ΠΎΡ‰Π°Π΄ΡŒ пластин.(ΠΊΡ€Π΅Π΄ΠΈΡ‚: модификация Ρ€Π°Π±ΠΎΡ‚Ρ‹ Π ΠΎΠ±Π±ΠΈ Π‘ΠΏΡ€ΡƒΠ»Π°)

Π‘ΠΈΠΌΠ²ΠΎΠ»Ρ‹, ΠΏΠΎΠΊΠ°Π·Π°Π½Π½Ρ‹Π΅ Π½Π° рисункС 4.1.8, ΠΏΡ€Π΅Π΄ΡΡ‚Π°Π²Π»ΡΡŽΡ‚ собой схСмныС изобраТСния Ρ€Π°Π·Π»ΠΈΡ‡Π½Ρ‹Ρ… Ρ‚ΠΈΠΏΠΎΠ² кондСнсаторов. ΠžΠ±Ρ‹Ρ‡Π½ΠΎ ΠΌΡ‹ ΠΈΡΠΏΠΎΠ»ΡŒΠ·ΡƒΠ΅ΠΌ символ, ΠΏΠΎΠΊΠ°Π·Π°Π½Π½Ρ‹ΠΉ Π½Π° рис. 4.1.8 (Π°). Π‘ΠΈΠΌΠ²ΠΎΠ» Π½Π° РисункС 4.1.8 (c) прСдставляСт кондСнсатор ΠΏΠ΅Ρ€Π΅ΠΌΠ΅Π½Π½ΠΎΠΉ Смкости. ΠžΠ±Ρ€Π°Ρ‚ΠΈΡ‚Π΅ Π²Π½ΠΈΠΌΠ°Π½ΠΈΠ΅ Π½Π° сходство этих символов с симмСтриСй кондСнсатора с ΠΏΠ°Ρ€Π°Π»Π»Π΅Π»ΡŒΠ½Ρ‹ΠΌΠΈ пластинами. ЭлСктролитичСский кондСнсатор прСдставлСн символом Π½Π° рис. 4.1.8 (b), Π³Π΄Π΅ изогнутая пластина ΠΎΠ±ΠΎΠ·Π½Π°Ρ‡Π°Π΅Ρ‚ ΠΎΡ‚Ρ€ΠΈΡ†Π°Ρ‚Π΅Π»ΡŒΠ½Ρ‹ΠΉ Π²Ρ‹Π²ΠΎΠ΄.

(рисунок 4.1.8)

Рисунок 4.1.8 Π—Π΄Π΅ΡΡŒ ΠΏΠΎΠΊΠ°Π·Π°Π½Ρ‹ Ρ‚Ρ€ΠΈ Ρ€Π°Π·Π»ΠΈΡ‡Π½Ρ‹Ρ… схСмных прСдставлСния кондСнсаторов. Π‘ΠΈΠΌΠ²ΠΎΠ» Π² (Π°) являСтся Π½Π°ΠΈΠ±ΠΎΠ»Π΅Π΅ часто ΠΈΡΠΏΠΎΠ»ΡŒΠ·ΡƒΠ΅ΠΌΡ‹ΠΌ. Π‘ΠΈΠΌΠ²ΠΎΠ» Π² (b) прСдставляСт собой элСктролитичСский кондСнсатор. Π‘ΠΈΠΌΠ²ΠΎΠ» Π² (c) прСдставляСт кондСнсатор ΠΏΠ΅Ρ€Π΅ΠΌΠ΅Π½Π½ΠΎΠΉ Смкости.

Π˜Π½Ρ‚Π΅Ρ€Π΅ΡΠ½Ρ‹ΠΉ ΠΏΡ€ΠΈΠΊΠ»Π°Π΄Π½ΠΎΠΉ ΠΏΡ€ΠΈΠΌΠ΅Ρ€ ΠΌΠΎΠ΄Π΅Π»ΠΈ кондСнсатора взят ΠΈΠ· ΠΊΠ»Π΅Ρ‚ΠΎΡ‡Π½ΠΎΠΉ Π±ΠΈΠΎΠ»ΠΎΠ³ΠΈΠΈ ΠΈ ΠΈΠΌΠ΅Π΅Ρ‚ Π΄Π΅Π»ΠΎ с элСктричСским ΠΏΠΎΡ‚Π΅Π½Ρ†ΠΈΠ°Π»ΠΎΠΌ Π² плазматичСской ΠΌΠ΅ΠΌΠ±Ρ€Π°Π½Π΅ ΠΆΠΈΠ²ΠΎΠΉ ΠΊΠ»Π΅Ρ‚ΠΊΠΈ (рис. 4.1.9). ΠšΠ»Π΅Ρ‚ΠΎΡ‡Π½Ρ‹Π΅ ΠΌΠ΅ΠΌΠ±Ρ€Π°Π½Ρ‹ ΠΎΡ‚Π΄Π΅Π»ΡΡŽΡ‚ ΠΊΠ»Π΅Ρ‚ΠΊΠΈ ΠΎΡ‚ ΠΈΡ… окруТСния, Π½ΠΎ ΠΏΠΎΠ·Π²ΠΎΠ»ΡΡŽΡ‚ Π½Π΅ΠΊΠΎΡ‚ΠΎΡ€Ρ‹ΠΌ ΠΎΡ‚ΠΎΠ±Ρ€Π°Π½Π½Ρ‹ΠΌ ΠΈΠΎΠ½Π°ΠΌ ΠΏΡ€ΠΎΡ…ΠΎΠ΄ΠΈΡ‚ΡŒ Π²Π½ΡƒΡ‚Ρ€ΡŒ ΠΈΠ»ΠΈ ΠΈΠ· ΠΊΠ»Π΅Ρ‚ΠΊΠΈ.Π Π°Π·Π½ΠΎΡΡ‚ΡŒ ΠΏΠΎΡ‚Π΅Π½Ρ†ΠΈΠ°Π»ΠΎΠ² Π½Π° ΠΌΠ΅ΠΌΠ±Ρ€Π°Π½Π΅ составляСт ΠΎΠΊΠΎΠ»ΠΎ. ΠšΠ»Π΅Ρ‚ΠΎΡ‡Π½Π°Ρ ΠΌΠ΅ΠΌΠ±Ρ€Π°Π½Π° ΠΌΠΎΠΆΠ΅Ρ‚ Π±Ρ‹Ρ‚ΡŒ слишком толстой. Рассматривая ΠΊΠ»Π΅Ρ‚ΠΎΡ‡Π½ΡƒΡŽ ΠΌΠ΅ΠΌΠ±Ρ€Π°Π½Ρƒ ΠΊΠ°ΠΊ кондСнсатор Π½Π°Π½ΠΎΡ€Π°Π·ΠΌΠ΅Ρ€ΠΎΠ², ΠΎΡ†Π΅Π½ΠΊΠ° наимСньшСй напряТСнности элСктричСского поля Π½Π° Π΅Π³ΠΎ «пластинах» Π΄Π°Π΅Ρ‚ Π·Π½Π°Ρ‡Π΅Π½ΠΈΠ΅.

Π­Ρ‚ΠΎΠΉ Π²Π΅Π»ΠΈΡ‡ΠΈΠ½Ρ‹ элСктричСского поля достаточно, Ρ‡Ρ‚ΠΎΠ±Ρ‹ Π²Ρ‹Π·Π²Π°Ρ‚ΡŒ ΡΠ»Π΅ΠΊΡ‚Ρ€ΠΈΡ‡Π΅ΡΠΊΡƒΡŽ искру Π² Π²ΠΎΠ·Π΄ΡƒΡ…Π΅.

(рисунок 4.1.9)

Рис. 4.1.9 ΠŸΠΎΠ»ΡƒΠΏΡ€ΠΎΠ½ΠΈΡ†Π°Π΅ΠΌΠ°Ρ ΠΌΠ΅ΠΌΠ±Ρ€Π°Π½Π° биологичСской ΠΊΠ»Π΅Ρ‚ΠΊΠΈ ΠΈΠΌΠ΅Π΅Ρ‚ Ρ€Π°Π·Π½Ρ‹Π΅ ΠΊΠΎΠ½Ρ†Π΅Π½Ρ‚Ρ€Π°Ρ†ΠΈΠΈ ΠΈΠΎΠ½ΠΎΠ² Π½Π° Π²Π½ΡƒΡ‚Ρ€Π΅Π½Π½Π΅ΠΉ повСрхности, Ρ‡Π΅ΠΌ Π½Π° внСшнСй.Диффузия ΠΏΠ΅Ρ€Π΅ΠΌΠ΅Ρ‰Π°Π΅Ρ‚ ΠΈΠΎΠ½Ρ‹ (калия) ΠΈ (Ρ…Π»ΠΎΡ€ΠΈΠ΄Π°) Π² ΠΏΠΎΠΊΠ°Π·Π°Π½Π½Ρ‹Ρ… направлСниях, ΠΏΠΎΠΊΠ° кулоновская сила Π½Π΅ остановит Π΄Π°Π»ΡŒΠ½Π΅ΠΉΡˆΡƒΡŽ ΠΏΠ΅Ρ€Π΅Π΄Π°Ρ‡Ρƒ. Π’Π°ΠΊΠΈΠΌ ΠΎΠ±Ρ€Π°Π·ΠΎΠΌ, внСшняя Ρ‡Π°ΡΡ‚ΡŒ ΠΌΠ΅ΠΌΠ±Ρ€Π°Π½Ρ‹ ΠΏΡ€ΠΈΠΎΠ±Ρ€Π΅Ρ‚Π°Π΅Ρ‚ ΠΏΠΎΠ»ΠΎΠΆΠΈΡ‚Π΅Π»ΡŒΠ½Ρ‹ΠΉ заряд, Π° Π΅Π΅ внутрСнняя ΠΏΠΎΠ²Π΅Ρ€Ρ…Π½ΠΎΡΡ‚ΡŒ ΠΏΡ€ΠΈΠΎΠ±Ρ€Π΅Ρ‚Π°Π΅Ρ‚ ΠΎΡ‚Ρ€ΠΈΡ†Π°Ρ‚Π΅Π»ΡŒΠ½Ρ‹ΠΉ заряд, создавая Ρ€Π°Π·Π½ΠΎΡΡ‚ΡŒ ΠΏΠΎΡ‚Π΅Π½Ρ†ΠΈΠ°Π»ΠΎΠ² Π½Π° ΠΌΠ΅ΠΌΠ±Ρ€Π°Π½Π΅. ΠœΠ΅ΠΌΠ±Ρ€Π°Π½Π° ΠΎΠ±Ρ‹Ρ‡Π½ΠΎ Π½Π΅ΠΏΡ€ΠΎΠ½ΠΈΡ†Π°Π΅ΠΌΠ° для (ΠΈΠΎΠ½ΠΎΠ² натрия).

КандСла Π¦ΠΈΡ‚Π°Ρ‚Ρ‹

Π›ΠΈΡ†Π΅Π½Π·ΠΈΠΎΠ½Π½Ρ‹ΠΉ ΠΊΠΎΠ½Ρ‚Π΅Π½Ρ‚ CC, особая атрибуция

  • Π—Π°Π³Ρ€ΡƒΠ·ΠΈΡ‚Π΅ бСсплатно ΠΏΠΎ адрСсу http: // cnx.org/contents/[email protected]. ΠŸΠΎΠ»ΡƒΡ‡Π΅Π½ΠΎ с : http://cnx.org/contents/[email protected]. ЛицСнзия : CC BY: Attribution

ΠšΠΎΠ½Π΄Π΅Π½ΡΠ°Ρ‚ΠΎΡ€ с ΠΏΠ°Ρ€Π°Π»Π»Π΅Π»ΡŒΠ½Ρ‹ΠΌΠΈ пластинами

— ΠΎΠ±Π·ΠΎΡ€

УмСньшСниС ΠΌΠ΅ΠΆΠΎΠ±ΠΌΠΎΡ‚ΠΎΡ‡Π½ΠΎΠΉ Смкости трансформатора

Π•ΠΌΠΊΠΎΡΡ‚ΡŒ кондСнсатора с ΠΏΠ°Ρ€Π°Π»Π»Π΅Π»ΡŒΠ½Ρ‹ΠΌΠΈ пластинами прямо ΠΏΡ€ΠΎΠΏΠΎΡ€Ρ†ΠΈΠΎΠ½Π°Π»ΡŒΠ½Π° ΠΏΠ»ΠΎΡ‰Π°Π΄ΠΈ пластин ΠΈ ΠΎΠ±Ρ€Π°Ρ‚Π½ΠΎ ΠΏΡ€ΠΎΠΏΠΎΡ€Ρ†ΠΈΠΎΠ½Π°Π»ΡŒΠ½Π° Ρ€Π°ΡΡΡ‚ΠΎΡΠ½ΠΈΡŽ, Ρ€Π°Π·Π΄Π΅Π»ΡΡŽΡ‰Π΅ΠΌΡƒ ΠΈΡ…, поэтому, Ссли Π±Ρ‹ ΠΌΡ‹ ΠΌΠΎΠ³Π»ΠΈ ΡƒΠΌΠ΅Π½ΡŒΡˆΠΈΡ‚ΡŒ ΠΏΠ»ΠΎΡ‰Π°Π΄ΡŒ ΠΌΠ΅ΠΆΠ΄Ρƒ Π½ΠΈΠΌΠΈ ΠΌΠ΅ΠΆΠ΄Ρƒ ΠΏΠ΅Ρ€Π²ΠΈΡ‡Π½ΠΎΠΉ ΠΈ Π²Ρ‚ΠΎΡ€ΠΈΡ‡Π½ΠΎΠΉ ΠΎΠ±ΠΌΠΎΡ‚ΠΊΠ°ΠΌΠΈ ΠΈ ΡƒΠ²Π΅Π»ΠΈΡ‡ΠΈΠ² ΠΈΡ… Ρ€Π°Π·Π΄Π΅Π»Π΅Π½ΠΈΠ΅, ΠΌΡ‹ ΡƒΠΌΠ΅Π½ΡŒΡˆΠΈΠΌ ΠΌΠ΅ΠΆΠΎΠ±ΠΌΠΎΡ‚ΠΎΡ‡Π½ΡƒΡŽ Π΅ΠΌΠΊΠΎΡΡ‚ΡŒ.(Π­Ρ„Ρ„Π΅ΠΊΡ‚ ΠΎΡ‚ ΠΏΠΎΠΏΡ‹Ρ‚ΠΊΠΈ ΠΎΡ‚Ρ€Π΅Π³ΡƒΠ»ΠΈΡ€ΠΎΠ²Π°Ρ‚ΡŒ Π΄ΠΈΡΠ»Π΅ΠΊΡ‚Ρ€ΠΈΡ‡Π΅ΡΠΊΡƒΡŽ ΠΏΠΎΡΡ‚ΠΎΡΠ½Π½ΡƒΡŽ Π±ΡƒΠ΄Π΅Ρ‚ ΠΌΠΈΠ½ΠΈΠΌΠ°Π»ΡŒΠ½Ρ‹ΠΌ.)

Π’ качСствС ΠΏΡ€ΠΈΠΌΠ΅Ρ€Π°, ΠΊΠΎΠΌΠΏΠ»Π΅ΠΊΡ‚ трансформатора Maplin 100 VA EI ΠΈΠΌΠ΅Π΅Ρ‚ ΠΊΠΎΠ½ΡΡ‚Ρ€ΡƒΠΊΡ†ΠΈΡŽ Ρ€Π°Π·Π΄Π΅Π»ΡŒΠ½ΠΎΠΉ ΠΊΠ°Ρ‚ΡƒΡˆΠΊΠΈ с ΠΏΠ΅Ρ€Π²ΠΈΡ‡Π½ΠΎΠΉ Π½Π°ΠΌΠΎΡ‚ΠΊΠΎΠΉ Π½Π° ΠΎΠ΄Π½ΠΎΠΉ ΠΏΠΎΠ»ΠΎΠ²ΠΈΠ½Π΅ ΠΊΠ°Ρ‚ΡƒΡˆΠΊΠΈ, Π° другая ΠΏΠΎΠ»ΠΎΠ²ΠΈΠ½Π° остаСтся пустой Π½Π° врСмя ΠΏΠΎΠ»ΡŒΠ·ΠΎΠ²Π°Ρ‚Π΅Π»ΡŒ для Π½Π°ΠΌΠΎΡ‚ΠΊΠΈ Π²Ρ‚ΠΎΡ€ΠΈΡ‡Π½ΠΎΠΉ ΠΎΠ±ΠΌΠΎΡ‚ΠΊΠΈ. ΠŸΠ»ΠΎΡ‰Π°Π΄ΡŒ ΠΌΠ΅ΠΆΠΎΠ±ΠΌΠΎΡ‚ΠΎΡ‡Π½ΠΎΠ³ΠΎ кондСнсатора Π½Π° Ρ€Π°Π·Π΄Π΅Π»Π΅Π½Π½ΠΎΠΌ каркасном трансформаторС — это просто ΠΏΠ»ΠΎΡ‰Π°Π΄ΡŒ раздСлитСля ΠΌΠ΅ΠΆΠ΄Ρƒ ΠΏΠ΅Ρ€Π²ΠΈΡ‡Π½ΠΎΠΉ ΠΈ Π²Ρ‚ΠΎΡ€ΠΈΡ‡Π½ΠΎΠΉ ΠΎΠ±ΠΌΠΎΡ‚ΠΊΠ°ΠΌΠΈ, Π·Π° ΠΈΡΠΊΠ»ΡŽΡ‡Π΅Π½ΠΈΠ΅ΠΌ ΡƒΠ³Π»ΠΎΠ², ΠΏΠΎΡ‚ΠΎΠΌΡƒ Ρ‡Ρ‚ΠΎ ΠΌΠ΅Π΄Π½Ρ‹Π΅ ΠΎΠ±ΠΌΠΎΡ‚ΠΊΠΈ ΠΈΠΌΠ΅ΡŽΡ‚ Ρ‚Π΅Π½Π΄Π΅Π½Ρ†ΠΈΡŽ ΠΈΠ·Π³ΠΈΠ±Π°Ρ‚ΡŒΡΡ ΠΏΡ€ΠΈ ΠΈΠ·Π³ΠΈΠ±Π΅ Π²ΠΎΠΊΡ€ΡƒΠ³ ΠΊΠ°ΠΆΠ΄ΠΎΠ³ΠΎ ΡƒΠ³Π»Π° (см. Рисунок 5.56).

Рисунок 5.56. ΠœΠ΅ΠΆΠΎΠ±ΠΌΠΎΡ‚ΠΎΡ‡Π½Π°Ρ Π΅ΠΌΠΊΠΎΡΡ‚ΡŒ Π² Ρ€Π°Π·Π΄Π΅Π»Π΅Π½Π½ΠΎΠΌ ΠΊΠ°Ρ‚ΡƒΡˆΠ΅Ρ‡Π½ΠΎΠΌ трансформаторС.

Емкостная ΠΏΠ»ΠΎΡ‰Π°Π΄ΡŒ дСлитСля β‰ˆ1800 ΠΌΠΌ 2 . И Π½Π°ΠΎΠ±ΠΎΡ€ΠΎΡ‚, Ссли Π±Ρ‹ Ρ€Π°Π·Π΄Π΅Π»ΠΈΡ‚Π΅Π»ΡŒ Π±Ρ‹Π» ΡƒΠ΄Π°Π»Π΅Π½ ΠΈ Π±ΠΎΠ±ΠΈΠ½Π° Π±Ρ‹Π»Π° Π·Π°ΠΏΠΎΠ»Π½Π΅Π½Π° многослойными ΠΎΠ±ΠΌΠΎΡ‚ΠΊΠ°ΠΌΠΈ, ΠΏΠ»ΠΎΡ‰Π°Π΄ΡŒ ΠΌΠ΅ΠΆΠ΄Ρƒ ΠΏΠ΅Ρ€Π²ΠΈΡ‡Π½ΠΎΠΉ ΠΈ Π²Ρ‚ΠΎΡ€ΠΈΡ‡Π½ΠΎΠΉ ΠΎΠ±ΠΌΠΎΡ‚ΠΊΠ°ΠΌΠΈ Π±Ρ‹Π»Π° Π±Ρ‹ β‰ˆ 7200 ΠΌΠΌ 2 , увСличивая Π΅ΠΌΠΊΠΎΡΡ‚ΡŒ Π² 7200/1800 = 4 Ρ€Π°Π·Π°.

Если ΠΏΠΎΡΠΌΠΎΡ‚Ρ€Π΅Ρ‚ΡŒ Π½Π° Ρ‚ΠΎΠ»Ρ‰ΠΈΠ½Ρƒ ΠΌΠ΅ΠΆΠ΄Ρƒ пластинами, типичная полиэфирная изоляционная Π»Π΅Π½Ρ‚Π° трансформатора ΠΌΠΎΠΆΠ΅Ρ‚ Π±Ρ‹Ρ‚ΡŒ Ρ€Π°Π²Π½Π° 0.055 ΠΌΠΌ, Π½ΠΎ хотя ΠΎΠ΄Π½Π° Ρ‚ΠΎΠ»Ρ‰ΠΈΠ½Π° тСорСтичСски ΠΌΠΎΠΆΠ΅Ρ‚ Π±Ρ‹Ρ‚ΡŒ рассчитана Π½Π° напряТСниС пробоя 3 ΠΊΠ’, ΠΏΡ€ΠΎΠΈΠ·Π²ΠΎΠ΄ΠΈΡ‚Π΅Π»ΡŒ трансформатора Π΄ΠΎΠ»ΠΆΠ΅Π½ Π·Π½Π°Ρ‚ΡŒ, Ρ‡Ρ‚ΠΎ Π½Π° ΠΏΡ€Π°ΠΊΡ‚ΠΈΠΊΠ΅ ΠΌΠΎΠΆΠ΅Ρ‚ ΠΏΠΎΡ‚Ρ€Π΅Π±ΠΎΠ²Π°Ρ‚ΡŒΡΡ Ρ‡Π΅Ρ‚Ρ‹Ρ€Π΅ слоя этой Π»Π΅Π½Ρ‚Ρ‹ ΠΌΠ΅ΠΆΠ΄Ρƒ ΠΏΠ΅Ρ€Π²ΠΈΡ‡Π½Ρ‹ΠΌ ΠΈ Π²Ρ‚ΠΎΡ€ΠΈΡ‡Π½Ρ‹ΠΌ слоями, Ρ‡Ρ‚ΠΎΠ±Ρ‹ Π³Π°Ρ€Π°Π½Ρ‚ΠΈΡ€ΠΎΠ²Π°Ρ‚ΡŒ ΡΠΎΠΎΡ‚Π²Π΅Ρ‚ΡΡ‚Π²ΡƒΡŽΡ‰ΡƒΡŽ Π΄ΠΈΡΠ»Π΅ΠΊΡ‚Ρ€ΠΈΡ‡Π΅ΡΠΊΡƒΡŽ ΠΏΡ€ΠΎΡ‡Π½ΠΎΡΡ‚ΡŒ для Π°Π΄Π΅ΠΊΠ²Π°Ρ‚Π½ΠΎΠΉ изоляции сСтСвого напряТСния. , давая ΠΎΠ±Ρ‰ΡƒΡŽ Ρ‚ΠΎΠ»Ρ‰ΠΈΠ½Ρƒ ΠΌΠ΅ΠΆΠ΄Ρƒ ΠΎΠ±ΠΌΠΎΡ‚ΠΊΠ°ΠΌΠΈ 0,22 ΠΌΠΌ. Напротив, Π΄Π΅Π»ΠΈΡ‚Π΅Π»ΡŒ Π² трансформаторС с Ρ€Π°Π·ΡŠΠ΅ΠΌΠ½ΠΎΠΉ ΠΊΠ°Ρ‚ΡƒΡˆΠΊΠΎΠΉ ΠΈΠΌΠ΅Π΅Ρ‚ Ρ‚ΠΎΠ»Ρ‰ΠΈΠ½Ρƒ 1,07 ΠΌΠΌ, поэтому ΠΎΠ½ сниТаСт Π΅ΠΌΠΊΠΎΡΡ‚ΡŒ Π² 1,07 / 0,22 = 4 Ρ€Π°Π·Π°.9.

Π’ совокупности ΡƒΠΌΠ΅Π½ΡŒΡˆΠ΅Π½Π½Π°Ρ ΠΏΠ»ΠΎΡ‰Π°Π΄ΡŒ пластины ΠΈ ΡƒΠ²Π΅Π»ΠΈΡ‡Π΅Π½Π½ΠΎΠ΅ Ρ€Π°Π·Π΄Π΅Π»Π΅Π½ΠΈΠ΅ ΠΏΡ€ΠΈΠ²Π΅Π΄Π΅Π½Π½ΠΎΠ³ΠΎ Π² качСствС ΠΏΡ€ΠΈΠΌΠ΅Ρ€Π° трансформатора с Ρ€Π°Π·Π΄Π΅Π»Π΅Π½Π½ΠΎΠΉ ΠΊΠ°Ρ‚ΡƒΡˆΠΊΠΎΠΉ ΡƒΠΌΠ΅Π½ΡŒΡˆΠ°ΡŽΡ‚ ΠΌΠ΅ΠΆΠΎΠ±ΠΌΠΎΡ‚ΠΎΡ‡Π½ΡƒΡŽ Π΅ΠΌΠΊΠΎΡΡ‚ΡŒ Π² 4 Γ— 4,9β‰ˆ19,5 Ρ€Π°Π·, Ρ‡Ρ‚ΠΎ ΠΎΠ·Π½Π°Ρ‡Π°Π΅Ρ‚ ΡƒΠΌΠ΅Π½ΡŒΡˆΠ΅Π½ΠΈΠ΅ ΠΏΠΎΠΌΠ΅Ρ… Π½Π° β‰ˆ26 Π΄Π‘.

ΠŸΡ€Π΅ΠΆΠ΄Π΅ Ρ‡Π΅ΠΌ сСтСвыС ΠΏΠΎΠΌΠ΅Ρ…ΠΈ достигнут ΠΌΠ΅ΠΆΠΎΠ±ΠΌΠΎΡ‚ΠΎΡ‡Π½ΠΎΠΉ Смкости, ΠΎΠ½ΠΈ Π΄ΠΎΠ»ΠΆΠ½Ρ‹ ΠΏΡ€ΠΎΠΉΡ‚ΠΈ Ρ‡Π΅Ρ€Π΅Π· ΠΈΠ½Π΄ΡƒΠΊΡ‚ΠΈΠ²Π½ΠΎΡΡ‚ΡŒ рассСяния трансформатора, Ρ‚Π°ΠΊ Ρ‡Ρ‚ΠΎ эти Π΄Π²Π° ΠΊΠΎΠΌΠΏΠΎΠ½Π΅Π½Ρ‚Π° ΠΎΠ±Ρ€Π°Π·ΡƒΡŽΡ‚ Ρ€Π΅Π·ΠΎΠ½Π°Π½ΡΠ½ΡƒΡŽ ΡΠ΅Ρ‚ΡŒ. Если ΠΌΡ‹ ΠΏΡ€Π΅Π΄ΠΏΠΎΠ»ΠΎΠΆΠΈΠΌ (для Ρ†Π΅Π»Π΅ΠΉ сравнСния), Ρ‡Ρ‚ΠΎ Ρ€Π΅Π·ΡƒΠ»ΡŒΡ‚ΠΈΡ€ΡƒΡŽΡ‰ΠΈΠΉ Ρ‚ΠΎΠΊ Ρ€Π°Π·Π²ΠΈΠ²Π°Π΅Ρ‚ напряТСниС Ρ‡Π΅Ρ€Π΅Π· сопротивлСниС зазСмлСния 1 Ом ΠΌΠ΅ΠΆΠ΄Ρƒ ΠΎΠ±ΠΌΠΎΡ‚ΠΊΠΎΠΉ нагрСватСля ΠΈ шасси, ΠΌΡ‹ ΠΌΠΎΠΆΠ΅ΠΌ ΡΠΌΠΎΠ΄Π΅Π»ΠΈΡ€ΠΎΠ²Π°Ρ‚ΡŒ Ρ‚Ρ€ΠΈ основных конструкции трансформатора.

Π”ΠΎΠ±Π°Π²ΠΈΡ‚ΡŒ ΠΊΠΎΠΌΠΌΠ΅Π½Ρ‚Π°Ρ€ΠΈΠΉ

Π’Π°Ρˆ адрСс email Π½Π΅ Π±ΡƒΠ΄Π΅Ρ‚ ΠΎΠΏΡƒΠ±Π»ΠΈΠΊΠΎΠ²Π°Π½. ΠžΠ±ΡΠ·Π°Ρ‚Π΅Π»ΡŒΠ½Ρ‹Π΅ поля ΠΏΠΎΠΌΠ΅Ρ‡Π΅Π½Ρ‹ *